You are on page 1of 247

1.

An 80 year old man previously fit healthy presents severe flinging movements of
left arm.Where is the neurological lesion?

1 ) Caudate nucleus

1.
An 80 year old man previously fit healthy presents severe flinging movements of
left arm.Where is the neurological lesion?
1 ) Caudate nucleus
2 ) Globus pallidus
3 ) Ipsilateral thalamus
4 ) Substantia nigra
5 ) Subthalamic nucleus
Comments:
The presence of severe flinging movements indicating hemibalistic movements. The site of
lesion contralateral subthalamic nucleus. The commonest cause infarction. Usually flinging
movements stop spontaneously in next 4 8 weeks. Tetrabenazine treatment of choice. 5 )

2.
A 52 year old man presented acute dyspnoea. His past history includes 3 vessel coronary
artery bypass surgery Ischaemic Heart Disease hypertension. Examination widespread expiratory
crackles chest X ray confirming pulmonary oedema. He was treated intravenous nitrates
frusemide symptomatic improvement. Investigations at this stage revealed:
Serum sodium
138 mmol/L
Serum potassium
4.2 mmol/L
Serum urea
8.7 mmol/L
Serum creatinine
170 umol/L
Random plasma glucose
10.1 mmol/L
Urinalysis Protein
++
The following day he was switched oral frusemide at a dose of 80 mg daily began Captopril 12.5
mg twice daily, increased 25 mg twice daily. Repeat investigations one week later revealed:
Serum sodium
Serum potassium
Serum urea
Serum creatinine
Fasting plasma glucose

134 mmol/L
5.1 mmol/L
15.7 mmol/L
220 umol/L
6.0 mmol/L

Which of following likely be responsible deterioration in renal function?


1 ) Captopril
2 ) Cholesterol emboli
3 ) Diabetic nephropathy
4 ) Frusemide
5 ) Hypertension
Comments:
This patient coronary artery atheroma therefore could have a renal artery stenosis by same
pathophysiological mechanism. A rise in serum creatinine more than 20% above baseline after
starting an ACEI should prompt clinician stop drug, monitor renal function. The patient does
have diabetes based upon a fasting plasma glucose of only 6 mmol/l. 1 )
3.
A 14 year old girl admitted headache, nausea vomiting. She had previously well but
her symptoms deteriorated over last 12 hours.
On admission, she noted be slightly confused a temperature of 39 degrees centigrade,
3
stiff neck positive Kernigs sign a faint purpuric
rash on knees. Her pressure 90/60
mmHg a pulse of 120 beats per minute.

2 ) Globus pallidus
3 ) Ipsilateral thalamus
4 ) Substantia nigra
5 ) Subthalamic nucleus
Comments:
The presence of severe flinging movements indicating hemibalistic movements. The site of
lesion contralateral subthalamic nucleus. The commonest cause infarction. Usually flinging
movements stop spontaneously in next 4 8 weeks. Tetrabenazine treatment of choice. 5 )

2.
A 52 year old man presented acute dyspnoea. His past history includes 3 vessel coronary
artery bypass surgery Ischaemic Heart Disease hypertension. Examination widespread expiratory
crackles chest X ray confirming pulmonary oedema. He was treated intravenous nitrates
frusemide symptomatic improvement. Investigations at this stage revealed:
Serum sodium
138 mmol/L
Serum potassium
4.2 mmol/L
Serum urea
8.7 mmol/L
Serum creatinine
170 umol/L
Random plasma glucose
10.1 mmol/L
Urinalysis Protein
++
The following day he was switched oral frusemide at a dose of 80 mg daily began Captopril 12.5
mg twice daily, increased 25 mg twice daily. Repeat investigations one week later revealed:
Serum sodium
Serum potassium
Serum urea
Serum creatinine
Fasting plasma glucose

134 mmol/L
5.1 mmol/L
15.7 mmol/L
220 umol/L
6.0 mmol/L

Which of following likely be responsible deterioration in renal function?


1 ) Captopril
2 ) Cholesterol emboli
3 ) Diabetic nephropathy
4 ) Frusemide
5 ) Hypertension
Comments:
This patient coronary artery atheroma therefore could have a renal artery stenosis by same
pathophysiological mechanism. A rise in serum creatinine more than 20% above baseline after
starting an ACEI should prompt clinician stop drug, monitor renal function. The patient does
have diabetes based upon a fasting plasma glucose of only 6 mmol/l. 1 )
3.
A 14 year old girl admitted headache, nausea vomiting. She had previously well but
her symptoms deteriorated over last 12 hours.
On admission, she noted be slightly confused a temperature of 39 degrees centigrade,
stiff neck positive Kernigs sign a faint purpuric rash on knees. Her pressure 90/60
mmHg a pulse of 120 beats per minute.
A diagnosis of meningococcal meningitis confirmed following CT headscan lumbar
puncture. She admitted ITU treated IV cefotaxime 2g tds Benzylpenicillin 2.4g qds. She a
rather stormy admission requiring intubation, ventilation hypotensive episodes. On day 2
of her admission, her urine output falls hourly urine output of approximately 10 mls/hr.
4

Investigations reveal:
Haemoglobin
16.7g/dL
White cell count
16.8 x109/L
Platelets
100 x109/L
Serum sodium
125 mmol/L
Serum potassium
5 mmol/L
Serum urea
6.7 mmol/L
Serum creatinine
100 umol/L
Plasma osmolality
300 mosmol/L
Urine osmolality
325 mosmol/L
Urine urea
120 mmol/L
Urine sodium
75 mmol/L
Select two reasons why this acute tubular necrosis is pre renal failure?
1 ) Her pressure low
2 ) Her haemoglobin 16.7
3 ) Her plasma sodium 125 mmol/l
4 ) Her urine plasma osmolality ratio >1.1
5 ) Her urine plasma urea ratio elevated
6 ) Her urine plasma osmolality ratio reduced
7 ) Her urine output falls
Comments:
In ATN, urine plasma osmolality should be < 1.1, urinary sodium excretion typically
>60mmol/L urinary urea excretion <160mmol/L. If this patient had a physiological oliguria,
there would still be preservation of urine concentration, low urinary sodium. Both ATN pre
renal failure can present a fall in urine output. There such a marked variation in urine urea
concentration, it seldom used as a clinical guide. 3 ) 6)
4.
A 52 year old man presented acute dyspnoea. His past history includes 3 vessel
coronary artery bypass surgery Ischaemic Heart Disease. Examination widespread
expiratory crackles chest X ray confirming pulmonary oedema. He was treated
intravenous nitrates frusemide symptomatic improvement. Investigations at this stage
revealed:
Serum sodium
138 mmol/L
Serum potassium
4.2 mmol/L
Serum urea
8.7 mmol/L
Serum creatinine
170 umol/L
Random plasma glucose
10.1 mmol/L
Urinalysis Protein
++
The following day he was switched oral frusemide at a dose of 80 mg daily began
Captopril 12.5 mg twice daily, increased 25 mg twice daily. Repeat investigations one week
later revealed:
Serum sodium
134 mmol/L
Serum potassium
5.1 mmol/L
5

Serum urea
15.7 mmol/L
Serum creatinine
220 umol/L
Fasting plasma glucose
6.0 mmol/L
Which next best investigation in determining the cause of his renal failure?
1 ) Captopril renogram
2 ) Doppler of renal arteries
3 ) Renal angiography
4 ) Renal biopsy
5 ) Renal isotope scan
Comments:
This patient coronary heart disease. Renal function deteriorated in presence of an ACEI it
important exclude atheroma of renal arteries causing renal artery stenosis. Renal angiography,
particularly MR angiography a good way diagnose renovascular disease without having
administer nephrotoxic contrast, but as gold standard, percutaneous angiography investigation
of choice, also enables angioplasty, if indicated. Renal artery Doppler prone more false
negative results than angiography. 3 )
5.
A 40 year male ulcerative colitis(UC) of 10 years duration presented feeling unwell.
He complained of right upper quadrant pain recently noticed he started itch. His stools
are pale coloured. He on sulphasalazine had only 2 previous minor relapses.
On examination he 5 spider naevi on upper trunk. In his abdomen he a 5 cm
hepatomegaly a tipable spleen, but no ascites.
Platelets
110 x 109/L
Prothrombin time
20 sec
Serum bilirubin
55 umol/L
Serum aspartate aminotransferase
101 IU/L
Serum alanine aminotransferase
38 IU/L
Serum alkaline phosphatase
482 IU/L
Hepatitis B surface antigen
Negative
What is the likely diagnosis?
1 ) Colonic carcinoma
2 ) Drug cholestasis
3 ) Cholangiocarcinoma
4 ) Primary biliary cirrhosis
5 ) Primary sclerosing cholangitis
Comments:
This man cirrhosis portal hypertension, cholestasis liver synthetic failure. Primary sclerosing
cholangitis(PSC) a strong association ulcerative colitis
likely diagnosis. There an increased
risk of cholangiocarcinoma PSC, it possible his deterioration may be due development of
cholangiocarcinoma. Drug cholestasis would explain stigmata of chronic liver disease (spiders,
splenomegaly)., neither would colonic carcinoma. Primary biliary cirrhosis associated UC
uncommon in men. 5 )
6.
A 56 year long standing rheumatoid arthritis presented pain in neck radiating
left hand. She noted wasting in right hand muscles. She was well controlled on
Penicillamine. On examination there was inversions of biceps supinator jerks. The triceps
6

jerk was brisk. Knee reflexes were present. The plantars were equivocal because of
rheumatoid arthritis of Hallux. Fasciculation's were noted, neck movements were
restricted, position sense was slightly impaired.Where is the lesion located?
1 ) Cervico medullary junction
2 ) C1,2
3 ) C5,6
4 ) C7,8
5 ) C8,T1
Comments:
Neck pain radiating hand indicating a radiculopathy. The fact there was inversion of biceps
(C5) supinator (C6) jerks indicating level of lesion at C5,6. The reflexes below lesion (i.e.)
triceps (C7) are brisk. 3 )

7.
A 14 year boy of white Irish parents admitted haematemesis. Gastroscopy demonstrates
bleeding oesophageal varices. Despite being born prematurely at 32 weeks he completely well
until presentation.On examination there are peripheral stigmata of chronic liver disease, but there
a palpable spleen 4 cm below costal margin. nvestigations showed:

Hepatic wedge pressure


6 mmHg (normal <7)
Inferior vena cava
3 mmHg (normal <5)
Which best initial investigation perform?
1 ) Ultrasound Doppler flow of hepatic portal veins
2 ) Splenoportogram
3 ) Liver biopsy
4 ) Laparoscopy
5 ) MRI scan liver
Comments:
This child likely have portal vein thrombosis resulting from neonatal cannulation of his
umbilical vein following his premature delivery. PV thrombosis should be easily demonstrated on
Ultrasound which a cheap non invasive procedure, one would hope this would be performed
prior considering hepatic venous pressure studies ! 1 )
8.
A 68 year old man was admitted nausea general malaise He was a hill farmer
continued work his farm his sons on which he kept mostly sheep. Over last two weeks,
since returning from Holiday in Spain, he had become increasingly fatigued. The only other
symptoms of note were a 3 month history of poor appetite a 8 kg weight loss. He was
receiving thyroxine 100 mcgm daily having diagnosed hypothyroidism by his GP 9 years
previously. He was a smoker of 5 cigarettes per day had drunk more alcohol than usual
whilst on holiday but usually drank about 12 units of alcohol daily.
On examination, he was sun tanned, slightly confused, appeared dehydrated had a pulse
of 92 bpm regular, a temperature of 37.2oC a pressure of 100/80 mmHg. Cardiovascular
respiratory examination were unremarkable. He had a slight liver edge on palpation
neurological examination was normal. Investigations revealed:
Serum sodium
125 mmol/L
Serum potassium
5.6 mmol/L
Serum corrected calcium
2.73 mmol/L
Serum standard bicarbonate
15 mmol/L
Serum urea
22 mmol/L
Plasma TSH
6 mU/L
(NR 0.4 4.0)
7

Which of following
1
2
3
4
5

)
)
)
)
)

likely diagnosis?

Bronchogenic carcinoma Syndrome of Inappropriate ADH secretion.


Hypoadrenalism
Hypothyroidism
Primary hyperparathyroidism
Sarcoidosis

Comments:
This farmer a three month history of weight loss, anorexia fatigue. He recently returned from
Spain but this a distractor as his occupation.
The investigations show hyponatraemia, hyperkalaemia, uraemia, hypercalcaemia slightly
elevated TSH which suggest diagnosis of Hypoadrenalism. He known have an autoimmune
disease hypothyroidism a diagnosis of Addisons suggested. Slight elevation of TSH a mild
hypercalcaemia are typical of hypoadrenalism. Bronchogenic Carcinoma SIADH would be
expected produce a hyponatraemia but potassium urea.
The patients symptoms do fit hypothyroidism (weight loss) uraemia hyperclacaemia would
be expected. Although sarcoidosis may produce hypercalcaemia, hyponatraemia would be a
typical finding respiratory signs may be expected. Primary hyperparathyroidism would produce
such problems such a calcium concentration. 2 )
9.
A 27 year old refugee from Cambodia presents a short history of fatigue jaundice.
He lived in UK last 12 months.
His mother died of liver disease age 55 his older brother died of liver cancer 3 years
previously in Cambodia age 40. His brother was known have hepatitis B. He consumes 30
40 units of alcohol a week.
Investigations show:
Haemoglobin
WBC
Platelets
Serum bilirubin
Serum gamma glutamyl transferase
Serum alkaline phosphatase
Serum aspartate aminotransferase
Serum alanine aminotransferase
Serum alpha fetoprotein
HbsAg
Positive
HBeAg
Positive
Anti HBc (IgM)
Positive
Anti HAV (IgM)
Negative
Anti HAV (IgG)
Positive
Anti HCV
Negative

14.5 g/dL
7.6 x 109/L
156 x 109/L
213 umol/L
200 IU/L
150 IU/L
912 IU/L
1450 IU/L
45 kU/L

(NR < 10)

An ultrasound scan of his liver shows mild hepatomegaly an echogenic liver but other
abnormality.What is the likely diagnosis?
1
2
3
4
5

)
)
)
)
)

Alcohol related hepatitis


Acute Hepatitis B
Flare of chronic Hepatitis B
Hepatocellular cancer
Hepatitis E

Comments:
This man's brother was known have HBV it likely their mother was also infected given her
demise from liver disease at a young age. In developing countries, vertical transmission from
mother child commonest mode of acquiring chronic HBV infection. It likely this patient
chronic infection now in Immune clearance phase of chronic infection a severe flare of his
hepatitis. He likely clear HBeAg during this illness. Although Anti HBc (IgM) usually associated
acute infection it often seen in acute flares of chronic disease. The ALT too high alcohol
related hepatitis. The AFP commonly elevated during acute hepatitis due hepatic regeneration
HCC unlikely in a 27 yr would account raised ALT. Hepatitis E endemic in UK a very
unlikely diagnosis. 3 )
10. A 56 year long standing rheumatoid arthritis presented pain in neck radiating
left hand. She noted wasting in right hand muscles. She was well controlled on
Penicillamine. On examination there was inversions of biceps supinator jerks. The triceps
jerk was brisk. Knee reflexes were present. The plantars were equivocal because of
rheumatoid arthritis of Hallux. Fasciculation's were noted, neck movements were
restricted, position sense was slightly impaired.What is the likely diagnosis?
1 ) Atlanto axial subluxation
2 ) B12 deficiency
3 ) Cervical cord tumour
4 ) Cervical myelopathy
5 ) Motor neurone disease
Comments:
The presence of inversion of biceps supinator reflexes indicating cervical myelopathy at C5, 6.
This a very important sign which distinguish cervical myelopathy from motor neuron disease. 4 )
11. A 50 year old man presents tingling in left upper limb. The pain originated in neck
radiated down left arm. He proceeded have numbness paraesthesia in left lower limb.
On examination he had restriction of neck movements there was a mild wasting be noted
in left biceps. There was inversion of supinator biceps jerks. His knee jerk ankle jerk
were hyper reactive he a positive extensor plantar response. He then developed
paraesthesia numbness of right lower limb. A diagnosis of cord compression was made
he underwent a surgical decompression.
Post surgery was complicated by septicaemia urinary tract infection he remained in bed 4
days. He subsequently developed inability dorsiflex his right foot right big toe. There was
numbness on outside of foot there was decreased eversion, but inversion was normal.
His reflexes remained as before.What is the cause cause of his problem?
1 ) Common peroneal nerve palsy
2 ) L4 root lesion
9

3 ) Recurrence of original cord compression


4 ) Spinal cord infarction
5 ) Sciatic nerve palsy
Comments:
The commonest cause of acute foot drop after prolonged bed rest entrapment common
peroneal neuropathy at neck of fibula. Typically there weakness of ankle dorsiflexion, eversion,
diminished sensation of lateral aspect of leg dorsum of foot. The ankle reflex remains intact. 1 )
12. A 40 year male ulcerative colitis of 10 years duration presented feeling unwell. He
complained of right upper quadrant pain recently noticed he started itch. His stools are
pale coloured. He on sulphasalazine had only 2 previous minor relapses.
On examination he five spider naevi on upper trunk. A liver edge palpable 5cm below
right costal margin. The tip of spleen also palpable but there are signs of ascites.
Platelets
110 x 109/L
Prothrombin time
20 sec
(Control 11.5 15.5)
Serum bilirubin
55 umol/L
Serum aspartate aminotransferase 101 IU/L
Serum alanine aminotransferase
38 IU/L
Serum alkaline phosphatase
482 IU/L
Hepatitis B surface antigen
Negative
Which investigation is likely to confirm the diagnosis ?
1 ) Liver ultrasound
2 ) Liver biopsy
3 ) ERCP
4 ) Anti mitochondrial antibodies
5 ) Withdrawal of sulphasalazine
Comments:
This man PSC, a diagnosis made on ERCP ( intra extra hepatic bile duct stricturing). Liver
ultrasound likely show a cirrhotic looking liver but confirm a cause. Liver biopsy will confirm
biliary cirrhosis but does provide information on whether there a dominant extra hepatic
stricture which may be amenable endoscopic intervention. AMAs are associated PBC PSC.
Withdrawal of his medication will be of benefit as this chap established liver disease due PSC
rather than drug related cholstasis. 3 )
13. A 28 year trainee accountant presents weight loss of one stone in 4 weeks,
diarrhoea mild abdominal pain. He had not travelled abroad recently. On examination he
had an aphthous ulcer on soft palate.
Haemoglobin
MCV
Serum corrected calcium corrected
Serum total protein
Serum albumin
Serum IgG
Serum IgA

9.7 g/dL
70 fL
2.05 mmol/L
65 g/L
34 g/L
15 mg/L
< 0.1 mg/L

10

(NR 6 13)
(NR 0.8 3.0)

Serum IgM
Anti endomesial IgA antibody

2.0g/L

(NR 0.4 2.5)

What is the likely diagnosis?


1 ) Coeliac disease
2 ) Crohns disease
3 ) Ulcerative colitis
4 ) Intestinal lymphoma
5 ) Whipples disease
Comments:
This patient selective Ig A deficiency, a known association coeliac disease. The symptoms, iron
deficiency anaemia, aphthous mouth ulceration are all features of coeliac disease. The anti
endomysial IgA negative because patient IgA deficient. Anti Tissue transglutaminase (IgG)
testing should be performed. 1 )
14. A 42 year old widow presents a one week history of progressive confusion unsteady
gait. She works as a barmaid lives in poor social circumstances.
On examination she malnourished disorientated. She nystagmus unable abduct either
eye. The pupils are sluggish unequal. Ankle jerks are absent but upper limb reflexes are
present. Shortly after her admission you are called ward as she become very drowsy
collapsed on floor.
Investigations on admission showed:
Haemoglobin
MCV
White cells
Platelets
Serum sodium
Serum potassium
Serum bilirubin
Serum gamma glutamyl transferase
Serum alkaline phosphatase
Serum aspartate aminotransferase
Serum alanine aminotransferase
Serum albumin
Prothrombin time

11.4 g/dL
99 fL
5.6 x109/L
230 x109/L
129 mmol/L
3.2 mmol/L
27 umol/L
440 IU/L
180 IU/L
90 IU/L
45 IU/L
33 g/L
12 sec

The first investigation should be:


1
2
3
4
5

)
)
)
)
)

CT head
Blood glucose
EEG
Blood cultures
B12 folate levels

Comments:
11

Wernickes encephalopathy likely diagnosis given history examination findings on a


background of likely alcoholism. Although all of above investigations may need be considered
exclude other contributing aetiologies her presentation only glucose can be done quickly
cheaply at bedside. Hypoglycaemia may have precipitated her deterioration a finger prick
glucose estimation should be first investigation. 2 )
15. A 40 year male presents an elective ERCP a common bile duct stone. Post ERCP he
develops acute septicaemia.
Pre ERCP
Serum sodium
136 mmol/L
Serum potassium
4 mmol/L
Serum chloride
100 mmol/L
Serum bicarbonate
28 mmol/L
Serum urea
4 mmol/L
Serum creatinine
96 umol/L
Post ERCP
Serum sodium
140 mmol/L
Serum potassium
4 mmol/L
Serum chloride
100 mmol/L
Serum bicarbonate
26 mmol/L
Serum urea
40 mmol/L
Serum creatinine
720 umol/L
All physical findings are his chest clinically clear. He producing 40 ml of urine per hour.
Based on recent clinical trials which one of the following is evidenced based?
1 ) Give high dose frusemide
2 ) Give low dose dopamine
3 ) Give low dose dopamine, frusemide mannitol
4 ) Give frusemide mannitol
5 ) None of above
Comments:
This man post ischaemic, non oliguric renal failure due sepsis. There clear evidence from
clinical trials support use of any of above agents. There some evidence from animal studies
these agents given early in septicaemia may help prevent renal damage. The use of dopamine,
frusemide, mannitol in acute renal failure remains controversial. 5 )
16. A 78 year old man presented an unsteady gait. He was noted be becoming impaired
his memory agitated at nights. His GP started an antidepressant. He was incontinent of
urine. He was a heavy smoker had lost 2 stones in weight over 2 months. His sugar was
10 mmol/l.
Which of the following is the next best investigation?
1 ) CT Head
2 ) CXR
3 ) Glycosylated Hb
12

4 ) Thyroid function test


5 ) Urinary Sodium
Comments:
The triad of unsteady gait, memory impairment urinary incontinence suggests diagnosis of
pressure hydrocephalus. CT head of brain investigation of choice show enlarged ventricles out
of proportion of cerebral atrophy. 1 )
17. An 18 year old presented bilateral ptosis tiredness towards afternoons. She had a
short tensilon (edrophonium test which was positive). A diagnosis of myasthenia Gravis
was made she was stated on pyridostigmine. She now relapses given edrophonium
intravenously. However her condition deteriorates her forced expiratory volume falls 1.0.
She transferred high dependency units. An initial CT scan chest x ray were normal.
What should be the next management step?
1 ) Azathioprine
2 ) Emergency thymectomy
3 ) Intravenous methylprednisolone
4 ) Neostigmine
5 ) Plasmaphoresis
Comments:
The diagnosis Myasthenia crisis. The treatment of choice either iv immunoglobulins plasma
exchange. Transfer ITU essential because patients may deteriorate rapidly needing intubation
ventilation. 5 )
18. A 14 year boy of white Irish parents admitted haematemesis. Gastroscopy
demonstrates bleeding oesophageal varices. Despite being born prematurely at 32 weeks
he completely well until presentation. On examination there are peripheral stigmata of
chronic liver disease, but there a palpable spleen 4 cm below costal margin.
Invasive venous pressures are as follows:
Hepatic wedge pressure
6 mmHg (normal <7)
Inferior vena cava
3 mmHg (normal <5)
What is the likely diagnosis?
1
2
3
4
5

)
)
)
)
)

Sarcoidosis
Longstanding portal vein thrombosis
Hepatic vein thrombosis
Schistosomiasis
Alpha one antitrypsin deficiency

Comments:
The hepatic venous pressure gradient (normal HVPG = 1 5 mmHg) means portal hypertension
related post sinusoidal intrinsic liver disease such as cirrhosis (caused in children by metabolic
disorders such as A1ATD ) post hepatic venous obstruction (HV thrombosis). The obstruction
must be pre sinusoidal. Sarcoidosis a very rare cause of pre sinusoidal portal hypertension,
particularly in white children. Schistosomiasis leading cause of pre sinusoidal hypertension
worldwide but unlikely in an Irish boy. Thrombosis of portal vein a well recognised
complication in premature neonates due cannulation of umbilical vein during neonatal
intensive care. 2 )

13

19. A 50 year old male presents a 4 weeks history of exertional shortness of breath. He
a long history of depression which he originally took Lithium Carbonate but stopped this
medication over 5 years ago since then taking Paroxetine. He was also diagnosed
asthma 2 years ago by his general practitioner which he was prescribed salbutamol
inhalers taking ibuprofen over last six months Osteoarthritis of hips. The only other
relevant information two weeks ago he returned from a 6 week holiday in Australia where
he spent a week in outback. He had frequently been bitten by mosquitoes.
Examination reveals a rather ill and tanned individual a temperature of 37oC, a pressure
of 146/86 mmHg a pulse of 106 beats per minute. No specific abnormalities were noted
except scattered bibasal fine crackles occasional wheeze on chest examination.
Investigations reveal:
Haemoglobin
14g/dL
White cell count
6 x 109/L
Neutrophil count
3.5x109/L
Lymphocyte coun
2 x 109/L
Monocyte count
normal
Eosinophils
1 x 109/L
ESR (Westergren)
65 mm/1st hour
Serum sodium
136 mmol/L
Serum potassium
7.0 mmol/L
Serum chloride
106 mmol/L
Serum bicarbonate
15 mmol/L
Serum creatinine
600 umol/L
Ultrasound of kidneys:
Right kidney 12cm, left kidney 13cm. No obstruction seen
What is the likely diagnosis?
1
2
3
4
5

)
)
)
)
)

Amyloid
Analgesic nephropathy
Churg Strauss syndrome
Membranous glomerulonephritis
Rapidly progressive glomerulonephritis

Comments:
This patient may have an inflammatory pathology as indicated by ESR. He developed asthma in
presence of a raised peripheral eosinophil count. The presence of above renal impairment
lends a possible diagnosis of Churg Strauss syndrome, which can cause a pauci immune small
vessel vasculitis, glomerulonephritis. ANCA titres should be checked.3 )

14

1.
A 52 year old presents tiredeness, weight loss, anxiety difficulty sleeping. She was
receiving a combined cyclical oestrogen/progesterone hormone replacement therapy. Examination
a thin patient a pulse of 110 beats per minute, a fine tremor proximal myopathy. Her spleen tip
was barely felt. Initial investigations showed:
Serum total thyroxine
250 nmol/L
(NR 60 140)
Plasma TSH
< 0.1 mU/L
(NR 0.4 4.0)
Serum alkaline phosphatase
202 IU/L
(NR 50 100)
Serum gamma glutamyl transferase
40 IU/L
(NR 10 50)
The general practitioner commenced Carbimazole 20 mg twice daily together propranolol. On
review six weeks later she looked euthyroid. Repeat investigations showed:
Serum total thyroxine
180 nmol/L
Plasma TSH
2.2 mU/L
Serum alkaline phosphatase
160 IU/L
Serum gamma glutamyl transferase
35 IU/L

The dose of carbimazole was decreased 20 mg daily. After one years treatment where
patient felt increasingly tired, GP decided refer her outpatient. Two weeks prior her
appointment she had developed a chest infection which her GP had prescribed
erythromycin. At clinic following results were noted:
Serum total thyroxine
80 nmol/L
Plasma TSH
10.2 mU/L
Serum alkaline phosphatase
102 IU/L
Serum gamma glutamyl transferase
36 IU/L
What is the cause of her elevated Alkaline Phosphatase?
1 ) Osteoporosis
2 ) Oestrogen therapy
3 ) Pagets disease
4 ) Progestogen therapy
5 ) Thyotoxicosis
Comments:
High bone turnover osteoporosis may be associated thyrotoxicosis. Bone turnover involves
increased osteoclastic osteoblastic activity, leading elevated alkaline phosphatase levels derived
from bone. Pagets disease associated raised alkaline phosphatase but one should apply
Ockhams Razor find two diagnoses when there one unifying diagnosis. Osteoporosis
associated a raised alkaline phosphatase under circumstances. 5 )
2.
A 62 year old woman referred a three month history of increasing weakness affecting left
more than right arm. She also complains of tingling numbness affecting left thumb which tends
be worse at nights. She dropped cups of tea twice in last month but attributes this her own
clumsiness. She denies any visual, speech, bowel bladder symptoms. Occasionally she gets some

15

neck discomfort on turning her head but denies any recent past history of neck trauma. Past
history includes a left mastectomy 10 years ago breast cancer. Her sister died of a brain tumour.
On examination, her cranial nerves are intact. No increase in tone detectable in all limbs. She mild
weakness when flexing supinated forearm against resistance, diminished biceps brachioradialis
reflexes increased triceps reflexes bilaterally. The rest of her muscle power reflexes are normal.
Plantar responses are flexor bilaterally. Sensory examination unremarkable pinprick joint position
sense. What is the likely diagnosis in this woman?

1
2
3
4
5

) Amyotrophic lateral sclerosis


) Cervical myelopathy
) Hereditary spastic paraparesis
) Multiple sclerosis
) Ulnar nerve entrapment

Comments:
The symptoms in this patient best fit a spondylotic problem affecting C5/6 level, thus C6
cervical root. Symptoms signs of a C6 root lesion include paraesthesias in thumb lateral distal
forearm, weakness of brachioradialis, biceps, triceps diminished biceps brachioradialis
reflexes in conjunction an increased triceps reflex. Patients do always present all of these
features but key finding here inverted upper limb reflexes (brisk triceps diminished biceps
brachioradialis). The presence of sensory symptoms make amyotrophic lateral sclerosis unlikely.
Ulnar nerve entrapment does cause reflex changes. Hereditary spastic paraparesis usually
begins in lower limbs patient a bit it only start manifesting now. Multiple sclerosis also less
common in this age group. 2 )
3.
A 78 year referred out patient department hypertension. She generally very well
but recently seen gynaecologists incontinence at which point hypertension was noted.
Examination reveals a well patient a BMI of 25, a pulse of 80 beats per minute, a pressure
of 188/78 mmHg heart sounds. Fundal examination reveals silver wiring.
Which of following class of drugs would be the appropriate treatment of this patients
hypertension?
1
2
3
4
5

)
)
)
)
)

ACE inhibitor
Alpha Blockers
Beta blocker
Calcium antagonist
Thiazide diuretic

Comments:
This elderly isolated systolic hypertension (systolic greater than 160 diastolic below 90
mmHg). Evidence from studies such as Systolic Hypertension in Elderly Program Syst Eur
indicate both thiazides calcium antagonists are drugs of choice in terms of reducing morbidity
mortality in this patient group. In this patients case incontinence may be exacerbated by
diuretic therapy a calcium antagonist may be more appropriate. 4 )
4.
An eighteen year student presented sudden onset of left sided chest pain
breathlessness. He had a history of cardiac surgery as a child. He smoked ten cigarettes a
day. He denied any alcohol elicit drug use. He had returned from a holiday in Thailand ten
days ago. On examination he was tall thin. There was clubbing lymphadenopathy. He was
cyanosed. He had a resting tachycardia there was an audible click expiration. What is the
likely diagnosis?
16

1
2
3
4
5

)
)
)
)
)

Marfans' Syndrome
Mitral Valve Prolapse
Tzeize's syndrome
Pulmonary Embolism
Viral pericarditis

Comments:
Sudden onset chest pain breathlessness are likely due a PE pneumothorax. Hamman's Sign (or
'Crunch') a crunching systolic sound heard over sternal edge in mediastinal emphysema left
apical pneumothoraces. It can be dependent on patients position when auscultating. 1 )
5.
A 55 year male consulted his General Practitioner a three month history of lethargy
weight loss. Six years previously he was diagnosed diabetes mellitus was receiving
Glibenclamide 10 mg daily Metformin 1g twice daily.
On examination he was noted have a BMI of 25.6 kg/m2, a pulse of 88 beats per minute a
pressure of 164/102 mmHg. Fundal examination numerous dot haemorrhages in temporal
retina of both eyes occasional hard exudates. Loss of position vibration sensation were
also noted mid tibia bilaterally.
Investigations revealed:
Haemoglobin
White cell count
Platelets
Serum sodium
Serum potassium
Serum urea
Serum creatinine
HbA1c
Urinalysis

14g/dL
4.8 x 109/L
195 x 109/L
137 mmol/L
4.6 mmol/L
16.7 mmol/L
220 umol/L
9.3%
Protein++, Blood +

Which of the following is appropriate therapeutic strategy this patient?


1
2
3
4
5

)
)
)
)
)

Change Glibenclamide insulin


Maximise his current oral hypoglycaemic therapy
Rosiglitazone
Stop Metformin
Stop metformin Glibenclamide start insulin

Comments:
In this patients case weight loss, modest BMI poor glycaemic control established retinopathy
nephropathy should be switched insulin. Most authorities recommended metformin should be
stopped in patients a creatinine above 150 micromol/l although this universal policy many
patients continue on metformin creatinines much higher than 150 without any ill effect.
Although Rosiglitazone could be added either metformin glibenclamide, there would be little
benefit gained in this manoeuvre as his problem now appears be weight loss osmotic
17

symptoms suggesting insulinopaenia. Similarly there would be little benefit in maximising his
oral hypoglycaemic agents which are already at reasonably top dose. 5 )
6.
A 22 year man, normally fit well, was referred hospital by his GP. His only
symptoms were of blurred vision headache had present preceeding two days. On
further questioning patient also said at he had passing much urine he had noticed his
breathing had worse on exertion.
There was history of ankle swelling, but he fell over 3 days previously hurt his leg he had
unable go out of house since. There was history of chest pain.
On examination he had a regular pulse of 110/minute a pressure of 200/120 mmHg JVP
of 5cm. His heart sounds were normal. Chest on auscultation fine basal crepations a
respiratory rate of 22/minute a sighing pattern. He had oedema of his right leg side. His
abdomen was soft non tender masses. Fundoscopy was normal.
An urinary catheter was instered. He was found have a residual volume of 50 mls of dark
urine. Urinalysis showed Blood+++ Protein++. Microscopy showed organisms, but scanty
hyaline casts fewer than 10 red cells per high powered field.Investigations revealed:
Serum sodium
135mmol/l
Serum potassium
7.4mmol/l
Serum urea
19.5 mmol/l
Serum creatinine
1044 mmol/l
Serum calcium
2.0 mmol/l
Serum phosphate
2.6 mmol/l
Bicarbonate
17 mmol/l
What treatment should he receive immediately?
1 ) Intravenous fluids
2 ) Intravenous insulin + dextrose + salbutamol
3 ) Intravenous sodium bicarbonate
4 ) Oral calcium resonium
5 ) Dialysis
Comments:
This man Rhabdomyolysis immediate treatment correction of his Potassium then correction of
fluid balance acidosis. IV insulin Dextrose number one thereafter if urine output cannot be
improved Potassium remains elevated. Dialysis maybe necessary. 2 )
7.
A 58 year male was reviewed by his General Practitioner a three month history of
lethargy weight loss. He had a 10 year history of type 2 diabetes which he was treated
Glibenclamide 10 mg daily Metformin 1g twice daily. He was receiving amlodipine 10mg
daily as treatment his hypertension. He confessed poor compliance diet, he smoked 5
cigarettes daily drank approximately 12 units of alcohol weekly.
On examination he was noted have a BMI of 25.6 kg/m2, a pulse of 88 beats per minute a
pressure of 164/102 mmHg. Fundal examination dot haemorrhages in temporal retina of
both eyes occasional hard exudates. Loss of position vibration sensation were also noted
mid tibia bilaterally. Peripheral pulses were all preserved.
Investigations revealed:
Haemoglobin
White cell count
Platelets

14g/dL
4.8 x 109/L</TD
195 x 109/L

18

Serum sodium
Serum potassium
Serum urea
Serum creatinine
HbA1c
Urinalysis

137 mmol/L
4.6 mmol/L
16.7 mmol/L
220 umol/L
9.3%
Protein ++ Blood +

(NR < 6%)

Which of the following is appropriate treatment for this patients blood pressure?
1 ) ACE inhibitor (+Frusemide) keep BP < 160/80
2 ) ACE inhibitor (+Frusemide) keep BP < 140/80
3 ) Increase antihypertensives (but exclude ACE) keep BP < 160/80
4 ) Increase antihypertensives (but exclude ACE) keep BP < 140/80
5 ) Lifestyle advice
Comments:
This patient appears have an established nephropathy ++ protein on urinalysis poor
hypertensive control. Without appropriate investigation it difficult know whether creatinine of
220 due renal artery stenosis. He requires investigation of protein in urine, but in first
instance patient needs urgent control of his pressure. Target BP hypertensive patients should
be less than 140/80, as described in Joint Guidelines lower in patients established renal
disease. Frusemide recognized as part of armoury in treatment of hypertension whereas
Thiazide diuretics are. 4 )
8.
A 48 year man adult Polycystic Kidney disease who on maintenance Haemodialysis
called Transplantation. He had his usual dialysis on evening before. On examination he a
JVP of 3cm, BP 140/85 a clear chest on auscultation. His weight 1kg above Dry Weight.
Investigations show:
Serum sodium
138 mmol/L
Serum potassium
5.5 mmol/L
Serum bicarbonate
19 mmol/L
Electrocardiogram
Normal
The transplant a good match surgery planned later on day.
What treatment should he receive prior theatre?
1 ) Intravenous fluids
2 ) Intravenous insulin + dextrose +/ salbutamol
3 ) Intravenous sodium bicarbonate
4 ) Oral calcium resonium
5 ) Two hours of hemodialysis
Comments:
Haemodialysis will correct his acid base electrolyte disturbance. 5 )
9.
A 52 year old man comes outpatient clinic his wife. Over last two months, he
complains of hearing strange noises occasionally non threatening voices. His wife says he
also hears music. On more detailed questioning, he admits becoming more withdrawn
recently would spent of his time now doing nothing. His sleep poor he commonly wakes
up at 2 3 a.m. in mornings. His appetite fallen off he eating very little, consequently
losing about 10 kg in last three months. He admits drinking one a half bottles of whisky
a day.
19

During conversation, he appears calm, his speech clear articulate, but his attention poor.
He tremor. His three minute recall of a given address impaired. He does exhibit any
clouding of consciousness there suggestion of delusions paranoid symptoms.
What is the likely diagnosis in this man?
1 ) Alcoholic hallucinosis
2 ) Korsakoffs psychosis
3 ) Major depression psychosis
4 ) Psychotic depression
5 ) Schizophrenia
Comments:
The presentation suggestive of major depression because of psychomotor retardation. Typical
vegetative symptoms include anorexia, weight loss insomnia, particularly early morning
awakening. Psychotic symptoms such as delusions hallucinations may occur in depression,
when they do, treatment both an antidepressant an antipsychotic indicated. In alcohol induced
psychotic disorder hallucinations, patient may have auditory hallucinations, usually voices. The
voices are characteristically maligning, reproachful threatening. The hallucinations usually last
less than a week. After episode, patients realise hallucinatory nature of symptoms.
Korsakoffs psychosis characterised by both anterograde retrograde amnesia, confabulation
early in course. In psychotic depression, depression of psychotic intensity delusional
convictions of disease, putrefaction poverty, contaminating others causing evil. There may also
be hallucinations, typically accusing derogatory voices. Core symptoms of schizophrenia are
delusions, hallucinations, disorganised speech, negative symptoms (e.g. blunted affect poverty
of speech) disorganised behaviour. 3 )
10. A 55 year old man presents left loin pain. His pain radiates groin from his left loin
severe in nature coming in spasms. He previously had a resection of small bowel a
jejunocolic anastomosis Crohns disease. Prior admission he had eating well a diet, high
in fibre usually drinks 3 litres of fluid per day. His Crohns disease had quiescent last
twelve months on steriods. His bowel habit was him two loose stools per day.
On examination his pressure 180/70 mmHg. His abdomen soft, but he tender in left
loin.Investigations show:
Urinalysis
Blood +++, protein, nitrates
Urine pH
5.5
A plain abdominal kidney/ureter/bladder (KUB) x ray shows a radio opaque area over left
ureter. An intravenous urogram (IVU) confirmed presence of a small calculi.
What is the likely cause of his renal stone?
1 ) Calcium
2 ) Cysteine
3 ) Oxalate
4 ) Urate
5 ) Xanthine
Comments:
Oxalate stones are uncommon in dietary excess of oxalate. However enteric oxaluria may occur
in a number of disorders in which malabsorption results in excessive colonic absorption of
oxalate. These include coeliac disease, Crohns disease, chronic pancreatitis short bowel
syndrome. High fluid intake calcium carbonate are mainstay of prevention. 3 )
11. A 62 year male referred impotence. He was diagnosed diabetes mellitus 10 years
ago was initially treated diet but required metformin over last three years. Five years
20

previosuly he underwent a left hip replacement. Over last two years he aware of
deteriorating erectile dysfunction now totally impotent. He shaves daily
aware of any
change in body hair. He a non smoker drinks approximately ten units of alcohol weekly.
Examination reveals an obese male a pressure of 146/88 mmHg. And secondary sexual
characteristics. Testicular examination reveals testes of approximately 15 mls in volume.
There are abnormalities on cardiovascular, respiratory abdominal examinations.
Investigations reveal:
Haemoglobin %
14.2 g/dL
White cell count
9.0 x 109/L
Platelets
188 x 109/L
Serum sodium
145 mmol/L
Serum potassium
4.5 mmol/L
Serum urea
7.2 mmol/L
Serum creatinine
110 mol/L
Serum alkaline phosphatase
88 IU/L
(NR 50 120)
Serum aspartate aminotransferase
40 IU/L
(NR 20 50)
Serum gamma GT
42
(NR 10 50)
HbA1c
7.8 %
Fasting plasma glucose
7.8 mmol/L
Plasma testosterone
7.1 nmol/L
(NR 9.8 33)
Plasma FSH
4.1 mU/L
(NR 3 12)
Plasma luteinizing hormone
5.1
(NR 3 10)
Which of following would you select as further investigation of this patient?
1 ) Ferritin
2 ) MRI scan head
3 ) Oestradiol concentration
4 ) Prolactin concentration
5 ) Ultrasound testes
Comments:
This patient Hypogonadotrophic hypogonadism (HH) LH/FSH low testosterone concentrations.
HH a relatively common scenario associated type 2 diabetes. The exact mechanism responsible
unknown. Haemochromatosis seems unlikely in absence of suggestive symptoms signs
(arthritis, pigmentation, hepatomegaly, deranged LFTs). Hyperprolactinaemia may be associated
a HH signs such as galactorrhoea may be present. However, as it appears
pituitary/hypothalamic axis functioning properly it may be worth radiological imaging MRI
would be best imaging technique.2)
12. A 45 year lady Chronic Renal failure secondary Systemic Lupus Erythromatosis
seen in a low clearance clinic as a routine follow up. Her Joints have causing some
discomfort she taking Naproxen PRN as well as Prednisolone 2.5mg od Azathioprine
50mg od disease control. She stable renal function a Creatinine of 300mmol/l a
Creatinine clearance of 18ml/min. Associated her Chronic renal failure she controlled
secondary hyperparathyroidism. She Anaemic last 6 months investigation of dyspepsia
an OGD showed only mild gastritis. She already on oral ferrous sulphate 200mg tds 3
months.Investigations show:
21

Haemoglobin%
9.4g/dL
hypochromic red cells
12%
Platelets
180 x 109/L
White cell count
6.4 x 109/L
Serum folate
4.0ug/L
(NR 2 11)
Serum ferritin
230ug/L
(NR 15 300)
Transferrin saturation
17%
What therapeutic intervention should now be considered?
1 ) Folate
2 ) GM CSF
3 ) Intravenous iron
4 ) Subcutaneous erythropoetin
5 ) Vitamin B12
Comments:
Intravenous iron first intervention as her percentage of hypochromic red cells 12% ferritin
represents an acute phase protein. She responded oral iron. Epo would be a valid alternative
but only once iron deficiency corrected. You would check a B12 level but lots of Iron studies
here."Iron also essential hemoglobin formation. The iron status of patient chronic Kidney
Disease (CKD) must be assessed adequate iron stores. (National Kidney Foundation
guidelines)3)
13. A 27 year referred by her GP. being 10 weeks pregnant. Three months ago she was
diagnosed thyrotoxicosis an elevated T4 concentration suppressed TSH concentration. At
stage her GP started her on carbimazole. At presentation she a pulse of 90 beats per
minute, a fine tremor lid lag. Blood pressure 118/80 mmHg she a palpable goitre.
From following select appropriate treatment this patient?
1 ) Continue carbimazole
2 ) Radioactive iodine
3 ) Stop all drugs during pregnancy
4 ) Switch propylthiouracil
5 ) Thyroidectomy
Comments:
This patient thyrotoxicosis now pregnant. Thyrotoxicosis itself associated poor pregnancy
outcome IUGR miscarriage. Therefore thyrotoxicosis needs be treated during pregnancy anti
thyroid medication. The patient should be rendered euthyroid then this should be maintained
on lowest dose of anti thyroid medication maintain euthyroidism. A block replacement regime
contra indicated as both carbimazole propylthiouracil cross placenta far better than Thyroxine
so may induce fetal hypothyroidism. There little choose between carbimazole propylthiouracil.
It was once considered carbimazole induced aplasia cutis in fetus but this more recently
disputed as aplasia cutis may be an effect of thyrotoxicosis rather than a side effect of
carbimazole. Consequently, patient should continue treatment Carbimazole. Radioactive iodine
absolutely contra indicated in pregnancy. Any surgery should be reserved last resort in
pregnancy as it associated increased risk of miscarriage. The pregnancy can progress without
problems if thyrotoxicosis adequately treated. 1 )
14. A 20 year student presents sleepiness, weakness vivid dreams which have
occurred over last two months.
22

She a six year history of type 1 diabetes using basal bolus insulin consisting of three
times daily Lyspro insulin evening long acting insulin last six months. Prior she had
receiving twice daily insulin but had noted rather eratic control after she attended
University. She generally adhered a good diet regularly monitored her BMs twice daily
which tend be below 10. She receiving other treatment except insulin. She does smoke,
lives in a flat two other student colleagues binge drinks often on Saturday nights.
Examination reveals a well a BMI of 23 kg/m2, a pulse of 80 bpm a pressure of 112/70
mmHg. No abnormalities are noted on examination.Investigations reveal:
Haemoglobin
15.2 g/dL
White cell count
6.8 x 109/L
Platelets
280 x 109/L
Serum sodium
146 mmol/L
Serum potassium
3.9 mmol/L
Serum urea
5.5 mmol/L
Serum creatinine
88 umol/L
Plasma glucose
7.9 mmol/L
HbA1c
6.2%
(NR < 6%)
What is the likely explanation of her symptoms?
1 ) Hypoglycaemic episodes
2 ) Narcolepsy syndrome
3 ) Schizophrenia
4 ) Sleep apnoea
5 ) Temporal lobe epilepsy
Comments:
Seizures at night are associated sleep disruption deprivation of REM sleep, are thus generally
associated vivid dreams. Narcolepsy associated hypersomnolence, cataplexy, sleep paralysis
hypnagogic hallucinations classically associated vivid dreams. Sleep apnoea associated vivid
dreams, but would be very unlikely in a young, non obese female. Schizophrenia an unlikely
cause of this symptom. The likely cause of this symptom of vivid dreams in this patient
nocturnal hypoglycaemia. REM sleep disruption may lead daytime weakness somnolence. 1 )
15. A 38 year old man presents an episode of right sided weakness affecting his right
arm leg. The weakness occurred while he was eating breakfast resolved completely in 30
minutes. Three months earlier he an episode of slurred speech lasting a few minutes had
being investigated extensively in hospital. Aspirin 75 mg had started as treatment.
On examination, he overweight a BMI of 38, pulse 88 beats/min regular BP 140/85
mmHg. Heart sounds are carotid bruits are detectable. The neurological examination
unremarkable except an upgoing plantar response on right side.
A Doppler ultrasound of carotid arteries reveal 50% stenosis in proximal carotid arteries
bilaterally.What evidence based intervention is likely to prevent further episodes of
patients condition?
1 ) Add clopidogrel aspirin
2 ) Add dipyridamole aspirin
3 ) Increase dose of aspirin 150mg daily
4 ) Stop aspirin start clopidogrel alone
5 ) Stop aspirin start dipyridamole alone
23

Comments:
This patient having recurrent episodes of anterior circulation transient ischaemic attacks (TIAs)
despite being on aspirin. If aspirin alone ineffective in preventing TIAs, then a combination of
low dose aspirin dipyridamole modified release recommended. There trial data as yet
evaluating use of clopidogrel as add on therapy aspirin in cerebrovascular disease, although
clopidogrel extensively evaluated in cardiovascular disease. However, clopidogrel shown be
an appropriate alternative patients a contraindication aspirin. 2 )
16. A 29 year old woman was referred intake by her General Practitioner because of
appearance of a rash on her legs. Ten days previously she had seen her GP complaining of
a sore throat had given a seven day course of amoxicillin.
On examination she appeared well. She was febrile (38oC), pulse 90 beats per minute in
sinus rhythm pressure 135/80 mmHg. Palpable purpura were seen on her buttocks back
of both thighs, extending down ankles.
Urinalysis showed protein (+) (+). What is the likely cause of her purpura?
1 ) Allergy amoxicillin
2 ) Epstein Barr virus infection
3 ) Group A Streptococcus infection
4 ) Infective endocarditis
5 ) Meningococcal septicaemia
Comments:
The description of rash its distribution highly suggestive of Henoch Schonlein purpura (HSP).
The presence of protein onurine dipstick testing suggests co existing nephritis. The likely
precipitant a Group A Streptococcal infection caused sore throat ten days earlier.
The main initial concern would be exclude meningococcal septicaemia. The patient had this
illness several days does appear be critically ill. Overwhelming infection DIC therefore
unlikely.
There are insufficient features be able diagnose infective endocarditis. EBV infection associated
a rash if ampicillin administered; rash this phenomenon typically maculopapular vasculitic.
Allergic reactions penicillins usually manifest as a maculopapular rash vasculitis. Allergy does
explain haematuria proteinuria. 3 )
17. A 42 year old widow presents a one week history of progressive confusion unsteady
gait. She works as a barmaid lives in poor social circumstances.
On examination she malnourished disorientated. She nystagmus unable abduct either
eye. The pupils are sluggish unequal. Ankle jerks are absent but upper limb reflexes are
present. Shortly after her admission you are called ward as she become very drowsy
collapsed on floor.
Investigations on admission were as follows:
Haemoglobin
11.4 g/dL
MCV
99 fL
White count
5.6 x 109/L
Platelets
230 x 109/L
Serum sodium
129 mmol/L
Serum potassium
3.2 mmol/L
Serum bilirubin
27 umol/L
24

Serum gamma GT
Serum alkaline phosphatase
Serum AST
Serum ALT
Serum albumin
Prothrombin time

440 IU/L
180 IU/L
90 IU/L
45 IU/L
33 g/L
12 sec

(Control 11.5 15.5 sec)

What was likely cause of her presentation drowsiness?


1 ) Hyponatraemia
2 ) Brain stem CVA
3 ) Central pontine myelinolysis
4 ) Liver failure
5 ) Wernickes encephalopathy
Comments:
This lady presents classic triad of WE (encephalopathy, gait ataxia occulomotor dysfunction).
Lower limb neuropathy also a feature of WE. Her occupation, poor nutrition, social situation,
results all suggest underlying alcoholism. The hyponatraemia mild unlikely cause symptoms
CPM (related rapid correction of Na+) unlikely. A brainstem CVA unlikely due gradual onset
over 2 weeks. Liver function appears well preserved (normal PT reasonable Albumin). 5 )
18. A 45 year old woman attends outpatient clinic follow up of her multiple sclerosis.
During conversation, she asks your advice on fluoxetine which she taking her depression.
She concerned about newspaper reports linking fluoxetine suicidal ideation therefore
keen discontinue drug. On further questioning, you establish she a long history of
depression. Initially her depression was severe but since starting on fluoxetine 20 mg
daily, her symptoms have dramatically improved. She does have regular follow up
psychiatrists feels her GP does take her concern seriously.
What would you do her treatment?
1 ) Change another class of antidepressant
2 ) Reassure her continue fluoxetine
3 ) Reduce dose of fluoxetine
4 ) Refer her a psychiatrist
5 ) Stop fluoxetine altogether
Comments:
You should explain patient although suicidal ideation linked fluoxetine, causality
established. Therefore it would be unwise stop fluoxetine altogether. Reassure her since her
depression well controlled by fluoxetine, she should continue it she likely relapse. In
addition, you should point out her selective serotonin reuptake inhibitors (SSRIs) as a group
have fewer antimuscarinic side effects than older tricyclics. Also bear in mind SSRIs are less
cardiotoxic than tricyclics in overdose. Apart from that, SSRIs are better than monoamine
oxidase inhibitors (MAOIs) because they are more effective do show dangerous interactions
some foods have fewer dangerous interactions drugs are characteristic of traditional MAOIs. 2 )

25

19. A 45 year old labourer referred a three month history of tingling paraesthesiae of
left hand. On two occasions he dropped his cup of tea but put these down a bit of
clumsiness. His symptoms tend be more noticeable in evenings when he resting
watching TV. He denies any muscle twitching. Bladder bowel function are reported as
normal. Past history unremarkable he involved in any accidents recently. His job means
he often use vibrating tools do heavy manual lifting.
n examination, he obvious loss of muscle bulk in his upper limbs. Tone normal. Power
appears in all muscle groups. Apart from an absent triceps reflex on left, his reflexes are
all present symmetrical. Sensory examination reveals loss of pinprick sensation on palmar
aspect of index, middle ring fingers of left hand. Tapping of palmar aspect of wrist fails
elicit any tingling sensations in hand distally. Joint position sense vibration sense are
both normal. Cranial nerve examination unremarkable.
What is the likely diagnosis?
1
2
3
4
5

)
)
)
)
)

C7 root neuropathy
Carpal tunnel syndrome
Erbs palsy
Klumpkes palsy
Medial cord lesion

Comments:
The sensory disturbance in C7 dermatome absent triceps reflex (C7, C8) makes a C7 root
neuropathy best answer. In carpal tunnel syndrome, sensory disturbance involves thumb,
index, middle fingers lateral half of ring finger, triceps reflex will be affected. Erbs palsy
usually result of traumatic avulsion of C5 C6 roots (commonly occurring during delivery at
birth) causes loss of shoulder abduction elbow flexion loss of biceps brachioradialis reflexes.
Klumpkes palsy, often result of a fall stopped by grasping a fixed object one hand, involves
C8 T1 roots causes weakness of small muscles of hand of long finger flexors extensors, a
sensory disturbance affecting medial half of ring finger little finger. A medial cord lesion also
affects C8 T1 roots. 1 )
20. 48 year teacher admitted a two day history of increasing breathlessness cough
productive of purulent sputum. He smoked 20 cigarettes a day since age of eighteen. He
in hospital before but was recently diagnosed as having chronic obstructive pulmonary
disease by his general practitioner. He taking an inhaled agonist on an as required basis.
On examination he breathless at rest, alert orientated. He cyanosed a respiratory rate of
26 breaths per minute. His temperature 37.8 C. His pulse 100/minute pressure 150/100.
Auscultation of his chest reveals bilaterally reduced air entry.
His chest radiograph demonstrates a heart size but lung fields are hyperinflated. There
pneumonic consolidation.
Arterial gases on admission on 24% oxygen by nasal cannulae show:
pH
7.34
pO2
6.5
kPa
pCO2
6.8
kPa
Standard bicarbonate
27
kPa
He treated nebulised bronchodilators his FIO2 increased 28%. The results of arterial
gases repeated after 30 minutes are:
26

pH
7.30
pO2
7.0
kPa
pCO2
8.5
kPa
Standard bicarbonate
28
kPa
What further management is required now?
1 ) Reduce FiO2 24%
2 ) Intubation mechanical ventilation
3 ) Non invasive positive pressure ventilation (NIPPV/NIV)
4 ) Continuous positive pressure ventilation (CPAP)
5 ) Give oxygen by face mask
Comments:
This patients ABG are deteriorating he developing an increasingly severe respiratory acidosis.
He still alert haemodynamically stable therefore NIV (such as BiPAP) treatment of choice
should be instigated without delay. 3 )
21. A 54 year sales representative was referred by his general practitioner complaining
of feeling tired all time. He had a history of depression which he was taking anti
depressant tablets prescribed by GP. He had recently resigned from his job as he was too
tired do large amount of driving required had nearly involved in a car accident when his
car had swerved across road apparent reason. He was overweight admitted 3 stone
increase in weight over last three years. His pressure was elevated at 170/100.What is the
likely diagnosis?
1 ) Obstructive Sleep Apnoea Syndrome
2 ) Chronic Hyperventilation Syndrome
3 ) Hypothyroidism
4 ) Absence Seizures
5 ) Infectious Mononucleosis
Comments:
Obstructive sleep Apnoea (or Sleep Apnoea/Hypopnoea) Syndrome occurs when episodes of
partial complete obstruction of pharyngeal airway occurs during sleep. This causes (a)
repetative apnoeas (cessation of airflow > 10seconds) hypopnoeas (50% reduction in airflow
greater than 10 seconds) (b) loud snoring (c) excessive daytime somnolence as a result of
repeated arousals. The gold standard diagnostic test overnight polysomnography. Increasingly
though simpler sleep monitoring systems simple overnight oximetry are being used often
studies undertaken in patients home. The treatment of choice weight loss, avoid sedatives
drugs/excess alcohol nasal CPAP. 1 )
22. A 23 year old woman presents complaining of a 'droopy face' occurring over last 24
hours. Two weeks previously she had several days of headache mild photophobia. The
headache was of acute onset moderately severe. Over last week she also noticed mild
lower back discomfort occasional tingling in feet which she put down fact she had
'pulled a muscle' when walking.Two months ago she had returned from a holiday in
Thailand.
27

She takes oral contraceptive pill. She smokes 20 cigarettes a day consumes alcohol
socially. She a family history of migraine her mother died of a 'brain haemorrhage'.
On examination, she looks well. Pulse 88 beats/min, BP 105/75 mmHg, temperature 37C.
On examination of her eyes, she pupillary responses light a full range of eye movements.
Fundoscopy unremarkable. Corneal reflexes are present equally on both sides. She
difficulty closing her left eye it rolls it upwards in attempt. When cheeks are puffed, left
side balloons more than right. The reset of cranial nerve examination unremarkable.
Power symmetrical in upper lower limbs tone appears normal. She absent reflexes
throughout, even reinforcement. Plantar responses are downgoing bilaterally. She able do
finger to nose heel to shin testing adequately. Sensory examination of light touch, pinprick
vibration sense are all normal.What is the likely diagnosis?
1 ) Cerebrovascular infarction
2 ) Guillain Barre syndrome
3 ) Migraine
4 ) Multiple sclerosis
5 ) Ruptured berry aneurysm
Comments:
The history of lower back pain, subjective sensory symptoms in absence of objective signs, left
lower motor neurone facial palsy absent reflexes make Guillain Barre syndrome likely
diagnosis. The time delay between onset of facial weakness initial headache makes diagnosis
of a subarachnoid haemorrhage, cerebrovascular infarction migraine extremely unlikely. To
make a clinical diagnisis of multiple sclerosis, we will need two separate symptoms at two
separate times lesions disseminated in space in time if other problem can be found explain
patient's condition conditions which patient does fulfill. 2 )
23. A 22 year old obese man presents mild ankle oedema urinalysis shows protein +++
blood. A diagnosis of Nephrotic Syndrome made on basis of a cholesterol of 6.9, an
albumin of 30g/dl proteinuria of 8g/24hours.
He ankle oedema a BP of 145/90. A renal biopsy arranged following week. A protein
selectivity index of 15% found on analysis of his urine. He empirically started on
prednisolone 60mg daily.
His 24 hour urine collection repeated prior renal biopsy this shows a reduction in his
urine protein output 1.5g/24 hours.What is the likely diagnosis?
1 ) Amyloid
2 ) Focal Segmental Glomerulosclerosis
3 ) Membranoproliferative Glomerulonephritis
4 ) Membranous Nephropathy
5 ) Minimal Change Nephropathy
Comments:
In this age group haematuria, minimal change nephropathy likely. Steriod responsiveness
near 100% in Minimal Change but only about 40% in FSGS. A protein selectivity index of <10%
highly selective a ratio of serum urine IgG albumin. High selectivity suggests Minimal Change
disease but less reliable in adults. 5 )
24. A 70 year old man referred a six month history of increasing unsteadiness. He an
irregular swaying gait a tendency drift right when walking. His wife comments he tends
keep his feet apart when standing. In addition he noticed problems urinary urgency
28

frequency. Multiple urine samples sent by his GP have failed detect infection. His muscles
sometimes feel stiff but he puts this down age.
On examination, pulse 72 beats/min, BP 140/85 mmHg lying, 110/60 mmHg standing.The
muscles of upper lower limbs show increase tone in opposing muscle groups when
joints are passively moved. There obvious loss of muscle bulk. Power appears be in all
muscle groups. Gait broad based a tendency lean right. Reflexes were brisk throughout
plantar responses were downgoing bilaterally. Finger to nose testing impaired in upper
limbs. Sensory examination unremarkable.
What is the likely diagnosis?
1 ) Amyotrophic lateral sclerosis
2 ) Chronic inflammatory demyelinating polyneuropathy
3 ) Freidreichs ataxia
4 ) Multiple system atrophy
5 ) Polymyositis
Comments:
Multiple system atrophy a degenerative disorder characterised by parkinsonian features,
autonomic insufficiency (leading postural hypotension, anhidrosis, disturbance of sphincter
control, impotence) signs of a cerebellar deficit. Chronic inflammatory demyelinating
polyneuropathy clinically similar Guillain Barre syndrome (hyporeflexia areflexia, paraesthesiae
mild sensory deficits in upper lower extremities, weakness) except it follows a chronic
progressive course. Friedreichs ataxia characterised by progressive gait ataxia, depressed knee
ankle reflexes, cerebellar signs, impairment of joint position vibration sense clinical
manifestations almost always begin appear before puberty. Polymyositis often presents muscle
weakness wasting, especially of proximal girdle muscles, muscle pain tenderness, weight loss
a low grade fever. 4 )
25. A 56 year male a 12 year history of diabetes mellitus presents annual review. He
currently receiving gliclazide at a dose of 80 mg twice daily. Examination reveals a pulse of
76 beats per minute regular a pressure of 152/90 mmHg. Fundal examination reveals
bilateral dot haemorrhages scattered hard exudates. He loss of vibration sensation in
ankles but all pulses are palpable.
Investigations reveal:
Serum sodium
138 mmol/L
Serum potassium
3.8 mmol/L
Serum urea
10.2 mmol/L
Serum creatinine
160 mol/L
Glucose
12.1 mmol/L
HbA1c
9.5%
Cholesterol
5.5 mmol/L
Triglycerides
2.8 mmol/L
Which of following measures would you adopt improve this patient's prognosis?
1 ) ACE inhibitor
2 ) Beta blocker
3 ) Increased dose of gliclazide
4 ) Insulin
29

5 ) None
Comments:
This patient microvascular complications related his diabetes particularly Nephropathy,
Neuropathy Retinopathy. However, subjects type 2 diabetes have a 2 4 fold increased
cardiovascular mortality from which this patient probably prone in view of his established
chronic renal impairment. Studies such as UKPDS reveal improving glycaemic control would
reduce microvascular complications but this significant impact upon cardiovascular morbidity
mortality. However, lowering pressure significantly reduced morbidity from both microvascular
macrovascular disease. In this study ACEI compared beta blockers results were similar. But,
HOPE study (using Ramipril) suggested mortality in patients at risk of cardiovascular disease
(inc diabetics) may be further reduced by addition of an ACEI their standard regime. ACEI may
have a superior efficacy in delaying progression of nephropathy. 1 )
26. A 48 year lady presents increasing breathlessness cough. This getting worse over
last year she had repeated chest infections over last six months. She smoked 10
cigarettes a day until eight years ago. She known allergies. She works as a hairdresser. A
chest x ray was reported as being normal.
Pulmonary function testing demonstrated:
FEV1
1.60 L
(53% predicted)
FVC
2.86 L
(78% predicted)
Total lung capacity 4.83 L
(110% predicted)
TLCO
6.63%
(93% predicted)
KCO
1.36
(120% predicted)
What is the likely diagnosis?
1 ) Emphysema
2 ) Chronic Bronchitis
3 ) Asthma
4 ) Pulmonary Embolism
5 ) Obesity
Comments:
This lady moderate airways obstruction: FEV1/FVC = 56% predicted. Transfer factor transfer co
efficient can be elevated in patients asthma but always reduced in emphysema. Patients extra
pulmonary restrictive defects such as morbidity show an elevated Kco Tlco but restrictive
defect produces a elevated FEV1/FVC reduced lung volumes. 3 )
27. A 40 year old woman referred you complaining of a three month history of
dizziness a ringing noise in her left ear. The dizziness intermittent in nature usually
lasts up two minutes. She noise in her ear as being having a regular rhythm. Over last
two weeks, she also noticed mild difficulty closing her left eyelid her friends have
commented her face slightly asymmetrical.
On examination, she looks relatively anxious. Cranial nerve examination reveals a mild
difficulty in closing eyelid of her left eye. She also a slight droop over left side of her face
when she asked smile. Hallpikes manoeuvre negative. Webers test reveals lateralisation
of sound left ear. Rinnes test reveals bone conduction better than air conduction in left
ear vice versa in right ear. Testing of gag reflex reveals reduced a sensation on left side.
The rest of neurological examination unremarkable.
Where site of this patients lesion?
30

1 ) Cerebellopontine angle
2 ) Glomus jugulare
3 ) Internal capsule
4 ) Midbrain
5 ) Pons
Comments:
Cranial nerves IX, X XI run through jugular foramen. Therefore a glomus jugulare tumour
affects one, two all three of these cranial nerves. The tumour can enlarge sufficiently damage
cranial nerves VII XII. The clue this question lies in clinical features of a pulsatile tinnitus
(suggested by regular rhythmic ringing noise) ipsilateral conductive deafness. Glomus jugulare
tumour a highly vascular tumour which will account pulsatile tinnitus. The conductive deafness
can be accounted by fact tumour commonly presents as a mass behind tympanic membrane.
Most nonfamilial tumours have a preponderance an onset in fourth decade. The tumours may
also occur in an autosomal dominant fashion. 2 )
28. A 56 year builder presents cough breathlessness. He known have chronic
obstructive pulmonary disease Ulcerative Colitis. He smokes up 20 cigarettes a day. The
cough productive of clear sputum up 500ml a day. He had haemoptysis. The
breathlessness now restricts his exercise tolerance 50 metres. He lost over 2 stone in 2
months. Examination dullness percussion at right lung base. His abdomen was generally
tender but there was guarding. What is the likely diagnosis?
1 ) Bronchiolitis obliterans organizing pneumonia
2 ) Bronchioloalveolar cell carcinoma
3 ) Alveolar Proteinosis
4 ) Adenocarcinoma of lung
5 ) Bronchopleural fistula
Comments:
Bronchiolalveolar cell carcinoma of lung account around 5% of all primary lung carcinomas. The
classic massive clear frothy sputum produced by patients this cancer a late manifestation but
can be up one litre a day. Other symptoms are dyspnoea, weight loss chest pain. Almost a half
of patients are diagnosed on routine CXR, usually demonstrating a peripheral lesion. Its name
arises from its pattern of growth along alveolar walls without actually destroying them. It an
adenocarcinoma. In those whose tumour resectable prognosis poor. 2 )
29. A 26 year diabetic patient presented accident emergency department at 2am. He
had a party but complained of sudden worsening of a three day history of increasing
breathlessness. The following results were obtained from an arterial gas sample.
What your interpretation of these gas results?
pH
7.66
pO2
7.4
pCO2
4.7
Standard bicarbonate
30
H+
21
1 ) Metabolic alkalosis
2 ) Respiratory alkalosis
3 ) Mixed metabolic respiratory alkalosis
4 ) Metabolic acidosis respiratory alkalosis
5 ) Laboratory error
31

Comments:
The patient severely alkalotic a bicarbonate. He a pCO2 hypoxic. The only explanation must
be a laboratory error. 5 )
30. A 52 year old presents weight loss, anxiety difficulty sleeping. She had taking
combined cyclical oestrogen/progesterone hormone replacement therapy over last two
years.
On examination she was noted have a body mass index of 26.5 kg/m2, a pulse of 104
beats per minute a pressure of 112/72 mmHg. No goitre was palpable eye movements
were entirely normal. She was noted have weakness of proximal musculature of shoulder
hip girdles.
Initial investigations revealed:
Serum total thyroxine
250 nmol/L
(NR 60 140)
Plasma TSH
< 0.1 mu/L
(NR 0.4 4.0)
Serum alkaline phosphatase
202 IU/L
(NR 50 100)
Serum gamma glutamyl transferase
30 IU/L
(NR 10 50)
Her general practitioner commenced her on Carbimazole 10 mg tds together propranolol
120 mg bd. At review six weeks later patient appeared clinically euthyroid. Repeat
investigations showed:
Total thyroxine
180 nmol/L
Plasma TSH
2.2 mU/L
Serum alkaline phosphatase
160 IU/L
Serum gamma glutamyl transferase
36 IU/L
The dose of carbimazole was decreased 20 mg daily. After 1 year GP decided refer her
endocrine outpatients. Two weeks before she had a chest infection treated erythromycin.
Her test results showed:
Serum total thyroxine
80 nmol/L
Plasma TSH
12.8 mU/L
Serum alkaline phosphatase
102 IU/L
Serum gamma glutamyl transferase
42 IU/L
What is the cause of her thyroid function test results at her outpatient visit?
1 ) Interaction between erythromycin carbimazole
2 ) Interaction between HRT carbimazole
3 ) Patient stopped HRT
4 ) Over treatment carbimazole
5 ) Recent chest infection
Comments:
This patient was hyperthyroid was treated carbimazole now made her hypothyroid as reflected
by high TSH. Although her total thyroid hormone concentration it likely her free T4 would be
low. Although this might be due increased Thyroid Binding Globulin associated HRT, it more
likely effect of carbimazole potentiated by liver enzyme inhibiting effect of erythromycin was
prescribed her URTI. 1 )
32

31. A 45 year presents depression, constipation, polyuria and thirst. Over last six
months she become increasingly aware of tiredness arthralgia since being diagnosed
hypertension treated bendrofluazide 2.5 mg daily. Physical examination proves be
entirely except a pressure of 162/94 mmHg.
Investigations show:
Haemoglobin
14.4 g/dL
White cell count
7.1 x 109/L
Platelets
200 x 109/L
Serum sodium
148 mmol/L
Serum potassium
4.2 mmol/L
Serum chloride
105 mmol/L
Serum bicarbonate
28 mmol/L
Serum urea
8 mmol/L
Serum creatinine
105 umol/L
Serum corrected calcium
3.14 mmol/L
Serum bilirubin
16 umol/L
Serum alanine aminotransferase
10 IU/L
Serum aspartate aminotransferase
17 IU/L
Serum alkaline phosphatase
130 IU/L
Plasma parathyroid hormone
17 pmol/L
Which of the following is appropriate initial therapy?
1 ) Calcitonin
2 ) Frusemide
3 ) Intravenous saline
4 ) Pamidronate
5 ) Steroids
Comments:
This patient primary hyperparathyroidism hypercalcaemia should initially be treated IV saline.
She dehydrated requires appropriate fluid replacement. Once corrected patient could then be
offered surgery as appropriate therapeutic option. Calcitonin reserved severe hypercalcaemia
effects tend be transient. Pamidronate effective at reducing calcium over a couple of days but it
important first ensure patient adequately hydrated. Steroids are effective in certain types of
hypercalcaemia sarcoid but ineffective in primary hyperparathyroidism. Frusemide often
used induce a hypercalciuria in severe hypercalcaemia once patient adequately rehydrated. 3 )
32. A previously well 46 year old man presents a two day history of progressively
worsening headaches, dizziness, double vision, dry mouth swallowing difficulties. His wife
also noticed his face slightly asymmetrical over last day so. He denies any sensory
gastrointestinal symptoms. Three days ago he injured his left hand while gardening
wound on his little finger red tender.
On examination, he alert orientated. Pulse 60 beats/min, BP 130/65 mmHg, temperature
38?C. He ptosis, large poorly reactive pupils, diplopia on looking extremities
horizontally bilaterally, weakness of closing eyelids (right worse than left) inability
whistle properly. He also chokes when asked swallow a little water. Power mildly
33

generally reduced in upper limbs lower limbs. Deep tendon reflexes are generally
depressed sensation normal.Investigations reveal:
Haemoglobin
14.0 g/dL
White count
10.0 x 109/L
Platelets
200 x 109/L
Serum sodium
139 mmol/L
Serum potassium
4.0 mmol/L
Serum urea
6.8 mmol/L
Plasma glucose
7.5 mmol/L
CSF examination
Opening pressure
15 cm H2O
Cell count
< 2 per mm3
CSF protein
0.3 g/L
CSF glucose
6.1 mmol/L
What is the likely diagnosis?
1 ) Botulism
2 ) Guillain Barre syndrome
3 ) Lyme disease
4 ) Myasthenia gravis
5 ) Tetanus
Comments:
Botulism occurs either from gut colonisation (e.g. ingestion of contaminated home canned food)
an infected wound. Clostridium botulinum spores are widespread in soil aquatic sediment.
Typical initial features include diplopia, ptosis, facial weakness, dysarthria dysphagia. Later,
respiratory difficulty limb weakness occur. Neuromuscular blockade causes clinical features. In
botulism, impaired cholinergic transmission also involves autonomic synapses, causing poorly
reactive dilated pupils, dry mouth, paralytic ileus occasionally bradycardia. Reflexes are
depressed absent, sensation CSF in botulism. In Miller Fisher variant of Guillain Barre
syndrome, CSF often shows elevated protein. Lyme disease tends spare extraocular muscles.
Pupillary abnormalities do occur in myasthenis gravis. In tetanus, clinical features include jaw
stiffness, spasm of jaw muscles hyperreflexia. 1 )
33. A 62 year male presents recurrent episodes of weakness of right arm numbness of
face. He was admitted one year ago a similar episode. He was diagnosed a transient
ischaemic attack was commenced on aspirin 75mg daily. However, since this initial
episode he had two further episodes. Examination reveals an obese subject a BMI of 38, a
pulse of 76 beats per minute regular a pressure of 136/82 mmHg. Cardiovascular
examination normal, all pulses are palpable he carotid bruit audible.
Which evidence based intervention would be likely to prevent further episodes?
1 ) Aspirin plus clopidrogel
2 ) Aspirin plus dipyridamole
3 ) Increase dose of aspirin 150mg daily
34

4 ) Switch clopidogrel alone


5 ) Switch dipyridamole alone
Comments:
The 2002 guidelines from Royal College of Physicians suggest addition of dipyridamole
aspirin may have additive effects in reducing stroke based on ESPS 2 data. There little support
use of doses of aspirin greater than 75mg daily. 2 )
34. A 46 year old man admitted feeling generally unwell. He complains of increasing
stiffness in his arms jaws. He a mild throbbing frontal headache which he says typical of
migraine from which he known suffer from. He also a history of schizophrenia last
visited psychiatrist a month ago. Medications include sumatriptan fluphenazine, both of
which he on approximately two years.
On examination, his pulse 90 beats/min, BP 180/85 mmHg temperature 38.5C. Pulsatile
temporal arteries are noted bilaterally. Neurological examination reveals mild generalized
increase in tone throughout but otherwise unremarkable.
Investigations:
Hb
12.6 g/dl
WCC
4.9 x 109/l
Platelets
200 x 109/l
ESR
5 mm/hr
Plasma sodium
145 mmol/l
Plasma potassium
3.7 mmol/l
Plasma urea
4.9 mmol/l
Which of following drug treatments would you consider this patient's condition?
1 ) Benztropine
2 ) Bromocriptine
3 ) Lithium
4 ) Prednisolone
5 ) Procyclidine
Comments:
The diagnosis neuroleptic malignant syndrome (NMS), which can occur at any time during
treatment of antipsychotic medications. Concomitant treatment lithium anticholinergics may
increase risk of NMS. It manifested as by fever, rigidity, altered mental status autonomic
dysfunction. Treatment includes withdrawal of offending agent, reduction of body temperature
antipyretics. Dantrolene, bromocriptine levodopa preparations may be beneficial. Temporal
arteritis uncommon below age of 60. Benztropine, procyclidine lithium may all precipitate NMS. 2 )

35.
A 26 year man presents haemoptysis. He had a productive cough since childhood suffered
from recurrent sinusitis. He was known be infertile. Investigations immunoglobulins, sweat
sodium negative skin prick tests grass pollen, house dust mite aspergillus. What is the likely
diagnosis?

1
2
3
4
5

)
)
)
)
)

Bronchiectasis
Bronchiolitis
Cystic Fibrosis
Situs Inversus
Primary Ciliary Dyskinesia
35

Comments:
Primary Ciliary Dyskinesia a hereditary condition in which there partial complete deficiency of
outer inner dynein arms of cilia causing slow poorly co ordinated ciliary beating throughout
body. Patients suffer from Bronchiectasis Sinusitis. They are infertile because of reduced
motility of sperm. It associated in 50% of cases dextrocardia situs inversus when it called
Kartagener's syndrome. Patients Cystic Fibrosis also have bronchiectasis sinusitis are infertile
as vas deferens fails develop. Patients cystic fibrosis have an abnormal sweat test high levels
of sodium chloride although care needs be taken interpreting test in adults. The diagnosis
usually confirmed by determining patient's genotype. 5 )
36. A 68 year man dialysis via a haemodialysis line. He seen on renal unit complaining
of general malaise. He lost weight over last four months sleeping well due sweats.
Sometimes he joint aches back pain which mainly lumbar in origin. The back pain
getting worse but does stop him mobilising. He had a course of Flucloxacillin an infection
of his dialysis line 4 weeks ago which appeared clear infection.He dialysis three times
per week 4 hours per session had recorded pyrexias. He takes a combination of calcium
carbonate tablets Alucaps hyperphospataemia which now controlled. His aluminium
levels are his plasma parathyroid hormone level only mildly elevated having started 1
alphacalcidol. He on erythropoietin six months receives intravenous iron on dialysis
days.
On examination he apyrexial heart sounds a clear chest oedema. Abdomen soft non
tender. PR was normal. He some lumbar spine tenderness. Investigations reveal:
Haemoglobin
7.8g/dL
MCV
86 fL
White cell count
8.0 x 109/L
Platelets
180 x 109/L
ESR (Westergren)
78 mm/1st hour
What is the likely cause of her unresponsiveness erythropoietin?
1 ) Aluminium toxicity
2 ) Septicemia
3 ) Under dialysed
4 ) Hyperparathyroidism
5 ) Occult gastrointestinal bleeding
Comments:
The likely explanation Sepsis secondary Osteomyelitis of lumbar spine. Hyperparathyroidism
unlikely be cause only a mildly elevated result. 2 )
37. A 54 year sales representative was referred by his general practitioner complaining
of feeling tired all time. He had a history of depression which he was taking anti
depressant tablets prescribed by GP. He had recently resigned from his job as he was too
tired do large amount of driving required had nearly involved in a car accident when his
car had swerved across road apparent reason. He was overweight admitted 3 stone
increase in weight over last three years. His pressure was elevated at 170/100.Which
investigation likely provide correct diagnosis?
1 ) 24 hour monitoring
2 ) Overnight oximetry
3 ) TSH level
4 ) Paul Bunnell test
36

5 ) EEG
Comments:
Obstructive sleep Apnoea (or Sleep Apnoea/Hypopnoea) Syndrome occurs when episodes of
partial complete obstruction of pharyngeal airway occurs during sleep. This causes (a)
repetative apnoeas (cessation of airflow > 10seconds) hypopnoeas (50% reduction in airflow
greater than 10 seconds) (b) loud snoring (c) excessive daytime somnolence as a result of
repeated arousals.
The gold standard diagnostic test overnight polysomnography. Increasingly though simpler
sleep monitoring systems simple overnight oximetry are being used often studies undertaken
in patients home. The treatment of choice weight loss, avoid sedatives drugs/excess alcohol
nasal CPAP. 2 )
38. A 74 year old presents as an acute admission confusion diarrhoea. Little known of
her past history except it noted on GP letter she receiving treatment manic depression
hypothyroidism.
Examination reveals she a Glasgow Coma Scale of 15 but confused. She thin, unkempt
dehydrated a temperature of 370C. She a pulse of 82 beats per minute in a regular rhythm
a pressure of 112/72 mmHg. She noted have a coarse tremor dysarthric speech.Which of
the following is appropriate investigation assist in her management?
1 ) Blood gas analysis
2 ) CT head scan
3 ) Lithium concentration
4 ) Serum electrolytes
5 ) Thyroid function test
Comments:
This patient a number of possible causes of electrolyte disturbance. Dehydration from
diarrhoea alone may account some symptoms. However, you are told she receiving treatment
bipolar disorder this might consist of lithium therapy.
Lithium toxicity can cause diarrhoea, but may also be precipitated by dehydration. All of
symptoms may be due lithium toxicity (diarrhoea, tremor dysarthria). Lithium levels should be
taken, but may be of limited value in acute setting (rapid result may be available; levels always
reliable especially sustained release preparations).
The management of lithium toxicity largely supportive. The first step establish renal function
correct serum electrolytes. Renal function will determine patient's ability excrete lithium. 4 )
39. The following gases were obtained on a 58 year confused female:
pH
7.66
pO2
7.4 kPa
pCO2
4.6 kPa
HCO3
34 mmol/L
What your interpretation of these gas results?
1 ) Metabolic alkalosis
2 ) Respiratory alkalosis
3 ) Lab error
4 ) Metabolic alkalosis respiratory acidosis
5 ) Mixed metabolic respiratory alkalosis
Comments:
37

The pH high indicating an alkalosis high bicarbonate suggests a metabolic alkalosis. Therefore
there should be respiratory compensation reduced respiratory rate giving rise a high pCO2 low
pO2. However, this picture we see a type 1 respiratory failure low pO2 low pCO2. The results
therefore suggest a mixed respiratory metabolic alkalosis. This picture could be found in
association lung disease prolonged vomiting. 5 )
40. A 46 year old woman presents sudden onset of severe occipital headache associated
nausea vomiting. Cerebrospinal fluid examination a subsequent cerebral angiography
confirms diagnosis of an intracranial haemorrhage secondary a bleed from a posterior
communicating artery aneurysm. After consultation neurosurgeons, patient sent
surgical clipping of her aneurysm. She seems be doing well until third day post
operatively when she begins get drowsy confused. Urea electrolytes on her day of
admission her third post operative day are reviewed a 1 litre a day fluid restriction
instituted. However, her clinical condition fails improve further tests are carried out. The
summary of results as follows:
Day 1 Day 3

Day 4

Plasma sodium

136

113

114

mmol/l

Plasma potassium

4.9

3.5

3.3

mmol/l

Plasma urea

6.5

2.8

2.8

mmol/l

24 hour urinary sodium

70

mmol

Urine osmolality

710

mmol/kg H2O

What is the likely cause of this patient's confusion?


1
2
3
4
5

)
)
)
)
)

Addison's disease
Cranial diabetes
Fluid overload
Sick cell syndrome
Syndrome of inappropriate ADH secretion

Comments:
The low serum sodium level associated high urine osmolality suggestive of syndrome of
inappropriate antidiuretic hormone (SIADH) secretion. SIADH essentially occurs when water
retention accompanies water intake, leading plasma hyponatraemia hypo osmolality. Urine
more concentrated than plasma. In Addison's disease, plasma sodium will be low plasma
potassium level will be high. In cranial diabetes insipidus, urine osmolality low plasma
osmolality high. In fluid overload, urine osmolality low. Sick cell syndrome occurs when
hyponatraemia due a subnormal setting of hypothalamic osmoreceptors, associated usually a
chronic debilitating disease. 5 )
41. A 50 year male presents outpatient clinic a routine check up. He a three year
history of Type II diabetes mellitus currently diet controlled. He takes other medication.
Examination reveals he obese a BMI of 34 kg/m2, his pressure 180/90 mmHg he a
pulse of 80 beats per minute regular. A check of his joint management record shows
previous week his pressure was 160/90 mmHg 170/95 mmHg. He evidence of any end
organ damage his pinprick sensation using a monofilament normal.
38

Investigations show:
Serum sodium
Serum potassium
Serum urea
Serum creatinine
Serum bicarbonate
Cholesterol
Urine albumin
HbA1c

136 mmol/L
4 mmol/L
5 mmol/L
110 mmol/L
24 mmol/L
5.4 mmol/L
220 umol/d
7.4%

(NR 5 6.8%)

All of following drugs have evidence reduction of risk of developing a cardiac event in
this man EXCEPT:
1 ) Angiotensin Converting Enzyme inhibitor
2 ) Aspirin
3 ) Insulin
4 ) Metformin
5 ) Statins
Comments:
This man an obese type 2 diabetic (T2DM), fair diabetic control, a raised cholesterol,
hypertension, microalbuminuria. Thus we are looking at primary prevention of a cardiac event in
this patient. He at high risk of a cardiovascular event. WOSCOPS AFCAPS/TEXCAPS provide
evidence primary prevention of a cardiac event statin treatment. In HOPE study, Ramipril
shown lower cardiovascular risk in patients two CV risk factors as well as in diabetics,
independently of BP lowering. Aspirin shown lower risk of CV events in high risk individuals
diabetes in HOT study name one of many. Evidence from UKPDS 34 shows treatment of
overweight, diabetic patients metformin, lowers relative risk of myocardial infarction by 40%,
respect treatment sulphonylureas insulin. There evidence commencing T2Ds on insulin
lowers risk of MI, even if HbA1c improves. However, DIGAMI showed risk reduction of sliding
scale insulin in patients a MI (secondary prevention). 3 )
42. A 45 year male referred advice regarding his impotence diabetes. He was
diagnosed by his GP diabetes 4 years ago after presenting him problems related erectile
dysfunction. On questioning he complete impotence, a poor libido shaves infrequently. He
tried sildenafil (Viagra) without any success. Currently he receiving glibenclamide 10 mg
daily, aspirin 75 mg daily amlodipine 5 mg daily hypertension.
Examination reveals secondary sexual characteristics, a pulse of 80 bpm irregular, a
pressure of 126/86 mmHg a BMI of 24.4 kg/m2. No abnormalities are noted in
cardiovascular, respiratory abdominal systems. His fundi appear there are abnormalities
of sensation. Investigations reveal:
Haemoglobin
13 g/dL
White cell count
6 x 109/L
Platelets
200 x 109/L
Plasma Glucose
6.8 mmol/L
Serum bilirubin
15 umol/L</TD
Serum aspartate aminotransferase
Serum alanine aminotransferase
Serum alkaline phosphatase

40 IU/L
60 IU/L
100 IU/L

39

(NR 10 40)
(NR 10 40)
(NR 50 100)

Serum testosterone
Plasma luteinizing hormone
Plasma follicle stimulating hormone

4 nmol/L
1.2 U/L
1.3 U/L

(NR 9 30)
(NR 1 10)
(NR 1 7)

Which of following single appropriate investigation this patient?


1 ) HbA1C
2 ) MRI head
3 ) Serum Prolactin concentration
4 ) Serum Ferritin concentration
5 ) Ultrasound liver
Comments:
This patient diabetic, thin, deranged liver function tests hypogonadotrophic hypogonadism. A
unifying diagnosis would be Haemochromatosis therefore ferritin concentrations would be a
useful screening test. HbA1c would be of little value in investigating hypogonadism. Although
hyperprolactinaemia may cause hypogonadotrophic hypogonadism it would explain
hyperglycaemia deranged LFTs. Similarly MRI of head required initially. Ultrasound of liver
would be expected add little. 4 )
43. A 19 year lady presents gross oedema frothy urine. She investigated found have
an albumin of 10 heavy proteinuria of 11.5g/l a Cholesterol of 9. She undergoes a renal
biopsy diagnosised as having Minimal Change nephropathy. She initially goes into
remission oral prednisolone but on withdrawal of pteriods relapses quickly.
She readmitted a painful left leg further significant oedema. D Dimers are elevated a
deep vein thrombosis extending her external iliac vein diagnosised on doppler
ultrasonography. She commenced on Heparin but symptoms fail respond. She back on
oral Prednisolone diurectics. Her albumin remains at 20.
On examination she gross oedema of both legs some mild facial swelling. Her Left leg
painful erythematous a dusky hue. JVP raised. Her BP lying 110/70 standing 102/62.
Investigations show:
APTT ratio
1.86
INR
1.5
What is the likely diagnosis?
1 ) Antithrombin III deficiency
2 ) Hodgkins Disease
3 ) Dehydration
4 ) Inadequate Heparin
5 ) Anti phospholipid Syndrome
Comments:
ATIII deficiency likely as Nephrotics often lose Antithrombin in urine. Hodgkins Disease can
cause Minimal change disease but there nothing in history suggest this. She postural drop so
dehydration unlikely in presence of oedema. Anti phospholipid syndrome does cause minimal
Change nephropathy. 1 )
44. You are asked see a 25 year woman on labour ward. She had a vaginal delivery of a
baby boy who was severely hypotonic required resuscitation ventilation. On examination
she bilateral ptosis, global weakness (more marked distally) bilateral catacacts. She
reports her father also had bilateral cataracts died aged 58 years.
What mothers likely diagnosis?
40

1
2
3
4
5

)
)
)
)
)

Galactoasemia
Guillain Barre syndrome
Hypothyroidism
Myasthenia gravis
Myotonic dystrophy

Comments:
Myotonic dystrophy (Steinerts disease) commonest adult muscular dystrophy. Myotonia often
evident at an early age, but severity variable many patient do present until adulthood. Distal
weakness often more marked than proximal weakness; there a typical facial appearance due
atrophy of temporalis, masseter facial muscles. Men often have frontal balding some women
also have hair loss. Cataracts are common. Congenital myotonic dystrophy a more severe form
of disease occurs in children born mothers who have established myotonic dystrophy. Babies
are hypotonic frequently require assisted ventilation.
Galactosaemia seldom presents problems at birth. Babies develop diarrhoea fail gain weight.
Liver disease ensues unless diagnosed. Cataracts are common.
Myasthenia gravis frequently presents in third fourth decade of life exacerbations are common
in pregnancy. While this requires special attention mother, baby unaffected. Similarly, Guillain
Barre syndrome would affect baby. 5 )
45. A 45 year lady Chronic Renal failure secondary Systemic Lupus Erythromatosis
seen in a low clearance clinic as a routine follow up. Her Joints have causing some
discomfort she taking Naproxen PRN as well as Prednisolone 2.5mg od Azathioprine
50mg od disease control. She stable renal function a Creatinine of 300mmol/l a
Creatinine clearance of 18ml/min. Associated her Chronic renal failure she controlled
secondary hyperparathyroidism. She Anaemic last 6 months investigation of dyspepsia
an OGD showed only mild gastritis. She already on oral ferrous sulphate 200mg tds 3
months.
Investigations show:
Haemoglobin %
9.4g/dL
hypochromic red cells
12%
Platelets
180 x 109/L
White cell count
6.4 x 109/L
Serum folate
4.0ug/L
(NR 2 11)
Serum ferritin
230ug/L
(NR 15 300)
Transferrin saturation
17%
What therapeutic intervention should now be considered?
1
2
3
4
5

)
)
)
)
)

Folate
GM CSF
Intravenous iron
Subcutaneous erythropoetin
Vitamin B12

Comments:
41

Intravenous iron first intervention as her percentage of hypochromic red cells 12% ferritin
represents an acute phase protein. She responded oral iron. Epo would be a valid alternative
but only once iron deficiency corrected. You would check a B12 level but lots of Iron studies
here.
"Iron also essential hemoglobin formation. The iron status of patient chronic Kidney Disease
(CKD) must be assessed adequate iron stores. (National Kidney Foundation guidelines 3 )
46. A 25 year presents annual review. She developed diabetes mellitus at age of 15
currently treated human mixed insulin twice daily. Over last one year she aware of
episodes of dysuria received treatment trimethoprim on 4 separate occasions cysytits.
Examination reveals specific abnormality except two dot haemorrhages bilaterally on
fundal examination. Her pressure 116/76 mmHg.
Investigations show:
HbA1c
9%
Fasting plasma glucose
12.1 mmol/L
Serum sodium
138 mmol/L
Serum potassium
3.6 mmol/L
Serum urea
4.5 mmol/L
Serum creatinine
90 mol/L
Urinalysis
Glucose +
24 hour urine protein
220 mg/d
What would be best therapeutic option prevent progression of renal disease?
1
2
3
4
5

)
)
)
)
)

Improve glycaemic control Insulin


Prescribe a low protein diet
Treat ACEI
Treat prolonged antibiotics
Treat steroids

Comments:
This a rather difficult question as you would probably elect improve this patients glycaemic
control, treat her an ACE inhibitor treat any infections ensue. However based on evidence
probably ACE inhibitors would be superior renal protection. This patient evidence of diabetic
nephropathy microalbuminuria as defined by albumin excretion of between 30 300 mg/day
poor glycaemic control but pressure. The absence of any infection on urinalysis important, as
UTI would contribute albumin excretion. Diabetic nephropathy develops in approximately 40%
of patients type 1 diabetes in 5% 40% of patients type 2 diabetes. Without intervention
nephropathy likely deteriorate development of macroalbuminuria. In association latter, renal
function declines about 10% per year, ending in end stage renal disease. Proven interventions in
treatment of nephropathy include ACE inhibitors, low dietary protein improved glycaemic
control. The evidence good glycaemic control in treatment of microalbuminuria in patients
type 1 diabetes suggests clear benefit (Diabetic Control Complications study (DCCT). However,
Meta analyses of effects of ACE I on development of nephropathy in type 1 diabetics show an
albumin excretion rate 50% lower at two years in treated versus untreated patients. The evidence
a low protein diet exists overt proteinuria but microalbuminuria. 3 )
42

47. A 52 year old presents weight loss, anxiety difficulty sleeping. She had taking
combined cyclical oestrogen/progesterone hormone replacement therapy over last two
years.
On examination she was noted have a body mass index of 26.5 kg/m2, a pulse of 104
beats per minute a pressure of 112/72 mmHg. No goitre was palpable eye movements
were entirely normal. She was noted have weakness of proximal musculature of shoulder
hip girdles. Abdominal examination a palpable splenic tip.
Initial investigations following:
Serum total thyroxine
250 nmol/L
(NR 60 140)
Plasma TSH
<0.1 mU/L
(NR 0.4 4.0)
Serum alkaline phosphatase
202 IU/L
Serum gamma glutamyl transferase
30 IU/L
Her general practitioner commenced her on Carbimazole 10 mg tds together propranolol
120 mg bd. At review six weeks later patient appeared clinically euthyroid. Repeat
investigations showed:
Free thyroxine
180 nmol/L
Plasma TSH
2.2 mU/L
Serum alkaline phosphatase
160 IU/L
Serum gamma glutamyl transferase
36 IU/L
The dose of carbimazole was decreased 20 mg daily. After 1 year GP decided refer her
endocrine outpatients. Two weeks before she had a chest infection treated erythromycin.
Her test results showed:
Serum thyroxine
80 nmol/L
Plasma TSH
10.8 mU/L
Serum alkaline phosphatase
102 IU/L
Which of following would be appropriate, next investigation?
1
2
3
4
5

) Serum calcium
) Fasting serum lipids
) Isoenzymes of Alkaline Phosphatase
) Serum free thyroxine
) Ultrasound Liver

Comments:
Her recent TSH high, suggesting hypothyroidism. The serum total thyroxine however within
range. It stated in question this patient receiving oestrogen/progesterone HRT. Thyroxine
mostly bound thyroxine binding globulin in circulation. Oestrogen therapy associated
elevation of thyroxine binding globulin in serum. Thus total serum thyroxine may be
misleading in this case, serum free thyroxine will confirm whether this patient hypothyroid
euthyroid. 4 )
43

48. A 40 year old woman referred you complaining of a three month history of
dizziness, a buzzing noise in her left ear pain over her left cheek. The dizziness
intermittent in nature usually lasts up two minutes. She pain in her cheek as occurring
intermittently several times a day, electric shock like lasting several seconds each time.
Over last two weeks, she also noticed mild difficulty closing her left eyelid her friends
have commented her face slightly asymmetrical.
On examination, she looks relatively anxious. Cranial nerve examination reveals a reduced
left corneal reflex, mild difficulty in closing eyelids of her left eye a slight droop over left
side of her face when she asked smile. Hallpikes manoeuvre negative. Webers test
reveals lateralisation of sound right ear. Rinnes test reveals air conduction better than
bone conduction in both ears. The rest of neurological examination unremarkable.
What is the likely diagnosis?
1 ) Acoustic neuroma
2 ) Craniopharyngioma
3 ) Glomus jugulare tumour
4 ) Parasagittal meningioma
5 ) Prolactinoma
Comments:
The combination of sensorineural deafness, facial nerve palsy cranial nerve V involvement
suggests a cerebellopontine angle tumour e.g. acoustic neuroma.
Craniopharyngiomas are more common in children than in adults. Patients usually present signs
of local compression, especially of visual system, hypothalamic dysfunction, example diabetes
insipidus growth failure.
Glomus jugulare tumours tends present pulsatile tinnitus conductive deafness. Cranial nerves
IX, X XI which run through jugular foramen are also commonly affected varying degrees.
Cranial nerves VII XII may be affected if tumour enlarges sufficiently.
Parasagittal meningiomas may produce seizures be entirely asymptomatic, often growing
enormous size before they are discovered. Lesions reach sufficient size may cause spastic
paraparesis incontinence.
Prolactinomas commonly present amenorrhoea galactorrhoea in women impotence in men.
They are also associated visual field defects. 1 )
49.

A 56 year seaman presents his general practitioner this appearance on front of his legs.
He had this lesion at least one year received treatment topical steroids courses of
44

antibiotics. Despite this lesion deteriorated. The lesion painless recently he noticed
centre broken down slightly oozing. He a 5 year history of diabetes mellitus receives
metformin 500 mg tds. He also undergone dermatological surgery skin malignancies on
his face over last six years.What this skin lesion?
1)
Arteriopathic ulcer
2)
Basal cell carcinoma
3)
Necrobiosis lipoidica diabeticorum
4)
Squamous cell carcinoma
5)
Yaws
Comments:
This patient type 2 diabetes mellitus this long standing skin rash. Necrobiosis often mistaken
eczema but rather than responding steroids may actually deteriorate. Occasionally ulceration of
lesion may occur. Necrobiosis typically painless. The lesion does have appearances of a
squamous cell carcinoma which would have an irregular raised edge more necrotic. This site
an arteriopathic ulcer which usually found on foot.
3)

1.
A 38 year man referred diarrhoea, dyspnoea weight loss of approximately 6
months duration. Prior this he had quite fit active. The patient aware of up 10 episodes
of diarrhoea daily. It also transpires he frequently flushes during day which may occur at
any particular time during these episodes he frequently wheezy breathless.
Examination reveals a pulse of 90 beats per minute regular, a pressure of 122/76 mmHg
saturations of 98% on air. His jugular venous pressure elevated approximately 8 cm above
sternal angle auscultation of heart reveals a soft pan systolic murmur at left sternal
edge. He noted have 8cm hepatomegaly on abdominal examination.Investigations reveal:
Serum bilirubin
17 mmol/L
Serum ALP
720 IU/L
Serum AST
50 IU/L
Serum ALT
62 IU/L
24hr Urine HIAA 750 umol/L
(NR <210)
Echocardiography reveals marked tricuspid regurgitation mild pulmonary stenosis.
Which of following worst prognostic feature in this patients case?
1 ) Deranged liver function tests
2 ) Elevated urinary HIAA
3 ) Valvular heart disease
4 ) Wheeze
5 ) Young age
Comments:
The symptoms elevated urinary HIAA concentration confirm diagnosis of Carcinoid syndrome.
Markedly elevated HIAA concentrations are typically found do reflect a worse prognosis. Mild
derangement of AST/ALT typically found Alkaline phosphatase often elevated as a
consequence of carcinoid infiltration mild obstruction. But liver function usually quite despite
45

often heavy hepatic infiltration. Wheeze again a typical feature as a consequence of release of
vasoactive compounds such as 5 HT bradykinin. Symptoms usually improve following treatment
somatostatin analogues. Relative youth actually reflects a better prognosis. Cardiac lesions are
reversible treatment, deteriorate time frequently require replacement. 3 )
2.
A 21 year woman referred by her general practitioner deteriorating hirsutism. Since
menarche at age of 16 she noted deteriorating facial truncal hirsutism. She had taking
oral contraceptive pill had regular withdrawal bleeds up until one year ago when she
stopped pill due weight gain. Since then she had only one period, three months ago. On
examination her pulse was 82 beats per minute, pressure 128/82 mmHg she had a BMI of
30.4 kg/m2. Investigations reveal:
Free T4
12.8 pmol/L
(NR 9 22)
TSH
1.2 mU/L
(NR 0.4 4 mU/L)</TD
17 Beta oestradiol
254 pmol/L
(NR 130 850 pmol/L)
LH
11.4 mU/L
(NR 2 10 mU/L)
FSH
6.2 mU/L
(NR 2 10 mU/l)
Prolactin
610 mU/L
(NR 50 450 mU/L)
Testosterone
3.2 nmol/L
(NR < 3)
Dehydroepiandrostenedione
Sulphate (DHEAS) 17.2 pmol/L
(NR 2 10 pmol/L)
17 Hydroxy progesterone
3.2 pmol/L (NR 2 20 pmol/L)
What is the likely diagnosis?
1 ) Adrenal Carcinoma
2 ) Congenital adrenal hyperplasia
3 ) Microprolactinoma
4 ) Polycystic ovarian syndrome
5 ) Pregnancy
Comments:
This patient obese, oligomenorrhoea hirsutism. Her investigations show a oestradiol
increased LH:FSH, mild hyperprolactinaemia mildly increased androgens typical of PCOs. Mild
hyperprolactinaemia a typical feature of PCOs this picture of Oestradiol secretion
hyperandrogenism does fit a microprolactinoma. An elevated 17 OHP would be expected in
association CAH. A testosterone secreting tumour of either ovarian adrenal origin would
typically cause a testosterone concentration above 7 nmol/l would switch off LH/FSH
consequent hypo oestrogenism. Markedly elevated Oestrogen Prolactin would be expected at 12
weeks gestation testosterone would be expected be normal. 4 )
3.
A 24 year woman presents an eight weeks history of headaches, which deteriorated
markedly over last two days resulted in her admission. These headaches are distressing
her have problematical in morning but she found some relief from paracetamol. However,
headache deteriorated quite markedly over last two days being constant intolerable. The
only other information she gained 6 kg in weight last 6 months. On examination she
noted be obese a BMI of 32 kg/m2 a pressure of 122/76 mmHg. Fundoscopy reveals
bilateral swelling of both optic discs loss of venous pulsation but otherwise neurological
examination normal. Investigations reveal MRI appearances of brain a lumbar puncture
reveals an opening pressure of 30 cm H2O but CSF analysis normal. What is the likely
diagnosis?
46

1
2
3
4
5

)
)
)
)
)

Benign Intracranial Hypertension


Subarachnoid Hemorrhage
Migraine
Tension type headache
Multiple sclerosis

Comments:
This a typical history of BIH. An obese patient headaches papilloedema, Brain imaging, CSF
analysis high pressure. The other possible differential diagnosis would be cerebral/sagittal vein
thrombosis but one might expect pick up evidence this on MRI. 1 )
4.
A 76 year old lady had a GI bleed 10 months ago related NSAID use osteoarthritis.
At time an endoscopy showed a Duodenal Ulcer which was treated. She now re presents
an acute haematemesis following represcription of diclofenac misoprostol.
Her Hb on admission 5.6 g/dL. OGD performed, showing a single bleeding vessel on
posterior wall of duodenum. Injection sclerotherapy attempted but fails stop bleeding.
What is the next step in her management?
1 ) Glypressin
2 ) IV omeprazole
3 ) Octreotide
4 ) Repeat attempt at sclerotherapy following day
5 ) Urgent referral on call surgeons
Comments:
This lady lost a lot of ongoing bleeding despite endoscopic intervention, she requires urgent
surgery oversewing of bleeding vessel. It would be appropriate use omeprazole post operative
promote ulcer healing. Omeprazole will stop bleeding from a visible vessel. Glypressin &
Octreotide decrease portal pressure have an established role in variceal haemorrhage but in
peptic ulcer disease. 5 )
5.
A 40 year old man seen in Accident & Emergency department complaining of
wheeze dyspnoea. He complains of a persistent progressively worsening wheeze. His GP
initially prescribed a salbutamol inhaler, which had little effect, patient eventually had
take time off work on sick leave. During this time his GP intensified therapy, adding oral
prednisolone, good initial symptomatic improvement, though his symptoms returned
shortly after returning work. He reports little effect from therapy thereafter stopped
using inhaler as he did feel any benefit. The steroids were tapered stopped.
There past history of note. A full systems review unremarkable. He married three
children works at a local industrial plant as a solderer. He a lifelong non smoker drinks
about 8 units of alcohol weekly.
The patient takes his family on holiday two weeks in summer. During vacation he finds
his wheeze improves considerably. Two days after returning work his colleagues call an
ambulance because he an acute asthma attack in work he brought A&E unit.
On examination he dyspnoeic at rest but able speak in short sentences. Respiratory rate
22/minute. There are widespread inspiratory expiratory polyphonic wheezes throughout
both lungs. BP 130/76 detectable paradox. He afebrile. The rest of examination
unremarkable.
He treated nebulized salbutamol makes a significant improvement within 48 hours.
Investigations:
Hb
15 g/dl
47

WCC
Platelets
Plasma sodium
Plasma potassium
Plasma urea
Plasma creatinine
PEFR #1
PEFR #2
Which of following
1
2
3
4
5

)
)
)
)
)

9 x 109/l
395 x 109/l
143 mmol/l
4.2 mmol/l
2.8 mmol/l
70 mmol/l
250 l/min (on admission)
650 l/min (48h after admission)
important investigation be performed?

Bronchoscopy bronchoalveolar lavage


Full pulmonary function tests
Lung biopsy
CT scan of thorax
Peak flow rates measured at home in work

Comments:
The history highly suggestive of occupational exposure an allergen, significant symptomatic
improvement when away from his work environment. 5 )
6.
A 36 year woman referred by her general practitioner weight loss agitation of two
months duration. The GP had noted she had a mild tacchycardia, fine tremor, lid lag a
small goitre sent thyroid function tests which showed:
Serum Free T4
38.5 nmol/L (NR 9 22)
Serum TSH
<0.02 mU/L
(NR 0.4 4.0)
The patient was started on carbimazole 40 mg per day by GP referred clinic.
She seen in outpatient clinic two weeks later. She now reports a tickly, sore throat but
otherwise well. Investigations show:
Haemoglobin
12.8 g/dL
MCV
79 fL
White cell count 4.8 x 109/L
Neutrophils
2.6 x 109/L
Lymphocytes
2.0 x 109/L
Basophils
0.08 x 109/L
Eosinophils
0.1 x 109/L
Platelets
210 x 109/L
What is the appropriate treatment strategy for this patient?
1 ) Admit hospital
2 ) Continue carbimazole reassure
3 ) Reduce dose of carbimazole
4 ) Stop carbimazole
5 ) Stop carbimazole start course amoxicilin
48

Comments:
Lots of people get a sore throat but everyone warned about a sore throat when taking
thionamides. However, incidence of leucopaenia/neutropaenia Carbimazole less than 1%. In
this particular case her WBC
differential low MCV may reflect a mild iron deficiency in a
menstruant female. There requirement stop therapy she should be reassured. 2 )
7.
A 47 year old man HIV disease presents hospital a tonic clonic seizure. He had
initially presented six months previously Pneumocystis carinii pneumonia a CD4 T
lymphocyte count of 10 cells/mm3 had subsequently started on highly active
antiretroviral therapy. His recent CD4 count, taken one month prior his new presentation,
was 50 cells/mm3. On examination he focal weakness, but both plantar responses are
extensor. Fundoscopy is normal. A CT scan of his brain is shown.What is the diagnosis?
1 ) AIDS related dementia
2 ) Brain abscess
3 ) Cerebral toxoplasmosis
4 ) Primary CNS lymphoma
5 ) Progressive multifocal leukoencephalopathy
Comments:
Toxoplasma encephalitis commonest cause of focal brain disease in HIV/AIDS, occurring at
CD4 counts of less than 100 cells/mm3. Alhough Toxoplasma gondii may also cause a retinitis
in association HIV/AIDS, it need occur concomitantly CNS disease. The typical appearance on
CT of multiple ring enhancing lesions, as shown here. Treatment pyrimethamine +
sulfadiazine. Patients must subsequently be maintained on long term suppressive therapy
prevent relapse.3 )
8.
A CT scan shows dilated lateral ventricles but third fourth ventricles. Where is the
site of the obstruction?
1 ) Foramnen magnum
2 ) Foramen of Magendie
3 ) Sylvian aqueduct
4 ) Foramen of Munro
5 ) Arachnoid villi
Comments:
There obstruction between lateral ventricles third ventricle. The connection between these
sites via foramen of Munro. Sylvian aqueduct connection between third fourth ventricle. 4 )
9.
A 33 year male presents anxiety attacks palpitations associated hypertension. As
part of screening secondary hypertension he noted have repeatedly high urinary
catecholamine concentrations. He an intermittent smoker of 10 cigarettes per week drinks
approximately 18 units of alcohol weekly. He otherwise quite well. He adopted family
history available. He referred local endocrine department where elevated urine
catecholamine concentrations are noted CT scan of his abdomen reveals bilateral adrenal
masses of 4 cm diameter. Amongst other investigations an elevated plasma calcitonin
concentration noted. What other abnormality likely be present in this patient?
1 ) Cerebellar haemangioma
2 ) Parathyroid hyperplasia
3 ) Pituitary adenoma
4 ) Neurofibromas
5 ) VIPoma
49

Comments:
This patient bilateral phaeochromocytomas elevated plasma calcitonin concentration
suggesting a medullary thyroid cancer so implying a diagnosis of MEN type 2. The missing piece
of triad MEN type 2 hyperparathyroidism hyperparathyroidism a likely finding in this patient.
MEN type 2 an autosomal dominant condition although many presentations are sporadic;
mapped chromosome 10 associated presence of RET proto oncogene. There are 2 different
types 2A 2B, of which 2A commonest may be distinguished from 2B which associated
Musculoskeletal abnormalities including Marfanoid habitus ganglioneuromas. In our particular
case there was mention of any abnormalities on examination suggest mucosal/truncal
neurofibromas. 2 )
10. A 60 year male presents 5 days history of tingling of his fingers toes. Over last 2
days he noticed progressive weakness of his upper lower limbs. On admission he unable
ambulate unaided.On examination he a tetraparesis, areflexia flexor plantar response.
CSF examination shows:
Red cells
0
White cells
0
CSF glucose
3.6 mmol/L
Plasma glucose
5.2 mmol/L
CSF protein
1.3 g/L
What is the appropriate treatment?
1 ) Administration of IV Immunoglobulin
2 ) L dopa
3 ) No treatment required just physiotherapy
4 ) Pyridostigmine
5 ) Steroid therapy
Comments:
Intravenous immunoglobulins (0.4g/kg 5 days) plasma exchange are treatment of choice in
guillain barre. They improve long term prognosis. They have same efficacy. 1 )
11. A 72 year was admitted a deteriorating dyspnoea fever which had deteriorated
over preceding three days. Prior this admission she had well having returned one week
ago from a Spanish holiday her husband. The only other history of note was three years
ago she was diagnosed type 2 diabetes which she was treated diet alone. She a smoker
of 5 cigarettes per day.
On examination she was suntanned, slightly confused saturations of 92% on air. She had a
pyrexia of 40C, a pulse of 118 bpm a pressure of 118/90 mmHg. Auscultation of chest
left basal crackles. Investigations revealed:
Haemoglobin
14.3 g/dL
White cell count
8.2 x 109/L
Platelets
320 x 109/L
Serum sodium
123 mmol/L
Serum potassium
3.6 mmol/L
Serum urea
4.2 mmol/L
Serum glucose
10.9 mmol/L
50

Urine sodium
35 mmol/L
Arterial gases:
pH
7.36
pCO2
5.1 kPa
pO2
10.7 kPa
Standard HCO3
30 mmol/L
How would you treat this patients hyponatremia?
1 ) Demeclocycline therapy
2 ) Fluid restriction
3 ) Frusemide therapy
4 ) Hypertonic saline
5 ) Salt tablets
Comments:
This patient presents with high temperature, dyspnoea and mild hyponatraemia following a
Spanish holiday. The likely diagnosis is Legionnaires disease. Legionella infection treated with
antimicrobials achieve high concentrations intracellularly (macrolides, quinolones, rifampicin,
tetracycline).
She a marked hyponatraemia, as defined by a sodium concentration below 125 mmol/l, which
due syndrome of inappropriate ADH secretion. This may contribute her confusion although it
would be regarded as sufficient merit hypertonic saline should be appropriately treated fluid
restriction treatment of underlying condition. Saline tablets are irrelevant. Demeclocycline
should be reserved chronic SIADH an appropriate therapy Legionnaires disease. Frusemide
therapy useful in treatment of severe hyponatraemia but due urinary salt losses could
exacerbate hyponatraemia unless combined hypertonic saline.2 )
12. A 56 year presents her GP tiredness of three years' duration. No specific
abnormalities are found eventually GP undertakes some tests. He surprised find an
elevated calcium concentration refers patient you.
She well but depressed of late since death of her elderly parents. In her past history she
states her elder brother his son had some calcium problem diagnosed approximately 6
years ago although her knowledge they have received any treatment. She only takes
atenolol hypertension but receives other therapy does take any supplements.
Examination reveals a pressure of 148/96 mmHg but other abnormalities are noted.
Investigations reveal following:
serum sodium
138 mmol/L
serum potassium
3.8 mmol/L
serum urea
7.1 mmol/L
serum calcium
2.76 mmol/l
serum phosphate
1.0 mmol/l
Alkaline Phosphatase
100 IU/L
PTH concentration
4.4 pmol/L
Urine calcium
1.2 mmol/24 hrs
What treatment would you advise?

(NR 135 145 mmol/L)


(NR 3.4 5.0 mmol/L)
(NR 2.5 7.0 mmol/l)
(NR 2.2 2.6 mmol/L)
(NR 0.8 1.4 mmol/L)
(NR 50 120 IU/L)
(NR 3.0 5.4 pmol/L)
(NR 2 10 mmol/24 hrs)

51

1
2
3
4
5

)
)
)
)
)

Bisphosphonate therapy
Low calcium diet
Loop diuretic therapy
Parathyroidectomy
No treatment required

Comments:
This patient mild hypercalcaemia PTH, alkaline phosphatase reduced urine calcium. Together
family history of calcium problems a diagnosis of familial hypocalcuric hypercalcaemia (FHH)
suggested. This condition autosomal dominant a consequence of down regulation of calcium
receptor resulting in compensatory hypercalcaemia. FHH often misdiagnosed as primary
hyperparathyroidism but may be distinguished from latter by positive family history together
markedly low urine calcium excretion. Unlike hyperparathyroidism it associated any specific
abnormality, benign requires treatment. 5 )
13. A 78 year male presents exertional shortness of breath palpitations. His symptoms
developed over last 24 hours. Previously he was active but was diagnosed angina 2 years
ago which he takes isosorbide mononitrate 60mg daily, atorvastatin 10 mg daily, diltiazem
200 mg daily aspirin 75 mg daily. Two months ago he presented his GP general apathy
was commenced on fluoxetine 20mg daily.
On examination he was noted have a heart rate of 122 beats per minute irregularly
irregular, a pressure of 120/80 mmHg but otherwise appears fine. In particular he
appeared clinically euthyroid goitre was palpable on examination. ECG confirmed atrial
fibrillation.
Investigations reveal:
Serum Free T4
26.5 nmol/L
(NR 9 22)
Serum Free T3
4.8 nmol/L
(NR 3.5 5.8)
Serum TSH
0.1 mU/L
(NR 0.4 4.0)
Thyroid autoantibodies
Negative
ESR (Westergren)
28 mm/1sthr
(NR 0 10)
What likely cause of his abnormal thyroid function tests?
1
2
3
4
5

)
)
)
)
)

DeQuervains Thyroiditis
Drug induced thyrotoxicosis
Graves' disease
Hashimotos toxicosis
Solitary toxic thyroid nodule

Comments:
This patient mild thyrotoxicosis as reflected by raised T4 low TSH but T3 his prior symptoms
of apathy probably relate this. In absence of any thyroid auto antibodies which argue against
both Graves disease Hashitoxicosis, likely diagnosis a solitary toxic nodule. DeQuervains
associated a tender goitre, weight loss general malaise. A markedly raised ESR (>50 usually
100) typical. As this persons ESR would be considered reasonable their age. None of drugs
52

mentioned will cause this picture. Typically drugs responsible deranged TFTs include
Amiodarone. 5 )
14. A 38 year old presents after injuring her arm whilst on holiday. The next day she
finds she cant wind down window of her car her friends note prominence of her
shoulder blade. Weakness of what muscle causes winging of scapula?
1 ) Serratus anterior
2 ) Subscapularis
3 ) Teres minor
4 ) Latissimus dorsi
5 ) Teres major
Comments:
The patient a winged scapula. Serratus anterior muscle affected innervated by long thoracic
nerve (C5, C6, C7). 1 )
15. A 71 years woman presents 18 months history of gait ataxia, dysarthria dysphagia.
On examination there downbeating nystagmus slurred speech. There past pointing in
both upper limbs a wide based ataxic gait. Reflexes sensation are normal. There wasting
fasciculations. Plantar response flexor bilaterally.What is the likely diagnosis?
1 ) Botulism
2 ) Arnold Chiari malformation
3 ) Syringomyelia
4 ) Cervical myelopathy
5 ) Motor neuron disease
Comments:
Downbeating nystagmus indicating a craniomedullary junction pathology. Arnold Chiari one of
commonest causes. 2 )
16. A 23 year old intravenous drug user presents bilateral groin abscesses. After
drainage of abscesses, he started on intravenous flucloxacillin gentamicin. Twelve hours
later he complained of diplopia which deteriorated, also developed dysphagia muscle
weakness needed ventilatory support.
What is the diagnosis?
1 ) Aminoglycoside induced neuromuscular blockade
2 ) Guillain Barre syndrome
3 ) Metastatic brain abscess
4 ) Tetanus
5 ) Wound botulism
Comments:
Aminoglycosides may impair neuromuscular transmission should be given patients
myasthenia gravis; large doses given during surgery have responsible a transient myasthenic
syndrome in patients neuromuscular function. Non depolarising muscle relaxants (also known
as competitive muscle relaxants) compete acetylcholine receptor sites at neuromuscular
junction. Non depolarising muscle relaxants may be divided into aminosteroid group (e.g.
pancuronium, vecuronium), benzylisoquinolinium group (e.g. atracurium). 1 )
17. A 69 year man reviewed at outpatient clinic complaining of tiredness. Six months
ago he was admitted angina associated atrial flutter which spontaneously settled
following intravenous digoxin. Since then he treated amiodarone 200 mg daily. He also
53

takes aspirin 75 mg daily atenolol 50mg daily together Pravastatin 40 mg daily. Recent 24
hr ECG shows sinus rhythm throughout occasional ventricular ectopics.
Examination of patient reveals a fine tremor, a pulse of 56 beats per minute a pressure
of 146/88 mmHg.
Investigations reveal:
Serum Free T4
33.1 nmol/L
(NR 9 22)
Serum TSH
<0.02 mU/L
(NR 0.4 4)
What best management strategy this patient?
1
2
3
4
5

)
)
)
)
)

Continue amiodarone start carbimazole


Stop amiodarone start carbimazole
Stop amiodarone, start carbimazole flecainide
Stop amiodarone start steroids
Stop amiodarone only

Comments:
This patient probable amiodarone induced hyperthyroidism. The amiodarone was used
maintain sinus rhythm in this patient who was admitted ischaemic heart disease atrial
fibrillation/flutter spontaneously settled. Therefore, amiodarone should be stopped if
necessary other anti arrhythmics could be used maintain sinus rhythm such as Sotalol. However,
Flecainide would be contra indicated in this patient ischaemic heart disease. Amiodarone a very
long plasma half life stopping this therapy alone would be sufficient in a patient a tendency
AF ischaemic heart disease. Thus ensure adequate control following withdrawal of amiodarone,
patient should also be commenced on carbimazole. 2 )
18. A 29 year man presents a 6 months history of nasal congestion. He was seen by an
ENT consultant who diagnosed sinusitis. He presents now a two day history of right
periorbital swelling diplopia.
Examination reveals him be unwell, neck stiffness photophobia his temperature 37.4C.
He marked drooping of right eyelid right eye congested deviated right an enlarged
right pupil. Left eye appears normal. Fundoscopy normal. There also loss of sensation of
right forehead.
What is the likely diagnosis?
1 ) Cavernous sinus thrombosis
2 ) Epidural abscess
3 ) Meningitis
4 ) Pituitary adenoma
5 ) Tolosa Hunt syndrome
Comments:
The history typical cavernous sinus thrombosis possibly secondary sinusitis. This a life
threatening condition. Urgent CT brain antibiotics are needed. 1 )
19. A 33 year old male presents marked haemetemesis. He noted have a pulse of 128
beats per minute a pressure of 100/60 mmHg. An urgent endoscopy reveals a large
duodenal ulcer which does respond injection therapy he undergoes laparoscopy oversew
of his duodenal ulcer. He makes a good immediate post operative recovery receives
treatment IV omeprazole. No further bleeding noted his full count remains stable.
54

However, on twelfth post operative day he notes persisting weakness of right hand. On
examination there wasting of small muscles of right hand.
What likely site of lesion?
1 ) Wrist
2 ) Elbow
3 ) Neck
4 ) Brachial plexus
5 ) Brain
Comments:
This an entrapment ulnar neuropathy at elbow (the commonest site entrapment of ulnar
nerve). A common complication in ill patients in hospital. Nerve conduction studies will confirm
site of lesion. 2 )
20. A 55 year woman presents deteriorating wheeze dyspnoea. She diagnosed
asthma over last 10 years treated inhaled salbutamol, inhaled fluticasone received
numerous courses of prednisolone. Currently, she receiving 15 mg of prednisolone daily
as a reducing dose. Four years ago, she had also received cyclical hormone replacement
therapy post menopausal flushes but his was stopped after one year of therapy when she
developed a deep vein thrombosis.
What treatment would you select as prophylaxis against osteoporosis?
1 ) Alendronate
2 ) Calcium
3 ) Calcium ergocalciferol
4 ) Raloxifene
5 ) Salmon calcitonin
Comments:
This patient at high risk of developing osteoporosis being post menopausal receiving high
doses of steroids. Of therapies listed appropriate bisphosphonate, alendronate. National
guidelines suggest use of bisphosphonates where steroids are used over longer term
prevention of osteoporosis. Caltinonin administered subcutaneously appropriate. Raloxifene
a selective estrogen receptor modulator (SERM) an appropriate therapy prevention treatment
of post menopausal osteoporosis but like HRT may be associated an increased risk of
thromboembolism should be used in this case. Cacium vitamin D are useful adjuncts but
would prevent steroid induced osteoporosis. 1 )
21. A 32 year lady, recently returned from a holiday in Kenya, presents a fever (39.8C),
sore throat, lymphadenopathy a widespread macular rash.
What is the likely diagnosis?
1 ) Dengue fever
2 ) Herpesvirus infection
3 ) Infectious mononucleosis
4 ) Malaria
5 ) Primary HIV disease
Comments:
Her symptoms are suggestive of either infectious mononucleosis acute HIV infection. Acute EBV
less common in adults as it usually acquired in teenage years a diffuse macular rash often
occurs following administration of ampicillin. Acute HIV likely answer, particularly in light of
55

her recent travel sub Saharan Africa. Although details of specific 'risky' exposure are given, this
remains best answer. 5 )
22. A 24 year old presents an eight weeks history of headaches, which have
deteriorated over last week. She aware of a frontal headache, which constant in nature
unrelieved by paracetamol. These headaches have tended be present when she wakes in
morning lately have present all day. Over last six months she also explains she gained
more than 7 kg in weight. On examination, she rather tearful, a BMI of 32 kg/m2 but
nuchal rigidity neurological examination except bilateral optic disc swelling on fundal
examination. Her pressure noted be 122/88 mmHg her temperature 37C. An MRI scan of
her brain LP reveals an opening pressure of 30 cm H20 but CSF analysis. What treatment
would you offer this patient if her vision were threatened?
1 ) IV dexamethasone
2 ) Acetazolamide
3 ) Lumbo peritoneal shunt
4 ) IV mannitol
5 ) Frusemide
Comments:
Visual loss single threatening complication of BIH. Urgent LP shunt treatment of choice Optic
nerve fenestration an alternative. There are comparative studies between two interventions. 3 )
23. A 62 year old presents visual loss of right eye, which developed quite suddenly one
day ago. Fundoscopy reveals an embolus right superior temporal branch of retinal artery.
Where would you expect find visual field defect?
1 ) Superior nasal
2 ) Superior temporal
3 ) Inferior nasal
4 ) Inferior temporal
5 ) Inferior caudal
Comments:
Superior segmental optic nerve pathology will give inferior nasal visual field defect. 3 )
24. A 23 year old Indian man diagnosed pulmonary tuberculosis started on rifampicin,
isoniazid, pyrazinamide. Four weeks later he presents fever, myalgia, lethargy.
Investigations:
Haemoglobin
WBC

14.5 g/dL
7.5 x 109/L

Neutrophils
Lymphocytes
Monocytes
Eosinophils
Basophils
Platelets
Serum sodium
Serum potassium
Serum urea
Serum creatinine
Serum bilirubin

5.5 x 109/L
0.8 x 109/L
0.05 x 109/L
1.14 x 109/L
0.01 x 109/L
350 x 109/L
141 mmol/L
5.1
27.9 mmol/L
400 umol/L
7umol/L

56

Serum aspartate transaminase


Serum alkaline phosphatase
Serum albumin

31 IU/L
97 IU/L
39 g/L

Urinalysis
Protein +, Leucocytes +, bacteria
Urine culture
Negative
What is the likely cause of his renal impairment?
1 ) Acute interstitial nephritis
2 ) Isoniazid toxicity
3 ) Plasmodium vivax malaria
4 ) Renal tuberculosis
5 ) Rhabdomyolysis
Comments:
Interstitial nephritis may be caused by infection, autoimmunity glomerular disease as well as
hypersensitivity medicines. A large number of medicines are reported cause acute interstitial
nephritis. The agents commonly implicated are ? lactams, vancomycin, rifampicin, co
trimoxazole, sulphonamides, ciprofloxacin, NSAIDs ranitidine, cimetidine, frusemide, thiazides,
phenytoin. An acute allergic reaction, infiltration of immune cells, occurs in response causative
drug, causing direct cytotoxicity. Typical clinical features include renal failure fever, arthralgia,
eosinophilia eosinophiluria. 1 )
25. 52 year old male presents acute weakness drowsiness. At admission he noted have
a Glasgow Coma Scale of 4 out of 15. Other observations reveal he a temperature of 37C.
A pulse of 92 beats per minute regular, respirations of 26 per minute saturations of 96% a
pressure of 162/102 mmHg. Neurological examination reveals generalized weakness but
left sided hyper reflexia a left extensor plantar response. An urgent CT scan reported as
showing a deep intracerebral hemorrhage without evidence of hydrocephalus. What is the
appropriate next step in his management?
1 ) Lower Blood pressure
2 ) Urgent surgical evacuation
3 ) Dexamethasone
4 ) Ventriculo peritoneal shunt
5 ) Observe
Comments:
Most likely haemorrhage secondary his hypertension. Hypertensive cerebral hemorrhage
usually affects basal ganglia, thalamus pons. Deep heamatoma are difficult evacuate. Since
there hydrocephalus, observation will appropriate next step of management. Acutely reducing
this patients pressure may produce paradoxical effect of exacerbating ischaemic territory. 5 )
26. A 26 year presents fatigue weight loss. Six years previously she had diagnosed
type 1 diabetes mellitus following diabetic ketoacidosis. She had well up until last one
year, since when she admitted on two occasions diabetic ketoacidosis. She currently
receiving soluble insulin three times daily long acting insulin in evening. Over last year
she had lost approximately 10kg in weight over last 3 months had generally lost her
appetite. She had also amenorrhoeic over last 3 months.
Examination reveals a thin, pale a pulse of 76 beats per minute a pressure of 116/80
mmHg. Cardiovascular, respiratory abdominal examination were normal. Sensation was
intact fundal examination normal.
Investigations reveal:
57

Serum sodium
Serum potassium
Serum urea
Serum creatinine
Serum glucose
HbA1c
Serum calcium
Serum phosphate
Serum Free T4
Serum TSH
Serum oestradiol
Serum LH
Serum FSH
Serum prolactin
Which of the following is
1 ) Pituitary CT scan
2 ) Pregnancy test
3 ) PTH concentration
4 ) Synacthen test
5 ) Thyroid uptake scan

128 mmol/L
5.0 mmol/L
6.8 mmol/L
110 umol/l
11.6 mmol/L
11.4%
2.95 mmol/L
(NR 2.2 2.6)
0.8 mmol/L
(NR 0.8 1.2)
8.2 nmol/L
(NR 9 22)
1.2 mU/L
(NR 0.4 4)
80 pmol/L
(NR 130 850
4.4 mU/L
(NR 2 10)
2.2 mU/L
(NR 2 10)
400 mU/L
(NR 50 450)
appropriate investigation this patient?

Comments:
This patient type 1 diabetes developed deterioration in glycaemic control, nausea weight loss.
Investigations reveal hyponatraemia, hypercalcaemia, low T4 TSH hypogonadotrophic
hypogonadism. This all fits a diagnosis of primary hypoadrenalism. The patient pregnant as
oestradiol concentrations would be elevated. Although low oestrogen T4 concentrations
LH/FSH TSH suggest a pituitary problem these are typical of severe hypoadrenalism would
respond steroid replacement therapy. A pituitary problem unlikely be responsible this picture
as firstly patient young, secondly prolactin concentration thirdly she already an autoimmune
disorder (T1DM). T1 DM Addisons disease are features of Schmidts disease (type 2
autoimmune polyendocrine syndrome). Primary ovarian failure also associated this condition
like primary hypothyroidism but this patient appears have developed these as yet. 4 )
27. A 44 year old Afro Caribbean woman presents hospital after returning from a
holiday South America. She a past history of sarcoidosis maintained on prednisolone
5mg once daily. She lost 5kg in weight over 4 weeks complains of fatigue aching
muscles. She febrile (38.5C). Examination reveals 4cm hepatomegaly multiple palpable
small lymph nodes.Investigations
Haemoglobin
13.5 g/dl
WBC
10.3 x 109/l
Neutrophils
5.5 x 109/l
Lymphocytes
1.5 x 109/l
58

Monocytes
0.05 x 109/l
Eosinophils
3.2 x 109/l
Basophils
0.01 x 109/l
Platelets
350 x 109/l
Serum sodium
141 mmol/l
Serum potassium
5.1 mmol/l
Serum urea
27.9 mmol/l
Serum creatinine
400 umol/l
What is the likely diagnosis?
1 ) Acute HIV
2 ) Hodgkin's lymphoma
3 ) Relapsed sarcoidosis
4 ) Strongyloidiasis
5 ) Tropical eosinophilia
Comments:
Tropical eosinophilia an allergic reaction microfilaria of Wuchereria bancrofti. Characteristic
features include: myalgia; fatigue; weight loss; cough dyspnoea wheeze; fever; current
previous residence in an area endemic filariasis (southern Asia, Africa, India, South America);
lymphadenopathy; marked peripheral eosinophilia high titres of anti filarial antibodies. The
chest x ray shows bilateral reticulonodular shadowing. This condition commonly accompanied
by false positive serological tests syphilis high titres of cold agglutinins. There typically a rapid
response treatment diethylcarbamazine.
The marked eosinophilia essentially excludes all of other options. Acute HIV usually consists of
fever, pharyngitis, widespread rash lymphadenopathy may be associated lymphopoenia, but
eosinophilia. Strongyloidiasis may be associated eosinophilia, but not theother clinical features
given5 )
28. A 70 years woman presents paroxysmal dizziness where she episodes where room
spins. These symptoms are exacerbated when she rolls over in bed right when she
reaches above her head. There are other symptoms.
Neurological general examination otherwise normal. The vertigo can be reproduced by
turning head of patient 45 degrees right then take patient supine position. There
nystagmus (upbeating torsional ), which last only a few seconds.
What is the diagnosis?
1
2
3
4
5

)
)
)
)
)

Benign positional vertigo


Vertebrobasilar insufficiency
Multiple sclerosis
Drug toxicity
Meniers disease

Comments:
This a typical history of benign positional vertigo. Usually occurs on lying turning head one
side. Hallpike manoeuvre reproduce symptoms. 1 )
29. A 76 year old male presents visual loss. Examination reveals a pressure of 168/102
mmHg fundoscopy shows an embolus right superior temporal branch of retinal artery.
59

Carotid dopplers are arranged these show left sided carotids, but 90% Right External
Carotid Artery stenosis approximately 50% Right Internal Carotid Artery stenosis. What
would be appropriate treatment?
1 ) Aspirin
2 ) Dipyridamole
3 ) Right External Carotid Artery endarterectomy
4 ) Right Internal Carotid Artery endarterectomy
5 ) Warfarin
Comments:
Endarterectomy only beneficial if internal carotid artery stenosis greater than 70%. If less than
aspirin control of vascular risk factors treatment of choice. In this case hypertensive control
advised. 1 )
30. A 22 year old presents an eight weeks history of headaches, which have
deteriorated over last week. She aware of a frontal headache, which constant in nature
unrelieved by paracetamol. These headaches have tended be present when she wakes in
morning lately have present all day. Over last six months she also explains she gained
more than 7 kg in weight. On examination, she rather tearful, a BMI of 32 kg/m2 but
nuchal rigidity neurological examination except bilateral optic disc swelling on fundal
examination. Her pressure noted be 122/88 mmHg her temperature 37C. Which of
following single best investigation this patient?
1 ) Lumbar puncture
2 ) MRI brain
3 ) CT brain
4 ) EEG
5 ) Fluorescin angiogram
Comments:
The history suggests BIH. However, brain imaging needed exclude a space occupying lesion,
hydrocephalus cerebral venous thrombosis. MRI brain better than CT brain look cerebral
venous thrombosis where a pyramidal defect in venous flow noted. 2 )
31. A 29 year old woman was referred clinic an opinion regarding a three month history
of fatigue, generalized weakness, easy bruising joint pains. On examination she appeared
pale. The spleen was palpable 3cm below left costal margin. The remainder of
examination was normal.
Investigations showed:
Haemoglobin
9.5 g/dL
MCV
95 fL
Platelets
55 x 109/L
White cell count
2.1 x 109/L
What is the likely diagnosis?
1
2
3
4
5

)
)
)
)
)

Acute myeloid leukaemia


Feltys syndrome
Idiopathic thrombocytopoenic purpura
Myelodysplastic syndrome
Systemic lupus erythematosus
60

Comments:
The combination of pancytopoenia, arthralgia splenomegaly makes SLE likely diagnosis.
Acute leukaemias are associated elevated white cell counts often accompanied by leukaemia
thrombocytopoenia; hepatosplenomegaly usually a feature of ALL rather than AML. ITP may be
associated thrombocytopoenia splenomegaly, but associated poenias of myeloid lymphoid
cell lines. Patients longstanding rheumatoid arthritis may develop Feltys syndrome
(leucopoenia splenomegaly; often accompanied by anaemia thrombocytopoenia) but this
follows years of active rheumatoid disease. Myelodysplastic syndromes present anaemia,
neutropoenia thrombocytopoenia but associated macrocytosis; MDS a disease of older people. 5 )
32.
A 75 year old retired farmer from Mid Wales found have very mildly deranged LFT's
during a routine health checkup. He an ultrasound scan which demonstrates multiple
cystic lesions in right lobe of liver ranging in size from 1 8cm in diameter. The larger
cysts have a calcified appearance. He had a melanoma removed from his back 20 years ago had
multiple basal cell cacinomas removed from his head, face, arms in recent years. What is the next
investigation?

1
2
3
4
5

) CT scan
) ERCP
) Hydatid serology
) Laparoscopy
) Ultrasound guided aspiration

Comments:
Asymptomatic, calcified cystic lesions in liver are typical of hydatid cysts. Hydatid infection was
endemic in Sheep farming regions (such as Wales NZ) in past sheep dogs were infected by
eating infected offal. Humans contract Hydatids via faecal oral spread from dogs. The liver
cysts are usually asymptomatic calcification usually denotes a non viable cyst. Hydatid Serology
a sensitivity of 80 90%. If hydatid serology was negative then further imaging (CT/MRI) +/
aspiration may be required make a diagnosis. Melanoma metastases are typically cystic
calcified. BCC's are very common in those who work outside in southern hemisphere, liver mets
are very uncommon. 3 )
33. A 28 year presents recurrent vulval candidiasis together polyuria. She gained
approximately 20kg in weight since birth of her last child five years ago. During this
pregnancy she recalls she was received insulin approximately four months although this
was discontinued after birth of her daughter who quite well. She also two older children
who are well although she did receive insulin during these pregnancies.
She a smoker of 12 cigarettes per day drinks approximately 12 units of alcohol weekly.
There a strong family history of coronary artery disease, her father paternal uncle died in
their fifties of heart disease. Her mother diabetic takes thyroxine a thyroid problem.
On examination she weighs 90kg a body mass index of 36.5 kg/m2. Her pulse regular, 82
beats/minute her pressure 140/88 mmHg. Otherwise examination of heart, chest
abdomen unremarkable. Fundal examination reveals three dot haemorrhages in left two
dot haemorrhages in right retina. There are abnormalities of sensation in feet hands.
Peripheral pulses are all normal.Investigations reveal:
HbA1c
9%
Fasting plasma glucose
12.3 mmol/L
Serum urea
10.1 mmol/L
61

Serum creatinine
125 umol/L
Serum alkaline phosphatase
150 IU/L
Serum HDL cholesterol
0.8 mmol/L
(Target > 1.55)
Serum total cholesterol
7.2 mmol/L
(Target < 5.2 mmol/L)
Fasting serum triglyceride
2.8 mmol/L
(Target 0.45 1.69 mmol/L)
Which of folllowing treatments should this patient receive?
1 ) Dietary advice
2 ) Glibenclamide
3 ) Insulin
4 ) Metformin
5 ) Rosiglitazone
Comments:
This patient typifies younger obese type 2 diabetic should initially be treated lifestyle dietary
advice. She should be encouraged lose weight adopt a high fibre, low fat, hypocaloric diet. This
approach should be adopted at least three months prior considering introduction of any other
therapy. Should this approach fail, as frequently case due poor compliance, then oral
hypoglycaemic therapy advised an appropriate drug would be metformin. This shown reduce
mortality morbidity in T2DM in UKPDS. Rosiglitazone recommended as first line therapy.
Although patient was treated insulin during her pregnancy implying she had gestational
diabetes, this would be considered at this stage as it would be likely induce weight gain.
Similarly, sulphonylurea, Glibenclamide may induce weight gain
appropriate drug her.
The elevated alkaline phosphatase a frequent finding in T2DM likely be due to hepatic
steatosis. Her mild renal impairment should be investigated may be a consequence of infection,
which may be asymptomatic, vascular disease, hypertension a combination of factors. The
deranged lipid profile typical of T2DM frequently improves diet, weight loss exercise. 1 )
34. A 40 year man presents casualty after playing in a charity rugby match. This first
time he played several years. He complains of headache neck pain.
On examination there loss of sensation over left side of his face, horizontal diplopia
looking towards left, a left lower motor neuron facial weakness. He also right
hemiparesis loss of sensation.
What is the likely diagnosis?
1 ) Cerebral hemorrhage
2 ) Vertebral artery dissection
3 ) Wallenbergs syndrome
4 ) Fractured cervical vertebra
5 ) Basilar artery aneurysm
Comments:
Head neck pain in a patient stroke suggests dissection. Preceded minor major injury common.
The clinical signs are typical brain stem CVA (limb weakness contralateral cranial nerve signs).
2)

35. A 33 year old woman presents a two month history of difficulty swallowing liquids
also aware of nasal regurgitation. Her friends have noted on occasions she looked rather
tired lost facial expression.
What bedside test might be employed help establish diagnosis?
1 ) Shake hands patient
62

2
3
4
5

)
)
)
)

Count numbers aloud


Examine tongue fasciculation
Attempt elicit a jaw jerk
Check plantar response

Comments:
The history suggests a diagnosis of myasthenia gravis. Fatigability best sign elicit during
examination. 2 )
36. A 39 year presents following an injury her right arm shoulder. The next day she
finds she cant wind down window of her car her friends note prominence of her
shoulder blade. What is the diagnosis?
1 ) Neuralgic amyotrophy
2 ) Lyme disease
3 ) Giullain Barre syndrome
4 ) C7 radiculopathy
5 ) Multiple sclerosis
Comments:
Neuralgic amyotrophy a brachial neuritis caused by a preceded infection (viral), vaccination,
trauma. It can be total segmental affecting only specific nerves of brachial plexus. In this case
long thoracic nerve (C5, C6, C7). 1 )
37. A 16 year old girl brought Casualty by her parents at 7am complaining of headache,
fever diarrhoea. She woke from her sleep at 2am had an episode of watery diarrhoea.
Over following three hours she become increasingly unwell. She past history of note
had perfectly well previous day. There history of recent travel. She lives her parents
younger brother, all of whom have well. There history of recent illness in any of her
school friends. Her periods are regular; her last period finished two days previously. Her
predominant complaint of a headache.
On examination she unwell a fever of 40C. Her pressure 70/30 mmHg heart rate 120
per minute. She complains of a headache mild photophobia but there nuchal rigidity. Her
chest clear on auscultation her abdomen soft non tender detectable organomegaly
masses. A diffuse macular rash noted over her over trunk.
Investgations:
Haemoglobin
WBC
Platelets
Serum sodium
Serum potassium
Serum urea
Serum creatinine
Serum bilirubin
Serum aspartate transaminase
Serum alkaline phosphatase
Serum albumin

10.5 g/dl
19.5 x 109/l
80 x 109/l
137 mmol/l
3.2 mmol/l
16.6 mmol/l
216 umol/l
7umol/l
61 IU/l
115 IU/l
41 g/l
63

Serum lactate dehydrogenase


195 IU/l
What is the diagnosis?
1 ) Haemolytic uraemic syndrome
2 ) Meningiococcal meningitis
3 ) Meningiococcal meningitis septicaemia
4 ) Toxic shock syndrome
5 ) Typhoid fever
Comments:
Toxic shock syndrome (TSS) caused by toxic shock syndrome toxins (TSS 1, TSS 2) produced by
certain types of Staphylococcus aureus. Although earliest described cases involved mostly
menstruating women using highly absorbent tampons, only 55% of current cases are associated
menstruation. The illness can also occur in children, postmenopausal women, men. Risk factors
include recent menstruation, recent use of barrier contraceptives such as diaphragms vaginal
sponges, vaginal tampon use (especially prolonged), recent childbirth, recent surgery, current S.
aureus infection. The diagnosis of toxic shock syndrome based on several criteria: fever,
hypotension, rash (resembling sunburn) peels after 1 2 weeks, at least 3 organs signs of
dysfunction. Blood cultures may be positive S. aureus. 4 )
38. An 81 year old woman presented a lobar pneumonia. She successfully treated
Erythromycin nebulised salbutamol. She taken frusemide 40 mg/day digoxin 125
mcg/day a number of years.Whilst on ward she developed a brief episode of chest pain.
An Electrocardiogram was taken.

What is the diagnosis?


1)
Atrial fibrillation
2)
Complete (third degree) heart block
3)
Hypokalaemia
4)
Second degree AV block
5)
Sinus bradycardia
Comments:
There are two P waves each QRS. The conducted PR intervals are constant. 2:1 AV block cannot
be classified as Mobitz type 1 type 2 AV block since it would be known whether second P wave
would be conducted same a prolonged PR interval. 4 )
39. A 38 year old woman presents a four week history of painless jaundice pruritis. Her
only past history of note a pan colectomy life threatening colitis as a teenager. She
consumes 20 units (200g) of alcohol per week. Her LFTs cholangiogram are shown.

64

Bilirubin
85 (1 22 umol/l)
ALP
450 (5 35 U/l)
GGT
325 (4 35 U/l)
AST
78 (1 31 U/l)
ALT
56 (5 35 U/l)
What is the likely diagnosis?
1)
Alcoholic cirrhosis
2)
Caroli's disease
3)
Choledocholithiasis
4)
Primary biliary cirrhosis (PBC)
5)
Primary sclerosing cholangitis (PSC)
Comments:
The ERCP shows stricturing beading of bile ducts, features clasically seen in PSC. 80% of those
PSC have a history of colitis. PSC occurs in 4% of those a history of ulcerative colitis. Caroli's
disease (a rare congenital disorder) also causes a beaded appearance of intrahepatic bile ducts
can cause recurrent cholangitis, less likely diagnosis in someone a history of UC, doesnt involve
CBD. The bile ducts appear in alcoholic cirrhosis PBC. 5 )
40. A 66 year old retired foundry worker was referred Medical Admissions Unit after
developing sudden onset of right sided chest pain. For past 24 hours he felt unwell,
malaise, headache myalgias. The GP's letter states patient become mildly confused over
past 3 4 hours. On examination he febrile (39C) confused. Pulse 62 per minute, pressure
110/75.
Investigations

65

Haemoglobin
WBC
Neutrophils
Lymphocytes
Monocytes
Platelets
Serum sodium
Serum potassium
Serum urea
Serum creatinine
Serum bilirubin
Serum aspartate transaminase
Serum alkaline phosphatase
Serum albumin
pH
pO2
pCO2
HCO3
Urinalysis:

16.5 g/dl
20.1 x 109/l
18.5 x 109/l
0.8 x 109/l
0.8 x 109/l
390 x 109/l
121 mmol/l
4.3 mmol/l
6.2 mmol/l
99 umol/l
7 umol/l
63 IU/l
100 IU/l
39 g/l
7.42
9.9 kPa (75 mmHg)
3.9 kPa (29 mmHg)
22 mmol/l
Protein (+)

His chest x ray shown below.

66

Which of following antibiotic regimes would be effective in treating this specific


infection?
1)
Amoxicillin clarithromycin
2)
Amoxicillin metronidazole
3)
Cefotaxime erythromycin
4)
Clarithromycin
5)
Erythromycin rifampicin
Comments:
This patients' symptoms, signs investigations are highly suggestive of Legionella pneumonia.
The illness may start abruptly a brief prodrome of malaise, myalgia headache. High fever non
productive cough are common may be accompanied by pleuritic chest pain. Confusion may
represent toxic encephalopathy. A marked neutrophil leukocytosis may be associated
concomitant lymphopoenia. Hyponatremia occurs more commonly than other pneumonias. Liver
function abnormalities are common but non specific. Proteinuria (sometimes myoglobinuria)
common. Chest x ray usually shows lobar consolidation progresses bilateral involvement in
50% of cases.
Macrolide antibiotics (erythromycin clarithromycin) are antibiotics of choice legionellosis,
rifampicin should be added in patients severe illness in immunocompromised patients. 5 )
41. A 47 year old man HIV disease presents hospital a tonic clonic seizure. He had
initially presented six months previously Pneumocystis carinii pneumonia a CD4 T
lymphocyte count of 10 cells/mm3 had subsequently started on highly active
antiretroviral therapy. His recent CD4 count, taken one month prior his new presentation,
was 50 cells/mm3. On examination he focal weakness, but both plantar responses are
extensor. Fundoscopy normal.
A CT scan of his brain shown.
67

What
1)
2)
3)
4)
5)

is the diagnosis?
AIDS related dementia
Brain abscess
Cerebral toxoplasmosis
Primary CNS lymphoma
Progressive multifocal leukoencephalopathy

Comments:
Toxoplasma encephalitis commonest cause of focal brain disease in HIV/AIDS, occurring at
CD4 counts of less than 100 cells/mm3. Alhough Toxoplasma gondii may also cause a retinitis
in association HIV/AIDS, it need occur concomitantly CNS disease. The typical appearance on
CT of multiple ring enhancing lesions, as shown here. Treatment pyrimethamine +
sulfadiazine. Patients must subsequently be maintained on long term suppressive therapy
prevent relapse.3 )
42. A 44 year old man presents Casualty a two week history of progressive dyspnoea,
dry cough fever.
Investigations
Haemoglobin
12.5 g/dl
WBC
7.2 x 109/l
Neutrophils
5.9 x 109/l
Lymphocytes
0.4 x 109/l
Monocytes
0.8 x 109/l
Platelets
130 x 109/l
Sodium
144 mmol/l
Potassium
4.1 mmol/l
Urea
7.5 mmol/l
Creatinine
75 mol/l
Arterial gases breathing air
pH
7.42
pO2
8.9 kPa (66.8 mmHg)
pCO2
4.7 kPa (35 mmHg)
HCO3
22 mmol/l
68

His chest x ray shown below.

What
1)
2)
3)
4)
5)

is the likely diagnosis?


Cytomegalovirus pneumonitis
Mycoplasma pneumonia
Pneumocystis carinii pneumonia
Pulmonary oedema
Severe acute respiratory syndrome (SARS)

Comments:
His presentation highly suggestive of PCP, a subacute onset predominant symptoms of dry
cough dyspnoea. The lymphopoenia very suggestive of PCP AIDS (and therefpre low CD4
lymphocyte count), although patients may have low lymphocyte counts other acute viral
infections indeed may be in HIV/AIDS due a compensatory increase in CD8 lymphocyte
subset. Hypoxia exercise desaturation a characteristic feature of PCP. The first choice treatment
PCP high dose co trimoxazole (iv po). Alternatives include trimethoprim + dapsone,
intravenous pentamidine. Steroids should be co administered all patients pO2 < 70 mmHg. 3 )
43. A 66 year old retired foundry worker was referred Medical Admissions Unit after
developing sudden onset of right sided chest pain. For past 24 hours he felt unwell,
69

malaise, headache myalgias. The GP's letter states patient become mildly confused over
past 3 4 hours.
On examination he febrile (39C) confused. Pulse 62 per minute, pressure 110/75.
Investigations
Haemoglobin
White cell count
Neutrophils
Lymphocytes
Monocytes
Platelets
Serum sodium
Serum potassium
Serum urea
Serum creatinine
Serum bilirubin
Serum AST
Serum ALP
Serum albumin
ABGs on air
pH
pO2
pCO2
Bicarbonate

16.5 g/dL
20.1 x 109/L
18.5 x 109/L
0.8 x 109/L
0.8 x 109/L
390 x 109/L
121 mmol/L
4.3 mmol/L
6.2 mmol/L
99 umol/L
7 umol/L
63 IU/L
100 IU/L
39 g/L

7.42
9.9 kPa
3.9 kPa
22 mmol/L

Urinalysis: Protein +
His chest x ray shown below.

70

Which of following investigations is likely to establish the identity of the causative


organism?
1)
Blood cultures
2)
Chlamydia pneumoniae serology
3)
Immunocytological staining of bronchoalveolar lavage fluid
4)
Legionella urinary antigen
5)
PCR
Comments:
This patients symptoms, signs investigations are highly suggestive of Legionella pneumonia.
The illness may start abruptly a brief prodrome of malaise, myalgia headache. High fever non
productive cough are common may be accompanied by pleuritic chest pain. Confusion may
represent toxic encephalopathy. A marked neutrophil leukocytosis may be associated
concomitant lymphopoenia. Hyponatremia occurs more commonly than other pneumonias. Liver
function abnormalities are common but non specific. Proteinuria (sometimes myoglobinuria)
common. Chest x ray usually shows lobar consolidation progresses bilateral involvement in
50% of cases.
Although diagnosis may be made by culturing organism from sputum, tests Legionella
antigens in urine offer a rapid test. 4 )

44. A 63 year old man presented 1 hour of heavy, central chest pain associated
shortness of breath sweating. He significant past history. His father had angina aged 50.
He an ex smoker of 20 cigarettes day stopped 10 years ago. He drinks 3 units of alcohol
per week.
71

Examination reveals an obese man a pulse of 84 bpm BP 145/80. Chest clear


auscultation. Heart sounds are soft added sounds. He was given 300 mg Aspirin in
ambulance also received oxygen analgesia in Accident Emergency Department.

His Electrocardiogram shown.


What is the appropriate next step?
1
2
3
4
5

)
)
)
)
)

Coronary angiography
Low molecular weight heparin
Non steroidal anti inflammatory drugs
Observation
Thrombolysis

Comments:
There a prominant R wave in lead V1 ST depression in V1 V3. There also ST elevation in leads
V5 V6. The clinical history, age ECG all suggest an acute myocardial infarction in this case a
postero lateral MI. The appropriate management thrombolysis. 5 )
45. A 14 year old girl presented hospital a 24 hour history of nausea, vomiting
abdominal cramps. On examination, she was afebrile but had a tender abdomen guarding.
Three hours after administration of a specific agent, she passed a large quantity of
material in her stools (pictured).

72

What
1)
2)
3)
4)
5)

is the diagnosis?
Ancylostoma duodenale
Ascariasis lumbricoides
Necator americanus
Strongyloides stercoralis
Trichuris trichuria

Comments:
Ascaris lumbricoides a large roundworm, growing up 35cm in length,
common nematode
parasite of humans. Infected patients are often asymptomatic. Symptoms may develop as a
result of pneumonitis caused by worm's migration through lungs, obstruction of
gastrointestinal tract biliary/pancreatic duct obstruction. Piperazine treatment of choice in
patients presenting bowel obstruction; mebendazole may be used treat other infections.
Strongyloides stercoralis a nematode infection. Symptoms may be cutaneous (pruritis due
invasion by nematodes), pulmonary (eosinophilic pneumonia due passage through lungs)
gastrointestinal (abdominal pain). Necator americanus Ancylostoma duodenale are hookworms
(`1cm in length). The major manifestation of chronic disease iron deficiency anaemia. Trichuris
trichuria (whipworm) a helminth approximately 4cm in lenth. Infected patients are largely
asymptomatic. 2 )
46. An elderly man presents an ulcer over his lower leg. He had previously served in
navy had spent a lot of time in Thailand. There was a history of high alcohol intake was a
heavy smoker many years. He had a past history of Rheumatoid arthritis, but was lost
follow up 10 years previously. He developed this ulcer 4 weeks previously despite
treatment oral erythromycin presumed infective causes continued increase in size.

73

Investigations revealed
Haemoglobin
White cell count
Platelet count
MCV
Liver function tests
Chest X ray
What
1)
2)
3)
4)
5)

12.4g/dl (NR 12 14.5)


12x109/l (NR 5 10)
344 (NR 150 450)
104 fl (NR 85 96)
Normal
No abnormality detected

is the likely diagnosis?


Cutaneous tuberculosis
Pyoderma gangrenosum
Squamous cell carcinoma
Tertiary syphilis
Yaws

Comments:
Pyoderma gangrenosum typically an expanding ulcer a polycyclic serpigenous outline a
characteristic undermined bluish edge. It can be associated inflammatory bowel disease,
rheumatoid arthritis, monoclonal gammopathies leukemia. It usually responds oral steroids.
The pathogenesis unknown, presumed be immunological. Both yaws syphilis can cause ulcers
but typically affect palms soles of feet. Squamous cell carcinomas usually are hyperkeratotic
tumours although they can ulcerate. The dorsum of foot an unusual site. Cutaneous TB can
present as ulceration but there usually a more chronic course. This question many clues away
from diagnosis an ulcer of infective etiology, but pyoderma gangrenosum likely ulcer. 2 )
47. A 50 year wine grower presented a 6 month history of recurrent blisters on dorsum
of his hand. These tended be worse in summer were occasionally itchy. He had
previously lived in tropics when he was in army. His only medication was bendrofluazide
hypertension had on same treatment previous 3 years. A picture of hands shown.

74

What is the diagnosis?


1)
Drug reaction
2)
Eczema
3)
Erythema multiforme
4)
Porphyria cutanea tarda
5)
Scabies
Comments:
Out of possible answers porphyria cutanea tarda likely induce a blistering rash on dorsum of
hand. It usually seen in men who have damaged their liver by drinking too much alcohol. This
patient was a vintner who may suggest regular alcohol intake blisters formed on face backs of
hands, in response sunlight minor trauma. These areas can become scarred hairy. The urine
pink fluoresces coral pink under woods light. Treatment by avoidance of alcohol oestrogens
venesection. The pruritic nature of lesions should make you think of scabies, as classical
hallmark of scabies are burrows. Bendrofluozide can induce photosensitivity but history of new
treatment against this being a drug reaction. Pompholyx eczema can cause vesicles mainly on
palmar aspect of hands. Erythema multiforme presents target lesions. 4 )
48. A young woman presents clinic a 3 week history of intermittent joint pain raised
rash over lower legs (picture). She complained of pain on palpation of rash on lower legs.
There was history of recent travel other recent illnesses. She had a previous history of
type 1diabetes, which she receives basal bolus insulin therapy. She had a family history of
diabetes on her maternal side psoriasis on her paternal side.

75

What investigation should be performed next?


1)
Chest X ray
2)
Complement levels
3)
Rheumatoid factor
4)
Skin Biopsy
5)
X Ray wrists ankles
Comments:
The picture shows erythema nodosum. This inflammation of subcutaneous tissue (panniculitis)
tender palpable. It a number of causes. The important causes exclude are TB sarcoidosis. It
therefore important perform a CXR. Other causes include infections (such as streptococcus
infections), drugs (sulphonamides, OCP), inflammatory bowel disease Bechets. Apart from CXR,
a throat culture, ASOT, Mantoux test, biochemistry are important. Lesions tend last 6 8 weeks.
Apart from treating underlying cause, Non steroidal anti inflammatory drugs bed rest are
helpful. The history of diabetes family history of psoriasis are coincidental. 1 )
49. A 55 year old man a history of asthma presents upper GI endoscopy investigate
chest pain, reflux symptoms, mild dysphagia solids. The picture below from mid
oesophagus.

What is the diagnosis?


1)
Achalasia
2)
Barrett's oesophagus ulceration
3)
Hiatus Hernia
4)
Oesophageal candidiasis
5)
Oesophageal carcinoma
Comments:

76

The photograph shows a long segment of Barrett's epithelium which replaced squamous epithelium
(normal mucosa seen in bottom right corner of photo). There an ulcerated circumfrencial stricture a
linear ulcer scar extending up oesophagus in 5 O'clock position. He requires biopsies exclude dysplasia
malignancy acid suppression a PPI. 2 )
50.
An 18 year old young girl presented a pruritic rash This rash had started as a single lesion
on her abdomen two weeks previously multiple smaller lesions developing over chest abdomen.
She was known have asthma hay fever, was taking salbutamol inhalers. There was a family
history of diabetes. A urine examination was negative glucose.

Examination reveals a scaly rash over the abdomen and the chest wall (see slide). What is the
likely diagnosis?
1)
Eczema
2)
Erythema multiforme
3)
Erythema migrans
4)
Guttate psoriasis
5)
Pityriasis rosea
Comments:
This classical history of pityriasis rosea. There a history of a herald patch, which presents as a single
large erythematous plaque on trunk. This followed by multiple erythematous plaques along rib lines on
chest abdomen, within a few weeks of herald patch. The condition self limiting, can be pruritic
asymptomatic usually lasts 6 8 weeks. The aetiology unknown. No particular treatment necessary,
although moisturizers can help pruritus. Pustular psoriasis as name suggests characterized by crops of
pustules. There are some misleading information regards history of atopy family history of diabetes.
Classical eczema affects flexures. This misleading information make you think of eczema, which what
picture shows. 5 )

1.
A 14 year old presents a two week history of deteriorating puffiness of face. She
aware of tiredness lethargy over last two months since having acquired a throat infection.
Over this period she gained at least 5 kg in weight over this time. She had a similar throat
infection aged 10 from which she made an uneventful recovery. There nothing else of note
in history she takes medication.
Examination reveals generalised puffiness pitting oedema of lower limbs. Her pressure
133/86 mmHg a pulse of 88 bpm. Chest, cardiovascular abdominal examinations are
normal.
77

Haemoglobin
14.6g/dL
White cell count
6 x 109/L
Platelets
250 x 109/L
Serum sodium
136 mmol/L
Serum potassium
4.0 mmol/L
Serum chloride
103 mmol/L
Serum bicarbonate
24 mmol/L
Serum urea
4.2 mmol/L
Serum creatinine
93 umol/L
Serum albumin
25 g/L
Urine dipstick
Protein +++
24 hour urine protein
4.3 g/l
Which best treatment this patient?
1 ) Observe
2 ) Salt poor albumin
3 ) Intravenous frusemide
4 ) High dose corticosteroids
5 ) Cyclophosphamide
Comments:
This patient nephrotic syndrome as defined by a triad of hypoalbuminaemia, proteinuria >
3g/24hrs oedema. The commonest cause of nephrotic syndrome in a child minimal change
disease, which usually responds a course of high dose corticosteroids. Cyclophosphamide may
hasten a remission, but given its cytotoxic profile, should be reserved steroid resistant cases. If
possible it should be avoided in childhood. Salt poor albumin iv frusemide may be useful
adjuncts managing oedema, but curative of proteinuria. If nephrotic syndrome left
unchecked, complications include Streptococcal sepsis, venous thromboembolism
hypercholesterolaemia. 4 )
2.
A 23 year old woman gave a 4 year history of intermittent diarrhoea constipation.
This had become more problematical since return from a back packing trip around
Thailand eight months ago. Her past history was otherwise unremarkable. Her family
history only her mother was suffering from hyperthyroidism.
On examination she had a body mass index of 18.5 kg/m2 but without any other specific
abnormality.
Investigations revealed:
Haemoglobin
MCV
White Cell Count
Platelets
ESR (Westergren)
Serum sodium
Serum potassium

10.3 g/dL
72.4 fL
6.7 x 109/L
192 x 109/L
5 mm/hr
139 mmol/L
4.2 mmol/L
78

Serum urea
6.4 mmol/L
Serum creatinine
75 umol/L
Corrected calcium
1.92 mmol/L
Serum Free T4
14.5
Serum TSH
0.8
What is the likely diagnosis?
1 ) Coeliac disease
2 ) Collagenous colitis
3 ) Crohns disease
4 ) Tropical sprue
5 ) Sick euthyroid

(NR 2.2 2.6)


(NR 10 21)
(NR 0.5 4.0)

Comments:
The likely explanation this patient's 4 year history plus hypocalcaemia iron deificiency anaemia
inflammatory markers Coeliac disease. In particular, family history of autoimmunity would also
favour this diagnosis. A macrocytic anaemia would be more in keeping tropical sprue history
precedes trip Thailand would argue against this. 1 )
3.
A 68 year male referred accident emergency department deteriorating
breathlessness. This deteriorated over last 2 weeks he also noticed leg swelling
discomfort. He a past history of ischaemic heart disease congestive cardiac failure, gastro
oesophageal reflux gout. He takes omeprazole 20 mg daily, allopurinol 100 mg daily,
atorvastatin 10 mg daily, digoxin 0,125 mg daily, frusemide 80 mg daily, spironolactone 10
mg daily, carvedilol 2.5 mg twice daily, Ramipril 10 mg daily recently commenced
trimethoprim 200 mg bd a urine tract infection.
Examination reveals him be slightly breathless at rest a pressure of 108/80 mmHg. His
jugular venous pressure elevated he peripheral oedema up his knees. Auscultation of
heart reveals a soft systolic murmur at apex he bilateral basal chest crackles.
Investigations reveal:
serum sodium
131 mmol/l
serum potassium 4.0 mmol/l
serum urea
26.5 mmol/l
serum creatinine
400 micromol/l
Plasma glucose
5.9 mmol/l
Which drug does require dose reduction due his renal impairment?
1 ) Allopurinol
2 ) Digoxin
3 ) Omeprazole
4 ) Spironolactone
5 ) Trimethoprim
Comments:
Omeprazole principally dependant upon hepatic clearance safe even marked renal impairment.
Spironolactone should probably be avoided this degree of renal impairment owing risk of
hyperkalaemia. Allopurinol toxicity increased in moderate severe renal impairment. Although
reduction in dose trimethoprim advocated in renal impairment, there are seldom any significant
problems at a full dose. 3 )
79

4.
A 20 year old woman a history of paroxysmal episodes of dysarthria, ataxia diplopia
lasting 20 30 minutes followied by right sided severe headache associated vomiting
lasting 1 3 days. The episodes occur once every month. MRI brain MR angiogram are
normal.
What is the likely diagnosis?
1 ) Basilar artery Transient ischaemic attacks
2 ) Basilar migraine
3 ) Brain stem Arteriovenous malformation
4 ) Giant cell arteritis
5 ) Multiple sclerosis
Comments:
The likely diagnosis Basilar Migraine. The MRI brain MR angiogram excludes other
possibilities. 2
5.
A 36 year old presents thirst frequency of micturition. These symptoms have
deteriorated over last three months she now aware of twice nightly nocturia occasionally
needs drink during night. She aware of intermittent frontal headaches over this time
which are generally relieved by paracetamol are without any relationship time of day. She
rather stressed of late since break up of her marriage six months ago. Since separation
she taking Fluoxetine 20 mg daily. No abnormalities are detected upon examination.
She undergoes an 8 hour water deprivation test which reveals following:
Time
8am
9am
10am
11am
12 midday
1 pm
2 pm
3 pm
4 pm

Weight (kg)
66.5

Plasma Osm
285

66

300

65.2

303

Urine Osm Urine Vol mls


110
200
270
310
352
320
0
420

200
290
220
200
120
80
50

DDAVP 1 microgram sc given at 4 pm permitted drink


6pm
66
295
660
20
P]range plasma osmolality 275 290 mosmol/kg]
What is the diagnosis?
1 ) Complete central diabetes insipidus
2 ) Fluoxetine induced diabetes insipidus
3 ) Nephrogenic diabetes insipidus
4 ) Partial diabetes insipidus
5 ) Primary polydipsia
Comments:
This patient developed haemoconcentration following 8 hrs of dehydration but although begun
concentrate her urine this inadequate constitutes partial DI. She responds well DDAVP
80

indicating she central DI rather than Nephrogenic DI. Fluoxetine does cause DI but
infrequently associated SIADH. 4 )

6.
A 40 year old man chronic alcohol abuse presented in accident emergency
department confusion, agitation ataxia. He had found wandering streets at 2 am was
brought in by police. He had admitted on numerous occasions previously related alcohol
abuse.
On examination he appeared disheveled, confused smelt of alcohol. He was disoriented in
time place a mini mental score of 16/30. He was apyrexial a pressure of 138/90 mmHg.
He had bilateral 6th nerve palsies, gaze evoked nystagmus gait ataxia.
What treatment should this patient receive?
1
2
3
4
5

)
)
)
)
)

Diazepam
Immunoglobulins
Penicillin
Steroids
Thiamine

Comments:
The likely diagnosis Wernickes encephalopathy. It characterized by confusion, ataxia gaze
palsies. Intravenous thiamine should be given immediately in those situations. The commonest
cause chronic alcohol abuse. 5 )
7.
A 51 year man presents a dry irritating unproductive cough. It often keeps him
awake at night. He a heavy snorer. He admits breathlessness on exertion feels tired
during day. He smoked 20 cigarettes a day since age of 20 years recently diagnosed as
having hypothyroidism. He overweight (BMI = 34). His lung function shows:
Observed
Normal Range
FEV1
2.8 L
(3.3 4.8)
VC
2.9 L
(3.7 5.6)
TLC
3.4 L
(6.0 8.2)
RV
0.9 L
(1.3 2.9)
TLCO
4.3 mmol/min/kPa
(8.2 12.8)
KCO
1.1 mmol/min/kPa/L
(1.2 2.0)
What is the likely diagnosis?
1 ) Asthma
2 ) Bronchiolitis Obliterans Organising Pneumonia
3 ) Chronic Obstructive Pulmonary Disease
4 ) Cryptogenic Fibrosing Alveolitis
5 ) Sleep Apnoea
Comments:
81

This mans lung function demonstrates a restrictive defect an increased ratio of FEV1/FVC
decreased lung volumes. Both Tlco Kco are reduced. The Tlco a sensitive (but specific)
indicator of integrity of alveolar capillary membrane gas exchange function of lung. He Kco
corrects any reduction in lung volume (eg after pneumonectomy). 4 )
8.
A 32 year old floor manager a paint warehouse presented with shortness of breath.
Recently, whilst on holiday in Lake District her symptoms had completely resolved. She
presented in early hours of morning having used her Salbutamol inhaler 12 times. She
was treated a Salbutamol nebuliser steroids completely recovered.
Which best investigation determine diagnosis?
1
2
3
4
5

)
)
)
)
)

Bronchial Provocation testing


CXR
PEFRs measure at home work
RAST IgE
Skin Prick Testing

Comments:
This patient occupational asthma. She symptoms of asthma improve when she on holiday
from her job.
A good history essential diagnosing occupational asthma. All jobs a patient had need be
noted as well as materials which they have exposed, when they were exposed interval
between exposures development of symptoms.
Objective measurements are key confirming diagnosis are best performed using serial PEFR
measured 2 hourly from waking sleeping at least over a 4 week period which should include at
least three periods away from work at least 2 consecutive days, although patients more severe
disease may require more than 10 days away from work environment before improvements are
noted.
Whilst serial PEFR measurement diagnositic test of choice Bronchial provocation tests specific
IgE (RAST) are required identify specific agent at work causing asthma. 3 )
9.
A 19 year woman seen following recurrent episodes of cystitis despite trimethoprim
therapy. As well as these symptoms she diagnosed diabetes mellitus from 12 years of
age receives twice daily mixed insulin. She also takes oral contraceptive last two years
smokes 10 cigarettes daily. She two elder brothers who are both well. Examination reveals
a thin but well looking a BMI of 21.5 kg/m2, a pressure of 108/76 mmHg cardiovascular,
respiratory abdominal examination.
Investigations reveal:
Serum Sodium
140 mmol/l
Serum Potassium
4.5 mmol/l
Serum Urea
5.6 mmol/l
Serum Creatinine
95 micromol/l
Fasting plasma glucose
10.2 mmol/l
HbA1c
10.1%
Ultrasound abdomen suggests reflux nephropathy
What best treatment preserve her renal function?
82

1
2
3
4
5

)
)
)
)
)

ACEi
Prophylactic antibiotics
Stop smoking
Strict glycaemic control
Surgical intervention correct reflux

Comments:
Strict glycaemic control would reduce frequency of recurrent infections reduce risk of
progression diabetic nephropathy. If she had proteinuria in absence of proven UTI, she would
benefit from an ACEI, even if she was normotensive. However, this would decrease frequency of
UTIs. Stopping smoking would be beneficial in reducing risk of developing renovascular disease.
Prphylactic antibiotics are of limited benefit in preserving renal function recurrent UTIs, but
should be offered if patient symptomatic her infections. Vesico ureteric reflux usually occurs
early in childhood, it at this juncture any surgical intervention would be beneficial. When
picked up in adulthood, mainstay of management would be pressure control, prompt treatment
of UTI careful surveillance during pregnancy. 4 )
10. A 60 year old man presented 6 months history of pulsatile tinnitus, dysarthria,
wasting of his tongue. On examination, there are palsies of tenth, eleventh twelfth cranial
nerves on right. The remainder of his neurological examination normal. What is the likely
diagnosis?
1
2
3
4
5

)
)
)
)
)

acoustic neuroma
glomus jugulare tumor
hypoglossal neuroma
motor neuron disease
Nasopharyngeal carcinoma

Comments:
The combination of X, XI, XII nerve palsies localize lesion in jugular foramen. Pulsatile tinnitus
consistent glomus tumour. 2
11. A 26 year who 13 weeks pregnant seen in outpatient clinic noted have a
sustained pressure of 170/92 mmHg. She past history of note otherwise well
asymptomatic. This her first pregnancy. Examination otherwise generally abnormalities
are noted on fundoscopy. Ultrasound examination of kidneys showed both kidneys be of
equal size 9 10 cm. Urinalysis reveals protein (+) (+).
What is the appropriate anti hypertensive therapy this patient?
1 ) ACE inhibitor
2 ) alpha Methyldopa
3 ) Beta blocker
4 ) Hydralazine
5 ) Labetolol
Comments:
Methyldopa safest agent use in first second trimester of pregnancy. Beta blockers may cause
intrauterine growth retardation. Manufacturers recommend avoidance of hydralazine ACEI in
pregnancy. 2 )
12. A 44 year male a 4 year history of alcoholic liver disease presents abdominal pain
confusion. On examination he mildly confused a Glasgow Coma Scale of 15. He appears
jaundiced, spider naevi clubbing of fingers. His temperature 37C, a pressure of 122/88
83

mmHg. Gynaecomastia noted abdominal examination reveals a mildly distended abdomen


left hypochondrial tenderness. No organomegaly noted.Investigations reveal:
Haemoglobin
14.9 g/dL
White cell count
7.2 x 109/L
Platelets
279 x 109/L
Serum sodium
139 mmol/L
Serum potassium
3.9 mmol/L
Serum urea
6.5 mmol/L
Serum creatinine
95 umol/L
Plasma glucose
5.5 mmol/l
Bilirubin
65 umol/L
AST
150 IU/L
ALT
110 IU/L
Alkaline Phosphatase
450 IU/L
Gamma GT
500 IU/L
Amylase
700 IU/L (NR < 150)
He was commenced on an intravenous 5% dextrose infusion was prescribed
dihydrocodeine regularly abdominal pain thought be associated chronic pancreatitis.
However, over next 24 hours he became increasingly drowsy associated a deterioration in
his Glasgow Coma Scale 8.
Which of following agents should be administered immediately?
1
2
3
4
5

)
)
)
)
)

Cefotaxime
Flumazenil
Naloxone
Thiamine
Vitamin K

Comments:
This patient chronic liver disease presents features of decompensation associated chronic
pancreatitis. Patients alcoholic liver disease are often surprisingly sensitive opiate analgesia
which should only be used caution. 3 )
13. A 22 year old obese woman presented an 8 week history of headaches, pulsatile
tinnitus transient visual loss on standing lasting a few seconds. She had otherwise well
history of note. She took oral contraceptive pill had taking this last 6 months used
salbutamol inhalers on an occasional basis her asthma which she had from childhood. She
also took vitamin A supplements which she bought over counter her general health.
On examination, only abnormality of note was bilateral papilloedema.
MRI brain MR Venogram are normal. Lumbar puncture showed an opening pressure of 38,
protein, glucose, cells.
What is the likely diagnosis?
1 ) Herpes simplex encephalitis
2 ) Intracranial hypertension secondary vitamin A
84

3 ) Malignant meningitis
4 ) Sagittal sinus thrombosis secondary OCP
5 ) Vertebral artery dissection
Comments:
The findings will be consistent benign intracranial hypertension (papilloedema, CSF analysis,
brain imaging). Vitamin A a well known cause of this condition (as well as tetracycline oral
contraceptive). 2 )

14. A 66 year admitted nausea vomiting of two days duration. Her her husband had
eaten a take away meal had both become ill. However, her husband a had recovered after a
day. Otherwise she had fit well except hypertension which she took amlodipine 5 mg
daily. On examination she appeared comfortable, a temperature of 36.5C, a pulse of 90
beats per minute regular a pressure of 146/98 mmHg. No other abnormalities were noted.
Investigations reveal following:
Sodium
146 mmol/l
Potassium
3.5 mmol/l
Urea
10.2 mmol/l
Creatinine
142 micromol/l
Corrected Calcium
3.01 mmol/l
Phosphate
0.7 mmol/l
Alkaline Phosphatase
110 iu/l
Aspartate transaminase
25 iu/l
Alanine transaminase
31 iu/l
What best treatment this patient?
1
2
3
4
5

)
)
)
)
)

(133 145)
(3.4 5.5)
(3 7)
(50 100)
(2.2 2.5)
(0.8 1.2)
(50 110)
(0 40)
(0 40)

Haemodialysis
IV 5% Dextrose
IV Normal saline
IV Pamidronate
Subcutaneous Calcitonin

Comments:
This patient dehydrated due gastroenteritis a moderate hypercalcaemia. The best initial
treatment rehydration 5% dextrose as she already hypernatraemic. This manoeuvre would be
likely produce a fall in her calcium concentration which, in conjunction hypertension
hypophosphataemia, likely be due primary hyperparathyroidism. Hyperparathyroidism usually
asymptomatic often picked up through routine biochemistry. Appropriate investigation of her
hypercalcaemia would include measurement of PTH which would be within range
(inappropriately normal) high. 2 )
15. A 65 year retired miner attends outpatient department a six month history of
breathlessness on exertion which got worse following a recent chest infection. He a
85

cough productive of mucoid sputum. He smoked 20 cigarettes a day until 5 years ago. He
keeps pigeons at home.
Lung function tests before after bronchodilators show:
Before
After
Normal range
FEV1
0.9
1.0
(2.2 4.4)
VC
2.1
2.3
(3.0 4.8)
TLC
7.2
(2.4 4.6)
Kco
0.6
(1.2 2.1)
What is the diagnosis?
1 ) Asthma
2 ) Chronic Obstructive Pulmonary Disease
3 ) Coal workers Pneumoconiosis
4 ) Pigeon Fanciers Lung
5 ) Extrinsic allergic Alveolitis
Comments:
This man's lung function demonstrates an obstructive pattern. The FEV1/VC usually greater
than 75% in young people but falls 70 75% in elderly subjects. Below this an obstructive
pattern. As airway obstruction progresses both FEV1 VC decrease. There an increased TLC. The
results also show there significant reversibility bronchodilators (i.e < 15% improvement in
FEV1) transfer co efficient reduced which consistent a diagnosis of COPD rather than asthma.
COPD divided into mild (FEV= 60 80%), Moderate (40 59%), severe (<40% predicted) (see BTS
guidelines: www.brit thoracic.org.uk) although international guidelines classification slightly
different: see www.goldcopd.com. 2 )
16. A 74 year male presents his General Practitioner dyspnoea. Over last three months
he aware of deteriorating breathlessness exercise capacity. He noticed he now develops
breathlessness after walking 100 yards on flat. He also noticed an approximate 7 kg
weight gain over this time. Up until this illness he well takes medication.
Examination reveals he appears comfortable at rest, a BMI of 29.5 kg/m2, a pressure of
124/88 mmHg a slight elevation of jugular venous pressure, peripheral oedema mid
tibia bibasal fine crackles on chest examination. Auscultation of heart normal.
Investigations reveal:
Serum urea
8.5 mmol/L
Serum creatinine
161 umol/L
His GP starts him on amiloride 10 mg frusemide 80mg od.
One week later, he returns markedly improved symptoms, being now asymptomatic
frusemide
stopped.
Repeat tests reveal:
Serum sodium
134 mmol/L
Serum potassium 5.9 mmol/L
Serum urea
12.1 mmol/L
Serum creatinine
190 umol/L
86

An ECG
normal.
What is the appropriate management of his hyperkalaemia?
1 ) Do nothing
2 ) Give calcium gluconate
3 ) Give calcium resonium
4 ) Give insulin + dextrose infusion
5 ) Give IV Normal saline
Comments:
Calcium gluconate insulin dextrose are reserved more severe hyperkalaemia, more marked
renal impairment, particularly, if there are ECG changes consistent hyperkalaemia. Calcium
resonium an unpleasant drug take commonly administered as a drug managing chronic
hyperkalaemia. This patient a mild hyperkalaemia, in first instance, should have close
monitoring of potassium levels. If this should rise further, it would be more appropriate
reintroduce a loop diuretic. 1 )
17. A 60 year old woman presented 24 months history of progressive back pain worse
on coughing, weakness of lower limbs, urinary incontinence nocturnal muscle spasms.
On examination, cranial nerves are intact, neck power normal, upper limbs are normal,
there a sensory level at T6, symmetrical spastic paraparesis muscle strength of 4 out of 5,
bilateral ankle clonus extensor plantar response. General examination normal.
What is the likely diagnosis?
1
2
3
4
5

)
)
)
)
)

spinal meningioma
motor neuron disease
multiple sclerosis
anterior spinal artery thrombosis
syringomyelia

Comments:
The long gradual history suggestive of a spinal tumor. Sensory involvement excludes motor
neuron disease. The age of patient atypical multiple sclerosis. The presentation of anterior
spinal artery occlusion acute. 1 )
18. A 19 year old student presents Casualty a seven day history of fever, sore throat
malaise. He managed eat food several days: partly because of severity of pharyngitis,
but also because of a loss of appetite. On examination, it noted he jaundiced.
Investigations reveal
Haemoglobin
14.9g/dL
MCV
89 fL
White cell count
6.5 x 109/L
Platelets
350 x 109/L
Reticulocytes
0.8% (NR 0.2 2.0%)
Serum bilirubin (total)
60 mmol/L
Conjugated bilirubin
10 mmol/L
Serum AST
19 IU/L
Serum ALP
117 IU/L
87

Serum albumin
42 g/L
What is the likely diagnosis?
1 ) Autoimmune hepatitis
2 ) Chronic hepatitis C virus infection
3 ) Gilberts syndrome
4 ) Primary biliary cirrhosis
5 ) Recurrent cholecystitis
Comments:
The level of unconjugated bilirubin elevated in absence of any other liver function
abnormalities. From history given, it appears hyperbilirubinaemia occurred following an upper
respiratory tact infection may have viral; this illness had associated anorexia. When calorie
intake falls, plasma levels of unconjugated bilirubin can double this rise may be sufficient
produce clinical jaundice in patients Gilberts syndrome. 3 )
19. A 19 year old student presents Casualty a seven day history of fever, sore throat
malaise. He managed eat food several days: partly because of severity of pharyngitis,
but also because of a loss of appetite. On examination, it noted he jaundiced.
Investigations reveal
Haemoglobin
14.9g/dL
MCV
89 fL
White cell count
6.5 x 109/L
Platelets
350 x 109/L
Reticulocytes
0.8% (NR 0.2 2.0%)
Serum bilirubin (total)
60 mmol/L
Conjugated bilirubin
10 mmol/L
Serum AST
19 IU/L
Serum ALP
117 IU/L
Serum albumin
42 g/L
What is the likely diagnosis?
1 ) Autoimmune hepatitis
2 ) Chronic hepatitis C virus infection
3 ) Gilberts syndrome
4 ) Primary biliary cirrhosis
5 ) Recurrent cholecystitis
Comments:
The level of unconjugated bilirubin elevated in absence of any other liver function
abnormalities. From history given, it appears hyperbilirubinaemia occurred following an upper
respiratory tact infection may have viral; this illness had associated anorexia. When calorie
intake falls, plasma levels of unconjugated bilirubin can double this rise may be sufficient
produce clinical jaundice in patients Gilberts syndrome. 3 )
20. A 70 year old woman admitted confusion dehydration. Her relatives claim she
deteriorating over last couple of months. They have noticed lately she become
incontinent of urine she become more withdrawn depressed. Her past history

88

unremarkable except hypetension diagnosed three years ago which she takes
bendrofluazide 2.5 mg daily amlodipine 10 mg daily.
On examination she confused but alert. She knows her name address but does know
where she is. She appears clinically dehydrated, a pulse of 110 bpm regular, a pressure of
194/88 mmHg apyrexial. Examination of cardiovascular system, abdomen chest are all
normal. No goitre was palpable but breast examination genrally nodular breasts.
Investigations reveal
Full count normal
Serum sodium
146 mmol/L
Serum potassium
3.3 mmol/L
Serum urea
12.1 mmol/L
Serum creatinine
250 umol/L
Serum calcium
4.2 mmol/L
Serum phosphate
0.8 mmol/L
Parathyroid hormone
1210 nmol/L
(NR 3.3 5.4)
What is the likely diagnosis?
1 ) Mutiple myeloma
2 ) Multiple bony metastases
3 ) Occult cancer ectopic PTH related pepetide secretion
4 ) Parathyroid adenoma
5 ) Parathyroid carcinoma
Comments:
This patient grossly elevated calcium in context of extremely high PTH concentration a
diagnosis of Parathyroid carcinoma should be considered. Parathyroid cancer rare only feature
may make you suspicious of a cancer rather than adenoma grossly elevated PTH. This ectopic
PTHrp secretion as, PTH would be expected be low ectopic PTH rp secretion latter does cross
react standard PTH assay. 5 )
21. A 15 year girl referred by her general practitioner agitation weight gain. Her
mother accompanies her during consultation explains over last 2 months she become
increasingly agitated poor sleep. Her progress at school up until recently, fine, although
of late she apathetic. She past history of note.
Examination reveals specific abnormalities a pressure of 112/70 mmHg a BMI of 20. Her
GPs letter reveals following results:
TSH
3.2 mU/L
Total T4
250 nmol/L
free T4
12.9 pmol/L
Total T3
3.2 nmol/L
Free T3
3.8 pmol/L
What is likely the diagnosis?
1 ) Anorexia nervosa
2 ) Apathetic thyrotoxicosis
3 ) Laboratory error

(NR
(NR
(NR
(NR
(NR

0.35 5.0)
55 144)
9 24)
0.9 2.8)
3.0 5.8)

89

4 ) Pregnancy
5 ) Secondary hyperthyroidism
Comments:
This patient a good story of something going awry in last 2 months thyroid function except
elevated total T4 T3 concentrations reflecting increased hormone binding, suggested diagnosis
pregnancy. 4 )
22. A 25 year old man presented diazepam overdose. He was found lie in his kitchen
semi conscious had probably there many hours. Two days after his admission into
hospital he complained of weakness of his right foot. On examination, he weakness of his
right dorsiflexion eversion. His reflexes are all intact. He decreased pain perception over
lateral aspect of his leg.
What is the diagnosis?
1
2
3
4
5

)
)
)
)
)

Common peroneal nerve palsy


L5 radiculopathy
Lumbar plexopathy
Sciatic nerve palsy
Tibial nerve palsy

Comments:
The clinical findings are consistent common peroneal nerve palsy. It commonest entrapment
neuropathy in lower limbs. It common after prolonged immobility supine position. The
entrapment site neck of fibula. 1 )
23. A 45 year retired coal miner simple silicosis presented with shortness of breath. He
retired early received a coal workers' pension. He had short of breath 3 months. He kept
budgerigars as pets last 3 months. On auscultation he had basal crepitations but CXR
showed fine nodular shadowing in apices.
What is the likely diagnosis?
1 ) Progressive silicosis
2 ) Tuberculosis
3 ) Allergic Bronchopulmonary Asperigilliosis
4 ) Psittacosis
5 ) Extrinsic allergic alveolitis
Comments:
Silicosis usually results in small nodular opacities in mid upper zones. There may be associated
hilar gland enlargement which may calcify producing characteristic eggshell calcification. These
radiological shadows are usually associated symptoms loss of lung function.
Extrinsic allergic alveolitis an inflammatory reaction inhaled organic dusts of which there are
many causes such as M.faeni, T. vulgaris (Farmer's Lung), Thermoactinomyces
sacchari(Bagassosis), A. clavatus (Malt workers' Lung) etc.
Bird Fancier's lung EAA caused by inhalation of avian serum proteins present particularly in
bloom of birds feathers. It occurs commonly among pigeon fanciers budgerigar owners latter
usually presenting a chronic EAA. Patients present a progressive dyspnoea on exertion.
90

Inspiratory craacles are heard on auscultation of lungs. CXR shows fine linear opacities in upper
lobes which may progress honeycombing.
The diagnosis of EAA based on typical clinical, radiological lung function changes in presence
of an identified source of antigen positive precipitating antibodies in patients serum causal
antigen. Improvement of clinical abnormalities following avoidance of causal antigen helps
confirm diagnosis. 5 )
24. A 78 year woman admitted hospital acute coronary syndrome discharged on day
6 after making a good recovery. She continues be under investigation of cardiologists
discharged on atenolol 50 mg daily, enalapril 10 mg daily, isosorbide mononitrate 30 mg
daily, Atorvastatin 20 mg daily aspirin 75 mg daily. Her investigations at admission a
serum urea concentration of 12.4 mmol/l a serum creatinine of 250 micromol/l. She re
admitted one week after discharge deteriorating dyspnoea. Investigations reveal a serum
urea of 28.9 mmol/l, a serum creatinine of 600 micromol/l, a serum bicarbonate of
18mmol/L a potassium concentration of 6.0 mmol/l T wave inversion in leads II, III V5
6.What best course of management if patient clinically fluid overloaded?
1 ) Dialysis
2 ) Frusemide infusion
3 ) Oral frusemide
4 ) Salbutamol nebulisers
5 ) Stop ACEI
Comments:
This lady fluid overloaded, running risk of overt pulmonary oedema, especially in context of
myocardial ischaemia. She also hyperkalaemic, which potentiates risk of a malignant arrhythmia
especially in a patient who more prone dysrhythmia after MI. The safest option dialyse her,
which will have added benefit of improving myocardial function improving hyperkalamia. 1 )
25. A 30 year presents tiredness poor appetite. She found increasing difficulty coping
at home, being particularly tired since birth of her son, 2 months ago. She one other child,
a girl aged 3. Both pregnancies were uneventful although she required iron supplements
anaemia during both pregnancies. She breast feeding quite normally her son quite well
There nil of note in her past history she takes regular medication. There family history
of note.
On examination she a BMI of 24 kg/m2, a pulse of 96 beats per minute a pressure of
124/70 mmHg. A small goitre palpable but bruit audible. She a slight tremor of her
outstretched hands. Cardiovascular, respiratory abdominal examinations are normal.
Investigations show:
Haemoglobin
10.5 g/dL
ESR (Westergren)
21 mm/1st hour
Serum sodium
136 mmol/L
Serum potassium
3.7 mmol/L
Serum urea
5.6 mmol/L
Serum creatinine
75 umol/L
Random plasma glucos
5.2 mmol/L
Serum T4
28.2 nmol/L
(NR 10 22)
Serum T3
6.8 nmol/L
(NR 3.0 5.5)
TSH
0.05 mU/L
(NR 0.5 4.0)
91

What is the likely diagnosis?


1 ) De Quervains thyroiditis
2 ) Graves disease
3 ) Hashimotos thyrotoxicosis
4 ) Postpartum thryoiditis
5 ) Toxic nodular goitre
Comments:
Post partum thyroiditis occurs in approximately 5% of females associated transient
hyperthyroidism usually 2 6 months postpartum followed by hypothyroidism which ialso usually
resolves but permanent hypothyroidism may occur. The exact aetiology unknown but
lymphocytic infiltration of thyroid typical suggesting auto immunity. Treatment
hyperthyroidism usually conservative as symptoms would resolve but, if required, beta blockers
are adequate. De Quervains thyroiditis associated tender enlargement of thyroid marked
constitutional symptoms weight loss in particular TFTs are often normal. A markedly elevated
ESR would also be expected mild elevation in this patient may be explained by her mild
anaemia. 4 )
26. A 17 year old woman presented ataxia, dysarthria inco ordination of her upper
limbs since age of 10. She now wheelchair bound. On examination, she scanning speech,
gaze evoked horizontal nystagmus, ataxia of upper limbs, spastic paraparesis absent knee
ankle reflexes, extensor plantar response, bilateral pes cavus.
What is the diagnosis?
1 ) Ataxia telengiectasia
2 ) Autosomal dominant spinocerebellar ataxia
3 ) Brain stem glioma
4 ) Friedreichs ataxia
5 ) Multiple sclerosis
Comments:
Friedreichs ataxia autosomal condition presenting in first decade. It associated
spinocerebellar degeneration. Bilateral pes cavus common. The typical clinical signs are
cerebellum dysfunction, spastic paraparesis, absent reflexes in lower limbs. Cardiomyopathy,
diabetes optic atrophy are also associated this condition. 4 )
27. A 52 year type 2 diabetic woman presents her annual review. She was diagnosed
diabetes 3 years ago was diagnosed hypertension 2 years ago. Currently she remains on
diet control alone her diabetes taking bendrofluazide 2.5 mg daily. There other past
history of note. She stopped smoking 5 years ago drinks approximately 5 glasses of wine
weekly.
On examination, she a body mass index of 33.1 kg/m2, a pulse of 88 beats per minute a
pressure of 160/92 mmHg. Her peripheral pulses are all present she a slight reduction of
light touch sensation in feet. Fundoscopy through dilated pupils reveals some hard
exudates close macula bilaterally. Her investigations reveal:
full count normal
Sodium
141 mmol/l (133 145)
Potassium
3.5 mmol/l (3.4 5.5)
Urea
10.2 mmol/l (3 7)
Creatinine
160 micromol/l (50 100)
Fasting plasma glucose
12.5 mmol/l (3.5 6)
92

HbA1c
8.1% (less than 6)
Which of following would be appropriate treatment reduce her cardiovascular risk?
1 ) Insulin
2 ) Metformin
3 ) Ramipril
4 ) Reduce dietary salt
5 ) Weight reduction Orlistat
Comments:
The appropriate treatment reduce her cardiovascular risk should focus on adequate pressure
control, supported by evidence from UKPDS which showed greater reductions in CV risk BP
control as compared overall reduction in CV mortality in patients tight glycaemic control on
insulin sulphonylureas. Similarly, HOPE study demonstrated addition of ACEi, ramipril, may
produce even greater benefit in subjects at high CV risk. In this patient addition of ramipril
bendrofluazide would be logical. Although metformin would be expected reduce CV risk in
obese type 2 diabetics, drug would be relatively contra indicated in this patient whose
creatinine exceeds 150 micromol/l. Weight reduction itself shown reduce CV risk as yet
studies have demonstrated this orlistat. 3 )
28. A 66 year retired plumber presents a two year history of progressive
breathlessness. He cough nor chest pain. He a 50 pack year smoking history drinks 40
units of alcohol a week. His appetite good his weight unchanged (BMI = 24). His
spirometry shows:

FEV1
FVC
PEF (l/min)
TLC
TLco (mmol/min/kPa)
Kco (mmol/min/kPa/l)
What is the diagnosis?
1
2
3
4
5

)
)
)
)
)

Actual
2.7 L
2.9 L
500
4.1 L
7.8
2.0

Predicted
3.3 L
4.2 L
550
7.2 L
8.0
1.4

Asbestosis
Chronic obstructive airways disease
Cryptogenic Fibrosing Alveolitis
Extrinsic allergic Alveolitis
Extensive Bilateral Pleural Thickening

Comments:
This man's lung function shows a restrictive pattern increased ratio of FEV1/FVC decreased
TLC. Extra pulmonary restriction produces a characteristic pattern where Kco greater than a
slight reduction in Tlco because of patients inability achieve a full inspiration. Causes of
extrapulmonary restriction include pleural disease, skeletal deformities respiratory muscle
weakness. Diffuse pleural thickening usually as a result of asbestos exposure although
relationship exposure dose as clearly defined as in asbestosis mesothelioma. The Kco
93

correlates degree of pleural thickening. The degree of breathlessness disability increases


increasing severity of pleural thickening. The condition tends progress although in itself an
additional risk factor development of malignancy. 5 )
29. A 15 year old girl complained of anxiety and excessive sweating. She was not taking
any medication.
Investigations showed:
TSH concentration
0.9 mU/L (0.5 3.4)
free T4 concentration
16 pmol/L (10 18)
total T4 concentration
180 nmol/L (55 145)
free T3 concentration
8.2 pmol/L (3.5 10.5)
total T3 concentration
3.3 nmol/L (0.9 2.5)
These results are compatible with which one of the following diagnoses?
1 ) Factitious thyrotoxicosis
2 ) Familial dysalbuminaemic hyperthyroxinaemia
3 ) Pregnancy
4 ) Sick euthyroid syndrome
5 ) Thyrotoxicosis
Comments:
The symptom complex intentionally misleading. The patient a TSH free T3 T4
concentrations, excluding thyrotoxicosis but elevated Total concentrations suggesting a rise in
binding globulins. This can occur in pregnancy. Sick euthyroidism would be typically associated
low thyroid hormone concentrations.3)
30. A 58 year man presents shortness of breath. His respiratory function tests are
shown below:
Lung function test
Actual
Predicted
FEV1(l)
2.4
3.0
FVC (l)
2.8
3.8
RV (l)
1.4
2.0
Tlco (mmol/min/kPa)
6.2
7.4
KCO (mmol/min/kPa/l)
1.7
1.4
What is the diagnosis?
1 ) Anaemia
2 ) EAA
3 ) Cryogenic fibrosing alveolitis
4 ) Pleural thickening
5 ) Emphysema
Comments:
This patients FEV1 FVC are both reduced producing a restrictive picture on spirometry. This
confirmed by small RV. Tlco also reduced in restrictive defects but elevated Kco in keeping
extrapulmonary restriction. This can be caused by pleural disease such as pleural thickening,
respiratory muscle weakness chest wall disease such as thoracoplasty scoliosis.
94

Emphysema produces obstructive spirometry increased RV but decreased Tlco.


CFA produces a restrictive picture but Kco normally reduced although it can be but never
elevated.
Anaemia does affect spirometry lung volumes but will result in a decreased Tlco Kco although
there are equations correct this. 4 )
31. A 15 year presents primary amenorrhoea. She accompanied by her mother who
explains she also lost approximately 10 kg of weight over last year occasional episodes
of diarrhoea. She recently become a vegetarian tends favour wheat snacks bread.
Her progress at school excellent, she plays piano in school orchestra she regularly goes
jogging. She a younger brother who well her mother maternal aunt have a past history of
hyperthyroidism. Her parents divorced about 2 years ago she sees her dad infrequently.
She takes medication.
On examination she thin a BMI of 17.6 kg/m2 appears phenotypically female. She breast
development galactorrhoea expression, absence of axillary scanty pubic hair.
Investigations reveal following
Plasma oestradiol
70 pmol/l (130 550)
LH
3.5 mU/l (2 10)
FSH
4.0 mU/l (2 10)
17 Hydroxyprogesterone
5.2 nmol/l (3 15)
free T4
12.4 nmoll/l (9 22)
TSH
2.2 mU/l (0.5 4.5)
Prolactin
520 mU/l (50 500)
What is the likely diagnosis?
1 ) Anorexia nervosa
2 ) Pregnancy
3 ) Microprolactinoma
4 ) Coeliac disease
5 ) Thyrotoxicosis
Comments:
This patient likely have anorexia nervosa as reflected by low BMI, excessive exercise
amenorrhoea due hypogonadotrophic hypogonadism. The mild hyperprolactinaemia often a
feature of Anorexia at this level would be unlikely be due a microptolactinoma. Pregnancy
would be associated elevated Oestradiol concentrations. This unlikely be Coeliac disease based
on endocrine abnormalities. 1 )
32. A 26 year who 13 weeks pregnant seen in outpatient clinic noted have a
sustained pressure of 170/92 mmHg. She past history of note otherwise well
asymptomatic. This her first pregnancy.
Examination otherwise generally abnormalities are noted on fundoscopy. Ultrasound
examination of kidneys showed both kidneys be of equal size 9 10 cm. Urinalysis reveals
protein (+) (+).
What is the likely cause of her hypertension?
1 ) Pre eclampsia
2 ) IgA nephropathy
3 ) Fibromuscular dysplasia
4 ) Membranous glomerulonephritis
95

5 ) Reflux nephropathy
Comments:
At 13 weeks of pregnancy, it too early present pre eclampsia. Fibromuscular dysplasia does
typically present dipstick haematuria. Membranous GN associated a nephrotic syndrome
therefore one would expect more proteinuria haematuria. This patient sized kidneys, which
would be unusual reflux nephropathy. IgA nephropathy can present all above features. 2 )
33. A 17 year old male admitted casualty after a night out friends at a nightclub. After
drinking eight pints of beer, he felt nauseated vomited. Vomitus was seen contain bright
red after vomiting a second time. Investigations:
Oesophago gastro duodenoscopy: Normal
What is the next step in his management?
1 ) Coeliac axis angiography
2 ) Discharge patient from hospital
3 ) Prescribe a proton pump inhibitor admit patient overnight observation
4 ) Refer surgeons review
5 ) Discharge patient arrange a repeat endoscopy in one month
Comments:
The history compatible a Mallory Weiss tear. No additional treatment follow up required
patient can be discharged from hospital. 2 )
34. A 27 year woman seen in clinic investigation of abdominal discomfort. She aware
of vague abdominal discomfort many months lower abdominal pain which occurred
intermittently. Her weight generally steady although occasionally she loses her appetite.
She otherwise well married. Her only medication combined oral contraceptive which
she taking many years.
Examination reveals her look well a BMI of 24.5 kg/m2 a pressure of 140/80 mmHg. No
specific abnormalities are noted on chest heart abdominal examination. No abnormal
neurology noted fundoscopic examination normal.
Investigations show:
Serum urea
5.9 mmol/L
Serum creatinine
90 umol/L
Urine dipstick
Blood + Protein +
An ultrasound scan of abdomen shows a small right kidney.
What is the likely cause of this patients presentation?
1 ) Chronic Glomerulonephritis
2 ) Congenital renal atrophy
3 ) Fibromuscular dysplasia
4 ) IgA nephropathy
5 ) Reflux nephropathy
Comments:
This lady probably experiencing abdominal discomfort because of recurrent urinary tract
infections. The small kidney on right hand side a consequence of chronic pyelonephritis due
vesicoureteric reflux, earlier in life. Renal asymmetry unlikely be consistent chronic
glomerulonephritis. With fibromuscular dysplasia, there might be evidence of more significant
hypertension, possibly end organ disease. As chronic GN, one would expect renal asymmetry
IgA nephropathy. 5 )
96

35. A 58 year man attends well man clinic because his father died of a heart attack at
age of 61. There past history of Diabetes Mellitus but he a smoker of 10 15 cigarettes
per day. On examination, he a BMI of 28 kg/m2 Blood Pressure of 162/88 mmHg.
Cardiovascular, chest abdominal examination are normal. Fundoscopy merely shows AV
nipping. He advised re attend his general practitioner who confirms sustained elevation
in pressure recordings averaging 170/94 mmHg. Dipstick urinalysis reveals urine protein
(+) Other investigations show:
Sodium
144 mmol/l
Potassium
4.2 mmol/l
Urea
12.1 mmol/l
Creatinine
186 micromol/l
Random plasma glucose concentration
10mmol/l
HbA1c
7%
Total cholesterol concentration
6.8 mmol/l
Triglycerides
2.6 mmol/l
Ultrasound abdomen normal size kidneys What is the likely cause of his renal
impairment?
1 ) Analgesic nephropathy
2 ) Chronic Glomerulonephritis
3 ) Diabetic nephropathy
4 ) Hypertensive nephropathy
5 ) Renovascular disease
Comments:
The absence of any changes in fundi consistent diabetic retinopathy makes diabetic
nephropathy unlikely. The presence of renal impairment hypertension would imply quite
advanced diabetic nephropathy, if this were diagnosis. He may well have diabetes mellitus
absence of family history does mean his father did have diabetes merely, it was diagnosed.
The raised HbA1c would suggest diagnosis of diabetes mellitus but this confirmed by random
glucose of 10. The diagnosis confirmed by a random glucose greater than 11.1 mmol/l a
fasting concentration above 7 mmol/l. The kidneys would be smaller a chronic GN analgesic
nephropathy. Hypertensive nephropathy can present a raised creatinine proteinuria. If kidneys
displayed an asymmetry in size on USS if there were a reduction in renal size bilaterally, one
would consider renovascular disease4 )
36. A 22 year old male referred by his General practitioner due problems related his
sex life. He recently entered into his first sexual relationship but problems achieving an
erection perturbed by poor sexual development.
He his pubertal development as poor: being aware of a paucity of pubic hair he
embarrassed about his gonadal development. He started shave at age of 18 but shaves
only twice weekly. Otherwise he quite fit active works as a labourer on a building site. He
takes medication drinks 20 units of alcohol weekly but mostly on weekends. He one
younger brother.
Examination reveals a phenotypically male, who tall but lean a BMI of 21.2 kg/m2. He
little beard growth, fine skin, a paucity of body hair scanty pubic hair. His penile length
97

approximately 6 cm testicular volumes of approximately 6 7 mls bilaterally (Normal 10 15


mls). Cardiovascular, respiratory abdominal examination are all normal. Fundal
examination he visual fields.
Investigations show:
Plasma testosterone concentration 6.2 nmol/l (10 30)
LH
20.2 mU/l (2 10)
FSH
22.2 mU/l (2 10)
Prolactin
433 mU/l (50 500)
Free T4
12.6 nmol/l (9 23)
TSH
2.3 mU/l (0.5 4.5)
What are chances of his brother developing this disorder?
1
2
3
4
5

)
)
)
)
)

<1%
25%
33%
50%
100%

Comments:
This patient Klinefelters syndrome as suggested by hypergonadotrophic hypogonadism, poor
secondary sexual characteristics plus tall stature suggested poor academic record. This due
47XXY specific genetic pattern of inheritance.1)
37. A 24 year old woman presented horizontal diplopia over last 6 weeks. 3 years ago
she had a painful visual loss of left eye, which recovered completely over next 3 months.
On examination, on looking right, there loss of adduction of left eye nystagmus of
abducting eye. What is the diagnosis?
1
2
3
4
5

)
)
)
)
)

Brain stem glioma


Brain stem stroke
Friedreichs ataxia
Multiple sclerosis
Wernickes encephalopathy

Comments:
The clinical signs are compatible a left intranuclear ophthalmoplegia. The commonest cause in a
young patient multiple sclerosis. The previous history of painful visual loss likely was due optic
neuritis. 4 )
38. A 40 year old man seen in Accident & Emergency department complaining of
wheeze dyspnoea. He complains of a persistent progressively worsening wheeze. His GP
initially prescribed a salbutamol inhaler, which had little effect, patient eventually had
take time off work on sick leave. During this time his GP intensified therapy, adding oral
prednisolone, good initial symptomatic improvement, though his symptoms returned
shortly after returning work. He reports little effect from therapy thereafter stopped
using inhaler as he did feel any benefit. The steroids were tapered stopped.
98

There past history of note. A full systems review unremarkable. He married three
children works at a local industrial plant as a solderer. He a lifelong non smoker drinks
about 8 units of alcohol weekly.
The patient takes his family on holiday two weeks in summer. During vacation he finds
his wheeze improves considerably. Two days after returning work his colleagues call an
ambulance because he an acute asthma attack in work he brought A&E unit.
On examination he dyspnoeic at rest but able speak in short sentences. Respiratory rate
22/minute. There are widespread inspiratory expiratory polyphonic wheezes throughout
both lungs. BP 130/76 detectable paradox. He afebrile. The rest of examination
unremarkable.He treated nebulized salbutamol makes a significant improvement within
48 hours.Investigations:
Hb
15 g/dl
WCC
9 x 109/l
Platelets
395 x 109/l
Plasma sodium
143 mmol/l
Plasma potassium
4.2 mmol/l
Plasma urea
2.8 mmol/l
Plasma creatinine
70 mmol/l
PEFR #1
250 l/min (on admission)
PEFR #2
650 l/min (48h after admission)
Which of the following important investigation be performed?
1 ) Bronchoscopy bronchoalveolar lavage
2 ) Full pulmonary function tests
3 ) Lung biopsy
4 ) CT scan of thorax
5 ) Peak flow rates measured at home in work
Comments:
The history highly suggestive of occupational exposure an allergen, significant symptomatic
improvement when away from his work environment. 5 )
39. A 17 year old girl presented a 2 day history of severe back pain. A plain x ray film of
her spine normal. Two days later, she complained of tingling in her fingers toes. The next
day she became generally weak. She presented in A/E.
On examination, she bilateral lower motor neurone facial weakness, tetraparesis
weakness in all limbs (4/5), areflexia, flexor plantars sensation.
A lumbar puncture was performed CSF analysis showed:
tein
1.4 g/l (<0.4)
Glucose
Normal
Cells
0
What is the likely diagnosis?
1 ) Botulism
2 ) Guillain Barre syndrome
3 ) Acute poliomyelitis
99

4 ) Myasthenia gravis
5 ) Hysteria
Comments:
The clinical history very typical of Guillain Barre syndrome. Back pain very common often
severe enough require opiate treatment. Acute onset of weakness areflexia high CSF protein
cell count are typical features of condition. 2 )
40. A 28 year old man was admitted casualty after attending his weekly Judo club
meeting. He had felt well had participating actively. Within a few minutes of his final fight,
he developed pain in his neck, vertigo, blurred vision diplopia. He had difficulty in
balancing himself a tendency fall right incoordination of right upper lower limbs. There
was accompanying facial asymmetry, decreased hearing from right side, difficulty in
swallowing, associated weakness of right side of body.
There was past history of note he was a non smoker. On examination, a right sided
Horner's syndrome was noted. There was nystagmus (fast phase right), sensorineural
hearing loss in right ear paresis of soft palate on right side. There was reduced tone
power in upper lower limbs on right. Deep tendon reflexes were brisk on right side
plantar response was extensor. Sensory examination a crossed hemianaesthesia
involvement of face on right side.
What is the likely diagnosis?
1 ) Atlanto axial dislocation
2 ) Cavernous sinus thrombosis
3 ) Posterior communicating artery aneurysm
4 ) Posterior inferior cerebellar artery infarction
5 ) Vertebral artery dissection
Comments:
Vertebral artery dissection well recognised cause of stroke in patients < 45 years associated a
10% mortality rate in acute phase. Death may occur due intracranial dissection, brainstem
infarction subarachnoid haemorrhage. Common causes include structural defects of arterial
wall, connective tissue disease, trauma (e.g. road traffic accident, sporting injury), chiropractic
manipulation of neck. The typical clinical presentation severe occipital headache followed by
focal neurological signs attributable ischaemia of brainstem cerebellum. 5 )
41. A 60 year old man presented acute weakness pain of his right lower limb. He had
awoken one morning after playing football his grandson had aware of marked pain
weakness of his right leg.
On examination there was weakness of ankle dorsiflexion, plantar flexion, eversion
inversion, thigh abduction extension.What is the likely diagnosis?
1 ) Common peroneal neuropathy
2 ) Femoral neuropathy
3 ) Sacral plexopathy
4 ) Sciatic nerve palsy
5 ) S1 radiculopathy
Comments:
The likely diagnosis sacral plexopathy. Thigh abduction extension weakness indicate gluteal
medius maximus involvement (L5,S1). This finding distinguishes it from sciatic nerve palsy. 3 )
42. A 45 year old man was referred hospital by his General Practitioner. Over a period of
three months he had developed a persistent progressively worsening wheeze. He brought
hospital by ambulance from his workplace where he became acutely breathless.
100

Two months previously his GP had tried treatment inhaled salbutamol, but little effect.
The patient was eventually forced take sick leave from his work as a solderer in a local
factory. While patients was off work, GP took opportunity intensifiy therapy, adding oral
prednisolone, good initial symptomatic improvement patient felt well enough return
work within five days. Unfortunately, his symptoms returned shortly after returning work.
The patient requested steroids be withdrawn as he did consider they had improved his
symptoms: they were tapered stopped.The patient then took two weeks annual leave
travelled his family on holiday Spain two weeks. During vacation he found his wheeze
improved significantly. Two days after returning work his colleagues call an ambulance
because he an acute asthma attack in work brought A&E unit.He a non smoker does
take any regular medication.
On examination he dyspnoeic at rest but able speak in short sentences. Respiratory rate
22/minute. There are widespread inspiratory expiratory polyphonic wheezes throughout
both lungs. BP 130/76 detectable paradox. He afebrile. The rest of examination
unremarkable.
He treated nebulized salbutamol makes a significant improvement within 48 hours.
Investigations reveal:On admission:
Haemoglobin
15.0 g/dL
White cells
9 x 109/L
Platelets
395 x 109/L
Serum sodium
143mmol/L
Serum potassium
4.2 mmol/L
Serum urea
2.8 mmol/L
Serum creatinine
70 mmol/L
PEFR
50l Lmin
48h after admission
PEFR
650 L/min
Which of following investigations should be performed?
1 ) Bronchoscopy bronchoalveolar lavage
2 ) CT scan of thorax
3 ) Full pulmonary function tests
4 ) Lung biopsy
5 ) Peak flow rates measured at home in work
Comments:
The history highly suggestive of occupational exposure an allergen, significant symptomatic
improvement when away from his work environment. 5 )
43. A 24 year male who works in a plastic factory presented with shortness of breath.
He went 2 weeks Spain was completely well. He presented at 0200 hours Accident &
Emergency Unit after having used his Salbutamol inhaler 8 times. He was treated a
Salbutamol nebuliser steroids completely recovered.The likely diagnosis is?
1 ) Broncho pulmonary aspergillosis
2 ) Chemical pneumonitis
3 ) Extrinsic allergic alveolitis
4 ) Late onset asthma
5 ) Occupational asthma
101

Comments:
Occupational asthma commonest industrial lung disease in western world more than 500
causes. It accounts up 10% of adult onset asthma. It caused by agents are encountered at
work.
The commonest occupations affected (with causes) in UK are Paint sprayers (isocyanates),
Bakers (Flour mainly but also enzymes such as amylase used in baking process), Chemical
processors (acids, detergents, bleaches), Plastics workers (polyethylene, polyvinyl chloride),
Solderers (Colophony), Laboratory technicians (rats, mice, rabbits, locusts).
The typical history usually one of breathlessness, wheeze cough which occur during working
week remit during absences from work such as holidays.
The symptoms do usually develop immediately on first exposure but begin days, months even
years later. Removal from exposure sensitising agent at an early stage can lead remission of
asthma although sensitisation agent usually permananent. 5 )
44. A 55 year old man presented 6 month history of proximal weakness of both upper
lower limbs, dry mouth impotence. He was a heavy smoker of 30 cigarettes per day drank
12 units of alcohol per week. Otherwise he had well.
On examination, he was an obese individual a pressure of 155/90 mmHg was apyrexial.
Chest heart abdominal examination were normal. There was proximal weakness grade 4/5
in both upper lower limbs. The reflexes were generally depressed. Plantars were flexor
sensation was normal.What is the likely diagnosis?
1 ) Lambert Eaton Myasthenic Syndrome
2 ) Motor neuron disease
3 ) Motor peripheral neuropathy
4 ) Myasthenia gravis
5 ) Polymyositis
Comments:
The combination of proximal weakness, depressed reflexes, autonomic dysfunction in a smoker
patient suggest diagnosis of Lambert Eaton Myasthenic syndrome (LEMS). Positive voltage gated
calcium antibodies EMG confirm diagnosis. Myasthenia gravis, motor neuron disease
polymyositis do cause autonomic impairment. 1 )
45. A 58 year male a 3 year history of type 2 diabetes mellitus which managed by his
general practitioner referred diabetic clinic. He generally well but aware of a 6kg
weight gain over last one year together 2 3 episodes of nocturia nights. He currently on
diet control alone his diabetes. He an ex smoker having stopped 10 years ago. He drinks
approximately 8 units of alcohol weekly.
On examination he a body mass index of 33.5 kg/m2, a pressure of 162/98 mmHg a
pulse of 78 beats per minute. Auscultation of his heart, chest are abnormalities are
detected on abdominal examination. Fundoscopic examination reveals scattered
microaneurysms in both eyes a crescent of hard exudates encroaching upon macular in
right eye. Neurological examination reveals reduced light touch sensation in both feet
ankles. Dipstick of his urine reveal glucose +.
Investigations show:
Fasting plasma glucose
7.8 mmol/l
Sodium
138 mmol/l
Potassium
4.2 mmol/l
Urea
7.8 mmol/l
102

Creatinine
HbA1c
Cholesterol
Triglycerides
Which of the following is the
1 ) Anti hypertensive therapy
2 ) Insulin
3 ) Metformin
4 ) Statin therapy
5 ) Weight reduction

90 micromol/l
7.8% (<6.2%)
4.0 mmol/l (<5.2)
2.5 mmol/l (1 2)
appropriate treatment to reduce his cardiovascular risk?

Comments:
A tough choice between a statin antihypertensive therapy but in this scenario antihypertensive
therapy probably wins, in view of normal cholesterol concentration. Adequate control of his
hypertension (<140/80) would be expected reduce cardiovascular risk both stroke MI reduce
microvascular risk far more than improved glycaemic control even Metformin. Weight reduction
per se demonstrated reduce CV risk. 1 )
46. A 16 year male presented hospital a 24 hour history of fever confusion. He was
known have epilepsy, which was well controlled on drugs. He also had a previous history
of drug overdose. He consumed 50 units of alcohol per week admitted using recreational
drugs. On examination his temperature was 39oC.Investigations:
Haemoglobin
White cell count
Platelets
Serum sodium
Serum potassium
Serum urea
Serum creatinine
Serum ALT
Serum AST
Serum ALP
Serum bilirubin
Serum GGT
Urine Protein +

11g/dL
11 x 109
156 x 109
127 mmol/L
4.1 mmol/L
12 mmol/L
160 umol/L
300 IU/L
250 IU/L
120 IU/L
20 IU/L
400 IU/L

His chest x ray on admission shown.

103

What best combination of antibiotics in first instance?


1)
Amoxicillin Metronidazole
2)
Ampicillin Gentamicin
3)
Cefotaxime Erythromycin
4)
Doxycycline
5)
Erythromycin Rifampicin
Comments:
The diagnosis aspiration pneumonia. The frequent conditions associated aspiration
pneumonia are decreased consciousness dysphagia. He several risk factors impaired
consciousness epilepsy, high alcohol intake use of recreational drugs a history of drug
overdose. Dental sepsis also increases risk by increasing anaerobic flora in mouth pharynx.
Infection, particularly in community, usually resultant from anaerobic organisms such as
Peptostreptococcus Bacteroides. Anaerobic infection can result in pneumonia, lung abscesses
empyema.Aspiration pneumonia can be indistiguishable from bacterial community acquired
pneumonia in early stages as foul smelling sputum does usually present until necrosis
occurred.The dependent segments of lung are usually involved posterior segments of upper
lobes apical segments of lower lobes when patient supine lower lobes when patient
upright.Amoxicillin appropriate but does cover all bacteriodes. Metronidazole good cover
against anaerobes but against aerobes therefore should be used alone.1 )
47.
A 19 year old woman presented her General Practitioner a history of general malaise,
lethargy fatigue. She was unsure when symptoms first started, but felt they had developing
slowly over a period of several months. She was referred a Cardiologist on basis of physical
findings.The data obtained from cardiac catheterisation are shown below:
Anatomical site
Oxygen
Pressure (mmHg) End
saturation (%)
systolic/End diastolic
Superior vena cava
77
Right atrium (mean)
79
7
Right ventricle
78
Pulmonary artery
87
52/17
Pulmonary capillary wedge pressure
16

104

Left ventricle
Aorta

96
97

120/11
130/60

What is the diagnosis?


1)
Patent ductus arteriosus
2)
Primary pulmonary hypertension
3)
Pulmonary stenosis
4)
Septum primum atrial septal defect
5)
Tetralogy of Fallot
Comments:
The data show an unexpected step up in oxygen saturation between RV PA. This associated
high pulmonary artery pressures a high wedge pressure.

1)

48.

A 44 year old HIV seropositive man presented clinic this rash.

105

What agent should be used to treat this condition?


1)
Aciclovir
2)
Flucloxacillin
3)
Ganciclovir
4)
Malathion
5)
Topical corticosteroids
Comments:
An easy question. The rash shown typical of herpes zoster. 1 )
49. This 40 year admitted a 2 month history of abdominal pains, watery diarrhoea
noted this extensive rash.

What is this rash?


1)
Acquired ichthyosis
2)
Erythema ab igne
3)
Erythema repens gyratum
4)
Erythema migrans
5)
Necrolytic migratory erythema
Comments:
106

The symptoms this rash which typical of Necrolytic Migratory Erythema suggests a
glucagonoma. Other features of glucagonoma include diabetes mellitus, hypoaminoacidemia,
cheilosis, normochromic normocytic anemia, venous thrombosis neuropsychiatric features. At
least 50% are metastatic at presentation so prognosis poor. Acquired ichthyosis appears as
symmetrical scaling of skin a cutaneous manifestation of underlying malignancy. Erythema
migrans associated Lyme disease. Erythema ab igne/livedo reticularis appearance of chain
mail. Erythema repens gyratum another paraneoplastic phenomenon commonly lung, breast,
stomach etc. 5 )
50. A 38 year old woman presents a four week history of painless jaundice pruritis. Her
only past history of note a pan colectomy life threatening colitis as a teenager. She
consumes 20 units (200g) of alcohol per week. Her LFTs cholangiogram are shown.

Bilirubin
85 (1 22 umol/l)
ALP
450 (5 35 U/l)
GGT
325 (4 35 U/l)
AST
78 (1 31 U/l)
ALT
56 (5 35 U/l)
What is the likely diagnosis?
1)
Alcoholic cirrhosis
2)
Caroli's disease
3)
Choledocholithiasis
4)
Primary biliary cirrhosis (PBC)
5)
Primary sclerosing cholangitis (PSC)
107

Comments:
The ERCP shows stricturing beading of bile ducts, features clasically seen in PSC. 80% of those
PSC have a history of colitis. PSC occurs in 4% of those a history of ulcerative colitis. Caroli's
disease (a rare congenital disorder) also causes a beaded appearance of intrahepatic bile ducts
can cause recurrent cholangitis, less likely diagnosis in someone a history of UC, doesnt involve
CBD. The bile ducts appear in alcoholic cirrhosis PBC. 5 )
51. A 56 year old presented a 4 day history of breathlessness, cough a high fever. She
gave a long history of indigestion intermittent dysphagia solids liquids. She would often
wake at night episodes of coughing spluttering. There was history of weight loss. On
examination she had a temperature of 39oC, resp rate of 30/min a HR of 102. On
auscultation of lung fields there were coarse basal cracklesHer chest x ray on admission
to hospital is shown.

What is the cause of her illness?


1)
Achalasia
2)
Benign oesophageal stricture
3)
Bronchiectasis
4)
Hiatus hernia
5)
Pharyngeal pouch
Comments:
Achalasia results in megaoesophagus which seen on plain CXR as smooth convex opacity
widening mediastinum right. A fluid level often present. If oesophagus full of air (eg after
108

oesophagoscopy) thickened wall of oesophagus can be clearly deliniated running parallel right
heart border.
Achalasia results in dysphagia both solids liquids retention of contents in body of
oesophagus regurgitation resultant aspiration.
Aspiration can result in (1) a chemical pneumonitis from gastric acid aspiration, (2) mechanical
obstruction from aspiration of particulate matter (3) Bacterial infection from aspiration of
oropharyngeal bacteria. 1 )

1.
A 68 year old man type II diabetes mellitus (insulin controlled) end stage renal
failure (haemodialysis dependent 4 years) was admitted coronary care unit 72 hours ago,
an acute inferior myocardial infarction. Despite appropriate therapy, including
thrombolysis, he continues have ischaemic symptoms, in pulmonary oedema. His last
haemodialysis session was three hours prior admission. His pressure 86/52 mmHg.
Investigations show:
Serum sodium
139 mmol/L
Serum potassium
6.7 mmol/L
Serum urea
49 mmol/L
Serum creatinine
950 umol/L
Haemoglobin
10.8g/dL
Troponin T
>25 ug/L
(NR <0.04)
A transthoracic echocardiogram shows a left ventricular ejection fraction of 20%.
Which of the following is appropriate management strategy?
1
2
3
4
5

)
)
)
)
)

Blood transfusion
Conservative management
Coronary angiography +/ angioplasty
Haemodialysis
IV furosemide

Comments:
This patient, multiple cardiovascular risk factors, sustained a large inferior myocardial
infarction. Despite appropriate treatment, he continues exhibit symptoms of ischaemia, he
cardiogenic shock, pulmonary oedema. Due his chronic renal failure, he probably does
produce any urine, therefore IV furosemide would work. A transfusion, although it may improve
his ischaemic symptoms, may worsen his pulmonary oedema. Due cardiogenic shock, he would
be unlikely tolerate haemodialysis, he too unstable any invasive cardiological procedures. He
could be admitted intensive care unit, commence haemofiltration inotropic support, although
this an option given in stem. The appropriate answer conservative management. He a very
poor prognosis, cardiogenic shock following a large acute inferior myocardial infarct,
complicated by haemodialysis dependent chronic renal failure. 2 )

109

2.
A 60 year old man presented 12 months history of forgetfulness, paroxysmal
drowsiness. He was examined by GP who found bilateral bradykinesia rigidity in upper
limbs. He was started on L dopa. Six weeks later he developed formed visual hallucinations.
The visual hallucinations persisted despite stopping L dopa. The GP started him on a small
dose of haloperidol (5mg). However, patient developed severe vomiting drowsiness; he
recovered when haloperidol was discontinued.
He had mild bradykinesias rigidity in upper limbs rest tremor of left arm. Gait was
normal. His mini mental examination was 20/30. The remainder of examination was
normal.
Investigations showed:
Haemoglobin
16.5 g/dL
White cell count
8.9 x 109/L
Platelets
290 x 109/L
Serum sodium
139 mmol/L
Serum potassium
3.8 mmol/L
Serum urea
6.7 mmol/L
Serum creatinine
91 umol/L
Serum albumin
39 g/L
Serum total bilirubin
19 umol/L
Serum aspartate aminotransferase
28 IU/L
Serum alkaline phosphatase
99 IU/L
Serum calcium (corrected)
2.4 mmol/L
Plasma glucose
4.2 mmol/L
Free T4
11 pmol/L
(NR 10 22)
Free T3
8 pmol/L
(NR 5 10)
Plasma TSH
3 mU/L
(NR 0.4 5.0)
Serum vitamin B12
228 ng/L
(NR 160 760 ng/L)
VDRL
Negative
CT brain EEG were reported as being normal.What is the likely diagnosis?
1 ) Idiopathic Parkinsons disease
2 ) Diffuse Lewy body disease
3 ) Alzheimers disease
4 ) Schizophrenia
5 ) Vascular dementia
Comments:
Diffuse Lewy body disease typically presents dementia, parkinsonism, visual hallucinations,
intermittent alteration of attention, sensitivity neuroleptics. Patients idiopathic Parkinsons
disease do usually manifest visual hallucination until many years after introduction of L dopa
treatment (> 5years). Dementia usually appears 5 10 years after onset of parkinsonism. 2 )
3.
A 42 year old a recent diagnosis of systemic sclerosis, referred hospital a
complaint of headaches blurred vision. She a past history of asthma. On examination, her
pressure 230/120, there bilateral papilloedema.Which of the following medications
should be prescribed immediately?
1 ) IV Furosemide
2 ) IV Labetolol
3 ) IV Sodium Nitroprusside
4 ) Oral Enalapril
110

5 ) Sublingual Nimodipine
Comments:
Systemic sclerosis a systemic disorder characterised by skin thickening due deposition of
collagen in dermis. Adverse prognostic features are renal, cardiac pulmonary involvement. A
major complication development of scleroderma renal crisis. This characterised by abrupt
onset of severe hypertension, usually retinopathy, together rapid deterioration of renal function
heart failure. It develops in 8 15% of patients diffuse systemic sclerosis, especially associated
rapid progression of diffuse skin disease. It usually presents early, within three years of
diagnosis. The pathogenic mechanisms leading renal damage are known. The clinical
presentation typically symptoms of malignant hypertension, headaches, blurred vision, fits
heart failure. Renal function impaired usually rapidly deteriorates. The hypertension almost
always severe a diastolic BP over 100 mmHg in 90% of patients. There hypertensive retinopathy
in about 85% of patients, exudates haemorrhages if severe, papilloedema. Scleroderma renal
crisis a emergency. The hypertension should be treated an ACE inhibitor. The aim reduce
pressure gradually, as an abrupt fall can lead cerebral ischaemia infarctions (as in any
accelerated hypertension). Calcium channel blockers may be added ACE inhibitors. Deterioration
in renal function can be rapid, gross pulmonary oedema; therefore patients scleroderma renal
crisis should be managed in hospitals facilities dialysis. 4 )
4.
A 40 year old male followed up in renal clinic. His creatinine clearance (measured)
76ml/min. A recent USS of his renal tract shows enlarged kidneys bilaterally, multiple
renal cysts. His father required dialysis at age of 45 years. On examination, his pressure
149/88 mmHg.Which of following statements incorrect?
1 ) An ACE inhibitor would be appropriate control his hypertension
2 ) He should maintain a high fluid intake
3 ) Ciprofloxacin should be prescribed upper urinary tract infections
4 ) Genetic counselling should be offered
5 ) He should be informed of probable need dialysis in 5 7 years
Comments:
This patient Autosomal Dominant Polycystic Kidney Disease (ADPKD). On average, patients
progress end stage renal failure between ages of 40 60 years. In these patients renal function
usually deteriorates in a gradual fashion, usually a drop in creatinine clearance of
5/6ml/min/year (at least 10 years this patient). Treatment should include a high fluid intake (to
prevent formation of renal stones clots) regular follow up of pressure renal function. Loin
pain should be treated symptomatically, hypertension should be managed standard anti
hypertensive medications. Haematuria should be treated conservatively. Urinary tract infections
should be treated lipophillic drugs (e.g. Ciprofloxacin, Trimethoprim sulphamethoxazole) as
they have best penetration into cyst fluid. It an autosomal dominant disease, therefore
offspring of an affected patient a 50% chance of inheriting disease. The patient should be
offered genetic counselling, despite fact disease a variable clinical course even between
affected family members. 5 )
5.
57 year old man presented deteriorating breathlessness over last one year. He had
received inhalers last 2 years prescribed by his GP which he has, up until last 3 months
used intermittently. He was also taking Ramipril 10mg od Bendrofluazide 2.5 mg daily a
six year history of hypertension. He stopped smoking 2 years previously consumed
approximately 14 units of alcohol weekly.
111

On examination he was cyanosed had a swollen face dilated superficial veins over
anterior chest wall fixed dilated neck veins. His pressure was 154/88 mmHg, pulse 88
beats per minut. Heart sounds were normal. There was pitting oedema of ankles.
Respiratory examination a hyperexpanded chest scattered expiratory wheeze. Abdominal
examination was normal. Investigations revealed:
Haemoglobin
14.8 g/dL
White cell count
12.91 x 109/L
Platelets
488 x 109/L
Serum sodium
130 mmol/L
Serum urea
10.8 mmol/L
Serum corrected calcium
2.81 mmol/L
The ECG was normal. Chest x ray showed hyperexpanded lung fields left paratracheal
shadowing. A CT scan of thorax showed an anterior mediastinal mass
Which single investigation would be helpful in making a diagnosis?
1 ) biopsy of mediastinal mass
2 ) bone marrow trephine biopsy
3 ) bronchoscopy
4 ) CT scan of abdomen
5 ) tuberculin skin test
Comments:
The patient presents symptoms signs of superior vena caval obstruction (SVCO). This due
anterior mediastinal mass single useful diagnostic investigation histological confirmation of
lesion from a biopsy. This can usually be done by a percutaneous CT guided biopsy. The
common cause of SVCO primary lung cancer. Other causes include lymphoma. Treatment of
underlying condition although in some cases of non small cell lung cancer stenting of SVCO
may be required relief of symptoms prior chemotherapy/palliative radiotherapy. 1 )
6.
A 78 year lady admitted increasing breathlessness a cough productive of mucoid
sputum on days. She had exertional dyspnoea two years but she become more
breathless over last five days. She was a smoker of 40 cigarettes per day until three
months ago. She takes Becotide 200mg bd, Salmeterol 50mg bd Ventolin PRN.
On examination she was obese a BMI of 32 kg/m2. She was cyanosed pale but there was
clubbing nor lymphadenopathy. She was breathless at rest a respiratory rate of 24/min.
Pulse was 110 beats per minute pressure 140/80 mmHg. Her chest was hyperinflated
expiratory wheezes she had bilateral swollen ankles.
She was treated nebulised bronchodilators, controlled oxygen therapy, oral prednisolone,
antibiotics commenced on diuretic therapy. She improved was discharged home five days
later.
On review six weeks later investigations revealed:
Arterial gas analysis (on air):
PaO2
6.9 kPa
PaCO2
6.8 kPa
pH
7.4
Pulmonary Function testing:
FEV1
0.9 L (3.2 predicted)
112

FVC
4.2 L (4.5 predicted)
Which one of following primary indication long term domiciliary oxygen therapy in this
patient?
1 ) cor pulmonale
2 ) inability get out of house
3 ) low FEV1
4 ) low paO2
5 ) raised paCO2
Comments:
This patient chronic obstructive pulmonary disease (COPD) as a result of her smoking. Her
investigations demonstrate it severe (FEV1 < 40% predicted). The reason why she a candidate
long term oxygen therapy (LTOT) she hypoxic. The criteria LTOT are paO2 < 7.3 kPa (55
mmHg) without hypercapnia paO2< 8.0kPa (60 mmHg) if there evidence of pulmonary
hypertension/cor pulmonale/polycythemia (Nocturnal Oxygen Therapy Trial Group Ann Int Med
1980;93: 391 398. Medical Research Council Working Party. Lancet 1981;1:681 686.) LTOT
smoking cessation are currently only interventions in COPD have shown prolong life. COPD
guidelines can be found at www.goldcopd.com4 )
7.
A 72 year old male presents a two month history of agitation weight loss. Four
months previously he was admitted paroxysmal atrial fibrillation was commenced on
amiodarone 200mg daily. Since then he had further palpitations. He otherwise well a
past history of hypertension which he takes Amlodipine.
On examination he look well had a slight tremor of outstretched hands, a pulse of 88
beats per minute regular a pressure of 164/88. No goitre was palpable there were eye
signs. Investigations reveal:
Haemoglobin
16.9 g/dL
White cell count
6.9 x 109/L
Platelets
310 x 109/L
ESR
18 mm/1st hour
Serum sodium
139 mmol/L
Serum potassium
3.7 mmol/L
Serum urea
6.5 mmol/L
Serum creatinine
95 umol/L
Free T4
32.1 pmol/L
(NR 10 22)
Free T3
6.8 pmol/L
(NR 5 10)
TSH
0.07 mU/L
(NR 0.4 5.0)
Thyroid autoantibodies
negative
What is the appropriate treatment this patient?
1 ) Start Atenolol
2 ) Start Carbimazole
3 ) Start Prednisolone
4 ) Stop Amiodarone
5 ) Treat Radioactive Iodine
Comments:
113

This patient amiodarone induced hyperthyroidism, a relatively common problem. The patient
commenced amiodarone due paroxysmal AF but other agents such as Sotalol may be more
appropriate. Therefore appropriate treatment would be withdrawal of amiodarone.
There evidence support a destructive thyroiditis (relatively ESR) slightly elevated TFTs so
steroids are necessary. The radio iodine uptake scan would probably any uptake as amiodarone
stuffed Iodine so radioactive iodine treatment would be ineffective as well as unnecessary. 4 )
8.
A 39 year old male who a general practitioner presents neurology clinic twitching
muscles. He particularly concerned about involuntary twitches he noticed in his muscles
over last six months. He aware of these twitches particularly at night when at rest. He
plays tennis three times per week notices twitches are worse after exercise. He noticed
deterioration in his ability exercise his fitness improved over last three months. He
otherwise fit well takes medication. His weight stable. He drinks approximately two
bottles of wine per week but does smoke.
On examination, he appears fit well a pressure of 122/78 mmHg. Cardiovascular,
respiratory abdominal examination are all normal. Neurological examination reveals tone
bulk but sporadic fasciculations are seen in quadriceps, brachioradialis, biceps, triceps,
calves pectorals muscles. No fasciculations are seen in tongue. He generally strong in all
muscle groups tested 5/5 strength. Reflexes are all preserved flexor plantar responses.
Sensation intact.
Investigations reveal:
Haemoglobin
17.5 g/dL
White cell count
8 x 109/L
Platelets
244 x 109/L
Serum sodium
136 mmol/L
Serum potassium
4.2 mmol/L
Serum urea
4.5 mmol/L
Serum calcium
2.3 mmol/L
Serum alkaline phosphatase
100 IU/L
(NR 45 - 105)
Serum aspartate aminotransferase
30 IU/L
(NR 1 - 31)
Serum gamma glutamyl transferase
80 IU/L
(NR 10 50)
What is the likely diagnosis
1 ) Alcohol related Myopathy
2 ) Amyotrophic Lateral Sclerosis
3 ) Benign Fasciculation syndrome
4 ) Chronic poliomyelitis
5 ) Hypokalaemic periodic paralysis
Comments:
This GP strength notices 'twitching' fasciculations at rest. The likely diagnosis benign
fasciculation syndrome a condition associated a reduced threshold action potentials at
Neuromuscular junction entirely benign. Typically patients are health professionals as they
translate fasciculations mean Motor Neurone Disease. The general public often do pay any
attention muscle twitches particularly when they feel well. 3 )

114

9.
A 22 year lady was planning emigrate Australia had a chest X ray undertaken as
part of her Visa requirements. This demonstrated bilateral hilar lymphadenopathy but clear
lung fields . It was reported as being suspicious of Sarcoidosis. She was referred
outpatients clinic. On systems review in outpatients a two month history of arthralgia a
dry cough was elicited. She had previous history of note but three months ago she had
had unprotected intercourse took an HIV test which was negative. She had other
symptoms. She lived in Zimbabwe until she was 16. She worked as a waitress.Which one
investigation likely confirm diagnosis?
1
2
3
4
5

)
)
)
)
)

Bronchoalveolar lavage
High resolution CT scan of chest
Serum angiotensin converting enzyme activity
Transbronchial lung biopsy
Tuberculin test

Comments:
A HRCT scan of chest may reveal pulmonary abnormalities in Sarcoidosis, such as multiple ill
defined opacities running along bronchovascular bundles, lymphatics interlobar septa even in
absence of plain CXR abnormalities. Serum ACE elevated in about 70% of patients active
Sarcoidosis but sensitive specific therefore helpful in diagnosis. Histological confirmation
required make diagnosis confidence. Transbronchial lung biopsy will provide positive histology
in about 80% of patients, safe can be done under sedation local anaesthesia therefore
diagnostic investigation of choice. 4 )
10. A 70 year male presents irritation of legs particularly at night of 6 months duration.
He aware of a vague irritation in both legs keeps him awake. He finds rubbing his legs
walking around gives him some relief but problem deteriorated over last two months,
getting little sleep due this problem. He a past history of hypertension which he takes
atenolol Ramipril. He stopped smoking 10 years ago drinks little alcohol.
On examination, he a pressure of 148/88 mmHg. No abnormalities are found on
neurological examination of legs. Both plantars are flexor, muscle power sensation
intact.Which of the following is appropriate management this patient?
1 ) Start amitriptyline
2 ) Start bromocriptine
3 ) Start phenytoin
4 ) Stop atenolol
5 ) Stop ramipril
Comments:
This patient restless leg syndrome, a condition characterized by uncomfortable, irritating
sometimes painful sensations which usually occur in legs when at rest usually at night. The
condition benign clinical examination usually reveals abnormalities. It may be exacerbated by
diuretics, Tricyclics, phenytoin calcium antagonists. Studies suggest Dopamine agonists l Dopa
may be useful treatments. 2 )
11. A 28 year old man was referred clinic by his General Practitioner. Over past two
months he had noticed increasing exertional dyspnoea. Previosuly he had physically active
jogged regularly. At time of referral he was breathless climbing one flight of stairs. He a
past history of asthma underwent an orchidopexy as a child an undescended right testis.
115

On examination, gynaecomastia was noted. No wheeze was audible in his chest.


Examination of genitals a firm mass 1cm x 1cm in right testis.
Investigations showed:
Serum alpha fetoprotein
90 kU/L (<10 kU/L)
Serum beta human chorionic gonadotrophin
1250 U/L (<5 U/L)
What is the likely diagnosis?
1 ) Choriocarcinoma
2 ) Gonadoblastoma
3 ) Leydig cell tumour
4 ) Seminoma
5 ) Sertoli cell tumour
Comments:
The classical presentation testicular tumors of a healthy male in third fourth decade of life a
painless, swollen, hard testis. Testicular cancer can be divided into germ cell non germ cell
tumours. Germ cell tumors are classified as either pure seminomas mixed non seminomatous
germ cell tumours (NSGCTs): these two groups comprise more than 90% of all tumors. Non
germ cell malignancies (Leydig Sertoli cell tumors, gonadoblastomas) make up less than 10% of
all testicular tumors.
Patients history of cryptorchidism have a 10 40 times increased risk of testicular cancer this
risk greater abdominal versus inguinal location of undescended testis. Orchidopexy does
reduce risk of subsequently developing a malignancy. An abdominal testis more likely be
seminoma, while a testis surgically brought scrotum by orchiopexy more likely be NSGCT.
Choriocarcinoma aggressive of NSGCTs. It disseminates hematogenously lungs, liver, brain,
bone, other viscera very early in disease process. Unlike classic seminoma mixed GCTs, pure
choriocarcinoma more likely present symptoms from metastatic disease. Most testicular GCTs
cause scrotal swelling, a palpable mass, choriocarcinoma different in local tumor may be
small nonpalpable.
Pure seminomas do cause a rise in AFP level (AFP only produced by tumors containing
embryonal yolk sac elements). Elevated AFP levels are consistent NSGCT, though AFP often
within reference range in pure choriocarcinoma. Beta HCG usually markedly elevated in pure
choriocarcinoma, but only elevated in 10 15% of seminomas. Gynecomastia occurs due
elevation of beta hCG levels therefore common in choriocarcinoma, but only rarely seen in
patients a seminoma.
On ultrasound scanning, choriocarcinoma associated hemorrhage necrosis may appear more
cystic, inhomogeneous, calcified than a seminoma. Calcifications cystic areas are less common
in seminomas than in nonseminomatous tumors. 1 )
12. A 17 year presents a two day history of vomiting, general lethargy giddiness. Over
last six months she had lost 5kg in weight, had a reduced appetite had feeling
increasingly lethargic. She had past history of note, was a non smoker took combined
oral contraceptive pill contraception. Her eldest brother was well there was a family
history of thyroid disease both her mother maternal grandmother taking thyroxine.
On examination, she was comfortable at rest, appeared slightly dehydrated was apyrexial,
had a body mass index of 18.5 kg/m2 oxygen saturations on air of 99%. Her pressure was
102/64 mmHg fell 86/60 mmHg on standing. Her oulse was 90 beats per minute regular
auscultation of heart chest were normal. No abnormalities were detected on abdominal
CNS examination. Investigations revealed:
116

Haemoglobin
Mean cell volume
White cell count
Neutrophils
Lymphocytes
Eosinophils
Serum sodium
Serum potassium
Serum urea
Serum creatinine
Plasma glucose
Free T4
TSH
Urinalysis
Which of the following
1
2
3
4
5

)
)
)
)
)

10.5 g/dL
88 fL
8.8 x 109/L
4.4 x 109/L
2.8 x 109/L
0.8 x 109/L
130 mmol/L
5.8 mmol/L
12.8 mmol/L
135 umol/L
3.8 mmol/L
8.8 pmol/L
(NR 9 20)</TD
1.2 mu/L (NR 0.5 4.0)
Ketones +
appropriate investigation this patient?

Adrenal autoantibodies
CT adrenals
MRI pituitary
Short synacthen test
Thyroid autoantibodies

Comments:
The salient features in this young patient's case long standing asthenia weight loss sudden
episode of vomiting. She appears clinically dehydrated as demonstrated by postural
hypotension but her results reveal a hyponatraemia, hyperkalaemia hyperuricaemia. Her full
count shows an eosinophilia. The likely diagnosis acute hypoadrenalism due probably
Addison's disease in view of strong family history of autoimmune disease. The diagnosis should
be confirmed a short synacthen test a cotisol response less than 550 nmol/l confirmatory.
Abnormal thyroid function tests low T4 TSH are quite commonly associated Addison's do
reflect secondary hypothyroidism but sick euthyroidism. Thyroxine replacement must be given
these patients as it can exacerbate adrenal crisis. The TFTs will normalise hydrocortisone
therapy. Even if this were hypopituitarism an MRI of pituitary would diagnose hypoadrenalism
again this could be confirmed a short synacthen test. 4 )
13. A 42 year old homeless man brought into accident emergency department at
approximately 11:00pm by emergency ambulance crew. He known have a history of
alcohol abuse on this occasion, he was found by a group of passers by outside a nightclub
having sustained a laceration his forehead following a fall onto pavement.
On arrival accident emergency department, he was described as being unkempt a strong
odour of alcohol. He appeared be moderately intoxicated alcohol on examination he was
found have a superficial laceration over his left forehead, which required suturing. His
117

Glasgow Coma Scale was 14/15. No localising signs were found on examination of central
nervous system. He was admitted overnight observations.
He took his own discharge at approx 9:00am next day but returned ward later on same
day complaining shortness of breath blurred vision. In addition, he had developed
abdominal pain associated vomiting diarrhoea.
On examination, he had a respiratory rate of 30/min. Pulse rate of 100/min regular,
pressure of 110/60 mmHg heart sounds normal. There were occasional coarse crepitations
in both lung fields which cleared on coughing. Examination of abdomen generalised
tenderness masses hepato splenomegaly. Rectal examination was normal. The only other
positive findings were hyperaemia blurring of optic discs bilaterally.
Initial investigations showed:
pH
7.25
pO2
14.3 kPa
pCO2
3.7 kPa
Bicarbonate
12 mmol/L
Base excess
12
Serum sodium
132 mmol/L
Serum potassium
4.1 mmol/L
Serum urea
7.2 mmol/L
Serum creatinine
113 mol/L
Serum chloride
96 mmol/L
Serum glucose
11.4 mmol/L
What is the likely cause his abnormal investigations?
1 ) Diabetic ketoacidosis
2 ) Ecstasy ingestion
3 ) Ethylene glycol ingestion
4 ) Methanol ingestion
5 ) Pancreatitis
Comments:
The obvious abnormality a metabolic acidosis respiratory compensation. He also a large anion
gap of 29.1, {anion gap = (Na + K) (Chloride + HCO3), normally 122} which indicates
presence of a large concentrations of cations. Possible diagnosis includes ethylene glycol
methanol ingestion diabetic ketoacidosis. The absence of an elevated glucose makes diabetic
ketoacidosis unlikely. Visual impairment typically occurs methanol in severe cases result in
permanent blindness. Initial presentation of methanol ethylene glycol mimics those of ethanol
ingestion when co ingested, protects patient from toxic effects of methanol ethylene glycol
(possibly delaying diagnosis). This due alcohol dehydrogenase a higher affinity ethanol hence
methanol ethylene glycol are excreted unchanged in kidneys; preventing formation of toxic
metabolites formate (methanol) oxalic acid (ethylene glycol). Severe pancreatitis can give rise a
lactic acidosis but does give rise hyperaemia blurring of optic discs. 4 )
14. A 16 year mentally handicapped boy brought clinic by his parents who are
concerned he lost 3kg over last 6 weeks had some problems lower abdominal pain. He
attended day centres last 1 year a long history of severe epilepsy which he takes
lamotrigine carbamazepine. He had one fit two weeks ago.
118

On examination he appears well, apyrexial abnormalities are noted on abdominal


examination. Examination of his faeces reveals fine, thin white worms approximately 1 2cm
in length. What is the appropriate treatment this patient?
1 ) Levamisole
2 ) Mebendazole
3 ) Nicosamide
4 ) Piperazine
5 ) Praziquantel
Comments:
This patient threadworms, which uncommon amongst institutionalised individuals. The
appropriate treatment mebendazole. Other drugs are used include piperazine but this drug
recommended use in epilepsy. Therefore mebendazole appropriate. Praziquantel may be used
tapeworms. 2 )
15. A 53 year male presents general lethargy, weight gain abdominal swelling. His
symptoms have deteriorated over last three months he confesses long standing heavy
alcohol consumption. Currently he taking medication past history of note.
Examination reveals he well orientated, apyrexial a pressure of 132/88 mmHg. He
numerous spider naevi present on upper chest gynaecomastia. Abdominal examination
reveals moderate ascites he oedema of legs up mid thigh. No organomegaly noted on
abdominal examination.
Investigations reveal:
Serum sodium
140 mmol/L
Serum potassium
4.4 mmol/L
Serum urea
7.8 mmol/L
Serum creatinine
135 umol/L
Serum total Bilirubin
35 umol/L
(NR 1 22)
Serum aspartate aminotransferase
70 IU/L
(NR 1 31)
Serum alkaline phosphatase
220 IU/L
(NR 45 105)
Serum albumin
30 g/L
(NR 37 49)
Which of following measures would be appropriate in initial management of this patient?
1 ) Drain 4L ascitic fluid
2 ) Liver biopsy
3 ) Peritoneovenous shunt
4 ) Start ACE inhibitor
5 ) Start spironalactone
Comments:
The initial management would be spironolactone which usually well tolerated in these patients.
Both ascitic urine culture would then be appropriate but transcutaneous liver biopsy
contraindicated ascites (use trans jugular biopsy if absolutely necessary).
Several large randomized, controlled trials have shown repeated large volume paracentesis (4 6
L) safer more effective treatment of tense ascites compared larger than usual doses of
diuretics.5 )
119

16. A 72 year male presents a three month history of tiredness a weight loss of
approximately 4 kg. Five months ago he was commenced on amiodarone paroxysmal atrial
fibrillation maintained on 200 mg daily. Examination reveals a pulse of 90 beats per
minute regular, he a slight tremor of outstretched hands but lid lag evident. No goitre
palpable nor any bruit heard over thyroid.
Investigations reveal:
Haemoglobin
15.2 g/dL
White cell count
8.9 x 109/L
Platelets
299 x 109/L
ESR (Westergren)
11 mm/1st hour
Free T4
28.6 nmol/L
(NR 10 22)
Free T3
6.2 nmol/L
(NR 3.0 5.5)
TSH
< 0.05 mU/L
(NR 0.5 4.0)
Thyroid autoantibodies
Negative
What is appropriate initial step in management of this patient?
1 ) No change in treatment required
2 ) Stop amiodarone
3 ) Treat Carbimazole
4 ) Treat prednisolone
5 ) Treat Radio Iodine
Comments:
The patient mild amiodarone induced hyperthyroidism which unlikely be due a thyroiditis as
inflammatory markers (reflected by ESR) insignificant. The appropriate action this patient
stop amiodarone although in view of prolonged half life anti thyroid therapy such as
carbimazole may be required. An alternative anti arrhythmic may bebused treatment of his
paroxysmal AF. 2 )
17.
A 42 year old businessman presented Casualty 6 weeks after returning from a
business trip Thailand. On examination he was febrile (38.5C) had generalised
lymphadenopathy a diffuse macular rash. Atypical lymphocytes were seen on a film. He
had taken mefloquine as malaria prophylaxis.What is the likely diagnosis?
1
2
3
4
5

)
)
)
)
)

Acute HIV disease (seroconversion illness)


Dengue fever
Plasmodium falciparum malaria
Glandular fever
Secondary syphilis

Comments:
Acute HIV disease (seroconversion illness) occurs 2 12 weeks following exposure HIV said
occur in 40 80% of individuals. The typical presentation fever, severe pharyngitis, generalised
lymphadenopathy a macular maculo papular rash. The presentation may mimic glandular fever,
though pharyngitis associated acute HIV usually more severe a rash only occurs in glandular
fever when ampicillin given. Dengue fever an incubation of 21 days. 1 )
18.
A 55 year male previously diagnosed alcoholic liver disease was admitted
increasing confusion. He consumed at least 40 units of alcohol weekly.
120

On examination he was confused a Glasgow Coma Scale of 14, had a slight flap of
outstretched hands. His temperature was 36.8C he had a pressure of 122/88 mmHg. He
was noted be jaundiced spider naevi palmar erytthema. Abdominal examination slight
distension but organomegaly.
Investigations revealed:
Serum sodium
139 mmol/L
Serum potassium
3.6 mmol/L
Serum urea
2.7 mmol/L
Serum creatinine
65 umol/L
Serum bilirubin
65 umol/L
(NR 10 - 20)
Serum aspartate aminotransferase
150 IU/L
(NR 10 - 40)
Serum alanine aminotransferase
110 IU/L
(NR 10 - 40)
Serum alkaline phosphatase
450 IU/L
(NR 30 - 100)
Serum gamma glutamyl transferase
500 IU/L
(NR 10 - 50)
At stage, he was commenced on rapid detoxification programme consisting of diazepam
20 mg at a minimum interval of 2 hourly (max dose 200mg on 24 hours, standard regime).
The following day, he was found collapsed in bathroom on ward.
Which of following agents should be administered immediately?
1 ) Cefotaxime
2 ) Flumazenil
3 ) Naloxone
4 ) Thiamine
5 ) Vitamin K
Comments:
The rapid detoxification programmes now used are excellent acute withdrawal from alcohol but
should be used in patients who are encephalopathic ( as suggested by his confusion flapping
tremor). Causes of his encephalopathy should be sought such as Spontaneous Bacterial
peritonitis, GI bleed infection. 2 )
19. A 68 year old retired plumber presents a six month history of dry nocturnal cough
increasing exertional breathlessness. The cough unproductive there haemoptysis. He
comfortable at rest but his breathing limits him 400 metres on flat he beginning have
difficulty climbing stairs. He sleeps four pillows. He had a myocardial infarction four years
ago a 15 year history of hypertension. His current treatment aspirin, atenolol, Simvastatin
bendrofluazide.
On examination he finger clubbing, cyanosis looks pale. His pulse 48 beats per minute
regular. Blood pressure I 158/ 78. On examining his chest he vesicular breath sounds
bilateral basal crackles.
Investigations reveal:
Arterial gases on air:
PaO2
8.2 kPa1
PaCO2
5.1 kPa
pH
7.41
Pulmonary function tests:
121

FEV1
2.3 (predicted 3.0)
FVC
2.8 (predicted 3.8)
FEV1/FVC
82%
The ECG showed a sinus bradycardia Q waves in anterior chest leads left ventricular
hypertrophy (by voltage criteria).
The chest x ray showed bilateral lower zone shadowing
Which one of following investigations likely establish diagnosis?
1 ) echocardiography
2 ) high resolution CT scan of chest
3 ) measurement of diffusion capacity
4 ) serum angiotensin converting enzyme (ACE) activity
5 ) bronchoalveolar lavage
Comments:
The patient presents symptoms signs are consistent pulmonary fibrosis. This supported by
his CXR which shows bilateral lower zone shadowing a reticulo rnodular pattern seen his
spirometry which demonstrates a restrictive defect. His gases show Type 1 respiratory failure,
again which consistent diagnosis. HRCT of chest investigation of choice. It will confirm
diagnosis of interstitial fibrosis can be diagnostic in a number of diseases such as Usual
Interstitial Pneumonia (CFA), Asbestosis Sarcoidosis preventing need lung biopsy. 2 )
20. A 74 year man presented his general practitioner a chronic cough. He smokes 10
cigarettes per day starting at age of 14. The GP had organised a chest x ray which was
reported as showing calcification on both hemi diaphragms clear lung fields. He was
therefore referred outpatients department. An occupational history he had worked in a
shipyard about eight years fifty years previously. He had previous history of note.
Systems review was negative apart from some intermittent heartburn nocturia. In
particular his appetite was there had weight loss. He lived alone but kept pigeons.
On examination his pulse was 74 beats per minute, pressure 155/75 mmHg there were
abnormalities found on respiratory examination.
Which one of following statements correct?
1 ) He does require long term follow up
2 ) He at high risk of developing asbestosis
3 ) He at high risk of developing mesothelioma
4 ) It unlikely x ray abnormality related his previous occupation
5 ) The x ray abnormality cause of his cough
Comments:
This patients CXR shows calcification on both hemi diaphragms which likely be pleural plaques
from previous asbestos exposure. He would have exposed asbestos in his job in ship yards.
Pleural plaques are benign simply a marker of previous asbestos exposure therefore are
common in anyone hasbeen exposed asbestos in past. They rarely cause symptoms
thereforeunlikely be cause of his cough. They require long term follow up. He unlikely
develop asbestosis 50 years after his last exposure asbestos patients developing disease within
20 years of exposure. Similarly average latent period from exposure diagnosis of mesothelioma
20 years. It as a result of this incidence of mesothelioma 1 )
21. A 67 year lady was admitted a productive cough confusion ten days after being in
hospital a stroke.
On examination her temperature was 39C. She had a residual left sided weakness but
other neurological deficit. Her pulse was 110 beats per minute pressure 110/60 mmHg.
122

Her respiratory rate was 22 breaths per minute. There was dullness percussion at right
base coarse crackles an area of bronchial breathing.
A chest x ray shadowing in right lower lobe a diagnosis of pneumonia was made. What is
the likely causative organism?
1 ) Anaerobic bacteria
2 ) Fungi
3 ) Gram negative bacteria
4 ) Gram positive bacteria
5 ) Legionella
Comments:
With a pneumonia occurring so soon after discharge from hospital this lady should be treated as
having a (late) hospital acquired pneumonia. Her age recent CVA are risk factors. The common
organisms are gram negative bacteria although anaerobic bacteria are common particularly if
there a history of aspiration. 3 )
22. A 15 year girl referred by her general practitioner agitation weight gain. Her
mother accompanies her during consultation explains over last 2 months she become
increasingly agitated poor sleep. Her progress at school up until recently, fine, although
of late she apathetic. She past history of note, although family history reveals her
mother was treated an 'overactive thyroid' now takes thyroxine tablets. Examination
reveals specific abnormalities a pressure of 112/70 mmHg a BMI of 20.
The GP's letter includes results of following investigations:
TSH
3.2 mU/L
(NR 0.35 - 5.0)
Total T4
250 nmol/L
(NR 55 - 144)
Free T4
12.9 pmol/L
(NR 9 - 24)
Total T3
3.2 nmol/L
(NR 0.9 - 2.8)
free T3
3.8 pmol/L
(NR 3.0 - 5.8)
What likely explanation of her presentation results?
1 ) Bulimia Nervosa
2 ) Dysthyroglobulinaemia
3 ) Factitious Thyrotoxicosis
4 ) Graves Disease
5 ) Pregnancy
Comments:
This young girl TFTs as reflected by free T4, T3 TSH. However, her total T4 T3 are elevated
suggesting elevated Thyroid Hormone Binding, a feature of pregnancy. Dysthyroglobulinaemia
Pendred's syndrome a combination of Hypothyroidism, goitre Deafness. 5 )
23. A 67 year lady presented a seven month history of exertional breathlessness
cough. She had a reduced appetite had lost half a stone in weight. She had previous
history of note. She had worked as a hairdresser. She kept a cat at home. She lived alone
but had coping well until now. She smoked 20 cigarrettes a day.
On examination she was clubbed cyanosed. She was pale. Pulse rate was 80 beats per
minute. BP was 138/80 mmHg. Heart sounds were normal. There were bilateral fine
inspiratory crackles heard at lung bases.
123

Investigations revealed:
FEV1
2.8 L
(3.6 predicted)
FVC
3.1 L
(4.5 predicted)
Diffusion capacity
5.1 mmol/min/kPa (NR 6.3 11.9)
The chest X ray showed slight increase in basal lung markings What is the likely
diagnosis?
1 ) bronchiectasis
2 ) fibrosing alveolitis
3 ) left ventricular failure
4 ) lymphangitis carcinomatosis
5 ) sarcoidosis
Comments:
The history examination findings are suggestive of interstitial lung disease (ILD). The pulmonary
function tests demonstrate a reduction in both FEV1 FVC (FEV1/FVC = 90%) a low diffusion
capacity i.e. restrictive defect which consitent ILD. The likely diagnosis CFA (or Usual
Interstitial Pneumonia). Bronchiectasis usually results in a productive cough an obstructive
pattern on lung function. Sarcoidosis usually affects mid zones distributed more uniformly
throughout lungs. The history CXR findings are suggestive of LVF lymphangitis
carcinomatosis typically produces hilar enlargement diffuse streaky midzone infiltrates. The
diagnosis can be confirmed on HRCT in cases although lung biopsy may be required. 2 )
24. A patient chronic renal failure, treated regular haemodialysis, attends renal clinic.
He treated six months oral ferrous sulphate, 200mg three times a day. His haemoglobin
at this clinic attendance 7.6. His previous result was 10.6 six months ago.Which of the
following is appropriate treatment?
1 ) Blood transfusion
2 ) Commence SC erythropoietin
3 ) Increase dose of oral ferrous sulphate
4 ) IV Iron
5 ) IV iron subcutaneous erythropoietin
Comments:
The aim in this patient maintain haemoglobin concentration above 11g/dl (K DOQI guidelines).
It important correct anaemia, as it leads ventricular hypertrophy, which increases
cardiovascular morbidity mortality. This patients haemoglobin dropped despite appropriate
doses of oral ferrous sulphate. The iron demands will increase further if erythropoietin started.
The patient should be prescribed IV iron therapy (following a small IV test dose, exclude
anaphylaxis) SC erythropoietin, enhance erythropoiesis increase haemoglobin concentration.
If patient does respond this dual regime, he should be investigated sepsis, chronic loss, non
compliance of treatment, severe hyperparathyroidism (which can cause marrow fibrosis). 5 )
25. A 19 year old lady developed increasing breathlessness hypoxia two days after
admission severe burns. She had previous history of note. Her father had had a
myocardial infarction at age of 40. She was a non smoker. On examination she had a
respiratory rate of 26 breaths per minute a pulse rate of 110 beats per minute. On
listening her chest crackles over both lung fields. A chest x ray bilateral hazy
shadowing.What is the likely diagnosis?
1)
adult respiratory distress syndrome
2)
chemical pneumonitis
124

3)
left ventricular failure
4)
nosocomial pneumonia
5)
pulmonary haemorrhage
Comments:
This lady likely have developed adult respiratory distress syndrome (ARDS) as a complication of
her severe burns. The patient becomes tachypnoeic, increasingly breathless cyanosed develops
refractory hypoxia. The CXR classically shows bilateral peripheral interstitial alveolar infiltrates
become progressively more confluent but spare costophrenic angles. Normal heart size, absent
septal lines, air bronchograms a peripheral distribution are helpful in differentiating ARDS from
other conditions such as LVF. 1 )
26. A 72 year old man undergoes an emergency abdominal aortic aneurysm repair.
Three days later, you are asked review him on surgical high dependency unit. On
examination, he drowsy but a Glasgow Coma Scale of 15/15, his pressure 150/90 mmHg
a Central Venous Pressure recording +14 cm H2O. Investigations reveal:
Serum sodium
132 mmol/L
Serum potassium
5.9 mmol/L
Serum urea
32.8 mmol/L
Serum creatinine
520 umol/L
Urinary sodium
58 mmol/L
Urine volume
320ml/24hrs
Urine osmolality
280 mOsmol/L
An ultrasound scan shows left kidney measures 12.7cm in length right kidney 11.5cm in
length. Which one of following would point a diagnosis of acute tubular necrosis (ATN)
rather than pre renal failure?
1 ) A serum potassium of 5.9 mmol/l
2 ) Renal USS results
3 ) Serum urea of 32.8 mmol/l
4 ) Urine osmolality 280 mOsm/l
5 ) Recent surgery
Comments:
80 90% of acute renal failure seen by physicians will fall into category of pre renal failure ATN.
In pre renal failure, diagnosis made by rapid resolution of renal impairment appropriate fluid
resuscitation.The diagnosis of ATN based on clinical context, which often involves circulatory
compromise, exclusion of obstruction renal inflammatory conditions. The urinary
abnormalities in ATN suggest tubular dysfunction (i.e. urinary sodium >40mmol/l, low urinary
osmolality < 350 mosmol/l). In this case patient appropriately fluid resuscitated post
operatively (CVP +14). The urinary biochemistry in keeping ATN, there evidence obstruction
inflammation. The management involves correcting any underlying electrolyte disturbance,
either by renal replacement therapy if severe, by managing his input/output of fluids
electrolytes. High does furosemide (0.5 2g/day) may help control fluid balance, but there
evidence furosemide renal dose dopamine reduces length of renal impairment, need dialysis
mortality. If this patient does succumb his underlying illness, he survives acute renal failure
episode, his renal function should recover completely over days weeks (following a period of
relative polyuria). 4 )
125

27. A 44 year old HIV seropositive patient seen in a routine outpatient appointment. He
was diagnosed HIV disease two years ago when he presented Pneumocystis carinii
pneumonia (PCP) a CD4 T lymphocyte count of 40 cells/mm3. After being treated PCP he
was started on combination antiretroviral therapy. His initial response antiretroviral
therapy had promising, an increase in his CD4 count (210 cells/mm3) achieving an
undetectable HIV viral load in peripheral blood.
One year ago, his CD4 count started fall HIV RNA became detectable in peripheral blood.
An HIV viral resistance test confirmed resistance all of his antiretroviral drugs. The patient
admitted over preceeding three months he had only taking his therapy intermittently.
Despite risks, he decided he did wish have any further antiviral therapy. At this point,
his CD4 count was 20 cells/mm3 a very high HIV viral load of 120,000 copies/ml.
Seven months ago he presented malaise, weight loss (8kg), fevers night sweats.
Mycobacterium avium intracellulare (MAI) was isolated from a culture he was started on
therapy ethambutol rifabutin. He subsequently agreed re start antiretroviral therapy.
During his clinic consultation, he states he noticed a steady deterioration in his visual
acuity over past four weeks. His current medications include stavudine (d4T), didanosine
(ddI), nevirapine, ethambutol, rifabutin. On examination, fundoscopy normal.
His last CD4 count, taken one month ago was 30 cells/mm3, an HIV viral load of 2500
copies/ml.What is the likely cause of his reduction in visual acuity?
1 ) Antiretroviral therapy
2 ) Cytomegalovirus retinitis
3 ) Ethambutol
4 ) HIV retinopathy
5 ) Toxoplasma retinitis
Comments:
Loss of visual acuity in this case consistent ethambutol induced optic neuritis. Ethambutol may
produce optic neuritis which decreases visual acuity which appears be related dose duration
of treatment. Symptoms generally start between 4 months one year after starting therapy. The
effects are generally reversible when administration of ethambutol discontinued promptly. In
rare cases recovery may be delayed one year more effect may possibly be irreversible in these
cases. Although patient advanced HIV a low CD4 count might be susceptible CMV
toxoplasma retinitis, both of these conditions are usually associated retinal lesions would be
visible on fundoscopy. HIV retinopathy Reduced visual acuity typically seen any of
antiretrovirals listed. 3 )
28. A 65 year old man presented a four week history of pleuritic chest pain associated
shortness of breath dry cough. He also reported weight loss of nearly 10kg in past six
months.
He had a past history of myocardial infarction 20 years earlier from which he had made a
good recovery. He did suffer from any exertional chest pain subsequently. He lived alone
had seen his general practitioner two years. He seldom saw his General Practitioner, but
had attended surgery twice recently mild recurrent pain in his left knee responded well
treatment simple analgesia. He was an ex smoker of 15 cigarettes per day, having given
up smoking 20 years previously. His only medication was aspirin.
On examination of his chest he had reduced expansion, dull percussion note decreased
breath sounds on right. A chest X ray confirmed a right sided pleural effusion.
Analysis of a pleural aspirate revealed:
Pleural fluid protein content
42 g/L
126

Pleural fluid glucose


What is the diagnosis?
1
2
3
4
5

1.3 mmol/L

) bronchial carcinoma
) cardiac failure
) mesothelioma
) rheumatoid arthritis
) tuberculosis

Comments:
The pleural fluid protein greater than 30g/l which demonstrates it an exudate effectively
excludes cardiac failure. If pleural fluid protein 25 35 g/l then Lights Criteria more accurate in
determining whether effusion an exudate transudate. It an exudate if 1 more of following
criteria are met (a) pleural fluid protein divided by serum protein > 0.5 (b) pleural fluid LDH
divided by serum LDH > 0.6 (c) pleural fluid LDH > 2/3rds upper limits of serum LDH.
The pleural glucose level very low. Levels less than 3.3 mmol/l are found in empyema,
rheumatoid arthritis, lupus, malignancy, oesophageal rupture tuberculosis. The lowest levels are
found in rheumatoid effusions empyema pleural glucose in rheumatoid effusions rarely being
above 1.6 mmol/l. 4 )
29. A 73 year old lady presented lethargy weight loss of one stone over last four
months. She had had an irritating cough repeated haemoptysis. She had a smoker of 20
cigarrettes a day but gave up four months previously when her symptoms began. She had
treated pulmonary tuberculosis 15 years previously. She was sure what medication she
was given how long.
Investigations revealed:
Haemoglobin
12.0 g/dL
White cell count
4.1 x 109/L
Platelets
140 x 109/L
Serum sodium
134 mmol/L
Serum potassium
3.7 mmol/L
Serum urea
8.5 mmol/L
Aspergillus fumigatus precipitins
positive
RAST Aspergillus
negative
Her chest x ray showed a solid lesion at left lung apex What is the likely diagnosis?
1
2
3
4
5

)
)
)
)
)

allergic bronchopulmonary aspergillosis


aspergilloma
bronchial carcinoma
invasive aspergillosis
reactivation of tuberculosis

Comments:
The lesion in left apex likely be an aspergilloma which developed in an tuberculous cavity.
Aspergillomas are often asymptomatic but can result in haemoptysis in up three quarters of
patients. In some patients haemoptysis can be massive fatal. Systemic symptoms of weight loss,
127

lethargy fever are less common. The CXR appearances are of a solid opacity within a cavity often
associated a rim of air. Thses features are seen more clearly on computed tomography.
Precipitating antibodies help confirm diagnosis are present in 95% of cases. 2 )
30. A 33 year referred clinic by her GP tiredness lethargy of approximately three
months duration. She was diagnosed an thyroid 2 years ago by her GP was commenced
on 150 micrograms of thyroxine daily. She two young children had well up until birth of
her last child 6 months ago since feeling increasingly tired weight gain. She a smoker of
5 cigarettes daily drinks modest quantities of alcohol. Her mother also receiving
thyroxine replacement therapy an thyroid.
Examination reveals a pulse of 82 beats per minute, a pressure of 124/74 mmHg goitre
palpable. No abnormalities are noted. Investigations reveal:
Haemoglobin
12.0 g/dL
White cell count
6.1 x 109/L
Platelets
235 x 109/L
Serum sodium
141 mmol/L
Serum potassium
3.4 mmol/L
Serum urea
3.7 mmol/L
Serum creatinine
87 umol/L
Free T4
18.1 nmol/>
(NR 10 22)
Free T3
3.8 nmol/L
(NR 3 8)
TSH
25.8 mU/L
(NR 0.5 4.0)
Thyroid peroxidase antibody
Strongly
positive
What is the likely cause this womans presentation?
1 ) De Quervains thyroiditis
2 ) Dysthyroglobulinaemia
3 ) Non compliance thyroxine
4 ) Secondary hypothyroidism
5 ) TSH feedback adenoma
Comments:
Non compliance likely cause this womans deranged TFTs T4 T3 but elevated TSH reflecting
she just remembered take thyroxine prior her outpatient visit. A TSH feedback adenoma
exceedingly rare associated long term untreated hypothyroidism resulting in development of a
pituitary TSH adenoma. Secondary hypothyroidism would be associated a raised TSH. De
Quervains thyroiditis a subacute thyroiditis 3 )
31. A 19 year old man a 10 year history of asthma presents a six week history of
worsening symptoms of exertional breathlessness wheeze. He a history of eczema
hayfever as a child. He smokes 10 cigarettes a day works as an electrician. In addition he
a cough which wakes him up nights. The cough unproductive. Systems review was
negative apart from some occasional diarrhoea. His current treatment inhaled
Beclomethasone 800 ug per day inhaled Salbutamol 200 ug when required via a metered
dose inhaler.Which one of following next appropriate step in management?
1 ) add Aminophylline
2 ) add Montelukast
128

3 ) add Salmeterol
4 ) change a dry powder inhaler
5 ) double dose of Beclomethasone
Comments:
This patient poorly controlled asthma. His current regime places him at step 2 of BTS guidelines
but there a clear need an increase in therapy control symptoms. Many RCT meta analysis of
RCT (eg. Pauwels RA et al. N Eng J Med 1997;337:1405 1411. Shrewsbury et al BMJ
200;320:1368 1373) have shown addition of a long acting b agonist (LABA) such as salmeterol
eformeterol improves symptom comtrol, lung function, reduce exacerbations over increasing
dose of inhaled corticosteroids whilst other studies show clear benefits of LABA over leucotriene
receptor antagonists such as montelukasts theophyllines.. 3 )
32. In HIV disease, patients first become susceptible infection Pneumocystis carinii
when CD4 cell count falls to:
1 ) < 1000 cells/mm3
2 ) < 500 cells/mm3
3 ) < 350 cells/mm3
4 ) < 200 cells/mm3
5 ) < 50 cells/mm3
4)

33.
A 22 year old was admitted hospital, three days after a sore throat. The patient complained
of abdominal pain, loose stools, wrist ankle pain.
On examination, there was a non blanching macular papular rash over back, buttocks lower legs.
Her pressure was 175/90 mmHg.Investigations showed:
Serum urea
8 mmol/L
Serum creatinine
118 umol/L
Serum albumin
30 g/L
Urinalysis:
Protein ++ Blood +
What would be appropriate first treatment?
1 ) ACE inhibitor
2 ) Azathioprine
3 ) Cyclophosphamide
4 ) High dose Prednisolone
5 ) Plasma Exchange
This lady probably Henoch Schonlein nephritis (similar IgA nephritis). It should be diagnosed on renal
biopsy. It can present 1 3 days following infection of an IgA secreting mucous membrane (commonly
following pharyngitis, but can occur following infection of GI tract, bladder breast). The main
presentation painless macroscopic haematuria, but in others it can be microscopic haematuria
proteinuria, less commonly nephrotic syndrome. The rash due a cutaneous vasculitis, abdominal pain
due gut vasculitis, which may be severe in some cases, leading bloody diarrhoea. Arthralgia a common
symptom.
This lady ++protein on dipstick, +blood. She therefore a worse prognosis than patients haematuria
alone. All patients hypertension proteinuria (>1g/day which this patient probably has) should be started
on an ACE inhibitor, which may control BP proteinuria. Once BP controlled she should have a renal
biopsy, if this showed changes of a crescentic GN, then an immunosuppression regime similar used in
renal vasculitis should be started (probably high dose steroids in first instance +/ Cyclophosphamide).
In children, Henoch Schonlein Purpura (HSP) common cause of vasculitis affecting kidneys. In adults
presenting similar symptoms signs, cause necessarily HSP, but may be any type of vasculitis. 1 )

129

34.
A 32 year old man presented hospital a four week history of progressively worsening
dyspnoea on exertion. He also complained of a non productive cough. Over two days preceeding
admission patient had become breathless at rest was started on oral co amoxiclav by his general
practitioner.On examination he was febrile 38C looked unwell. Candida was noted on tonsilar
pillars. No wheeze crackles were heard in his chest. His chest radiograph shown. Oxygen
saturation was 95% on room air, but fell 85% following about of coughing. Arterial gases show
pO2 of 59 mmHg.

What
1)
2)
3)
4)
5)

treatment shold be given?


Co amoxiclav + clarithromycin
Co trimoxazole + prednisolone
Vancomycin + ceftazidime
Cefuroxime + metronidazole
Benzylpenicillin + flucloxacillin

Comments:
Oral candidiasis never should always raise possibility of AIDS history in this case
characteristic of Pneumocystis carinii pneumonia. There are several agents can be used treat
PCP, though co triomoxazole clearly effective treatment of first choice. Patients often
deteriorate after starting therapy PCP as pneumonitis worsens due inflammation associated
dying pneumocysts. In patients a pO2<60 mmHg, oral prednisolone added reduce
inflammatory effect. 2 )
35. A 35 year old woman brought casualty by her husband. She drowsy confused.
There a three day history of malaise low grade fever, but she become increasingly
confused over past eight hours apparently drifting in out of conciousness. On
examination she drowsy but rousable. GCS 8/15. There are focal neurological signs. While
in Emergency Unit she a generalized tonic clonic seizure. An MRI scan of her brain shown.

130

A lumbar puncture was performed analysis of cerebrospinal fluid revealed:


Appearance
Clear, colourless
Cells
500 mm3 (>90% lymphocytes)
Protein
0.7 g/L
Glucose
4.5 mmol/L
Plasma glucose was measured as
5.3 mmol/L
Which of following investigations is likely to reveal the correct diagnosis?
1)
Blood cultures
2)
CSF HSV PCR
3)
CSF Cryptococcal neoformans
4)
Magnetic resonance angiogram
5)
Serum viral titres
Comments:
This patient's MRI presentation typical of Herpes simplex encephalitis. The MRI demonstrates a
temporo parietal dense lesion. The diagnosis reliably confirmed by presence of HSV in CSF by
PCR. Read more about HSE 2 )
36. A 65 year male presents increasing breathlessness weight gain over last three
months. He treated heart failure by his GP, receiving frusemide Lisinopril. On
examination he a pulse of 88 beats per minute regular a pressure of 120/80 mmHg his
JVP measured at 4 cm above manubrio sernal angle. Auscultation of hear reveals an
added diastolic sound. His chest examination clear. His abdomen slightly swollen he
shifting dullness percussion. He leg oedema knees. His lateral chest x ray shown.
131

What
1)
2)
3)
4)
5)

is the diagnosis?
Amyloidosis
Constricitve pericarditis
Endomyocardial fibrosis
Ischaemic cardiomyopathy
Sarcoidosis

Comments:
The right sided failure, ascites pericardial calcification on X ray suggest a diagnosis of
constrictive pericarditis. Sarcoidosis would, like amyloidosis be expected cause a restricitive
cardiomyopathy. There left sided failure suggest an ischaemic cardiomyopathy. The added
sound pericardial 'knock' rather than a third/fourth heart sound. 2 )
37. This serum specimen was obtained on a 46 year male admitted under care of
surgeons.

132

What is the likely explanation this appearance?


1
2
3
4
5

)
)
)
)
)

Apolipoprotein CII deficiency


Dysbetalipoproteinaemia
Hepatic LDL receptor deficiency
Lipoprotein Lipase Deficiency
Tangier's disease

Comments:
This milky looking serum sample due hyperchylomicronaemia/hypertriglycerideamia a
consequence of deficiency of Lipoprotein Lipase. Xanthomas, Lipaemia Retinalis are features
Pancreatitis Gout occur. Can occur as primary condition due a rare autosomal loss of LPL
more commonly secondary diseases such as pancreatitis, hypothyroidism, type 1 diabetes,
alcoholism Cushings syndrome. Apolipoprotein C II Deficiency (rare autosomal hereditary
disorder) an inhibitor lipoprotein lipase may cause this type of appearance but extremely rare
manifests in childhood. Tangier Disease an extremely rare autosomal metabolic disorder.
Characteristics of Tangier Disease include increased levels even a complete absence of high
density lipoproteins (HDL) concentrations in one's plasma, low cholesterol levels in plasma,
increased cholesteryl esters in tonsils, spleen, liver, skin lymph nodes. One easily visual
characteristic usually found in children Tangier disease presence of enlarged, yellow orange
tonsils. Hepatic LDL receptor deficiency a feature of Familial Hypercholesterolaemia. 4 )
38. A 47 year old lady presented two weeks after returning from a holiday in Caribbean
complaining of an intensely pruritic rash on her abdomen.
133

What
1)
2)
3)
4)
5)

is the causative organsim?


Ancylostoma braziliense
Chironex fleckeri Southcott (Box Jelly Fish)
Borrelia burgdorferi
Onchocerca volvulus
Treponema pallidum

Comments:
The figure shows cutaneous larva migrans (creeping eruption). Usually caused by an animal
hookworm, commonly Ancylostoma braziliense. The infection acquired by direct contact dog
cat faeces often acquired when sunbathing on contaminated sand, etc. The larvae burrow in
dermo epidermal junction. Symptoms include pruritis a raised, serpiginous erythematous rash
migrates at a rate of up 1 cm/day. Treatment topical thiobendazole, oral albendazole. Acute
infection human nematodes Strongyloides stercoralis, Necator americanus Ancylostoma
duodenale, may produce a similar appearance. (Further reading Archives of Dermatology) 1 )

1.
A 23 year presents acutely unwell. She a three month history of weight loss,
tiredness lethargy which deteriorated over last week. Six weeks previously she had
diagnosed hypothyroidism by her general practitioner.
Investigations at time showed:
Free T4
8.8 nmol/L
(NR 9.8 22.0)
Plasma TSH
5.5 mu/L
(NR 0.5 5.0)
She had started thyroxine 50 micrograms daily but had deteriorated over last 2 weeks.
She a non smoker, drinks alcohol takes oral contraceptive pill. Her mother maternal
grandmother have both diagnosed hypothyroidism take thyroxine.
On examination she appears unwell mildly dehydrated. She a temperature of 37.5oC a
BMI of 21.3 kg/m2. Her pressure 72/44 mmHg, a pulse of 100 beats per minute.
Examination of cardiovascular system otherwise normal. No abnormalities are
encountered on respiratory abdominal examination. Brief neurological examination both
plantars are flexor.
134

As yet, investigations requested by House Officer are unavailable. In meantime What is


the appropriate immediate management of this patient?
1 ) Intravenous Cefotaxime
2 ) Intravenous Glucose
3 ) Intravenous Hydrocortisone
4 ) Intravenous saline
5 ) Intravenous thyronine (T3)
Comments:
The patient had a long history of weight loss fatigue was diagnosed hypothyroidism based
upon a slightly low T4 slightly high TSH. Thyroxine was prescribed but this precipitated a
deterioration of underlying condition such patient presents features suggesting an addisonian
crisis. Thus a emergency should be treated intravenous hydrocortisone. An appropriate test
would be a short synacthen test which could be completed in 30 minutes. Adrenal
autoantibodies are likely be positive in over 80% of cases. Sick euthyroidism a recognized
feature of Addisons disease treatment thyroxine may exacerbate condition precipitate acute
hypoadrenalism. 3 )
2.
A 15 year old girl referred clinic complaining of generalised muscle weakness,
fatigue polyuria. Her pressure in clinic measured at 90/74 mmHg.
Investigations reveal:
Serum sodium
127 mmol/L
Serum potassium
3.0 mmol/L
Serum urea
7.2 mmol/L
Serum creatinine
110 umol/L
Serum chloride
92 mmol/L
Serum bicarbonate
34 mmol/L
Serum magnesium
0.82 mmol/L
(NR 0.8 1.1)
Urine sodium
160 mmol/L
(NR 40 130)
Urine potassium
150 mmol/L
(NR 30 110)
Urine calcium
8.0 mmol/24hr
(NR 2.5 8.0)
Which of following likely diagnosis?
1 ) Addisons disease
2 ) Bartters syndrome
3 ) Laxative abuse
4 ) Liddles syndrome
5 ) Thiazide diuretic abuse
Comments:
A difficult question! This patient a hypochloraemic alkalosis, high urinary sodium potassium
loss.
Laxative abuse tends cause low urinary sodium potassium, low serum bicarbonate, due GI
losses.
Thiazide diuretic abuse would fit serum findings, but urinary sodium potassium (as water
will follow electrolytes).
Liddles syndrome a congenital form of salt sensitive hypertension characterised by a very high
rate of renal sodium uptake, despite low levels of aldosterone, secondary hypokalaemia
135

metabolic acidosis. It caused by a congenital mutation, which causes a constitutive hyper


reactivity in epithelial sodium channel (ENaC). The increased sodium uptake accompanied by an
increased water uptake, leading an increase in volume, secondary hypertension.
Addisons disease primary adrenal insufficiency. The common cause in western world
autoimmune adrenalitis, worldwide causes include TB, fungal infections (histoplasmosis,
cryptococcus) cytomegalovirus. Patients Addisons disease have glucocorticoid
mineralocorticoid deficiency. This leads hypotension, hyponatraemia (in 90%) hyperkalaemia (in
65%).
Bartters syndrome a rare, autosomal disorder, caused by one of three mutations of ion
transporter ion channel present in thick ascending limb of distal nephron.
Type I II mutations present in infancy (often following premature birth polyhydramnios) severe
dehydration, hypokalaemic alkalosis, hypercalciuria nephrocalcinosis. Mortality high.
Type III mutations present a more varied clinical picture type I II, ranging in severity from near
fatal volume depletion hypokalaemic alkalosis respiratory arrest, mild disease presenting in
teenagers weakness polyuria. Nephrocalcinosis described in type III mutations, therefore it
can differentiate between type I II disease, type III disease. Management long term potassium
supplementation care avoid dehydration. The long term prognosis uncertain. 2 )
3.
A 45 year male diabetic was seen in clinic at diabetic clinic annual review. He had a
diabetic 8 years attended clinic regularly. He was also being treated hypertension was
taking metformin 500mg tds, gliclazide 80mg daily, atorvastatin 10mg/d, ramipril 10mg/d
bendrofluazide 2.5 mg/d.
He was noted be obese (130kg). Physical examination was otherwise unremarkable.
Investigations reveal:
HbA1c
8.1%
(NR < 6.5)
Fasting glucose
9 mmol/L
(NR 3.5 6.0)
24hr Urine free cortisol 354 mmol/d
(NR < 250)
9am Plasma ACTH
12 ng/dL
(NR 10 50)
CT abdomen
3cm right adrenal mass
What adrenal mass?
1 ) Aldosterone secreting adenoma
2 ) Cortisol secreting adenoma
3 ) Ectopic CRF producing Phaeochromocytoma
4 ) Incidentaloma
5 ) Metastasis
Comments:
The patient likely have Cushing's syndrome as suggested by elevated UFC lowish ACTH plus
adrenal mass on CT would suggest this due a functional adrenal adenoma. 2 )
4.
A 33 year nurse admitted prolonged fasting. She originally presented clinic a
history of episodic sweating light headedness which had developed over a six month
period symptoms being entirely relieved by eating. She had developed one of these
episodes whilst on ward a BM monitor showed a value of 2 mmol/l. She took some
glucose tablets had quickly recovered.
On examination specific abnormalities were found a pressure of 118/74 mmHg, a pulse
of 72 beats per minute a BMI of 22 kg/m2.
She was admitted a 72 hour fast at 3 am, 16 hours into fast she develops typical
symptoms. Her BM measured at 2.2 mmol/l, fast stopped bloods taken. Her results show:
136

Plasma Glucose
1.8
(NR
Plasma 3 Beta HydroxyButyrate
0.5 mmol/L
(NR
Plasma Insulin
450 pmol/L
(NR
C peptide
0.2 nmol/L
(NR
What is the likely diagnosis?
1 ) Adult Glycogen storage disease
2 ) Factitious hypoglycaemia due Insulin treatment
3 ) Factitious hypoglycaemia due Sulphonylurea treatment
4 ) Insulinoma
5 ) Non islet cell tumour hypoglycaemia

3
>
<
<

6)
1 mmol/l)
21)
0.5)

Comments:
This patient developed hypoglycaemia suppression of her 3betaHydroxybutyrate (a ketone),
elevated insulin yet suppressed C peptide. This would suggest there insulin induced
hypoglycaemia as C peptide suppressed indicates exogenous administration of insulin.
Sulphonylureas would produce raised insulin C peptides could be assessed in suspicious cases
by measuring a sulphonylurea concentration. Again insulinoma would be associated
proportionately elevated insulin C peptide.2)
5.
A 32 year old man referred general outpatient clinic. He his wife had referred by
his GP infertility clinic consideration assisted conception. He generally fit well works
as a central heating engineer. He referred because he a chronic productive cough a
history of recurrent chest infections since childhood in addition recurrent siunsitis.
Investigations show:
Sodium sweat test
Serum IgG
Serum IgA
Serum IgM

Normal
7.4 g/L
1.2 g/L
3.1 g/L

(NR 6 13)
(NR 0.8 3.0)
(NR 0.4 2.5)

What is the likely diagnosis?


1 ) Bronchiectasis
2 ) Chediak Higashi syndrome
3 ) Cystic fibrosis
4 ) Primary ciliary dyskinesia
5 ) Situs inversus
Comments:
Primary ciliary dyskinesia inherited as an autosomal disorder characterized by abnormal ciliary
motion impaired mucociliary clearance. The lack of effective ciliary motility, causes abnormal
mucociliary clearance. This leads recurrent persistent respiratory infections (which may lead
bronchiectasis), sinusitis, otitis media, male infertility. In 50% of patients, PCD associated
situs inversus (Kartageners syndrome).
The principal differential diagnoses consider in this case are cystic fibrosis primary ciliary
dyskinesia. The diagnosis of CF based on typical pulmonary and/or gastrointestinal tract
137

manifestations positive results on sweat test (pilocarpine iontophoresis). A negative sweat test
sufficient evidence exclude CF as a diagnostic possibility.
While patient may have bronchiectasis, this alone sufficient account his other symptoms. 4 )
6.
A 45 year old man presented diplopia, dysarthria difficulty swallowing. Over next
few days he developed weakness of upper lower limbs. On day 4 he was unable walk
unaided. He denied any sensory symptoms bladder disturbances. His previous history
unremarkable. He a non smoker, does drink alcohol excessively. He does take any drugs.
On examination he was apyrexial. His general examination was normal. His higher mental
function was unremarkable. There were signs of meningism. Cranial nerve examination
showed bilateral dilated fixed pupils. He had binocular diplopia but obvious
ophthalmoplegia. He was dysarthric weak cough. His vital capacity was 3.15 standing 2.00
lying flat. He had lower motor neuron tetraparesis of power 3/5. He was hyporeflexic
sensation was unable walk unaided.
Investgations including full count, urea electrolytes, liver function tests, serum calcium,
thyroid function tests, autoantibody screen, erythrocyte sedimentation rate serum C
reactive protein were normal.
His electrocardiogram chest radiograph were normal. A CT scan of his brain was normal,
as were nerve conduction studies an electromyogram (EMG).
What is the likely diagnosis?
1
2
3
4
5

)
)
)
)
)

Guillain Barre Syndrome


Lyme disease
Myasthenia gravis
Botulism
Vasculitis

Comments:
The clinical presentation of descending weakness autonomic dysfunction (fixed dilated pupils)
typical of botulism. It a neuromuscular junction disorder therefore nerve conduction studies
EMG are normal. Repetitive nerve stimulation showing incremental responses, which diagnostic
of Botulism. CSF analysis usually normal. 4 )
7.
A 45 year presents feeling unwell weight loss, throat pains palpitations. These
symptoms have developed over last two weeks she lost approximately 3kg in weight.
There other past history of note. She a smoker of 10 cigarettes per day drinks
approximately 10 units of alcohol weekly. She employed as a cleaner. Of note in her family
history a maternal grandmother who receives treatment an thyroid.
On examination she a temperature of 37.5oC, a fine tremor of outstretched hands, a pulse
of 98 beats per minute regular a pressure of 120/80 mmHg. She evidence of lid lag but
exophthalmos. Examination of her neck reveals a tender goitre but palpable
lymphadenopathy. No bruit audible over goitre. Auscultation of heart lungs are both
masses are palpable on abdominal examination.
Investigations reveal:
Haemoglobin
White cell count
ESR (Westergren)

14.5 g/dL
7.9 x 109/L
88 mm/1st hour
138

Serum sodium
139 mmol/L
Serum potassium
4.2 mmol/L
Serum urea
6.4 mmol/L
Serum creatinine
105 umol/L
Serum calcium
2.32 mmol/L
Free T4
45.4 nmol/L
(NR 10 23)
TSH
0.05 mU/L
(NR 0.5 5.0)
Anti thyroid peroxidase antibody positive
What is the likely diagnosis?
1
2
3
4
5

)
)
)
)
)

DeQuervain's (subacute) thyroiditis


Graves' Disease
Hashimoto's thyrotoxicosis
Papillary Thyroid cancer
Riedel's thyroiditis

Comments:
The salient features of this patient relatively acute onset of illness temperature,
thyrotoxicosis, tender goitre elevated ESR which all point a diagnosis of DeQuervains
thyroiditis. The high ESR would argue against a diagnosis of Hashimotos Graves. DeQuervain's
subacute thyroiditis a disease of unknown aetiology which associated inflammation of thyroid
follicles (thyroiditis) causing a liberation of their contents (thyrotoxicosis) which often transient.
A radio iodine uptake scan usually shows minimal Zero uptake. The condition treated steroids
and/or beta blockers. 1 )
8.
A 45 year old man a past history of multiple sclerosis was admitted hospital
following an overdose of baclofen. He was diagnosed relapsing remitting multiple
sclerosis 20 years ago usually mobile 2 sticks. He performs intermittent self
catheterisation his only medication baclofen 20mg three times a day.
He was found surrounded by empty packets previously containing baclofen by his son
after returning from a night out his friends. Earlier evening, patient his partner had an
argument this was thought have precipitated his actions. According his partner, there
were approximately 20 tablets left in packet, each containing 10mg of baclofen. (150mg of
baclofen associated severe toxicity).
He a non smoker tea total. The only other past history of note a previous admission 18
months ago severe community acquired pneumonia, which he needed mechanical
ventilation.
On examination, he was drowsy a respiratory rate of 5 breathes per minute. He had a
Glasgow coma scale (GCS) of 8/15 (Eye=2, Verbal=2, Motor=4) neurological examination
generalised hyporeflexia. Pulse rate was 60 beats per min pressure was 95 systolic 60
diastolic. Examination of respiratory, cardiovascular abdominal system was unremarkable.
His arterial gases on 50% inspired O2 were as follows:

pH
PO2

7.34
24.0 kPa
139

PCO2
HCO3
Base excess
What would be
1
2
3
4
5

)
)
)
)
)

7.2 kPa
27 mmol/L
0.3
your next step in management of this patient?

Increase concentration of inspired oxygen


Intravenous doxapram infusion
Intubation mechanical ventilation
Non invasive positive pressure ventilation (NIPPV)
Reduce concentration of inspired oxygen

Comments:
His CO2 retention likely be due CNS depression reduction in diaphragmatic contraction
secondary baclofen toxicity. Onset of toxicity rapid its effect can last up 35 40 hours post
ingestion. Features include drowsiness, coma, respiratory depression, hyporeflexia, hypotonia,
hypothermia hypotension. Bradycardia first degree heart block prolongation of Q T interval can
occur. Treatment usually supportive often requires intensive care. The presence of hypoxic
drive unlikely in view of patient a non smoker thus a reduction in inspired oxygen would
increase his respiratory drive. NIPPV would be an option but in a patient a GCS 8/15. Patients a
high risk of aspiration pneumonia are a contraindication non invasive ventilation. Doxapram
place in treatment of baclofen toxicity. 3 )
9.
A 47 year old man presented Accident & Emergency department following sudden
onset left posterior auricular pain while in his garage working under his car. On
examination he had a degree of neck pain stiffness also impaired coordination of left
hand weakness increased tone in his right limbs.
What is the likely diagnosis?
1
2
3
4
5

)
)
)
)
)

Cervical dislocation
Migraine
Subarachnoid haemorrhage
Tension headache
Vertebral artery dissection

Comments:
The typical presentation of vertebral artery dissection a young person (average age 40 years)
severe occipital headache neck pain following a recent head neck injury. The trauma often
trivial, but usually associated some form of cervical distortion.
About 85% of patients develop focal neurological signs due ischemia of brain stem cerebellum.
The commonest neurological manifestations are symptoms attributable lateral medullary
dysfunction (i.e. Wallenbergs syndrome).
Common symptoms & signs include:
Ipsilateral facial pain and/or numbness (the common symptom)
Vertigo (very common)
Dysarthria hoarseness (CN IX X)
Ipsilateral limb trunk numbness (cuneate gracile nuclei)
Ipsilateral loss of taste (nucleus tractus solitarius)
Hiccups
140

Vertigo
Nausea vomiting
Diplopia oscillopsia (image movement experienced head motion)
Dysphagia (CN IX X)
Depending upon which areas of brain stem cerebellum are affected, clinical signs may include:
Limb truncal ataxia
Nystagmus
Ipsilateral Horner syndrome (up 1/3rd patients affected)
Ipsilateral impairment of fine touch proprioception
Contralateral impairment of pain thermal sensation in extremities (ie, spinothalamic tract)
Contralateral hemiparesis
Lateral medullary syndrome
Tongue deviation side of lesion (impairment of CN XII)
Internuclear ophthalmoplegia (lesion of medial longitudinal fasciculus)
Risk factors associated development of vertebral artery dissection include: judo; yoga; ceiling
painting; nose blowing; minor neck trauma; chiropractitioner manipulation; hypertension; oral
contraceptive use; sex. 5 )
10. A 44 year male attends a health check at a mobile cardiovascular risk assessment
clinic. He takes medication but leads a sedentary lifestyle. He a non smoker family
history reveals his father had an MI at 60 years old.
Investigations reveal:
Total cholesterol
5.0 (NR < 5.5)
Triglycerides
4.0 (NR 1.0 2.2)
Which of following commonest cause of an isolated hypertiglycerdemia?
1 ) Alcohol
2 ) Diabetes Mellitus
3 ) Drug Therapy
4 ) Familial Hyperlipidaemia
5 ) Obesity
Comments:
The commonest cause of a mild hypertriglyceridaemia obesity secondary a reduced efficacy of
lipoprotein lipase activity overproduction of VLDL. Obesity (defined as a BMI above 30) present
in approximately 20% of subjects in UK rising hence this why it commonest cause of
hyperlipidaemia. Alcohol probably a close second. Other secondary causes of
hypertriglyceridaemia include pregnancy, hypothyroidism, diuretics pancreatitis. 5 )
11. A 42 year presented acute back pain after lifting a chair. The pain intense,
increases on movement, radiates bilaterally hypochondriacal region. Upon questioning
she denied previous fractures, but admitted a gradual loss of height (5cm from her young
adult height) occasional self limiting back pain.
Past history included spontaneous menopause at age of 37. She had never taken any
regular medications in past, calcium Vitamin D supplements.
Examination a thin woman who had a dorsal hyperkyphosis. Severe back pain was ellicited
on movement local percussion.
Investigations: Bone mineral density (BMD) was assessed by dual energy xray
absorptionmetry at hip lumber spine (L1 L4).
141

BMD T score at lumber spine:


3.0
BMD T score at total hip:
2.8
What is the diagnosis?
1 ) Osteopaenia
2 ) Osteomalacia secondary Vitamin D calcium deficiency
3 ) Post menopausal osteoporotic vertebral fractures
4 ) Senile osteoporosis
5 ) Vertebral fractures secondary malignant infiltration
Comments:
This lady several risk factors post menopausal osteoporosis, including early onset (<45 years)
menopause, absence of hormone replacement therapy, calcium Vitamin D supplemention low
body weight. Patients osteoporosis may have warning signs until first fracture occurs. Gradual
height loss dorsal kyphosis may result from microfractures complete fractures of vertebral
bodies.
A T score refers difference in standard deviation (SD) from mean bone mass. The World Health
Organisation (WHO) criteria diagnosis of osteoporosis as follows:
Diagnosis
T Score
Normal, > = 1.0
Osteopaenia
1.0 2.5
Osteoporosis
< = 2.5
Causes of secondary osteoporosis such as osteomalacia, bone metastasis, myeloma must be
excluded when a vertebral fracture found. 3 )
12. A 52 year old woman referred by an orthopaedic surgeon advice following a Colles
fracture 8 weeks ago. At time of her fracture, radiologist had reported significant
osteopenia. A dual energy x ray absorptiometry (DEXA) scan was carried out her T score
was 2.6 at hip 1.9 at lumbar spine.
She smokes approximately 15 cigarettes per day a body mass index of 21 kg/m2. She
post menopausal 2 years, unaware of any menopausal symptoms had a benign breast
lump removed 18 months ago. She currently taking aspirin, atenolol glyceryl trinitrate
(GTN) spray her angina, which she only uses occasionally.
What would be appropriate treatment?
1
2
3
4
5

)
)
)
)
)

Calcitonin
Calcium Vitamin D supplements
Hormone replacement therapy
Raloxifene
Risendronate

Comments:
Bisphosphonates (e.g. alendronate risendronate) act as a potent inhibitors of bone resorption by
decreasing osteoclast recruitment, activity, life span. Treatment bisphosphonates shown
significantly increase bone mineral density (BMD) in osteoporotic patients thus reduce fractures.
Problems adverse effects (mainly GI upset) can be reduced by a weekly administration of
bisphosphonates. It drug of choice treatment of osteoporosis. Calcium Vit D supplements are
more likely benefit women who are >5 years post menopause as their intake likely be low.
142

Postmenopausal women who wish reduce risk of osteoporosis should consume 1000 1500 mg
of elemental calcium 400 800 IU of vitamin D daily, ideally through calcium containing foods.
Excessive intake of calcium vit D may cause adverse effects such as hypercalcaemia
hypercalciuria. The use of Hormone replacement therapy (HRT) controversial should be
reserved those menopausal symptoms without overt cardiovascular disease as studies suggest
increased cardiovascular risk although it still a good agent in preventing fractures. 5 )
13. A 30 year old man presented a painless visual loss of his left eye over 24 hours.
Over previous two weeks he had lost vision in his right eye. There had any improvement
of his visual acuity over next 2 months. He denied any other symptoms.
His previous history was unremarkable. He smoked 10 cigarettes per day drank 20 units
of alcohol per week. There was relevant family history. He did take any drugs.
Physical examination was unremarkable. His visual acuity was 6/60 on right finger
counting on left. His colour vision iwa impaired in both eyes. He had bilateral optic
atrophy. The remainder of his neurological examination was normal.
Investigations showed:
Full count
Normal
Serum urea & electrolytes
Normal
Liver function tests
Normal
TFTs
Normal
Serum calcium
Normal
Serum glucose
Normal
Autoantibody screen
Normal
VDRL
Negative
Serum vitamin B12
Normal
Chest x ray
Normal
MRI brain orbits
Normal
CSF analysis
Unremarkable
What is the likely diagnosis?
1 ) Multiple sclerosis
2 ) Alcohol/tobacco induced optic neuropathies
3 ) Lebers optic atrophy
4 ) Giant cell arteritis
5 ) Glaucoma
Comments:
Lebers optic atrophy usually affects young men. It causes sequential optic neuropathies in days
weeks. It typical painless severe. Visual acuity fails improve. Optic neuritis usually painful,
visual acuity improves over a matter of weeks. Giant cell arteritis affects elderly patients.
Alcohol/Tobacco optic neuropathies are usually chronic. 3 )
14. 60 year old woman presented 3 months history of diplopia blurred vision of left eye.
She denied any pain other neurological symptoms. Her previous history unremarkable.
She smokes 20 cigarettes per day drinks alcohol in moderation.
143

Her general examination normal. Her visual acuity on right 6/6 on left 6/36. There left
partial ptosis mild proptosis conjunctival injection. The left pupil smaller than right but
reacting normally light. There some limitation of abduction of left eye. Fundoscopy
showed a pale left optic disk. The left corneal reflex reduced. The remaining of
neurological examination normal.
Routine tests including full count, urea electrolytes, liver function tests, thyroid function,
serum calcium, serum creatine kinase, autoantibody screen were normal. Her
electrocardiogram chest radiograph showed abnormalities.
Slit lamp examination was intra ocular pressures were within range.
Where likely cause of her symptoms?
1 ) Cavernous sinus
2 ) Superior orbital fissure
3 ) Orbital apex syndrome
4 ) Optic chiasm
5 ) Brain stem
Comments:
The combination of optic neuropathy, proptosis, chemosis, horners syndrome, ophthlmoplegia
(in this case due 6th nerve palsy), involvement of first branch of trigeminal nerve typical of
orbital apex syndrome. Proptosis chemosis are useful signs distinguish orbital apex syndrome
from cavernous sinus pathology. 3 )
15. A 23 year presents her GP problems related hay fever. He prescribes her an anti
histamine which caused her problems drowsiness. Which one of following anti
histamines likely cause sedation?
1 ) Cetirizine
2 ) Cyproheptadine
3 ) Fexofenadine
4 ) Loratadine
5 ) Terfenadine
Comments:
All antihistamines are potentially sedating but newer agents are said be less sedating than
older types such as chlorpheniramine cyproheptadine. Cyproheptadine can also be used in
treatment of migraine. Terfenadine (a pro drug) associated cardiac arrhythmias (torsades de
pointes) especially in individuals prolonged QT intervals. Fexofenadine active metabolite of
terfenadine do appear have same arrhythmogenic effects as terfenadine. 2 )
16. A 22 year old nursery school teacher referred clinic by her GP. She gives several
weeks history of swollen, tender finger joints. She past history of note entirely well
apart from a recent viral infection. She had significant travel history. The only sick
contacts she could recall were several of children in her class who had recently off sick a
mild self limiting illness consisting of a fever, rhinorrhoea a rash.
On examination metocarpo phalangeal joints proximal interphalangeal joints of both
hands are swollen tender.
What is the likely diagnosis?
1 ) Adult Stills disease
2 ) Lyme disease
3 ) Parvivirus B19 infection
4 ) Rheumatoid arthritis
144

5 ) Systemic lupus erythematosus


Comments:
Parvovirus B19 a single strand DNA virus. The widely known clinical manifestation of parvovirus
B19 erythema infectiosum (slapped cheek syndrome), a mild viral illness of childhood
characterized by a classic exanthem in which both cheeks appear bright red as though they had
slapped.
Although usually a benign self limiting viral illness, parvovirus B19 infection may have more
serious sequelae. The virus a tropism rapidly dividing erythrocyte precursors which they infect
destroy. Thus, reticulocytes (immature erythrocytes) are available replace aging damaged
erythrocytes as they are cleared by reticuloendothelial system. This may have any significant
impact on otherwise healthy individuals, but can trigger an aplastic crisis particularly in
patients haemoglobinopathies.
Parvivirus B19 infection may also be associated a symmetrical postinfectious arthritis, affecting
small joints of hands feet. The knees elbows are rarely involved. The arthritis much more
common in adults, particularly in women, may persist weeks months (even years in a small
number of patients). The arthritis may mimic rheumatoid arthritis. Unlike rheumatoid arthritis,
postinfectious arthritis associated parvovirus B19 does cause permanent damage bones
joints. 3 )
17. Which one of following conditions associated Autosomal Dominant Polycystic
Kidney Disease (APKD)?
1
2
3
4
5

)
)
)
)
)

Aortic stenosis
Dissecting aortic aneurysm
Hypertension
Mitral valve prolapse
Subarachnoid haemorrhage

Comments:
Hypertension a common, early finding in patients ADPKD, should be managed aggressively
standard antihypertensive medications. Intracerebral aneurysms are found in up 2% of patients
ADPKD. These may lead subarachnoid intracerebral haemorrhage, severe of complications
associated ADPKD. Mitral valve prolapse found in 20% of patients ADPKD. There an increased
incidence of aneurysms arterial dissections in patients ADPKD.
Aortic stenosis associated ADPKD. 1 )
18. A 70 year admitted acutely unwell. Six weeks prior admission she had presented
GP tiredness weight loss had diagnosed hypothyroidism based on results which
showed:
T4
7.8 nmol/L
(NR 9 22)
TSH
4 mu/L
(NR 0.5 4.5)
She was treated thyroxine 100 micrograms daily since deteriorated. She other past
history of note, does smoke drinks modest quantities of alcohol. She a widow self
caring. Her mother had hypothyroidism.
On examination, she drowsy, thin, a temperature of 37.5oC, a pulse of 98 beats per minute
a pressure of 84/50 mmHg. Cardiovascular, respiratory abdominal examination are
otherwise normal. There are neurological abnormalities.
145

The house officer sent some emergency bloods on this patient. Whilst awaiting results,
What is the appropriate immediate treatment this patient?
1 ) IV 50% Dextrose
2 ) IV Cefotaxime
3 ) IV Hydrocortisone
4 ) IV Thyronine (T3)
5 ) IV Thyroxine (T4)
Comments:
The specific features in this case are past history of weight loss associated a low T4 but TSH.
The latter would suggest either sick euthyroidism which can be associated hypoadrenalism
secondary hypothyroidism. In hypoadrenalism which either primary secondary, addition of
thyroxine can precipitate acute hypoadrenalim which appears be case here. The use of IV
steroids therefore life saving should be given without hesitation. 3 )
19. A 72 year male being treated hypertension, gout, Gastro oesophageal reflux a
three year history of type 2 diabetes. He takes a variety of medications. His general
practitioner concerned after requesting serum biochemistry on this patient.
These investigations have revealed:
Serum sodium
138 mmol/L
Serum potassium
4.4 mmol/L
Serum urea
12.8 mmol/L
Serum creatinine
162 micromol/l
Of following drugs he takes, which ones dose does NOT need be reduced this patient?
1
2
3
4
5

)
)
)
)
)

Allopurinol
Gliclazide
Lansoprazole
Lisinopril
Metformin

Comments:
Allopurinol useful in renal impairment, but dose should be reduced from 300mg/day
100mg/day in moderate severe renal impairment, as toxicity may occur, leading
hypersensitivity rashes hepatitis.
Gliclazide dosage should be reduced in mild renal failure, should be stopped in severe renal
disease.
Lansoprazole safe use in renal impairment (caution in liver impairment), at a dose of 15
30mg/day.
Lisinopril should be used caution in renal impairment. It may potentiate hyperkalaemia
hypotension, therefore dose should be reduced 10 20mg/day, rather than 20 40mg/day.
Metformin predisposes lactic acidosis, therefore should be stopped even mild renal impairment
general advice suggests stopping a creatinine above 150. 3 )
20. A 32 year woman seen in outpatient clinic three weeks after receiving a cadaveric
renal transplant. She currently taking prednisolone ciclosporin. Three days ago her serum
146

urea electrolytes were normal. She says she feels well. Repeat serum urea electrolytes
show:
Serum
Serum
Serum
Serum

sodium
potassium
urea
creatinine

139 mmol/L
3.9 mmol/L
11 mmol/L
168 umol/L

The renal transplant was HLA matched. The patient was CMV IgG negative; kidney donor
was CMV IgG positive.
Which of following best accounts change in renal function?
1 ) Acute cellular rejection
2 ) Ciclosporin toxicity
3 ) CMV infection
4 ) Dehydration
5 ) Pyelonephritis of transplanted kidney
Comments:
This patient had a sudden deterioration in renal function, three weeks following an
uncomplicated renal transplant. Despite this, she clinically well, symptoms.
This lady acute cellular rejection. Approximately 25% of transplant patients will have at least
one episode of rejection mostly between day 7 21, less commonly up three months post
operation. It often clinically silent, only a sharp rise in serum creatinine pointing towards
diagnosis. Doppler ultrasound studies may show a sharp deterioration in graft perfusion, kidney
biopsy will show invading lymphocytes penetrating tubular basement membrane, causing
tubilitis. Treatment IV bolus of high dose steroids. Long term graft function will be
compromised if rejection episode completely reversed.
This lady CMV seronegative receied a graft from a CMV seropositive donor. Although CMV
infection would cause a sudden deterioration in her renal function (as functioning kidney
CMV+), this lady does have a three five fold increased risk of developing severe CMV infection
while immunosuppressed. Typical manifestations of infection would include interstitial
pneumonitis, oesophagitis, peptic ulceration, colitis retinitis. CMV infection associated
increased graft rejection renal artery stenosis in renal transplant recipients. Because she well,
early onset of findings CMV+ transplant, this option can be discounted.
Ciclosporin a calcineurin inhibitor which a potent immunosuppressant nephrotoxin.
Ciclosporin can cause a dose dependent increase in urea creatinine in first few weeks of taking,
also long term graft failure. This probably related total amount of ciclosporin taken. There
evidence in question state dose recently increased, therefore this answer can be discounted.
This lady well there suggestion of intercurrent disease causing dehydration.
Pyelonephritis of transplanted kidney a particular problem in early immunosuppressed period.
Pyelonephritis would present low grade pyrexia, tender, swollen kidney deteriorating graft
function. This lady well, evidence of infection. 1 )
21. A 80 year presents A+E acutely unwell after being found unresponsive on floor of
her house by her son. She had a past history of hypothyroidism taking thyroxine daily.
However, her compliance treatment questionable.
147

On examination she was unrouseable a Glasgow Coma Scale of 6/15, had a central
temperature of 34oC, Oxygen saturations of 95% on air, a pulse of 44 beats per minute a
pressure of 100/80 mmHg. There were specific localising signs on neurological
examination but both plantars were extensor.
Prior results of her emergency tests being available, What is the appropriate immediate
treatment this patient?
1
2
3
4
5

)
)
)
)
)

Intravenous 50% Glucose


Intravenous Hydrocortisone
Intravenous Normal Saline
Intravenous Thyronine (T3)
Oral thyroxine via NG tube

Comments:
This patient typical features of Myxoedema coma which usually occurs in elderly who are
typically non compliant. It carries a high mortality should initially be treated IV thyroid hormone
either T4 T3 even before results are obtained. Steroids are often employed, in case of
associated hypoadrenalism but in this case would probably be immediate choice. Other
treatment includes rewarming. IV fluids should be used cautiously as these patients are typically
fluid overloaded. 4 )
22. A 47 year old lady presented two weeks after returning from a holiday in Caribbean
complaining of an intensely pruritic rash on her abdomen.

What causative organsim?


1)
Ancylostoma braziliense
2)
Chironex fleckeri Southcott (Box Jelly Fish)
3)
Borrelia burgdorferi
4)
Onchocerca volvulus
5)
Treponema pallidum
Comments:
The figure shows cutaneous larva migrans (creeping eruption). Usually caused by an animal
hookworm, commonly Ancylostoma braziliense. The infection acquired by direct contact dog
cat faeces often acquired when sunbathing on contaminated sand, etc. The larvae burrow in
dermo epidermal junction. Symptoms include pruritis a raised, serpiginous erythematous rash
148

migrates at a rate of up 1 cm/day. Treatment topical thiobendazole, oral albendazole. Acute


infection human nematodes Strongyloides stercoralis, Necator americanus Ancylostoma
duodenale, may produce a similar appearance. (Further reading Archives of Dermatology) 1 )
23. A 45 year old man presents an episode of loss of consciousness. He was fully alert
on arrival at Emergency Medicine Department a 12 lead electrocardiogram was taken.

Which two of the following electrocardiographic features are present in ECG?


1)
Acute myocardial injury
2)
Early repolarisation
3)
First degree heart block
4)
Prolonged QT interval
5)
Second degree heart block
6)
Third degree heart block
7)
Torsade de pointes VT
8)
Ventricular escape beats
9)
Ventricular pre excitation
10 ) Ventricular premature beats
Comments:
The ECG shows a long QT ventricular premature beats. The loss of consciousness may have due
ventricular arrhythmia, in particular, Torsade de pointes VT. This man warrants further
investigation.
4 ) 10)
24. A 32 year old man presented hospital a four week history of progressively
worsening dyspnoea on exertion. He also complained of a non productive cough. Over two
days preceeding admission patient had become breathless at rest was started on oral co
amoxiclav by his general practitioner.
On examination he was febrile 38C looked unwell. Candida was noted on tonsilar pillars.
No wheeze crackles were heard in his chest. His chest radiograph shown. Oxygen
saturation was 95% on room air, but fell 85% following about of coughing. Arterial gases
show pO2 of 59 mmHg.
149

What treatment shold be given?


1)

Co amoxiclav + clarithromycin
2)
Co trimoxazole + prednisolone
3)
Vancomycin + ceftazidime
4)
Cefuroxime + metronidazole
5)
Benzylpenicillin + flucloxacillin
Comments:
Oral candidiasis never should always raise possibility of AIDS history in this case characteristic of
Pneumocystis carinii pneumonia. There are several agents can be used treat PCP, though co
triomoxazole clearly effective treatment of first choice. Patients often deteriorate after starting
therapy PCP as pneumonitis worsens due inflammation associated dying pneumocysts. In patients a
pO2<60 mmHg, oral prednisolone added reduce inflammatory effect. 2 )
25.
A 35 year old woman brought casualty by her husband. She drowsy confused. There a
three day history of malaise low grade fever, but she become increasingly confused over past
eight hours apparently drifting in out of conciousness. On examination she drowsy but rousable.
GCS 8/15. There are focal neurological signs. While in Emergency Unit she a generalized tonic
clonic seizure. An MRI scan of her brain shown.

150

A lumbar puncture was performed analysis of cerebrospinal fluid revealed:


Appearance
Clear, colourless
Cells
500 mm3 (>90%)lymphocytes
Protein
0.7 g/L
Glucose
4.5 mmol/L
Plasma glucose was measured as 5.3 mmol/L
Which of following investigations likely reveal correct diagnosis?
1)
Blood cultures
2)
CSF HSV PCR
3)
CSF Cryptococcal neoformans
4)
Magnetic resonance angiogram
5)
Serum viral titres
Comments:
This patient's MRI presentation typical of Herpes simplex encephalitis. The MRI demonstrates a temporo
parietal dense lesion. The diagnosis reliably confirmed by presence of HSV in CSF by PCR. Read more
about HSE 2 )

1.
A 62 year old lady was seen in outpatient department a six month history of
increasing breathlessness on exertion. Her exercise tolerance was limited 80 metres on
flat she had started become breathless walking up stairs in her house. She slept four
pillows had noticed some swelling of both her ankles. She had a cough which was
occasionally productive of sputum. She had given up smoking when she first noticed her
dyspnoea had a 40 pack year smoking history. She was known have hypertension
ischaemic heart disease had coronary artery stenting 12 months ago.
151

Pulmonary funtion testing revealed:


FEV1
0.82 L(1.80 3.02 predicted)
FVC
1.84 L (2.16 3.58 predicted)
Diffusion capacity
2.40 mmol/min/kPa(5.91 9.65 predicted)
Total Lung Capacity
4.40 L(4.25 - 6.22 predicted)
Residual Volume
2.69 L(1.46 - 2.48 predicted)
What is the diagnosis?
1
2
3
4
5

)
)
)
)
)

Asthma
Chronic Obstructive Pulmonary Disease
Cryptogenic Fibrosing Alveolitis
Left Ventricular Failure
Sarcoidosis

Comments:
This lady a history consistent chronic obstructive pulmonary disease(COPD)
<Paedia.asp?id=214>. The diagnosis confirmed on lung function; she airways obstruction a
reduced FEV1 FEV1/FVC. She a total lung capacity but her residual volume increased
indicating a degree of air trapping. Finally her Diffusion capacity (transfer factor) decreased
which helps differentiate COPD from asthma. 2 )
2.
A 60 year old man was diagnosed motor neuron disease bulbar involvement. What
median survival time from onset of symptoms?
1
2
3
4
5

)
)
)
)
)

Death in 12 months
Death in 3 5 years
Death in 5 10 years
Death in 10 15 years
Variable prognosis

Comments:
Motor neuron disease a progressive neurodegenerative disease involving corticospinal,
corticobulbar tracts anterior horn cell. The median survival time from onset of symptom 3 5
years. However, some patients survive a longer time (mainly primary lateral sclerosis). Poor
prognostic factors include low FVC older age. 2 )
3.
A 79 year old lady presents casualty fast atrial fibrillation dyspnoea. She had a VVI
permanent pacemaker implanted one year previously sick sinus syndrome after she
presented syncope. She adequately anticoagulated referred DC cardioversion.
Which statement is correct?
1 ) Cardioversion will affect her pacemaker
2 ) Cardioversion contraindicated because of pacemaker in situ
3 ) Her VVI pacemaker should be upgraded a dual chamber system
4 ) Pacemaker function should be checked after cardioversion antiarrhythmic therapy added 5 )
She should be treated antiarrhythmic therapy cardioversion should be cancelled
Comments:
152

This lady sick sinus syndrome (tachy brady syndrome). She a VVI pacemaker in situ prevent
bradycardia induced syncope but she presented symptomatic fast AF. The aim treatment
initially cardiovert this patient. DC cardioversion contraindicated in patients pacemakers,
although it prudent have pacemaker function settings rechecked after cardioversion.
Subsequently patient should be treated antiarrythmic medication prevent recurrences of fast
AF. Dual chamber pacing would prevent AF in this case. 4 )
4.
A 20 year old male was found by police in an acutely confused state in early hours
of morning. He was taken local accident emergency department where he a frequent
attender a history of schizophrenia deliberate self harm. He found be have dilated pupils
was somewhat agitated visual hallucinations a sinus tachycardia of 110 beats per minute
on electrocardiogram.
He later told nursing staff he had taken some acid of unknown quantity because the
voice had told him to, so he can be protected against other people poisoning him. He was
observed overnight in hospital where became less agitated his heart rate had returned
normal. Prior discharge, he was reviewed by on call psychiatric team his anti psychotic
medication was changed olanzapine.
A week later, his mother found him semi conscious in his flat. His admission
electrocardiogram showed a sinus tachycardia of 115 beats per minute, his pupils were
dilated he had a temperature of 39C. His muscle tone was globally increased his reflexes
brisk throughout equivocal plantars. He had a palpable bladder was catheterised.
Investigations showed:
Haemoglobin
14.5 g/dL
White cell count
8.4 x 109/L
Platelets
310 x 109/L
Serum sodium
145 mmol/L
Serum potassium
5.5 mmol/L
Serum urea
32.3 mmol/L
Serum creatinine
410 mol/L
Serum chloride
108 mmol/L
Serum bicarbonate
15 mmol/L
Urine dipstick
Protein ++ Blood +++
What is the likely cause his biochemical abnormalities?
1
2
3
4
5

) Amphetamine overdose
) Neuroleptic malignant syndrome
) Rhabdomyolysis
) Serotonin syndrome
) Urinary tract infection

Comments:
Rhadomyolysis can be due prolonged immobilisation on a hard surface, prolonged muscle
rigidity (e.g. seizures, hyperthermia) a specific syndrome such as neuroleptic malignant
syndrome serotonin syndrome.

153

In this case, a drug overdose such as amphetamine likely cause. A markedly elevated serum
creatinine kinase would confirm diagnosis of rhabdomyolysis. Myoglobinuria would manifest as
haematuria on routine urinary dipstick test.
Neuroleptic malignant syndrome associated fever rare newer anti psychotic drugs such as
olanzapine. 3 )
5.
An 84 year old lady admitted hospital following a collapse at home transient loss of
consciousness. On examination she appears well. Her pulse measured at 48 beats per
minute a pressure of 110/65 mmHg. Her electrocardiogram on admission shows a sinus
bradycardia of 45 beats per minute acute ischaemic changes.
What is the appropriate next step in her management?
1 ) Administer intravenous atropine
2 ) Admit arrange a 24 hour tape as an inpatient
3 ) Admit insert a temporary pacing wire
4 ) Carotid sinus massage
5 ) Observe overnight arrange a 24 hour tape as an outpatient
Comments:
This patient likely have bradycardia induced syncope. The diagnosis however certain.
Therefore observation as an inpatient suitable heart rate monitoring best answer. Outpatient
observation appropriate given syncopal episode. The patient cardiovascularly stable therefore
a temporary wire, atropine carotid sinus massage are required. 2 )
6.
A 17 year old learning difficulties intractable epilepsy presented 24 hours of
confusion agitation. She was taking sodium valproate, phenytoin carbamazepine, but her
compliance was very poor.What is the appropriate treatment?
1 ) Intravenous acyclovir
2 ) Intravenous antibiotics
3 ) Intravenous diazepam
4 ) Intravenous thiamine
5 ) Psychiatric referral
Comments:
The clinical history suggests non convulsive status. It a common presentation in patients
intractable epilepsy poor compliance antiepileptic treatment. It usually presents as a
confusional state. Intravenous benzodiazepine treatment of choice. 3 )
7.
A 24 year old lady was referred your renal outpatient clinic from her general
practitioner. She presented him tiredness. She was taking any medications. On
examination, her pressure was measured at 180/95 mmHg.
Routine investigations by general practitioner showed:
Serum sodium
140
Serum potassium
3.0
Serum urea
7.0
Serum creatinine
92 umol/L
She was admitted hospital further investigations, which showed:
Plasma renin activity (recumbent)
5.2 pmol/ml/h
(NR 1.1 2.7)
Serum aldosterone (supine after 30 minutes) 900 pmol/L (NR 135 400)
Which of following correct diagnosis?
1 ) Barrters Syndrome
154

2
3
4
5

)
)
)
)

Conns Syndrome
Essential hypertension
Fibromuscular dysplasia
Liquorice ingestion

Comments:
This lady hypertension, associated a low serum potassium, high plasma renin serum
aldosterone activity.
Barrters syndrome usually presents in childhood severe muscle weakness. It a salt wasting
state, which due a defect in loop of Henle chloride reabsorption (at Na+ K+ 2Cl
cotransporter). Patients have low pressure severe hyperreninaemia.
Conns syndrome caused by an adrenal adenoma secreting aldosterone. The clinical features
include hypertension, hyperaldosteronism, hypokalaemia hyporeninaemia.
Essential hypertension associated endocrine electrolyte abnormalities. In patients younger
than 35 hypertension, an endocrine cause should be excluded. 4 )
8.
Fibromuscular dysplasia, a rare cause of hypertension hypokalaemia more common
in women. It causes hyperreninaemic hyperaldosteronism.
Liquorice ingestion causes a primary aldosterone type picture. It caused by glycyrrhizic
acid contained in liquorice, blocking enzyme 11b hydroxysteroid dehydrogenase. This
prevents inactivation of cortisol, which in turn activates mineralocorticoid receptors in
kidney.
You are asked advice by a young professional couple, Mr Mrs X. Mrs X 9 weeks pregnant.
Mr X's brother his partner had a child cystic fibrosis. As a result, Mr X was screened
found carry DF508 mutation cystic fibrosis. Mrs X declines be tested. What are chances
of Mr Mrs X's child having cystic fibrosis, given gene frequency this mutation in general
population 1/20.
1 ) 1/4
2 ) 1/20
3 ) 1/40
4 ) 1/80
5 ) 1/160
Comments:
The chance of Mrs X being a carrier of gene 1/20. The chances of two carriers of a gene having
a child homozygous disease (i.e. both genes are transmitted child) . Therefore, chances of
this couple having a child CF are x 1/20 = 1/80. 4 )
9.
A 68 year old lady attended rheumatology clinic, review of her rheumatoid arthritis.
She was found have symptoms signs of active synovitis, elevated inflammatory markers.
She had taking IM sodium aurothiomalate 50mg once a week previous six months. Other
medications included diclofenac 50mg three times a day.Investigations showed:
Serum sodium
138 mmol/L
Serum potassium
4.9 mmol/L
Serum urea
12 mmol/L
Serum creatinine
290 umol/L
Urine dipstick
Protein ++ Blood ++
24 hr urine collection
0.4g protein/24hr(NR <0.2g/24hr)
155

Her renal function was at her last clinic visit two months previously.
Which of following likely cause of deterioration in renal function?
1 ) Amyloid
2 ) Gold nephropathy
3 ) Interstitial nephritis
4 ) Myeloma
5 ) Vasculitis
Comments:
This lady longstanding rheumatoid arthritis treated gold. She currently an exacerbation of her
symptoms.
She renal impairment mild proteinuria haematuria.
Amyloid common in patients longstanding rheumatoid arthritis. The fibrils are amyloid A
protein. It presents proteinuria often nephrotic range.
Gold nephropathy causes proteinuria, which often in nephrotic range. Haematuria rare, as an
abnormal renal function.
Interstitial nephritis may account changes seen in this question. The patient taking Ibuprofen.
NSAIDs may cause a reversible reduction in glomerular filtration rate, acute tubular necrosis,
acute interstitial nephritis often heavy proteinuria, renal papillary necrosis chronic
tubulointerstitial nephritis.
Myeloma could account changes seen, but there evidence of this in question.
Vasculitis a rare cause of renal impairment in rheumatoid arthritis. The clinical presentation in
rheumatoid includes nail fold infarcts, a leucocytoclastic vasculitis, a peripheral neuropathy,
pericarditis, GI infarcts renal vasculitis. Renal abnormalities are found in 25% of patients
rheumatoid vasculitis, usually presenting proteinuria, microscopic haematuria renal
impairment. As this cause less common than interstitial nephritis associated NSAIDs, it
correct answer. 3 )
10. A 45 year old man was referred hospital by his General Practitioner. Over a period of
three months he had developed a persistent progressively worsening wheeze. He brought
hospital by ambulance from his workplace where he became acutely breathless.
Two months previously his GP had tried treatment inhaled salbutamol, but little effect.
The patient was eventually forced take sick leave from his work as a solderer in a local
factory. While patients was off work, GP took opportunity intensifiy therapy, adding oral
prednisolone, good initial symptomatic improvement - patient felt well enough return
work within five days. Unfortunately, his symptoms returned shortly after returning work.
The patient requested steroids be withdrawn as he did consider they had improved his
symptoms: they were tapered stopped.
The patient then took two weeks annual leave travelled his family on holiday Spain two
weeks. During vacation he found his wheeze improved significantly. Two days after
returning work his colleagues call an ambulance because he an acute asthma attack in
work brought A&E unit.
He a non smoker does take any regular medication.
On examination he dyspnoeic at rest but able speak in short sentences. Respiratory rate
22/minute. There are widespread inspiratory expiratory polyphonic wheezes throughout
both lungs. BP 130/76 detectable paradox. He afebrile. The rest of examination
unremarkable.He treated nebulized salbutamol makes a significant improvement within
48 hours.
Investigations reveal:
156

On admission:

Haemoglobin
15.0 g/dL
White cells
9 x 109/L
Platelets
395 x 109/L
Serum sodium
143mmol/L
Serum potassium
4.2 mmol/L
Serum urea
2.8 mmol/L
Serum creatinine
70 mmol/L
PEFR
50l Lmin
48h after admission
PEFR
650 L/min
Which of following investigations should be performed?
1
2
3
4
5

)
)
)
)
)

Bronchoscopy bronchoalveolar lavage


CT scan of thorax
Full pulmonary function tests
Lung biopsy
Peak flow rates measured at home in work

Comments:
The history highly suggestive of occupational exposure an allergen, significant symptomatic
improvement when away from his work environment. 5 )
11. A 19 year old girl was found by her father semi conscious in her bedroom. She was
surrounded by empty packets of his medication which consisted of digoxin aspirin. There
was also evidence she may have co ingested a large amount of alcohol. She was last seen
8 hours previously her father estimated she could have taken tablets at anytime since
then.
On arrival hospital she had a Glasgow Coma Score of 13/15, a pulse rate of 40 beats per
minute, pressure 80mmHg systolic 50 mmHg diastolic. A 12 lead ECG showed a
bradycardia of 38 beats per minute a 2:1 heart block.
Investigations showed:
Serum sodium
140 mmol/L
Serum potassium
5.9 mmol/L
Serum chloride
98 mmol/L
Serum bicarbonate
20 mmol/L
Serum urea
9.2 mmol/L
Serum creatinine
130 umol/L
Plasma glucose
5.2 mmol/L
Digoxin level
8 nmol/L (therapeutic range 1 2 nmol/L)
Salicylate level
<10mg/dL
157

Full count arterial gases were normal.


She had a good initial response intravenous atropine, which transiently increased her
heart rate 60 beats per minute but her pressure remained low at <90mmHg systolic.
Shortly after intravenous atropine she started having intermittent episodes of broad
complex tachycardia.
What would be appropriate treatment this patient?
1
2
3
4
5

)
)
)
)
)

Bicarbonate infusion
Digoxin specific antibodies (Digibind)
Haemodialysis
Insertion of a temporary pacing wire
Insulin dextrose infusion

Comments:
Clinical features of severe digoxin poisoning include hyperkalaemia, metabolic acidosis, both
brady tachyarrythmia. Hypotension can occur due negative chronotropic effects of digoxin.
Digoxin level useful but an absolute guide toxicity.
Indications digoxin specific antibodies:
Severe hyperkalaemia (>6mmol/L) resistant treatment insulin dextrose (NOT calcium gluconate
- risk of further ventricular arrythmias)
Bradyarrythmia unresponsive atropine cardiac compromise
Tachyarrythmia (esp. ventricular) associated cardiac compromise
Digoxin specific antibodies should be considered at an earlier stage if patient pre existing
cardiac disease
In absence of digoxin specific antibodies, insertion of a temporary pacing wire may improve
heart rate but unlikely improve pressure due negative inotropic effects of digoxin. 2 )
12. A 75 year old man presented a sudden onset of right sided monocular painless
visual loss lasting 1 minute.
He described visual loss as a curtain descending across his vision. His known be
hypertensive. An ECG showed sinus rhythm. Doppler of his carotids showed right external
carotid artery stenosis of 90% right internal carotid artery stenosis of 50%.
What is the appropriate treatment?
1 ) Aspirin
2 ) No treatment
3 ) Right carotid angioplasty
4 ) Right carotid endarterectomy
5 ) Warfarin
Comments:
The clinical presentation typical of amaurosis fugax. Carotid endartectomy only indicated if
there >70% stenosis of symptomatic carotid artery. Aspirin will be appropriate treatment in
this case. Warfarin indicated as ECG showed sinus rhythm. 1 )
13. A 70 year old man presented a fall. His regular medications included tramadol 50mg
qds, aspirin 75mg od, paroxetine 20mg od, amlodipine 10mg od, isosorbide mononitrate
20mg bd. Initial investigations showed his serum sodium was 123 mmol/l his
urine/plasma osmolality ratio was 3.5.Which of medication likely cause of his problems?
1 ) Amlodipine
2 ) Aspirin
158

3 ) Isosorbide mononitrate
4 ) Paroxetine
5 ) Tramadol
Comments:
The low sodium result of syndrome of inappropriate ADH secretion. Drugs remain simple
commonest cause of this syndrome. Potential drugs include carbamazepine, chlorpropadine,
antidepressants, lithium, phenothiazines. 4 )
14. A 30 year old woman went motor racing holiday in France. She woke up severe pain
around right shoulder one morning. A few days later she developed weakness of right
arm. At same time pain started improve. She denied any injuries.
Two weeks before her holiday she had suffered from an upper respiratory tract infection.
On examination, she had weakness of right shoulder abduction, elbow flexion reduced
sensation over C5 and C6 dermatomes. The right biceps reflex was absent.
What is the likely diagnosis?
1
2
3
4
5

)
)
)
)
)

C5 radiculopathy
C6 radiculopathy
Left internal capsule lacunar infarction
Multiple sclerosis
Neuralgic amyotrophy

Comments:
The clinical history is typical of neuralgic amyotrophy. It usually proceeded by an upper
respiratory tract infection. Pain around shoulder presenting symptom usually very severe. As
pain starts resolving, weakness begins usually affects muscles innervated by upper brachial
plexus (C5 6). Treatment conservative.It is usually a self limiting condition (improvement over
weeks months).5)
15. A 45 year old woman was admitted hospital difficulty breathing. She was admitted
from a local restaurant after becoming suddenly unwell while eating curry. On arrival in
Casualty, her face lips were noted be grossly swollen there was an audible inspiratory
wheeze. There was past history of allergies. She had diagnosed hypertension by her GP
started on captopril ten days previously.
Investigations:
Serum IgE
>1000kU/L(NR <120)
Mast cell tryptase Normal after 30 minutes
What is the most likely cause of this reaction?
1
2
3
4
5

)
)
)
)
)

C1 inhibitor deficiency
Captopril induced angioedema
Hyperimmunoglobulinaemia E
Mastocytosis
Nut allergy

Comments:
Angiotensin converting enzyme inhibitors affect metabolism of eicosanoids polypeptides,
including endogenous bradykinin. Patients receiving ACE inhibitors (including captopril) may be
159

subject a variety of adverse reactions, some of them serious. Angioedema involving


extremities, face, lips, mucous membranes, tongue, glottis larynx seen in patients treated
ACE inhibitors, including captopril. Most cases occur within one week of starting therapy. If
angioedema involves tongue, glottis larynx, airway obstruction may occur be fatal. Emergency
therapy, including but necessarily limited to, subcutaneous administration of a 1:1000 solution
of epinephrine should be promptly instituted. 2 )

16. A 64 year man noted have hypertension (180/100 mmHg). He a history of a


myocardial infarction two years ago a background history of asthma.What
antihypertensive of choice this patient?
1
2
3
4
5

)
)
)
)
)

Angiotensin converting enzyme (ACE) inhibitor


Alpha blocker
Amlodipine
Beta blocker
Thiazide diuretic

Comments:
The antihypertensive medication of choice an ACE inhibitor. This drug clear morbidity
mortality benefits in both patients impaired left ventricular function. Beta blockers are
contraindicated asthma. The other medications do have wealth of clinical benefits ACE
inhibitors offer. 1 )
17. A 39 year old man was referred renal clinic. Two weeks previously he had an
episode of loin pain, passed a small renal stone. He had one previous episode three years
previously, but did seek advice. He was otherwise fit well.
Investigations showed:
Haemoglobin
14.5 g/dL
White cells
7.5 x 109/L
Platelets
210 x 109/L
Serum sodium
137 mmol/L
Serum potassium
4.2 mmol/L
Serum urea
6.1 mmol/L
Serum creatinine
100 umol/L
Serum corrected calcium
2.3 (NR 2.2 2.6)
24 hour urine collection
Volume
1150 ml/24hr
Calcium
18 mmol/24hr(NR 2.5 7.5)
Analysis of stone showed it contain mostly calcium.
He initially advised increase his fluid intake, but he returns your clinic after one month,
having had two further episodes.
Which of following medications would you prescribe?
1 ) Allopurinol
2 ) Loop diuretic
160

3 ) Potassium bicarbonate
4 ) Potassium citrate
5 ) Thiazide diuretic
Comments:
This gentleman had a calcium urinary tract stone. He serum calcium, but raised urinary
excretion of calcium. Idiopathic hypercalciuria often familial, common cause being increased
gastrointestinal absorption of calcium. The common stones are calcium oxalate stones.
In this patient, his urinary output 1150 mls/24hr. He should aim a daily urinary output in
excess of 2000 ml. The correct answer therefore advise him increase his fluid intake. A high
protein intake associated urinary stones, by reducing his dairy dietary intake he may reduce
his GI absorption of calcium, but main initial treatment increase his urine volume.
Allopurinol would reduce incidence of uric acid stones, but would affect formation of calcium
stones.
Potassium citrate potassium bicarbonate can be used alkalinise urine, prevent formation of
cystine containing stones. They would have effect on formation of calcium containing stones.
Thiazide diuretics reduce renal tubular calcium excretion, therefore can prevent calcium stone
formation.
Loop diuretics increase urinary excretion of calcium, therefore would exacerbate calcium renal
stone formation. 5 )
18. A 28 year old man was referred clinic by his General Practitioner. Over past two
months he had noticed increasing exertional dyspnoea. Previosuly he had physically active
jogged regularly. At time of referral he was breathless climbing one flight of stairs. He a
past history of asthma underwent an orchidopexy as a child an undescended right testis.
On examination, gynaecomastia was noted. No wheeze was audible in his chest.
Examination of genitals a firm mass 1cm x 1cm in right testis.
Investigations showed:
Serum alpha fetoprotein
90 kU/L (<10 kU/L)
Serum beta human chorionic gonadotrophin
1250 U/L (<5 U/L)
What is the likely diagnosis?
1 ) Choriocarcinoma
2 ) Gonadoblastoma
3 ) Leydig cell tumour
4 ) Seminoma
5 ) Sertoli cell tumour
Comments:
The classical presentation testicular tumors of a healthy male in third fourth decade of life a
painless, swollen, hard testis. Testicular cancer can be divided into germ cell non germ cell
tumours. Germ cell tumors are classified as either pure seminomas mixed non seminomatous
germ cell tumours (NSGCTs): these two groups comprise more than 90% of all tumors. Non-germ
cell malignancies (Leydig Sertoli cell tumors, gonadoblastomas) make up less than 10% of all
testicular tumors.
Patients history of cryptorchidism have a 10 40 times increased risk of testicular cancer this
risk greater abdominal versus inguinal location of undescended testis. Orchidopexy does
reduce risk of subsequently developing a malignancy. An abdominal testis more likely be
seminoma, while a testis surgically brought scrotum by orchiopexy more likely be NSGCT.
161

Choriocarcinoma aggressive of NSGCTs. It disseminates hematogenously lungs, liver, brain,


bone, other viscera very early in disease process. Unlike classic seminoma mixed GCTs, pure
choriocarcinoma more likely present symptoms from metastatic disease. Most testicular GCTs
cause scrotal swelling, a palpable mass, choriocarcinoma different in local tumor may be
small nonpalpable.
Pure seminomas do cause a rise in AFP level (AFP only produced by tumors containing
embryonal yolk sac elements). Elevated AFP levels are consistent NSGCT, though AFP often
within reference range in pure choriocarcinoma. Beta HCG usually markedly elevated in pure
choriocarcinoma, but only elevated in 10 15% of seminomas. Gynecomastia occurs due
elevation of beta hCG levels therefore common in choriocarcinoma, but only rarely seen in
patients a seminoma.
On ultrasound scanning, choriocarcinoma associated hemorrhage necrosis may appear more
cystic, inhomogeneous, calcified than a seminoma. Calcifications cystic areas are less common
in seminomas than in nonseminomatous tumors. 1 )
19. A 30 year old man presented one week history of severe pain behind right eye,
waking him at 2 am every night, lasting 90 minutes. He also noted his right eye watery
red during attacks. What is the appropriate treatment during attacks?
1
2
3
4
5

)
)
)
)
)

Amitriptyline
Indomethacin
Pethidine
Tramadol
Sumatriptan

Comments:
The diagnosis cluster headache. It associated lacrimation of eye, ptosis, eyelid swelling,
injection of conjunctiva nasal congestion. It usually presents in male patients (M/F 9:1), occurs
in early hours of morning. The treatment of choice during acute attacks include s/c
sumatriptan, high flow oxygen. Prophylactic treatment includes lithium, verapamil, sodium
valproate, prednisolone ergotamines. 5 )
20. A 78 year old diabetic man had on regular haemodialysis (four hours three times a
week) twelve years. He presented complaining of increasing muscular aches, numbness
joint stiffness affecting his hands, arms shoulders. His lower limbs were affected. His
medications include calcium carbonate two tablets three times a day meals, ACE inhibitor,
aspirin 1 alpha calcidol 1mcg once a day. Which of following may account his symptoms?
1
2
3
4
5

) Aluminium toxicity
) Amyloidosis
) Cerebrovascular disease
) Diabetic neuropathy
) Uraemia

Comments:
All above options can cause neurological abnormalities in patients renal failure.
Aluminium toxicity can cause twitching, myoclonic jerks, motor apraxia, fits personality
changes. It now rare, due treatment of dialysis water reverse osmosis, remove aluminium,
use of calcium containing phosphate binders, instead of aluminium.
162

Amyloidosis a common cause of neurological impairment in patients on longstanding dialysis. It


caused by beta2 microglobulin accumulation. It causes joint pains stiffness, usually upper limbs
more than lower limbs. The only treatment renal transplantation. It can be reduced by using
high flux dialysis membranes in patients who are likely be on dialysis a prolonged period.
The above symptoms do fit cerebrovascular disease, as symptoms spare lower limbs.
Diabetic neuropathy again does fit above picture of symptoms sparing lower limbs.
Uraemia may cause neurological symptoms. This more common in men, mostly affects legs.
Sensory symptoms (paraesthesia, burning sensations pain) occur before motor symptoms
(muscle atrophy, myoclonus, paralysis). The sensory symptoms may improve starting
increasing frequency of dialysis, but motor symptoms are reversible. 2 )
21. A 65 year old man referred Respiratory clinic by his General Practitioner. The
patient gives a three month history of a productive cough steady weight loss of 10kg. His
sputum had initially yellow, but over past week he noticed streaks of blood. He also
complains of sweats at night sometimes had change his bedclothes.
He a longstanding history of bronchial asthma required admission hospital on several
occasions. On two occasions he admitted in respiratory distress required intubation
ventilation. His current therapy includes salbutamol beclomethasone inhalers
prednisolone 5mg once daily which he taken several years. He also a home nebuliser
which he uses as required.
His chest x ray in clinic shows right apical consolidation. The last chest radiograph, taken
six months previously on admission hospital, had normal. A sputum sample proves be
positive acid alcohol fast bacilli. A presumptive diagnosis of pulmonary tuberculosis
made he started on antituberculous therapy.
Five days later he presents Casualty breathlessness wheeze. On examination he afebrile.
Widespread expiratory wheezes are heard throughout his chest. His Peak Expiratory Flow
Rate (PEFR) measured at 110 l/min. His chest x ray does show any significant change from
last film taken in Chest Clinic. Which of following options would be appropriate step in
his management?
1
2
3
4
5

)
)
)
)
)

Continue anti tuberculous therapy steroids at current dose


Continue anti tuberculous therapy reduce steroid dose
Continue anti tuberculous therapy increase steroid dose
Stop anti tuberculous therapy continue steroids at current dose
Stop anti tuberculous therapy steroids start broad spectrum antibiotics

Comments:
The metabolism of corticosteroids increased by rifampicin. Patients on long term steroids
should have their dose of steroids increased when starting antituberculous therapy.
See also:
Edwards OM, Courtney Evans RJ, Galley JM, et al. Changes in cortisol metabolism following
rifampicin therapy. 3 )
22. A 76 year old male presents visual loss. Examination reveals a pressure of 168/102
mmHg fundoscopy shows an embolus right superior temporal branch of retinal artery.
Carotid dopplers are arranged these show left sided carotids, but 90% Right External
Carotid Artery stenosis approximately 50% Right Internal Carotid Artery stenosis. What
would be appropriate treatment?
1 ) Aspirin
2 ) Dipyridamole
163

3 ) Right External Carotid Artery endarterectomy


4 ) Right Internal Carotid Artery endarterectomy
5 ) Warfarin
Comments:
Endarterectomy only beneficial if internal carotid artery stenosis greater than 70%. If less than
aspirin control of vascular risk factors treatment of choice. In this case hypertensive control
advised. 1 )
23. A 25 year male referred hypertension, agitation sweats of approximately 6 months
duration. He specific family history of note, smokes 10 cigarettes per day drinks little
alcohol. Medication prescribed by his GP hypertension includes bendrofluazide 2.5 mg/d
Ramipril 10mg per day. His pressure on examination was 176/94 mmHg he a BMI of 23.5
kg/m2.
Further investigations showed:
Urine free metadrenaline
12 umol/24 hr (NR < 5)
Fasting plasma calcitonin
100 ng/L
(NR 0 11.5)
MRI scan of abdomen a 3.5 cm mass in right adrenal gland
Based upon this information, what other diagnosis likely be associated his condition?
1
2
3
4
5

)
)
)
)
)

Acoustic neuroma
Gastrinoma
Hyperparathyroidism
Insulinoma
Prolactinoma

Comments:
This patient features of MEN type 2 <http://www.onexamination.com/site/Paedia.asp?id=640>
suggested by phaeochromocytoma medullary cell thyroid neoplasia associated
hyperparathyroidism. MEN type 1 associated pancreatic pituitary neoplasia. MEN type 2
usually autosomal dominant but in this patients case there appears be a spontaneous mutation
as there family history of note. It important investigate phaeochromocytoma in young
potential precipitating disorders such as Neurofibromatosis, MEN VonHippel Lindau. 3 )
24. A 45 year old lady referred outpatient clinic by her General Practitioner following
finding of significant hypertension on a routine check up. Her pressure in clinic measured
at 180/100 mmHg. Her electrocardiogram reveals changes characteristic of left ventricular
hypertrophy. What is the likely cause of her hypertension?
1
2
3
4
5

)
)
)
)
)

Coarctation of aorta
Conns Syndrome
Essential hypertension
Phaeochromcytoma
Renal artery stenosis

Comments:
Essential hypertension likely diagnosis as this common cause of hypertension. However this
lady clearly needs investigation secondary causes of raised BP. 3 )
164

25. A 75 year old man presented three episodes of transient right monocular visual loss.
He described visual loss as a curtain descending down on right eye. On examination he
looked plethoric.
Investigations revealed:
Haemoglobin
18 g/dL
White cells
4.5 x 109/L
Platelets
350 x 109/L
Serum cholesterol 7.8 mmol/L
Carotid doppler scan showed bilateral internal carotid stenosis of 50%
What are two appropriate treatments this patient?
1)
2)
3)
4)
5)
6)
7)
8)
9)
10

Aspirin
Carotid angioplasty
Carotid endarterectomy
Clopidogrel
Dipyridamole
Perindopril
Prednisolone
Statin
Venesection
) Warfarin

Comments:
The two important treatments in this case will be aspirin venesection. They should reduce risk
of further TIAs stroke. Carotid endarterectomy considered if there significant carotid artery
stenosis i.e. >70%. Aspirin still first choice as antiplatelet treatment ahead of clopidegrol
dipyridamole. Statin also a role in management but as important as aspirin venesection in this
case. 1)9)
26. A 52 year presents a 6 month history of weight loss diarrhoea. Over this period of
time she had lost approximately 10 kg in weight was aware of watery diarrhoea three
four times daily. She was also aware of occasional flushes which she had experienced since
menopause at age of 49 but had become more frequent of late. She had previously well
other past history. She took medication. She was a non smoker drank approximately 12
units of alcohol weekly.
On examination, she had a reddish complexion had a BMI of 24 kg/m2. She had a pulse of
88 beats per minute regular a pressure of 122/88 mmHg. There were abnormalities on
cardiovascular respiratory examination but abdominal examination two finger breadths
hepatomegaly.
Investigations revealed:
Haemoglobin
14.5 g/dL
White cells
8.5 x 109/L
Platelets
301 x 109/L
Serum sodium
144 mmol/L
Serum potassium
4.1 mmol/L
Serum urea
3.9 mmol/L
165

Serum
Serum
Serum
Serum

creatinine
alkaline phosphatase
alanine aminotransferase
bilirubin

110 umol/L
125 IU/L
40 IU/L
12 umol/L

24hr Urine 5 Hydrox yindoleacetic acid (5 HIAA)


100 mg/d(NR < 5 mg/d)
Abdominal ultrasound scan showed numerous echo dense deposits within liver. What is
the appropriate treatment this patients diarrhoea?
1
2
3
4
5

)
)
)
)
)

Cyproheptadine
Ketanserin
Loperamide
Methysergide
Octreotide

Comments:
This patient Carcinoid syndrome <../site/paedia.asp?id=162> as by grossly elevated urine 5
HIAA concentrations. Carcinoid a slow growing neuroendocrine tumour symptoms of diarrhoea
flushes occur as a consequence of meatastasis liver hence systemic release of vasoactive
compounds such as 5 HT bradykinin. The best treatment symptoms of Carcinoid somatostatin
analogue, Octreotide which improves symptoms prognosis in Carcinoid syndrome. Other
potential treatments following resistance failure of octreotide includes hepatic artery
embolisation. 5 )
27. A 67 year man presents a two month history of persistent cough. The cough always
unproductive there haemoptysis. It never wakes him from his sleep. He chest pain but
noticed some mild breathlessness on exertion. He a smoker of 20 cigarettes a day since
age of seventeen. He lost a small amount of weight, a reduced appetite. He constipated
over last two weeks.
Examination was unremarkable. He clubbing nor lymphadenopathy. Examining his chest,
breath sounds were vesicular added sounds.
Investigations showed:
Haemoglobin
12.6 g/dL
White cell count
10.5 x 109/L
Platelets
160 x 109/L
Serum sodium
149 mmol/L
Serum potassium
4.4 mmol/L
Serum urea
9.4 mmol/L
Serum creatinine
120 umol/L
Serum corrected calcium
3.26 mmol/L
The chest x ray showed right hilar lymphadenopathy
What is the next investigation of choice?
166

1
2
3
4
5

)
)
)
)
)

Bone marrow biopsy


Bronchoscopy
CT chest
Isotope Bone scan
Kveim Test

Comments:
The differential diagnosis of this mans hilar lymphadenopathy hypercalcaemia include bronchial
carcinoma <Paedia.asp?id=143>, sarcoidosis <Paedia.asp?id=874> lymphoma.
The likely diagnosis squamous cell bronchial carcinoma. Hilar involvement usually a result of
metastatic spread hilar nodes.
CT chest next investigation of choice as it will help identify any further nodal involvement
allow planning of an approach histological confirmation of tumour. 3 )
28. A 35 year old man presented shooting pain of lateral aspect of his left arm. On
examination he had weakness of biceps diminished biceps reflex.
What likely explanation of his symptoms?
1
2
3
4
5

)
)
)
)
)

C5 radiculopathy
C6 radiculopathy
C7 radiculopathy
C8 radiculopathy
T1 radiculopathy

Comments:
Shooting pain cardinal symptom of a radiculopathy. The lateral aspect of arm represents C5
dermatome. C5 innervates biceps, C6 brachioradialis, C7 triceps, C8 finger flexors, T1 small
muscles of hand. 1 )
29. A 64 year old man a history of renal transplantation 8 years ago presented fever
pleuritic right sided chest pain. His symptoms started 3 days previously a flu like illness
associated generalised arthralgia.
He subsequently developed a cough productive of purulent green sputum occasional
haemoptysis on day of admission his wife found him collapsed in bathroom.
He was under 6 monthly review by renal physicians was taking azathioprine ciclosporin.
He was recently diagnosed having gout was started on indometacin followed by
allopurinol.
On examination he looked pale a tachycardia of 110/minute regular. His pressure was
100/60 his heart sounds normal. Examination of his chest dull percussion, increased
vocal resonance bronchial breathing over right base.
Investigations on admission showed:
Haemoglobin
6.4 g/dL
Neutrophils
0.2 x 109/L
Lymphocytes
0.1 x 109/L
Platelets
45 x 109/L
Serum sodium
138 mmol/L
Serum potassium
4.1 mmol/L
Serum chloride
100 mmol/L
167

Serum bicarbonate
Serum urea
Serum creatinine
Plasma glucose

18 mmol/L
22.4 mmol/L
253 umol/L
7 mmol/L

What is the likely cause of his illness?


1
2
3
4
5

)
)
)
)
)

Azathioprine toxicity
Ciclosporin toxicity
Type A adverse reaction indometacin
Type B adverse drug reaction due allopurinol
None of above

Comments:
Allopurinol acts by inhibition of xanthine oxidase thus inhibit metabolism of 6 mercaptopurine,
an active metabolite of azathioprine. Concomitant use of allopurinol azathioprine
recommended if concomitant use occur, a dose reduction in azathioprine by 25% advised
regular count monitoring. Azathioprine toxicity commonly causes bone marrow suppression
thus sepsis as seen in this scenario.
Ciclosporin levels can also rise concomitant administration of allopurinol but a lesser extent.
Ciclosproin toxicity more commonly cause renal toxicity.
Blood dyscrasias such as thrombocytopenia, leucopenia agranulocytosis can occur indometacin
but this an indiosyncratic reaction (type B adverse reaction). 1 )

30. A 34 year old man was referred renal clinic. He notice increased swelling of his
ankles over last six weeks. His pressure was 170/100 mmHg.
Initial investigations by his general practitioner showed:
Serum sodium
140 mmol/L
Serum potassium
5.0
Serum urea
12
Serum creatinine
128
Albumin
26 g/L
Urine dipstick
Protein ++++ Blood +
24 hour urine protein:
9 g/24hr (NR <0.2 g/24hr)
Prednisolone 60mg daily was commenced in clinic. Six weeks later, a 24 hour urine
collection was repeated.
24 hour urine protein: 0.8 g/24hr (NR <0.2g/24hr)
Which of following would likely be seen on renal biopsy?
1 ) Antiglomerular basement membrane disease
2 ) IgA glomerulonephritis
3 ) Membranous glomerulonephritis
168

4 ) Mesangiocapillary glomerulonephritis
5 ) Minimal change glomerulonephritis
Comments:
This gentleman a nephrotic syndrome impairment of his renal function, which improves
markedly oral prednisolone.
Antiglomerular basement membrane disease, Goodpastures, causes an aggressive type of renal
failure, which requires treatment plasma exchange cyclophosphamide. It would improve oral
prednisolone alone.
IgA glomerulonephritis an uncommon cause of nephrotic syndrome. High dose prednisolone
can be used in patients nephrotic range proteinuria in IgA nephropathy, but improvements
seen in question would occur.
In patients membranous glomerulonephritis, there clear agreement about relevant treatment.
Treatment high dose prednisolone (125mg/alternate days) can be given, but outcome probably
affected. This patient would be at high risk of developing chronic renal failure (male, heavy
proteinuria, renal impairment) therefore probably would respond prednisolone as in this
question.Mesangiocapillary glomerulonephritis treated antiplatelet drugs, anticoagulants,
corticosteroids alkylating agents. Steroids are given a prolonged period of time, may have
some benefit in some patients.
The correct answer minimal change glomerulonephritis. This extremely steroid sensitive, 80%
of adult patients achieving remission within 16 weeks prednisolone 60mg per day. 5 )
31. An 82 year old man was reviewed in diabetic clinic. He had type II diabetes mellitus,
which tablet controlled. He was taking gliclazide 80mg twice a day, bendrofluazide 2.5mg
once a day, ibuprofen 600mg three times a day. On examination his pressure was
measured at 160/95 mmHg. Fundoscopy showed grade II hypertensive retinopathy.
Investigations revealed:
Haemoglobin A1C
8.2%
(NR 3.8 6.4%)
Serum urea
11 mmol/L
Serum creatinine
114 umol/L
GFR
59 ml/min
(NR 70 140)
Urine dipstick
Protein +
Which of following likely cause of his findings?
1 ) Age related
2 ) Crescentic glomerulonephritis
3 ) Diabetes mellitus
4 ) Hypertension
5 ) Interstitial nephritis
Comments:
This gentleman mildly impaired renal function a GFR of 59ml/min. All five of options could
account above.
Age related changes in renal function are very common. The glomerular filtration rate falls as
does tubular reabsorptive secretory functions. These changes exceed decline in lean body mass
so serum urea creatinine rise slightly.
There evidence this patient a glomerulonephritis.
Diabetes mellitus an important cause of renal impairment, but in this case it less likely, as
patient does have any other evidence of diabetic complications (especially diabetic retinopathy).
169

Hypertension causes renal impairment, but seen commonly in accelerated phase hypertension,
when it associated grade III hypertensive retinopathy (flame haemorrhages soft exudates).
Interstitial nephritis can be caused by NSAID use. NSAIDs may cause a reversible reduction in
glomerular filtration rate, acute tubular necrosis, acute interstitial nephritis often heavy
proteinuria, renal papillary necrosis chronic tubulointerstitial nephritis. In this patient,
proteinuria minimal, therefore this option can be discounted.1)
32. A 34 year old man was admitted hospital macroscopic haematuria. He had recently
arrived in country from Nigeria. He was taking any medications, denied any other
symptoms. On examination, his pressure was 130/80 mmHg.
Investigations showed:
Haemoglobin
11.4 g/dL
MCV
72 fL
Reticulocyte count
4.9% (NR 0.5 2.4)
Serum sodium
134
Serum potassium
3.8
Serum urea
8.0
Serum creatinine
120 umol/L
C reactive protein
8 mg/L
Urine dipstick
Protein + Blood ++++
Which of options
1
2
3
4
5

)
)
)
)
)

likely cause of his haematuria?

Analgesic nephropathy
Loin pain haematuria syndrome
Papillary necrosis
Plasmodium infection
Pyelonephritis

Comments:
This gentleman from Africa presents macroscopic haematuria. He a mild anaemia raised
reticulocyte count low MCV. His inflammatory markers are normal. He probably sickle cell
anaemia; this presentation would fit haemoglobin sickle cell disease.
There nothing in question point towards a diagnosis of analgesic nephropathy.
Loin pain haematuria syndrome causes haematuria, but associated severe loin pain, either uni
bilateral. It a diagnosis of exclusion.
The likely cause of his symptoms papillary necrosis. This results from ischaemic lesions of
renal papilla, varying from minute ulcerations renal papillary necrosis. Treatment conservative,
although prolonged haematuria may require transfusion limited surgery. It a common finding
in sickle cell disease.
His inflammatory markers are normal; therefore there evidence of malaria pyelonephritis. 3 )
33. 50 year old gentleman known severe aortic stenosis develops syncopal symptoms
on exertion.What is the next step in his management?
1 ) Aortic valvuloplasty
170

2
3
4
5

)
)
)
)

Aortic valve replacement


Close outpatient follow up serial echocardiograms
Exercise test looking dysrhythmias
Observe until he develops symptoms of breathlessness

Comments:
Exertional syncope an indication surgery in patients severe aortic stenosis, even in absence of
exertional dyspnoea. Aortic valvuloplasty compares unfavourably aortic valve replacement in
medium long term in aortic stenosis. 2 )
34. A 62 year old presents visual loss of right eye, which developed quite suddenly one
day ago. Fundoscopy reveals an embolus right superior temporal branch of retinal artery.
Where would you expect find visual field defect?
1 ) Superior nasal
2 ) Superior temporal
3 ) Inferior nasal
4 ) Inferior temporal
5 ) Inferior caudal
Comments:
Superior segmental optic nerve pathology will give inferior nasal visual field defect. 3 )
35. A 35 year old woman previous history of severe depression, presented 40 minutes
of loss of consciousness associated violent shaking of her limbs. On examination, pupils
were reacting light. Her eyes were tightly shut, impossible open passively. Oxygen
saturation, pulse pressure are all within range. What is the appropriate management?
1 ) Intravenous diazepam
2 ) Intravenous phenytoin
3 ) Observation
4 ) Transfer ITU
5 ) Urgent CT brain
Comments:
The clinical presentation typical of pseudoseizure (non epileptic attack). Violent shaking,
resistance passive eye opening vital signs are useful indicators of pseudoseizures. Simple
observation appropriate management. Pseudoseizures are commonly misdiagnosed as seizures
treated inappropriately anti epileptic drugs. 3 )
36. A 55 year old man was admitted hospital a twenty four hour history of constant
central chest pain radiating his left arm. He had a past history of hypertension had had
exertional angina two years. His regular medication included aspirin, atenolol enalapril.
On examination he was pale, sweaty breathless at rest. His ECG showed anterior ST
segment elevation. Thrombolysis was administered due delay in his presentation.
Four weeks later he underwent an exercise tolerance test. His resting pressure was
150/90 mmHg. The patient developed central chest pain radiating left arm after 2 minutes
of exercise his pressure at this time was measured at 110/65mmHg. His pre exercise ECG
showed anterior Q waves, but there were ischaemic changes associated his chest pain
during exercise.
What should be the next step?
1 ) Stop exercise test repeat again in four weeks
171

2
3
4
5

)
)
)
)

Stop exercise test arrange a thallium exercise test


Stop exercise test arrange coronary angiography
Repeat test off all medication
Encourage patient continue on treadmill

Comments:
A response exercise includes a progressive increase in heart rate pressure. This patient
angina at very low work load a paradoxical BP response. Even in absence of ST segment
depression at peak exercise, these findings suggest severe coronary artery disease such
features are frequently seen in patients left main stem stenosis. Failure of BP increase, a
decrease by 15mmHg may reflect a decline in ventricular function due ischaemia. 3 )
37. An 80 year old woman presented Casualty after a fall minor head injury. On
examination her Glasgow Coma Scale (GCS) was 15/15 full physical examination was
unremarkable. She was discharged after overnight observation.Two days later she
presented confusion depressed conscious level (GCS 7/15). An urgent CT brain showed an
ovoid collection compressing temporal lobe. What is the likely diagnosis?
1
2
3
4
5

)
)
)
)
)

Cerebral abscess
Cerebral infarction ma
Exdradural haematoma
Glioma
Subdural haematoma

Comments:
Extradural haematoma an uncommon but potentially fatal complication of head injury. Prompt
recognition appropriate treatment essential. Classic lucid interval in <50%. On CT brain you
see a hyperdense biconvex extra axial mass. Herniation common. The commonest cause
laceration of middle meningeal artery. It usually occurs within first 36 hours. 3 )
38. A 47 year old lady was referred outpatients a four week history of cough productive
of sputum breathlessness. The cough had become so severe she was producing 500ml of
frothy mucoid sputum per day. She had noticed a reduction in appetite but weight loss.
What is the likely diagnosis ?
1 ) Alveolar Proteinosis
2 ) Broncholveolar Cell Carcinoma
3 ) Carcinoid Tumour
4 ) Langerhans Histiocytosis X
5 ) Lymphangitis Carcinomatosa
Comments:
Bronchoalveolar cell carcinoma classically presents progressive breathlessness production of
large amounts of sputum (bronchorrhoea). They account up 1% of all bronchial carcinomas. The
tumour spreads using alveolar walls as a frame alveoli are often filled mucin. The differential
diagnosis includes heart failure, persistent pneumonia cryptogenic organising pneumonia
Alveolar proteinosis, pulmonary langerhans cell histiocytosis X lymphangitis carinomatosis <
can all present progressive breathlesness cough but bronchorrhoea a feature. 2 )
39. A 65 year old lady admitted as an emergency after a choking episode. She
breathless a temperature of 37.8C. She a smoker of 20 cigarettes a day, but history of
previous respiratory disease; she had a stroke 6 months previously. She was started on
oral co amoxiclav by her General Practitioner.
172

A chest x ray showed a homogenous opacity at right base right hilum pulled
downwards.What is the next investigation of choice?
1 ) Blood cultures
2 ) Bronchoscopy
3 ) CT chest
4 ) Sputum Culture
5 ) Ventilation/Perfusion Scan
Comments:
This ladys clinical history suggests aspiration. Her chest x ray suggests collapse +/
consolidation of right lower lobe. The lower lobes are usual site of aspiration when patient
upright. She may therefore have aspirated a foreign body during choking episode a
bronchoscopy will identify this allow removal of it. 2 )
40. A 39 year old man was referred renal clinic. Two weeks previously he had an
episode of loin pain, passed a small renal stone. He had one previous episode three years
previously, but did seek advice. He was otherwise fit well.
Investigations showed:
Haemoglobin
14.5 g/dL
White cells
7.5 x 109/L
Platelets
210 x 109/L
Serum sodium
137 mmol/L
Serum potassium
4.2 mmol/L
Serum urea
6.1 mmol/L
Serum creatinine
100 umol/L
Serum corrected calcium
2.3 (NR 2.2 2.6)
24 hour urine collection:
Volume
1150 ml/24hr
Calcium
18 mmol/24hr
(NR 2.5 7.5)
Analysis of stone showed it contain mostly calcium
What advice would you initially give to this man?
1 ) Decrease dietary dairy product intake
2 ) Decrease dietary protein intake
3 ) Increase fluid intake
4 ) Increase dietary protein intake
5 ) Low fat diet
Comments:
This gentleman had a calcium urinary tract stone. He serum calcium, but raised urinary
excretion of calcium. Idiopathic hypercalciuria often familial, common cause being increased
GI absorption of calcium. The common stones are calcium oxalate stones.
In this patient, his urinary output 1150 mls/24hr. He should aim a daily urinary output in
excess of 2000 ml. The correct answer therefore advise him increase his fluid intake. A high
protein intake associated urinary stones, by reducing his dairy dietary intake he may reduce
his GI absorption of calcium, but main initial treatment increase his urine volume. 3 )
41. A 45 year old gardener was referred Dermatology clinic a well circumscribed,
raised erythematous lesion on her finger. The lesion had enlarged steadily over previous
three weeks, was tender bled easily when touched.What is the likely diagnosis?
1 ) Cutaneous anthrax
173

2 ) Keratoacanthoma
3 ) Malignant melanoma
4 ) Mycobacterium marinum infection
5 ) Pyogenic granuloma
Comments:
Pyogenic granuloma (lobular capillary haemangioma) benign vascular lesion of skin mucosa.
The cause unknown. The name a double misnomer the lesion neither pyogenic nor a
granuloma. Pyogenic granulomata are usually solitary lesions, appearing as a glistening red
papule nodule prone bleeding ulceration. Lesions often grow rapidly (over weeks), frequently
occurring at sites of trauma commonly involve digits, arms, head face. Pathologically, it an
inflammatory lesion composed of granulation tissue chronic inflammatory cells.
Keratoacanthoma a low grade malignancy originates in pilosebaceous glands. Clinically
pathologically lesions resemble squamous cell carcinoma. Lesions typically are solitary a
central area of ulceration may contain a characteristic keratin plug.
Cutaneous anthrax would be expected on basis of history given. Anthrax lesions begin as
reddened, indurated papules, later becoming necrotic a characteristic black centre (eschar).
Pictures of cutaneous anthrax appear in MRCP Part 2 exam, though usual history given of
someone from (or who visited) Middle East had contact cattle/sheep/goats.
Malignant melanomas may occur at any site, but given context provided in this case, a less
likely option.
Mycobacterium marinum cause of Fish Tank Granuloma. Lesions are ovoid usually occur on
hands (following contact with fish). Fishermen, fishmongers tropical fish enthusiasts are
susceptible.
*Sporotrichosis (caused by Sporothrix schenckii) usually acquired by contamination of wounds
soil characteristically a rose thorn. Although these patients can present skin lesions, they are
usually multiple associated regional adenopathy systemic symptoms/sepsis. This was felt be
too specialised inclusion in a MRCP Part 2 question, but candidates should consider this
diagnostic possibility in this epidemiological context (a gardener) but should take clinical
presentation & any images into account also. 5 )
42. A 55 year old man was referred outpatient clinic anaemia. He had presented his
General Practitioner a three month history of fatigue low back pain. There was
preceeding history of trauma. A plain x ray of his lumbar spine showed a lytic lesion in
body of fourth lumbar vertebra (L4).
Investigations revealed:
Haemoglobin
10.5 g/dL
WBC
4.0 x 109/l
Platelets
175 x 109/l
Serum sodium
137 mmol/L
Serum potassium
3.5 mmol/L
Serum urea
2.7 mmol/L
Serum creatinine
110 umol/L
Serum corrected calcium
2.4 mmol/L (2.2 - 2.6 mmol/L)
Bence Jones proteins were detected in urine
A skeletal survey showed increased uptake in L4
Which of following therapies will reduce risk of pathological fracture?
1 ) autologous stem cell transplantation
174

2 ) melphalan
3 ) pamidronate
4 ) plasmapharesis
5 ) steroids
Comments:
Pathological fractures bone pain are extremely common symptoms in patients myeloma. It
estimated 70% of patients have bone pain at time of presentation. Biphosphonates reduce bony
disease in myeloma: lowering frequency of pathological fractures. There also evidence
bisphosphonates modulate disease have some antitumour activity. As a result,
bisphosphonates should be given routinely patients myeloma, even in absence of
hypercalcaemia.3 )
43. A 45 year male presents a one year history of impotence reduced shaving
frequency. These symptoms have evolved over this period of time he unaware of any
erections whatsoever libido. He also aware he shaves just once weekly whereas
previously he was shaving daily. Together these symptoms he also feeling rather
lethargic reduced energy aware of vague joint aches. He takes medication otherwise
well except an appendicectomy at age of 20. He drinks approximately 20 units of alcohol
weekly smokes 5 cigarettes daily. He married but children.
On examination, he appears slightly pigmented, gynaecomastia a fine skin scant facial,
pubic axillary hair. Examination of testes reveal testicualar size of approximately 15 mls
bilaterally masses are felt. Examination of cardiovascular, respiratory abdominal
systems are normal. No abnormalities are noted on joint movements a full range of
movement.
Investigations reveal:
Prolactin
370 mu/l
(50 450)
Testosterone 3.5 nmol/l
(10 - 30)
LH
2.1 mu/l
(2 10)
FSH
2 mu/l
(2 10)
Free T4
12.8 nmol/l
(10 23)
TSH
2.1 mu/l
(0.5 4.5)
Which of following important investigation this patient?
1 ) Ferritin
2 ) Karyotype
3 ) MRI pituitary
4 ) Short synacthen test
5 ) Ultrasound examination of testes
Comments:
This man hypogonadotrophic hypogonadism (low testosterone but LH FSH are inappropriately
low normal) indicating pituitary involvement. But, features of this disorder in a middle aged
male a pigmented appearance, gynaecomastia asthralgia would suggest a diagnosis of
haemochromatosis a ferritin would be appropriate investigation. Testes appear issue
pituitary so ultrasound unnecessary. Addisons suggested by symptoms - weight loss
mentioned, postural features so a short synacthen test would be best choice. It could be
argued pituitary MRI appropriate as a pituitary tumour could produce these results. However a
macroadenoma stalk compression, prolactin may be slightly higher, visual fields may be
175

abnormal arthralgia gynaecomastia could be explained. Finally, LH FSH would be high in


Klinefelters history also suggests previously erectile function development were normal. 1 )
44. A 61 year old lady was referred clinic by her GP a six month history of fatigue.
There was history of weight loss change in bowel habit, cough sputum production. She
denied having any fevers sweats. Her only other complaint was of occasional pains in her
joints secondary rheumatoid arthritis, which had diagnosed three year previously.
She worked as an assistant in a bookshop lived her husband. She had three children, all of
whom were well. There was family history of note. She was a lifelong non smoker seldom
drank alcohol.
On examination she appeared well. She was afebrile. Xanthelasma were noted around both
eyes. A few faint scratch marks were visible on skin of her arms. She attributed this an
allergy a new washing powder. Her pulse was 70/min pressure 110/65. Heart sounds
were murmurs added sounds. Her chest was clear. The abdomen was soft non tender.
The spleen was palpable three centimetres below left costal margin. The liver kidneys
were palpable. Neurological examination was normal.What is the likely diagnosis?
1 ) Chronic lymphocytic leukaemia
2 ) Familial hypercholesterolaemia
3 ) Hodgkin's lymphoma
4 ) Primary biliary cirrhosis
5 ) Pseudoxanthoma elasticum
Comments:
Over 90% of cases of primary biliary cirrhosis (PBC) occur in women aged 35 70. The
presentation usually insidious. Pruritus, nonspecific fatigue, both are initial symptoms in >
50% of patients can precede other symptoms by months years. About 50% of patients present
an enlarged, firm, nontender liver; 25%, splenomegaly; about 15%, skin xanthomas; 10%,
hyperpigmentation. Jaundice present in 20% of patients tends develop in others time.
Clubbing, metabolic bone disease (ie, osteoporosis), peripheral neuropathy, renal tubular
acidosis, steatorrhea (from cholestasis pancreatic secretory failure) are possible. Later, all
features complications of cirrhosis can develop. PBC commonly associated autoimmune
disorders (eg, RA, scleroderma, sicca complex, autoimmune thyroiditis).
CLL usually associated symmetrical lymphadenopathy; splenomegaly detectable in about 50%
of cases.
Localised lymphadenopathy (usually above diaphragm) commonest presentation of Hodgkins
disease; hepatosplenomegaly usually associated NHL.
While familial hypercholesterolaemia might explain xanthelasma in this case, it does
adequately explain other symptoms of fatigue pruritis presence of splenomegaly. 4 )
45. A 52 year presents agitation weight loss. Examination reveals a fine tremor, goitre
tacchycardia. Investigations confirm thyrotoxicosis positive TSH receptor autoantibodies.
She elects receive radioactive iodine treatment.
Which of following statements concerning therapy this patient correct?
1)
There less than 30 percent chance of hypothyroidism in long term following radioactive
iodine treatment.
2)
There a long term 2 fold risk of thyroid neoplasia following radio iodine treatment.
3)
The risk of recurrence after anti thyroid drugs above 50%.
4)
A reduction in size of goitre would be expected in approximately 90% of cases following
radio iodine treatment.
5)
Long term studies do reveal a slightly increased risk of gastric neoplasia following
radioactive iodine.
176

Comments:
Current strategies treatment of thyrotoxicosis include a variable treatment period of anti
thyroid drugs such as carbimazole propylthiouracil, radioiodine surgery. Long term remission
following antithyroid drugs of order of 15%, vast majority relapsing. Thus frequently, radio
iodine advocated as a primary treatment - particularly multi nodular toxic solitary nodules.
However, approx 80% will have long term hypothyroidism following radio iodine. There
evidence of increased risk of thyroid neoplasia gastric neoplasia following RAI. Goitre shrinkage
may occur in up 30% following RAI. 3 )
46. A 56 year old patient chronic renal failure was admitted hospital following a gastro
intestinal bleed. On examination, his pressure was 148/90 mmHg, pulse 88 beats per
minute, regular. Rectal examination malaena stool.
Investigations showed:
Haemoglobin
10.9 g/dL
MCV
80 fL
Serum sodium
142 mmol/L
Serum potassium
5.1 mmol/L
Serum urea
14 mmol/L
Serum creatinine
450 umol/L
Investigations two weeks previously at clinic:
Haemoglobin
11.8 g/dL
MCV
84 fL
Serum sodium
139 mmol/L
Serum potassium
4.7 mmol/L
Serum urea
9 mmol/L
Serum creatinine
250 umol/L
Which of the following is appropriate next step in his management?
1 ) Blood transfusion
2 ) Haemodialysis
3 ) Hydration intravenous fluids
4 ) Observation on ward
5 ) Start proton pump inhibitor
Comments:
This patient chronic renal failure had an upper GI bleed. He haemodynamically stable. His
haemoglobin level appropriate. His creatinine risen from 250 450, which may reflect a small
change in renal function in a patient chronic renal failure.
The appropriate treatment careful hydration IV fluid monitoring of his renal function. He does
require transfusion (as he haemodynamically stable) there indication emergency dialysis. 3 )
47. A 33 year old male presents marked haemetemesis. He noted have a pulse of 128
beats per minute a pressure of 100/60 mmHg. An urgent endoscopy reveals a large
duodenal ulcer which does respond injection therapy he undergoes laparoscopy oversew
of his duodenal ulcer. He makes a good immediate post operative recovery receives
treatment IV omeprazole. No further bleeding noted his full count remains stable.
177

However, on twelfth post operative day he notes persisting weakness of right hand. On
examination there wasting of small muscles of right hand.
What likely site of lesion?
1
2
3
4
5

)
)
)
)
)

Wrist
Elbow
Neck
Brachial plexus
Brain

Comments:
This an entrapment ulnar neuropathy at elbow (the commonest site entrapment of ulnar
nerve). A common complication in ill patients in hospital. Nerve conduction studies will confirm
site of lesion. 2 )
48. The orthopaedic surgeons request a opinion on a 77 year old man who at post
operative day ten following a total hip replacement. The patients operation had
uncomplicated he had well in post operative period date. On day of referral he had
suddenly developed left sided chest pain shortness of breath. On examination his pulse
was 120/minute his respiratory rate 22/minute. His chest was clear auscultation his
heart sounds were normal. His ECG showed left bundle branch block.
Investigations:
Arterial gases on air:
pH
7.44
pO2
9.5 kPa </TD
pCO2
3.4 kPa
What is the likely diagnosis?
1 ) Acute left ventricular failure
2 ) Acute myocardial infarction
3 ) Fat embolism
4 ) Hospital acquired pneumonia
5 ) Pulmonary embolism
Comments:
This a common clinical conundrum! Despite an ECG showing LBBB, PE likeliest explanation
this patients chest pain dyspnoea. Being post op following a THR, there a significant risk of PE.
This backed up by gases which show hypoxia hypocapnoea (i.e. he unable oxygenate despite
good ventilation). There are clinical signs of either LVF pneumonia gases are compatible
either of these options. Fat embolus a risk intra operatively in immediate post op period, but
much less likely 10 days afterwards. The presence of LBBB raises possibility of MI, but its clear
whether this conduction defect new old; gases arent in keeping an AMI - so you have say PE
likely explanation. 5 )
49. A 22 year old woman presents recurrent epidodes of breathlessness cough
productive of foul smelling sputum. She a past history of recurrent episodes of colicky
abdominal pain last 4 years. Investigations:
Sputum culture Heavy growth of Pseudomonas aeruginosa Haemophilus influenzae
CXR Tramline ring shadows What is the likely explanation of her symptoms?
1 ) Bronchiectasis
178

2 ) Chronic Granulomatous Disease


3 ) Cystic Fibrosis
4 ) Hypogammaglobulinaemia
5 ) Primary ciliary dyskinesia
Comments:
Cystic Fibrosis (CF) usually presents in childhood, but it unusual some cases present in early
adult life. CF presenting in this fashion occurs frequently in Part 2 exam. Pseudomonas never a
pathogen in healthy lungs, it cause of foul smelling sputum implies there underlying
bronchiectasis. Similarly, H. influenzae typically a pathogen in chronic lung disease. Several of
alternative options may be associated bronchiectasis, but none adequately explains history of
abdominal pain, which likely be due distal intestinal obstruction syndrome (sludging of
intestinal contents leading intestinal obstruction by faecal impaction intussusception).
Chronic granulomatous disease (CGD) an inherited disorder of phagocytes cells, caused by an
inability of neutrophils undergo oxidative burst necessary kill some bacteria fungi. This leads
recurrent life threatening bacterial fungal infections. Fungal pneumonias, particularly those
caused by Aspergillus species are common. Most patients (75%) present within first five years of
life. Pseudomonas aeruginosa a rare pathogen in CGD because CGD neutrophils are able kill P
aeruginosa organisms by nonoxidative mechanisms.
T cell deficiencies (such as loss of CD4 T lymphocytes in HIV/AIDS)are typically associated viral
fungal pathogens.
Patients hypogammaglobulinaemia are susceptible recurrent upper lower respiratory tract
infections (otitis media, sinusitis, bronchitis, bronchiectasis, pneumonia). Encapsulated bacteria
(e.g. Streptococcus pneumoniae, Streptococcus pyogenes, Haemophilus influenzae,
Staphylococcus aureus) are common pathogens.
Primary ciliary dyskinesia associated recurrent upper lower respiratory tract infections due
inability clear secretions effectively. Patients are usually infertile. 3 )
50. A 32 year old woman presents exertional breathlessness. Echocardiography shows a
bicuspid aortic valve severe aortic stenosis. She says she her husband would like start a
family. What is the appropriate management strategy?
1 ) Refer aortic valve valvuloplasty
2 ) Refer bioprosthetic aortic valve replacement
3 ) Refer mechanical aortic valve replacement
4 ) Treat medically plan aortic valve replacement after delivery of her baby
5 ) Treat medically advise pregnancy be avoided
Comments:
This lady symptomatic severe aortic stenosis. This an indiction treatment surgery.
Valvuloplasty a short term treatment sometimes used in extreme circumstances, particularly if
there late presentation in pregnancy of aortic stenosis. There need anticoagulate patients
following implantation of a bioprosthethic - advantage in women of child bearing age (a
teratogen) need be used; however given her age she would be likely require a second operation
during her lifetime this therefore best solution. A mechanical valve replacement best long
term treatment but she would need be converted subcutaneous heparin instead of during
pregnancy therefore must have a planned pregnancy. 3 )
51. A 67 year old man was referred outpatient department a four month history of
increasing cough dyspnoea. He coughed mornings bringing up mucoid sputum. He denied
any haemoptysis. In addition he had become increasingly breathless on exertion so his
exercise tolerance was reduced 100 metres on flat.
179

He denied any chest pain but did have pain in his right shoulder was constant had started
keep him awake at night over last couple of months more recently pain of medial aspect
of his right arm. He was known have rheumatoid arthritis. His appetite was reduced he
had lost 5kg in weight. He had a forty pack year smoking history. He was a retired
engineer.
On examination there was nicotine staining of fingers. He had signs of rheumatoid
arthritis in his hands. There was bilateral finger clubbing. He had wasting of small muscles
of his right hand particularly thenar hypothenar eminences.
What is the diagnositic test of choice:
1
2
3
4
5

)
)
)
)
)

Bronchoscopy
CT Scan of Chest
Isotope Bone Scan
Nerve Conduction Studies
Sputum Cytology

Comments:
This patient a Pancoast superior sulcus tumour <Paedia.asp?id=705>. The tumour arises in
apex of lung infiltrates locally into brachial plexus, ribs mediastinum. Patients often have
signs of local extension on presentation such as neurological signs in arm hand (C8,T1
distribution), ipsilateral Horners syndrome radiological evidence of rib destruction.
CT scan investigation of choice. It can often be difficult get biopsy material as approaches in
this area are difficult. Bronchoscopy unhelpful as lesions are too peripheral. Tumours are
commonly squamous cell are usually inoperable on presentation. 2 )
52. A 73 year old man presents symptoms of breathlessness clinically heart failure. He
known have chronic atrial fibrillation. Normally he gets breathlessness on mild exertion
(NYHA class II). His heart rate at rest 90 beats/minute increases 150 beats/minute on
exertion. His current medications are Aspirin 150mg OD, Enalapril 10mg bd, Verapamil
180mg od, Digoxin 125mcg od. The serum digoxin level measured at 1.5 nmol/mL
(therapeutic range 1.0 - 2.6 nmol/mL).
What is the appropriate chage his medications?
1 ) Add spironolactone
2 ) Increase dose of digoxin
3 ) Increase dose of Enalapril
4 ) Increase dose of verapamil
5 ) Switch verapamil a beta blocker
Comments:
The diagnosis in this patient heart failure incomplete atrial fibrillation rate control. This can
result in exertional breathlessness. The appropriate therapy implementing beta blockade. This
evidence in NYHA II heart failure in terms of symptom improvement mortality benefit. In this
instance it will also improve AF rate control.
Verapamil negatively inotropic deleterious in heart failure this should be replaced. The
digoxin dose at present adequate as judged by plasma levels being within therapeutic range,
therefore do need increasing. Increasing Enalapril would be desirable but
appropriate first
step. 5 )
53. A 32 year old woman presented local emergency general practitioner whilst on
holiday in Isle of Wight complaining of increasing urinary frequency, lower abdominal pain
180

dysuria. A diagnosis of a urinary tract infection was made a course of trimethoprim


prescribed.
Three days later, she suddenly started feel unwell headache nausea in a restaurant
following a fish course. She was known suffer from any allergies. She had a past history
of asthma, which was well controlled a salbutamol inhaler on as required basis.
On arrival in accident emergency department she looked flushed a sinus tachycardia of
110 beats per minute confirmed 12 lead electrocardiogram. Her temperature was normal.
The pressure was 90mmHg systolic 60mmHg diastolic.
Auscultation of her chest a widespread expiratory wheeze a peak expiratory flow rate of
200L/min. A widespread diffuse macular rash was also noted over trunk limbs.
What is the likely diagnosis?
1
2
3
4
5

)
)
)
)
)

Acute allergic reaction trimethoprim


Acute severe asthma
Meningococcal septiacaemia
Saxitoxin poisoning
Scombroid poisoning

Comments:
Scombroid poisoning associated consumption of Scrombroidea dark meat fish such as tuna,
mackerel marlin. The symptoms are due ingestions of amines, predominantly histamines,
produced by bacterial decarboxylation of histidine in fish meat. The common cause of
scombroid poisoning due ingestion of spoiled fish following inadequate refrigeration
prolonged time at room temperature. Cooking does inactivate toxin/histamines. The illness
usually self limiting severity dependent on amount of fish ingested i.e. symptoms are worse
when a large amount of fish ingested.
Onset usually 10 30 minutes post ingestion of implicated fish but a delayed onset may occur
up 2 hours. The symptoms occur are typically associated histamine such as headache,
dizziness, abdominal pain, palpitations, nausea, diarrhoea, urticarial rash, bronchospasm
hypotension hypertension. Patients pre existing conditions such as bronchial asthma, those
taking isoniazid (a histaminase inhibitor) may be more symptomatic.
Specific treatment usually unnecessary symptoms subside between 2 36 hours. In severe
cases, symptoms respond rapidly antihistamines e.g. chlorpheniramine intravenous cimetidine
by slow intravenous injection over at least 5 minutes. Adrenaline required in rare, very severe
cases. There role routine corticosteroid use.
Saxitoxin a neurotoxin produced by dinoflagellates proliferates when sea temperatures rise in
spring summer often make sea look red in colour (red tide). Various molluscs such as
mussels, clams, cockles may concentrate saxitoxin during such red tide. Clinical symptoms
such as dizziness, incoordination, weakness, numbness paraesthesiae can occur within minutes
up 10 hours post ingestion of saxitoxin. Respiratory failure (due muscle weakness) may
develop.
Acute type 1 allergic reaction trimethoprim would be unlikely 3 days after starting first dose.
Lack of temperature absence of a macular papular rash would make meningococcal septicaemia
unlikely. 5 )
54. A 33 year woman was referred by her GP a 4 month history of weight loss agitation.
On examination she had a fine tremor, a resting tacchycardia a smooth goitre.
Free T4
32.1 nmol/l
(10 23)
181

Free T3
10.2 nmol/l
(3.5 5.5)
TSH
< 0.02
She was commenced on Carbimazole 40mg daily together propranolol 120mg daily. One
month later she re attended clinic feeling better but gave a three day history of a sore
throat. No abnormalities were found on examination of throat she was apyrexial.
Investigations reveal:
Haemoglobin
11.5 g/dL
MCV
80 fl
Platelets
155 x 109/L
White cell count 4.1x109/L
Neutrophils
2.1x109/L
Lymphocytes
1.5 x 109/L
What is the appropriate management this patient?
1 ) Admit ward barrier nursing
2 ) Repeat tests in one week
3 ) Reassure continue carbimazole
4 ) Stop Carbimazole
5 ) Treat antibiotics
Comments:
There are abnormalities on this young womans FBC except a probable iron deficiency
probably relates menstruation. Frequently lowish WBC noted in thyrotoxicosis per se as well as
following introduction of thionamides this an indication stop therapy. However, therapy
should be stopped if there a demonstrable neutropaenia/agranulocytosis neutrophil count
below 1.5 x 109/l. Carbimazole induced agranulocytosis
<http://qjmed.oupjournals.org/cgi/content/full/92/8/455> (neutrophil count below 0.5 x 109/l)
fortunately rare but life threatening. Thionamides should be withdrawn, infection treated
appropriate antibiotics (broad spectrum cephalosporin) occasionally, G CSF required when
white count fails respond. 3 )
55. A 38 year old presents after injuring her arm whilst on holiday. The next day she
finds she cant wind down window of her car her friends note prominence of her
shoulder blade. Weakness of what muscle causes winging of scapula?
1 ) Serratus anterior
2 ) Subscapularis
3 ) Teres minor
4 ) Latissimus dorsi
5 ) Teres major
Comments:
The patient a winged scapula. Serratus anterior muscle affected innervated by long thoracic
nerve (C5, C6, C7). 1 )
56. A 40 year old gentleman admitted severe central chest pain. His ECG shows an acute
anterior myocardial infarction. He was discharged from hospital two days ago following
ten day admission a bleeding peptic ulcer. His only medication a proton pump inhibitor.
Investigations show:
182

Haemoglobin
13.1 g/dL
White cell count
12.4 x 109/L
Platelets
399 x 109/L
Serum sodium
137 mmol/L
Serum potassium
4.2 mmol/L
Serum urea
6.4 mmol/L
Serum creatinine
95 umol/L
Faecal occult blood
Negative
What is the appropriate management this gentleman?
1 ) Primary angioplasty
2 ) Start aspirin
3 ) Start intravenous nitrates a beta blocker but heparin
4 ) Start low molecular weight heparin
5 ) Thombolyse tissue plasminogen activator (t PA)
Comments:
A history of recent bleeding peptic ulcer a contraindication thrombolysis. He needs reperfusion
therapy therefore primary angioplasty treatment of choice. None of alternative options offer
this. 1 )
57. A 57 year old lady was admitted hospital increasing thirst generalised abdominal
pain. She was diagnosed breast carcinoma three years previously, treated a radical
mastectomy.
Investigations showed:
Serum corrected calcium
3.5 (NR 2.2 2.6)
Serum alkaline phosphatase
1100 IU/L
Her serum calcium was still elevated following 4 litres of 0.9% saline intravenous infusion.
Which of the following is appropriate next step?
1 ) Dexamethasone 8mg
2 ) Intravenous 0.9% saline infusion over 4 hours
3 ) Low calcium diet
4 ) Pamidronate 60mg intravenously
5 ) Radiotherapy
Comments:
This lady hypercalcaemia, which probably related bony metastases secondary breast
carcinoma. The calcium still elevated despite adequate hydration saline. The appropriate next
step give pamidronate , a bisphosphonate , inhibit bone resorption formation, thus
reducing bone turnover. 4 )
58. An 81 year old man admitted Casualty after collapsing at home. His wife calls an
ambulance but he recovers spontaneously. He had admitted hospital chest pain three
months previously was diagnosed as having an acute anterior myocardial infarction. On
examination he a diffuse apex bea a pulse of 82 beats per minute a pressure of 110/70
mmHg. His electrocardiogram shows ST segment elevation across anterior chest leads
frequent ventricular extrasystoles.
What is the likely diagnosis?
1 ) Acute anterior myocardial infarction
183

2 ) Heart failure secondary ventricular tachycardia


3 ) Intermittent complete heart block post MI
4 ) LV aneurysm formation secondary ventricular tachycardia
5 ) Sinoatrial node disease
Comments:
The clinical presentation suggestive of VT secondary a specific cause. The persistant ST
elevation suggests formation of a LV aneurysm post anterior MI. Heart block post anterior MI
likely occur within first few days post MI rather than 3 months. 2, 4, 5 are less likely due
presenting symptoms ECG findings. 4 )
59. A 19 year mentally handicapped man was brought your clinic by his carers. They
had noticed he had drinking increasing amounts of fluids. His past history included
epilepsy, treated carbamazapine 600mg twice a day, asthma.
Baseline investigations showed:
Plasma osmolality
279 mosmol/Kg (NR 278 305)
Urine osmolality
165 mosmol/Kg (NR 350 1000)
Following a water deprivation test:
Plasma osmolality
289 mosmol/Kg (NR 278 305)
Urine osmolality
310 mosmol/Kg (NR 350 1000)
There was additional response intranasal desmopressin.
Which of the following is thelikely cause?
1 ) Carbamazapine
2 ) Compulsive (psychogenic) polydipsia
3 ) Cranial diabetes insipidus
4 ) Nephrogenic diabetes insipidus
5 ) Syndrome of inappropriate antidiuretic hormone (SIADH)
Comments:
This patient a low plasma osmolality low urine osmolality, which increases during water
deprivation test.
Carbamazepine causes a syndrome of inappropriate ADH. This would be associated
concentrated urine, dilute plasma. The urine osmolality would change during water deprivation
test.
Compulsive polydipsia fits this pattern of findings. His urine starts concentrate during water
deprivation test. The changes in his plasma urine osmolalities following test are large. This
because prolonged polyuria leads reduction in maximal concentrating ability of kidney by
removing renal medullary interstitial solute altering aquaporin 2 function.
Desmopressin therefore unable elicit its maximal effect of increasing water reabsorption.
Cranial diabetes insipidus(DI) caused by decreased secretion of vasopressin from posterior
pituitary. In cranial DI plasma osmolality following water deprivation test would rise
dangerously, an effect would be seen following administration of desmopressin (increased urine
osmolality).
Nephrogenic diabetes insipidus caused by renal tubular resistance vasopressin. In this case
plasma osmolality would rise during water deprivation test, but urine would remain dilute, even
after desmopressin administration. 2 )
60. A 23 year old student brought in Casualty department drowsy by her friend. The
patient lives alone in a one bedroom flat. She normally fit well although had recently
184

complaining of difficulty concentrating in lectures. She smokes 20 cigarettes a day. She


was on medication had previous history of note. She had vomited.
On examination she was flushed. She had a bounding pulse of 120 beats per minute. Her
pressure was 180/100 mmHg. Oxygen saturations were normal.Initial investigations
Haemoglobin
12.8 g/dL
White cell count
10.5 x 109/L
Platelets
280 x 109/L
Serum sodium
134 mmol/L
Serum potassium 3.6 mmol/L
Serum urea
7.4 mmol/L
Serum creatinine
80 umol/L
Drug screen
Negative
Arterial gases on air:
pO2
8.6 kPa
pCO2
4.7 kPa
pH
7.42
The chest x ray was normal. What investigation would confirm the diagnosis?
1 ) Blood Glucose
2 ) Blood Lactate
3 ) Carboxy Haemoglobin
4 ) Electroencephalogram
5 ) Lumbar Puncture
Comments:
This girl classical features of carbon monoxide poisoning <site/Paedia.asp?id=160>. Carbon
monoxide binds haemoglobin a greater affinity than oxygen displacing it from causing tissue
hypoxia. In addition carbon monoxide shifts oxygen dissociation curve left reducing tissue
delivery even more.
Symptoms of mild poisoning (Carboxyhaemoglobin levels = 10 30%) are headache, tiredness,
nausea, dizziness poor concentration. With increasing levels vomiting weakness then impaired
consciousness may occur hypertensin, tachycardia flushing.
With severe poisoning (Carboxyhaemoglobin levels > 50%) convulsions, coma, respiratory
depression death can occur.
Treatment 100% oxygen through a tight fitting, non re breathing face mask at a flow rate of 10
L/min. In severe cases intubation mechanical ventilation may be required in these patients
there a place hyperbaric oxygen. 3 )
61. A 45 year old woman presented 6 months history of slurring of her speech,
expressionless face, right partial ptosis diplopia. All her symptoms are worse in evening.
What is the appropriate diagnostic clinical examination?
1 ) Count numbers up 50 aloud
2 ) Demonstration of diminished reflexes
3 ) Demonstration of tongue fasciculations
4 ) Gait
5 ) Pupil reaction light
Comments:
185

Myasthenia gravis an autoimmune disease. It characterised by fluctuating skeletal muscle


weakness worsens use improves rest. Eye, facial, oropharyngeal, axial, limb muscles may be
involved in varying combinations degree of severity. The symptoms usually increase as day
progresses. The demonstration of fatigability cardinal feature of myasthenia gravis. 1 )
62. A 56 year old lady presents sudden onset occipital headache, vomiting depressed
conscious level. She was previously well significant past history. In particular there
history of exertional chest pain, asthma diabetes. She a non smoker.
Examination reveals neck stiffness, equal pupils a GCS of 12. Pulse 84 bpm, BP 140/90.
Chest clear auscultation.Laboratory findings are normal. An urgent CT brain scan reveals
an acute subarachnoid haemorrhage she transferred neurosurgery.After conducting
further investigations neurosurgery team plan take her theatre clipping of an aneurysm.
The anaesthetic team are concerned about her ECG shown below.

What likely explanation from list of options?


1)
acute myocardial infarction
2)
ECG changes due intracranial haemorrhage
3)
Left Bundle Branch Block
4)
pulmonary embolism
5)
ventricular pre excitation
Comments: The ECG shows a shortened PR interval a delta wave. In this case it probably best
seen in lateral chest leads V3 V6. Ventricular pre excitation (if there was a history of
tachycardia it would be Wolf Parkinson White syndrome) commonly masquerades as other
conditions such as bundle branch block ischaemia.
Intracranial haemorrhage can cause changes in ECG which are typically deep symmetrical T
wave inversion prolonged QT interval. Acute pulmonary embolism unlikely as there are none of
transient features one would expect see (S1Q3T3 pattern, RV strain tachycardia).5)

63.
A 47 year old man HIV disease presents hospital a tonic clonic seizure. He had initially
presented six months previously Pneumocystis carinii pneumonia a CD4 T lymphocyte count of 10
cells/mm3 had subsequently started on highly active antiretroviral therapy. His recent CD4 count,
taken one month prior his new presentation, was 50 cells/mm3. On examination he focal
weakness, but both plantar responses are extensor. Fundoscopy normal.A CT scan of his brain
shown. What is the diagnosis?

186

1
2
3
4
5

)
AIDS related dementia
)
Brain abscess
)
Cerebral toxoplasmosis
)
Primary CNS lymphoma
)
Progressive multifocal leukoencephalopathy
Comments: Toxoplasma encephalitis commonest cause of focal brain disease in HIV/AIDS,
occurring at CD4 counts of less than 100 cells/mm3. Alhough Toxoplasma gondii may also
cause a retinitis in association HIV/AIDS, it need occur concomitantly CNS disease. The typical
appearance on CT of multiple ring enhancing lesions, as shown here. Treatment
pyrimethamine + sulfadiazine. Patients must subsequently be maintained on long term
suppressive therapy prevent relapse.3)
64.
A 9 year old boy presents casualty visual disturbance. A visual field defect detected on
confrontational testing. Fundoscopy reveals appearance shown below. What is the likely cause of
this appearance?

1
2
3
4
5

)
Cytomegalovirus
)
Medullated nerve fibres
)
Retinoblastoma
)
Toxocara canis
)
Toxoplasma gondii
Comments: The slide shows typical appearance of toxocara retinitis a lesion at macula. In
retinitis due Toxocara canis, there usually only a single, well demarcated lesion.4)
65. A 72 year old lady complains of gradually increasing breathlessness over a period of
6 months. She complains of orthopnoea paroxysmal dyspnoea. She was previously well.
187

She had treated hypertension by her GP 10 years ago but lately diuretics were stopped
because of low pressure.
Examination reveals she breathless on minimal exertion. She an irregular pulse, BP 95/60.
Auscultation of heart a loud first heart sound a soft systolic murmur at apex. There were
fine crackles in both lung bases.
Her ECG shown.

What is the most likely diagnosis?


1)
alcoholic cardiomyopathy
2)
bronchiectasis
3)
hypertensive cardiac failure
4)
ischaemic heart disease
5)
mitral stenosis
Comments: The ECG shows atrial fibrillation Right Ventricular Hypertrophy (right axis deep S
waves in lateral chest leads usually there would be a dominant R wave in lead V1 but in this
case perhaps because of positioning of chest lead).
The prominent first heart sound also a clue diagnosis of mitral stenosis. The soft systolic
murmur a little mis leading; care needs be taken listen diastolic murmurs and, given this ECG,
you should return patient confirm features of mitral stenosis ... request an ECHO of course.

1.
A 60 year old man medication controlled essential hypertension returns clinic. He
being treated bendrofluazide 2.5mg od, enalapril 20mg od, ibuprofen 600mg tds. His
pressure in clinic 130/78 mmHg.
Investigations show today:
188

Serum
Serum
Serum
Serum

sodium
potassium
urea
creatinine

129 mmol/L
5.2 mmol/L
18 mmol/L
270 umol/L

Investigations show three months previously:


Serum sodium
137 mmol/L
Serum potassium 4.4 mmol/L
Serum urea
6.9 mmol/L
Serum creatinine
98 umol/L
Which one of following would be likely cause of change in his serum urea electrolytes?
1
2
3
4
5

) Hypertensive renal disease


) Hypovolaemia
) Interstitial nephritis secondary NSAIDs
) Treatment an angiotensin converting enzyme (ACE) inhibitor
) Urinary tract infection

Comments:
This gentlemans renal function deteriorated suddenly. All five choices could account
deterioration, but likely cause interstitial nephritis secondary NSAIDs.
Acute interstitial nephritis inflammation of renal tubulo interstitium, secondary a
hypersensitivity reaction drugs. The common drug related cause NSAIDs. Other precipitating
drugs include antibacterials (penicillins, cephalosporins, sulphonamides, rifampicin), loop
diuretics (furosemide), thiazide diuretics, amphotericin, cimetidine allopurinol.
Features include acute, commonly oliguric renal failure, without systemic features which
include fever, arthralgia skin rashes. Many patients have eosinophilia, raised serum IgE
eosinophiluria. Renal biopsy shows oedema of interstitum infiltration of plasma cells,
lymphocytes eosinophils, acute tubular necrosis variable tubular dilatation.
The treatment includes withdrawal of offending drug, may involve dialysis until renal function
returns.
This gentlemans BP 130/78; therefore there evidence he hypovolaemia.
ACE inhibitors can cause acute deterioration in renal function, mainly in patients bilateral
renovascular disease, commonly within first two weeks of treatment. ACE inhibitors may also
increase serum potassium, through impairment of angiotensin II mediated secretion of
aldosterone. The higher serum creatinine concentration, greater risk of hyperkalaemia.
Uncommonly, ACE inhibitors may cause progressive renal impairment in patients without
renovascular disease, especially elderly, which may be caused by a membranous
glomerulonephritis. There evidence this gentleman a urinary tract infection. The dose of
digoxin should be reduced in renal failure, but it a cause of deteriorating renal function 3 )
2.
A 47 year old lady a positive family history of hypertension premature coronary
artery disease referred outpatient clinic assessment of poorly controlled hypertension.
Her pressure in clinic measured at 200/100 mmHg. An MRI scan of her aorta renal
arteries shows severe atheromatous stenosis in both renal arteries.
189

What best way of treating her elevated pressure?


1
2
3
4
5

)
)
)
)
)

ACE inhibitors
Alpha blockers
Beta blockers
Bilateral renal artery stenting
Methyldopa

Comments:
The best long term treatment this lady bilateral renal artery stenting. ACE inhibitors are
contraindicated. Other pharmacological treatments are always going come off second best
stenting as there will be untreated bilateral renal artery stenoses. 4 )
3.
A 21 year old student presented hospital two weeks after returning from a three
month elective period in Africa. He gave a seven day history of a non productive cough a
fever. He had seen his General Practitioner previous day had started on a course of
amoxicillin.
He had spent his elective in Kenya, but had also spent time travelling in Malawi South
Africa. He developed a febrile illness diarrhoea two weeks before returning UK was seen
by a doctor in South Africa prescribed a course of metronidazole presumed amoebiasis
his symptoms settled within three days. There was other past history of note.
On examination he was febrile (38.5oC) an urticarial rash was visible over trunk. There
was palpable lymphadenopathy. His pulse was 90 beats per minute in sinus rhythm
pressure 115/65 mmHg. His chest was clear. His abdomen was soft slightly tender in left
hypochondrium, where tip of spleen could be palpated.
Investigations showed:
Haemoglobin
14.0 g/dL (NR 13.0 - 18.0)
White cell count
9.3 x 109/L (NR 4.0 - 11.0)
Neutrophils
5.3 x 109/L (NR 1.5 - 7.0)
thLymphocytes
1.5 x 109/L (NR 1.5 - 4.0)
Monocytes
0.1 x 109/L (NR <0.8)
Eosinophils
1.4 x 109/L (NR 0.04 - 0.4)
Basophils
0.1 x 109/L (NR <0.1)
Platelets
350 x 109/L (NR 150 - 400)
Chest x ray:
Normal
Amoebic serology:
Negative
What is the likely diagnosis?
1
2
3
4
5

)
)
)
)
)

Amoebiasis
HIV seroconversion illnes
Schistosomiasis
Trypanosomiasis
Typhoid fever

Comments:
190

The symptoms of acute schistosomiasis are very similar any acute viral bacterial infection it
therefore important establish a travel history an endemic area. Some patients develop a rash at
site of entry of cercariae into skin (swimmers itch) others develop systemic upset. This
systemic upset (Katayama syndrome) what seen in this case. Fever, lethargy, myalgia are
common symptoms; patients may also have cough, headache, anorexia, a generalized rash. If
acute schistosomiasis unrecognized, chronic infection occurs can lead significant morbidity
mortality. The other significant marker in this question presence of an eosinophilia - strongly
suggestive of parasitic infection - a a feature of any of other choices listed.
Schistosoma mansoni
Geographical distribution: Caribbean, eastern Mediterranean countries, South America,
countries in Africa Causes: Intestinal schistosomiasis liver disease (fibrosis, portal
hypertension). Some patients hepstosplenic disease develop schistosomal cor pulmonale. Spinal
schistosomiasis, presenting as transverse myelitis (travellers myelitis), primarily due S.
mansoni infection
Schistosoma japonicum
Geographical distribution: western Pacific countries (China, Philippines, Indonesia, Thailand).
Causes: Intestinal schistosomiasis liver disease (fibrosis, portal hypertension). Also associated
cerebral schistosomiasis.
Schistosoma haematobium
Geographical distribution: Africa eastern Mediterranean Causes: Urinary tract disease. Rarely
causes intestinal liver disease. Chronic infection associated development of bladder cancer. 3 )
4.
A 34 year man presents a swollen, painful right calf. A doppler ultrasound confirms
presence of a posterior knee occlusive deep vein thrombosis. Two weeks previously a
below knee plaster cast had removed on right side. This had in place 4 weeks following
a rugby injury. He had other significant past history. What best management plan this
man?
1 ) No treatment necessary
2 ) Low molecular weight heparin followed by 3 months.
3 ) Low molecular weight heparin followed by 6 months
4 ) TED support stockings repeat doppler in 1 week.
5 ) Warfarin only
Comments:
This gentleman a DVT secondary an acquired risk factor now removed. Although risk factor
removed, patient should be anticoagulated.
Candidates should know in this situation duration of treatment 3 months. They should also
know Warfarin should never be given alone, without heparin when treating a thrombotic event.
This because when started, there a pro thrombotic state created, by falling Protein c Protein
S, these are Vitamin K dependent natural anticoagulants, these fall before II, VII, IX ands X.2)
5.
A 50 year presents palpitations slight weight loss. She an uneventful past history
but takes supplements she buys in a health food shop. She a non smoker drinks little
alcohol. There a maternal aunt who an thyroid but nothing else of note in her history.
On examination she a pulse of 96 beats per minute a blood pressure of 122/76 mmHg.
She has fine tremor of the outstretched hands there slight lid lag but exophthalmos. No
goiter is palpable. Thyroid function tests reveal:
Free T4
29.3 nmol/L (NR 10 23)
Free T3
5.3 nmol/L (NR 3.5 5.5)
191

TSH

< 0.02 mu/L (NR 0.5 5.0)

What is the likely diagnosis?


1 ) Dysthyroglobinemia
2 ) Graves disease
3 ) Hashitoxicosis
4 ) Factitious thyrotoxicosis
5 ) Toxic nodule
Comments:
The two likely answers are either Graves disease a toxic nodule. As patient does have features
of Graves disease (no goitre, exophthalmos/proptosis) then toxic nodule probable diagnosis.
The family history of thyroid disease in maternal aunt a red herring. The features are
consistent factitious thyrotoxicosis. 5 )
6.
A 16 year girl was admitted hospital nausea vomiting due possible viral
gastroenteritis. She was given some intravenous anti emetic earlier in Accident Emergency
department. You have called urgently onto ward because nursing staff informs you she
fitting. On arrival, patient was looking upwards in a fixed stare, had a stiff neck spasm of
jaw muscles. What would be your treatment of choice?
1 ) Intravenous benzatropine
2 ) Intravenous dantrolene
3 ) Intravenous cefotaxime
4 ) Oral bromocriptine
5 ) Rectal diazepam
Comments:
Acute dystonic dyskinetic reactions mostly occur in children young adults about 70% of cases
are female. It occurs more commonly when excess of recommended dose administered.
Symptoms include oculogyric crisis, opisthotonus, torticollis, trismus tetanus like reactions. A
blue discolouration of tongue also described. The effects usually occur within 72 hours but
have reported occur within 30min of starting treatment. Although generally self limiting,
reaction can be reversed by an anticholinergic such as benzatropine procyclidine an
antihistamine such as diphenhydramine. 1 )
7.
A 26 year old man attends dental hospital a tooth extraction. He reports prolonged
bleeding after a tooth extraction 2 years previously. He had other surgical challenges
other history of prolonged bleeding. He otherwise fit well. On further questioning he said
his mother had recently referred a local hospital recurrent nose bleeds menorrhagia. He
was also aware his sister had seen in past menorrhagia.
The dental extraction cannot be delayed. What should dentist be advised give patient
reduce risk of significant bleeding?
1 ) Cryoprecipitate
2 ) Factor VIII replacement
3 ) Fresh frozen plasma
4 ) Tranexamic acid
5 ) von Willebrand Factor concentrate
Comments:
The bleeding described a defect of primary haemostasis. It inherited as mother sister are
affected. The likely diagnosis von Willebrand (vWB) disease. It likely be mild (Type I) as patient
family members have reached a good age major bleeding problems. The appropriate
192

management therefore good surgical technique, pressure oral tranexamic acid. The side effects
are minimal. The choice of treatment a mild vWB facing a more invasive procedure would be
DDAVP, providing contraindication. VWB factor concentrate would be reserved as 2nd line
treatment DDAVP. FFP cryoprecipitate are used in von Willebrand Disease.
Haemophilia diagnosis a defect of primary haemostasis generally females are not affected as
it is X linked. 4 )
8.
A 43 year old man who known smoke 20 cigarettes a day last 20 years admitted a
three day history of increasing breathlessness. He an associated cough productive of
purulent sputum noticed some associated left sided chest pain. His appetite poor over
last 48 hours as a result he admits drinking eating very much over this time.
He a history of hypertension which he currently taking Bendrofluazide 2.5 mg once daily.
His father died of bronchial carcinoma at age of 68 years. He works as a plumber. He
drinks four pints of beer a night.
On examination he had a temperature of 38.5oC. Respiratory rate 26 breaths per minute.
There was an area of bronchial breathing at left base associated coarse crackles.
Investigations revealed:
Haemoglobin
15.8 g/dL
White cell count
3.8 x 109/L
Platelets
100 x 109/L
Serum sodium
142 mmol/L
Serum potassium
4.2 mmol/L
Serum urea
14.8 mmol/L
Serum creatinine
160 umol/L
The chest x ray showed an area of dense consolidation in left lower zone.
Which one of following NOT associated a poorer prognosis in community acquired
pneumonia:
1
2
3
4
5

)
)
)
)
)

Total white cell count of less than 4 x 109/L


Serum creatinine of greater than 400 umol/L
Multilobar involvement on chest x ray
Platelet count of less than 100 x 109/L
Temperature greater than 40oC

Comments:
There are numerous predictors of increased severity risk of death from community acquired
pneumonia including white cell count less than 4 x 109/L greater than 20 x 109/L, co morbidity
such as renal disease, multi lobar involvement on CXR temperature < 35oC >40oC.
These have incorporated into severity scores such as pneumonia severity index (PSI) (1),
Modified American Thoracic Society rule (2) CURB 65 score (3). The CURB 65 score easily used
in practice derived from:
C
Confusion
U
Urea > 7 mmol/L
R
Respiratory rate >30/min
B
Blood pressure <60 mmHg diastolic <90 mmHg systoloic
65
Age >65
193

References:
1. Fine MJ, Auble TE, Yealy DM et al. A prediction rule identify low risk patients community
acquired pneumonia. N Engl J Med 1997;336: 243 250.
2. Ewig S, Ruiz M, Mensa et al. Severe community acquired pneumonia: assessment of severity
criteria. Am J Respir Crit Care Med 1998;158:1102 1108.
3. Lim WS, van de Eerden MM, Laing R et al. Defining community acquired pneumonia severity on
presentation hospital: an international derivation validation study. 4 )
9.
A 46 year old man was admitted Casualty from home a one week history of mild
global headache. On day of admission he found difficulty in expressing himself verbally.
He had a generalised seizure in ambulance on way hospital.
On examination he was fully conscious a Glasgow coma scale score of 15/15. He was
febrile (38oC) pulse 80 beats per minute in sinus rhythm pressure 130/75 mmHg. There
was nuchal rigidity. Neurological examination an expressive dysphasia mild right sided
weakness.
An MRI scan of his brain showed abnormal signals in both temporal lobes, but was
prominent on left where a degree of mass effect was noted.
A lumbar puncture was performed CSF analysis showed:
Opening pressure:
Normal
CSF protein:
Normal
CSF glucose:
Nomal
Cells:
9 lymphocytes/mm3
Gram stain:
Negative
What is the likely diagnosis?
1 ) Acute disseminated encephalomyelitis
2 ) Bacterial meningitis
3 ) Herpes simplex encephalitis
4 ) Lyme disease
5 ) Tuberculous meningitis
Comments:
This a typical history HSV encephalitis. Characteristic symptoms include a combination of :
fever, headaches, confusion, odd behaviour, dysphasia seizures. Patients may have olfactory
gustatory hallucinations. MRI may show oedema of temporal lobe(s). The diagnosis confirmed
by demonstrating presence of virus in CSF (PCR quickest method).
Acute disseminated encephalomyelitis (post infectious encephalomyelitis) may occur
approximately two weeks after a viral infection (measles, chickenpox, rubella mumps, influenza).
The clinical presentation similar encephalitis fever, convulsions coma +/ involuntary
movements. There prior history of viral infection in this case this option therefore less likely.
Both Borrelia burgdorferi (the causative agent in Lyme disease) can cause encephalitis but there
other supportive evidence Lyme therefore less likely.
The clinical presentation CSF results do support diagnosis of either bacterial meningitis
tuberculous meningitis. 3 )
10. A 66 year old gentleman known ischaemic heart disease sustains a cardiac arrest at
home whilst watching television. His wife calls 999 starts cardiorespiratory resuscitation.
On admission hospital he in sinus rhythm. A transthoracic echocardiogram reveals
significant left ventricular dysfunction.
What best long term management strategy this gentleman?
194

1 ) ACE inhibitor
2 ) Amiodarone
3 ) Beta blocker
4 ) Implantable defibrillator
5 ) Mexilitene
Comments:
This gentleman ischaemic cardiomyopathy.He presented an out of hospital cardiac arrest
successfully resuscitated. ICD implantation best long term management strategy this
gentleman as there a clear morbidity mortality benefit over above antiarrythmic therapy in this
condition, including amiodarone. Long term mexilitene therapy place at this stage in his
treatment probably proarrythmic in this situation. Beta blocker, statin ACE inhibitor therapy all
have a mortality benefit in either heart failure treament ischaemia may decrease incidence of
cardiac arrests but regardless of their usage ICD therapy first line treatment in this patient. They
are complementary therapies. 4 )
11. An 18 year old Zimbabwean man presented a two week history of massive painless
cervical lymphadenopathy.
Which of following investigations should be performed obtain diagnosis?
1 ) Bone marrow aspiration trephine biopsy
2 ) Chest x ray
3 ) CT scan chest
4 ) HIV antibody test
5 ) Lymph node biopsy
Comments:
Given lymphadenopathy massive painless, likely diagnosis a lymphoproliferative disorder
Hodgkins disease, rather than infection other cause. The mass appeared rapidly this would
make you think of a lymphoroliferative disorder a short doubling time e.g. Burkitts lymphoma.
The clue this he from Africa, Burkitts being endemic in parts of Africa, can also be seen
immunodeficiency states such as HIV. Only a lymph node biopsy will give you a definitive
diagnosis. A bone marrow full body CT done as part of staging a lymphoproliferative disorder;
but will necessarily provide you a definitive diagnosis. 5 )
12. A 27 year woman presents paracetamol poisoning 58 hours post ingestion of 30
paracetamol tablets. Her history considered be reliable her sugar by finger prick test
was 3.1 mmol/L. What would be your treatment of choice?
1 ) Haemodialysis
2 ) Intravenous dextrose infusion
3 ) Intravenous N acetylcysteine
4 ) Oral activated charcoal
5 ) Oral vitamin K
Comments:
Paracetamol concentration unhelpful in delayed presentation of paracetamol overdose. A dose
of >150mg/kg considered be toxic toxicity occurs at a lower concentration if patient thought
be in high risk group. Clotting screen (INR), liver function tests, acid base balance plasma
glucose should be taken without delay prior starting treatment N acetylcysteine. Serial
monitoring of clotting screen required N acetylcysteine should continue until INR returns
normal. Further advice on treatment of complicated cases should be sought from National
Poisons Information Service. Other supportive treatment guided by patients condition. 3 )
195

13. A 64 year old man a history of renal transplantation 8 years ago presented fever
pleuritic right sided chest pain. His symptoms started 3 days previously a flu like illness
associated generalised arthralgia.
He subsequently developed a cough productive of purulent green sputum occasional
haemoptysis on day of admission his wife found him collapsed in bathroom.
He was under 6 monthly review by renal physicians was taking azathioprine ciclosporin.
He was recently diagnosed having gout was started on indometacin followed by
allopurinol.
On examination he looked pale a tachycardia of 110/minute regular. His pressure was
100/60 his heart sounds normal. Examination of his chest dull percussion, increased
vocal resonance bronchial breathing over right base.
Investigations on admission showed:
Haemoglobin
6.4 g/dL
Neutrophils
0.2 x 109/L
Lymphocytes
0.1 x 109/L
Platelets
45 x 109/L
Serum sodium
138 mmol/L
Serum potassium
4.1 mmol/L
Serum chloride
100 mmol/L
Serum bicarbonate
18 mmol/L
Serum urea
22.4 mmol/L
Serum creatinine
253 umol/L
Plasma glucose
7 mmol/L
What is the likely cause of his illness?
1 ) Azathioprine toxicity
2 ) Ciclosporin toxicity
3 ) Type A adverse reaction indometacin
4 ) Type B adverse drug reaction due allopurinol
5 ) None of above
Comments:
Allopurinol acts by inhibition of xanthine oxidase thus inhibit metabolism of 6 mercaptopurine,
an active metabolite of azathioprine. Concomitant use of allopurinol azathioprine
recommended if concomitant use occur, a dose reduction in azathioprine by 25% advised
regular count monitoring. Azathioprine toxicity commonly causes bone marrow suppression
thus sepsis as seen in this scenario.
Ciclosporin levels can also rise concomitant administration of allopurinol but a lesser extent.
Ciclosproin toxicity more commonly cause renal toxicity.
Blood dyscrasias such as thrombocytopenia, leucopenia agranulocytosis can occur indometacin
but this an indiosyncratic reaction (type B adverse reaction).1)
14. A 43 year old gentleman presents Casualty a three hour history of palpitations. He
a history of frequent attacks of palpitations which are terminated by carotid sinus
massage. He also a history of asthma since childhood takes regular inhaled salbutamol
budesonide.
The 12 lead ECG shows a supra ventricular tachycardia.
196

What treatment should be given stop tachyarrhythmia?


1
2
3
4
5

)
)
)
)
)

Adenosine
Amiodarone
Digoxin
Lignocaine
Verapamil

Comments:
This patient' SVT should be terminated Verapamil. This drug second line treatment in SVT
treatment after adenosine. However in this patient, adenosine relatively contraindicated history
of asthma as it can precipitate an attack. Amiodarone digoxin do have a place routinely
terminating SVTs lignocaine mainly used treatment of ventricular tachycardias effect on AV
nodal conduction. 5 )
15. A 52 year male who a five year history of type 2 diabetes mellitus admitted chest
pain. His usual drug therapy includes metformin 500 mg tds he also takes bendrofluazide
2.5 mg daily hypertension.
On examination he obese a BMI of 32kg/m2, a pulse of 88 beats per minute a pressure
of 148/92 mmHg. His ECG shows ST elevation in leads II, III aVF. He receives streptokinase
his BMs show values between 7 - 10 mmol/l. His plasma glucose concentration obtained
from lab 8.8 mmol/l.
What is the appropriate treatment his glycaemic control?
1
2
3
4
5

)
)
)
)
)

Add gliclazide metformin


Change metformin gliclazide
Commence intravenous insulin infusion
Continue current dose of metformin
Increase metformin

Comments:
The DIGAMI study published in BMJ demonstrated significant reductions in mortality in subjects
diabetes MI treated sliding scale insulin (followed by 3 months of sc insulin) compared
conventional therapy their oral hypoglycaemic agents. 3 )
16. An otherwise fit well 76 year old gentleman presents syncope a nine second sinus
pause.
Which permanent pacemaker (PPM) should be implanted?
1
2
3
4
5

)
)
)
)
)

AAI
Biventricular
DDDR
VVI
VVIR

Comments:
This gentleman 76 years a 9 second asystolic pause causing syncope. With breakthrough
ventricular activity he should therefore have a dual chamber permanent pacemaker (DDDR). The
R in this code stands responsive, in an otherwise fit well 76 year he should have a responsive
element his PPM (ie increases his heart rate exercise). AAI/VVI/VVIR pacemakers alone are
197

insufficient as he needs both chambers paced. A biventricular pacemaker can be indicated in


heart failure patients reduced exercise capacity therefore warranted here.
Suggested references:
Schilling R, et al. Insertion of permanent pacemakers. British Journal of Cardiology 1999; 6:550
6.
Lamb D, et al. Who's a pacemaker? British Journal of Cardiology 3 )
17. A 32 year Nigerian lady sickle cell anaemia (Hb SS) a history of recurrent back pain.
She presents Casualty fever a worsening of back pain. There history of weight loss
night sweats.
Investigations show:
Haemoglobin
5.8g/dL
White cell count
475 x 109/L
Reticulocytes
12%
Serum total bilirubin
88 umol/L
What is the likely diagnosis?
1
2
3
4
5

)
)
)
)
)

Aplastic crisis
Haemolytic crisis
Malaria
Tuberculosis
Vasoocclusive event

Comments:
The results are suggestive of a sickling episode. The Hb slightly lower than would expect
normally, a raised reticulocyte count hyperbilirubinaemia. There evidence of an aplastic crisis
as Hb reasonable a good reticulocyte count. Conversely, Hb low enough reticulocyte count
bilirubin high enough a haemolytic crisis.
Vasoocclusive events can cause bone pain. Triggers include infection, dehydration , alcohol
change in temperature. Likely infection cause in this case as patient fever. TB malaria are as
likely from clinical scenario. Homozygotes HbS can get malaria. 5 )
18. A 52 year gentleman a known history of hypertension, attends outpatient clinic a
pressure of 150/90. He previously reduced his salt intake but continues drink 6 bottles of
wine per week. He on a beta blocker thiazide diuretic treatment.
What is the next step in treatment?
1 ) Commence an ACE inhibitor
2 ) Increase thiazide diuretic dose 5mg
3 ) Reassurance
4 ) Reduction of alcohol intake
5 ) Reduction of salt in cooking ingredients
Comments:
Reduction of alcohol intake is the next step in his treatment. Non pharmacological manoeuvres
are paramount the first line in hypertension management therefore an ACE inhibitor should be
next step. He is already reduced his salt intake. Reassuration will not bring his BP below
current guidelines. Increasing the diuretic dose gives very little further BP lowering effect but
increases the side effect profile. 4 )
198

19. A 55 year old male presents at diabetic annual review. He is generally well and
receiving metformin 500 mg tds. On examination, he has a BMI of 32, he a pressure of
144/88 mmHg a slight reduction sensation in his feet pinprick. There evidence of any
retinopathy.
Investigations show:
Serum sodium
138 mmol/L
Serum potassium 3.7 mmol/L
Serum urea
6.8 mmol/L
Serum creatinine
85 umol/L
Total cholesterol
6.8 mmol/l
Triglycerides
2.1 mmol/L
HbA1c
9.2%
Fasting glucose
9.8 mmol/L
What is the appropriate treatment this patients dyslipidaemia?
1 ) Diet
2 ) Ezetimibe therapy
3 ) Fibrate therapy
4 ) Insulin therapy
5 ) Statin therapy
Comments:
In this patient hypercholeserolaemia would predispose this patient an increased cardiovascular
risk which compounded by his diabetes. Studies suggest diabetes a coronary artery disease
equivalent i.e. cardiovascular risk associated diabetes similar of patients established IHD.
Therefore evidence from CARDS, HPS etc indicate subjects diabetes should receive statin
therapy as this significantly reduces cardiovascular events.
Fibrates are more effective on hypertriglyceridaemia, Ezetimibe would be associated an
approximate 20% reduction in cholesterol diet a paltry 10%. Statins such as Rosuvastatin may
reduce cholesterol by as much as 60%. 5 )
20. A 25 year of Bangladeshi origin presents with weight loss and fatigue of
approximately four months duration. She arrived back in UK 3 months ago after spending
one year in Bangladesh returned due ill health. She otherwise quite well other past
history, two children, a non smoker drinks alcohol.
On examination she thin a BMI of 20 kg/m2, obvious pigmentation of palmar creases,
pigmentation of buccal mucosa, a pulse of 77bpm a pressure of 100/62 mmHg. No other
abnormalities are evident on examination.
Investigations show:
Haemoglobin
11.2 g/dL
MCV
78 fl
White cell count
9 x 10 9/L
Serum sodium
130 mmol/L
Serum potassium
5 mmol/L
Serum urea
7.8 mmol/l
Serum creatinine
110 umol/L
Plasma glucose
5 mmol/L
199

ESR (Westergren)
60 mm/1sthr
9am plasma cortisol 90 nmol/L (NR 200 550)
What would be best investigation establish diagnosis in this patient?
1
2
3
4
5

)
)
)
)
)

CT abdomen
CT pituitary
CT thorax
PA Chest X ray
Radiolabelled white cell scan

Comments:
This young woman presents after returning from a long period in Bangladesh weight loss
lethargy. Her results are highly suggestive of a primary adrenal failure (pigmentation indicating
elevated ACTH hence primary adrenal dysfunction), low sodium, low BP low random cortisol. In
this case high ESR, TB adrenalitis should be considered in differential but also Addisons
disease still a possibility. The mostb appropriate initial investigation would be confirmation of
hypoadrenalism a short synacthen test. From list above a CT adrenals would be logical
absence/shrinkage enlargement of adrenals may be seen. Although a CXR would be an
appropriate initial investigation this may be despite possibility of TB. 1 )

21. In a patient aortic stenosis, what will lead an overestimation of severity of problem
when assessed by echocardiography?
1 ) Aortic regurgitation
2 ) Ebsteins anomaly
3 ) Mitral regurgitation
4 ) Mitral stenosis
5 ) Pulmonary hypertension
Comments:
An overestimation of severity of aortic stenosis can occur due large volumes of passing over
valve at high velocities, which occurs in aortic regurgitation. Ebsteins relationship pure aortic
stenosis nor does pulmonary hypertension. Mitral disease either in form of regurgitation
stenosis does increase trans aortic valve flow. 1 )
22. A 42 year old woman admitted hospital investigation of deteriorating renal
function. A renal biopsy shows focal necrotising glomerulonephritis, in keeping a
diagnosis of Wegeners granulomatosis. On examination she appears well. Her pressure
147/84 mmHg pulse 90 beats per minute, regular. Her respiratory rate 24 per minute
oxygen saturations of 96% on an inspired oxygen concentration of 28%.Investigations:
Serum sodium
134 mmol/L
Serum potassium
6.4 mmol/L
Serum urea
52 mmol/L
Serum creatinine
940 umol/L
Arterial gases
(28% FiO2)
pH
7.2
(NR 7.36 7.44)
200

PCO2
6.0 kPa
PO2
10.1 kPa
Actual bicarbonate
12 mmol/L
What should be first step in treating
1 ) Cyclophosphamide
2 ) Haemodialysis
3 ) Methylprednisolone
4 ) Plasma exchange
5 ) Positive pressure ventilation

(NR 4.7 6.0)


(NR 11.3 12.6)
(NR 20 28)
this patient?

Comments:
Wegeners granulomatosis a vasculitis affecting small vessels of kidney other organs,
especially lungs. It usually presents rapidly progressing renal failure (within 3 months),
proteinuria microscopic haematuria. It almost always associated upper airway involvement,
e.g. nasal ulceration, crusting, rhinorrhoea epistaxis, patients have evidence of granulomatous
lung disease at presentation, which often accompanied by alveolar capillaritis.
This lady indications dialysis. She acidotic, hyperkalaemic uraemic. She tachypnoeic
tachycardic, but her oxygenation normal. It likely she a degree of pulmonary involvement, but
this does require specific intervention at present.
This lady needs urgent treatment of underlying vasculitis. Without treatment Wegeners
invariably fatal, but immunosuppression 70% of patients focal necrotising glomerulonephritis
severe enough require dialysis at presentation regain independent renal function.
Methylprednisolone should be given immediately, followed by haemodialysis then
cyclophosphamide. Plasma exchange may have benefits in patients already requiring dialysis,
could then be commenced following this. 3 )
23. A 15 year girl referred urgently clinic because of a two day history of spontaneous
bruising. She past history of note taking any regular prescribed medication. She noticed
spontaneous appearance of bruises on her hips, thighs upper arms over past three days.
There history of trauma account their appearance. The largest of these measures 15cm
in diameter. Otherwise she feels well, though she reports having had a mild viral illness
two weeks previously.
Investigations show:
Haemoglobin
14.1 g/dL
White cells
7.3 x 109/L
Platelets
15.x 109/L
What is the next important next step?
1 ) Blood film examination
2 ) Bone marrow biopsy
3 ) Check coagulation screen.
4 ) Reassure this likely resolve see again in 5 days
5 ) Start prednisolone treatment
Comments:
The likely diagnosis immune thrombocytopenia. The clues this are well girl, post viral illness
quite marked thrombocytopenia; but other FBC parameters normal. The diagnosis one of
exclusion, in absence of major bleeding management would be observation, as it can resolve
201

spontaneously. The important investigation a film. Although diagnostic, this will confirm FBC
findings also exclude more sinister pathology such as leukaemia. 1 )
24. A 40 year woman presents tiredness, weight gain fatigue of over one years
duration. Two years ago she underwent trans sphenoidal resection of a non functioning
pituitary tumour. Post operatively she was confirmed have pan hypopituitarism amd
receiving treatment hydrocortisone 10 mg bd, thyroxine 100 micrograms daily, takes oral
contraceptive logynon.
On examination she a pressure of 110/64 mmHg, a pulse of 80 bpm appears clincally
euthyroid.Investigations show:
Free T4
12.5 pmol/L (NR 9 - 22)
Plasma TSH
0.2 mu/L (NR 0.5 5.0)
Serum oestradiol
<80 pmol/L (NR 130 550)
What is the appropriate treatment this patients fatigue?
1 ) DDAVP
2 ) Fludrocortisone
3 ) Growth hormone
4 ) Increase dose of hydrocortisone
5 ) Increase dose of oestrogen
Comments:
This woman underwent a hypophysectomy a non functioning pituitary tumour since then had
problems tiredness fatigue. All her hormonal deficiencies are replaced other than Growth
hormone. The Low TSH a reflection of hypopituitarism but T4 within range. Her dose of
hydrocortisone adequate. She taking a synthetic oestrogen - ethinyl oestradiol, estrogen
replacement this detectable on traditional estradiol assay. So this why oestradiol
concentration unrecordable also reason why oestradiol should be requested (but
unfortunately is) whilst patients are taking combined OCP. 3 )
25. A 50 year old gentleman presents Casualty chest pain. His electrocardiogram on
admission shows an inferior myocardial infarction. He develops complete heart block,
which accompanied by hypotension a pressure of 90/60 mmHg.
A temporary pacing wire inserted his pressure returns 11/75 mmHg. Four hours after
insertion of pacing wire he becomes cold clammy his pressure falls 70/40 mmHg. His
electrocardiogram shows a paced rhythm of 70 beats per minute.
What is the likely diagnosis?
1
2
3
4
5

)
)
)
)
)

Aortic dissection
Lead displacement
Pacing wire perforation causing ventricular rupture
Reinfarction
Ruptured chordae resulting in severe mitral regurgitation

Comments:
The likeliest diagnosis pacing wire induced rupture. The time course in this situation vital as
this often occurs through an infarcted RV wall which more friable thin walled regardless of
infarct setting. Reinfarction unlikely as mention made patient reperfused in first instance.
Ruptured chordae less likely happen an inferior MI than a anterior lateral MI. Aortic dissection
unlikely given presentation. ECG confirms temporary wire continues pace. 3 )
202

26. A 15 year presents with a 6 month history of secondary amenorrhoea. She


otherwise well also noticed slight galactorrhoea over the last 3 months.
She had menarche at age of 12 otherwise had regular periods. She has been sexually
active for approximately one year occasionally used condoms contraception. She smokes
five cigarettes daily and occasionally smokes cannabis. On examination she appears well,
appears clinically euthyroid, a pulse of 70 bpm a pressure of 112/70 mmHg.
Investigations show:
Serum oestradiol
130 nmol/L (NR 130 - 600)
Serum LH
4.5 mU/L (NR 2 20)
Serum FSH
2.2 mU/L (NR 2 20)
Serum prolactin
6340 mU/L (NR 50 450)
Free T4
7.2 pmol/L (NR 9 22)
TSH
2.2 mU/L(NR 0.5 5.0)
What is the likely diagnosis?
1
2
3
4
5

)
)
)
)
)

Drug induced
Non functional pituitary tumour
Polycystic ovarian syndrome
Pregnancy
Prolactinoma

Comments:
This young girl hyperprolactinaemia and, in general, a prolactin above 2000 mU/l suggestive of
a prolactinoma rather than a non functioning tumour stalk compression. Although
hyperprolactinaemia a feature, this pregnancy as elevated oestrdiol concentrations would
accompany hyperprolactinaemia. This level of hyperprolactinaemia would be found in PCOs as
concentrations are below 1000 oestradiol concetrations are high normal. 5 )
27. A 64 year old lady is reviewed in the outpatient clinic. She is known to have Chronic
Lymphocytic Leukaemia six months. Apart from three chest infections in last year, she
otherwise well.
Investigations show:
Haemoglobin
13.4 g/dL
White cell count
30.2 x 109/L
Lymphocytes
16.2 x 109/L
Neutrophils
3.0 x 109/L
Platelet count
350 x 109/L
Serum electrophoresis:
IgG
2.5 g/L (NR 6 13)
IgA
0.2 g/L (NR 0.8 3.0)
IgM
0.1 g/L (NR 0.4 2.5)

203

What is the appropriate management option at this time?


1
2
3
4
5

)
)
)
)
)

Chlorambucil
Fludarabine
Intravenous immunoglobulin infusions
Observe
Stem cell transplant

Comments:
CLL considered a low grade lymphoproliferative disorder. An observation policy usual during
early stages of disease. If disease control necessary B symptoms a rapidly rising white cell
count bone marrow compromise (anemia thrombocytopenia), chlorambucil would be choice.
Recurrent infections are recognized in CLL due hypogammaglobulinaemia immune paresis; but
are an indication disease control. She likely benefit from regular infusions of immunoglobulin
prevent infections. In a patient of this age fludarabine might be used as second line treatment in
CLL. Stem cell transplant initial treatment CLL. 3 )
28. An inpatient on haematology ward develops a fever five minutes after starting a
platelet transfusion. His temperature rises 1C from pre transfusion observations.
Other than feeling hot he well. His pressure, pulse oxygen saturations are all unchanged
from pre transfusion observations. An hour later he remains febrile, but well all other
observations stable.
What is the likely cause of his fever?
1
2
3
4
5

)
)
)
)
)

ABO incompatibility
Anaphylactoid reaction
Bacterial contamination of transfused platelets
Platelet destruction due IgG
Leucocyte cytokines present in unit transfused

Comments:
With such a rapid rise in temperature one needs consider a haemolytic transfusion reaction due
ABO incompatibility. However, several factors weigh against this as diagnosis. He receiving a
platelet transfusion, which does typically contain antigens necessary provoke a transfusion
reaction. It possible plasma in which platelets are suspended contains a high titre of anti A
anti B antibodies, but this would be unusual. In addition, clinical picture entirely consistent
ABO incompatibility: apart from a mild fever, patient very well. With ABO incompatibility
patients become shocked very quickly.
Febrile non haemolytic reactions are very common, are due presence of pyrogenic cytokines
released from leucocytes during storage of platelet units. This likely cause in this patients
case. The incidence decreasing due leucodepletion of products.
Bacterial contamination of platelets common, due fact they are stored at 22C. This supports
growth; but temperature rise on transfusion usually as rapid.
Destruction of platelets by IgG process behind immune thrombocytopenia. Platelets are usually
transfused in this condition.
The history does sound like an anaphylactoid reaction. 5 )
29. 50 year old gentleman known severe aortic stenosis develops syncopal symptoms
on exertion.
What is the next step in his management?
204

1
2
3
4
5

)
)
)
)
)

Aortic valvuloplasty
Aortic valve replacement
Close outpatient follow up serial echocardiograms
Exercise test looking dysrhythmias
Observe until he develops symptoms of breathlessness

Comments:
Exertional syncope an indication surgery in patients severe aortic stenosis, even in absence of
exertional dyspnoea. Aortic valvuloplasty compares unfavourably aortic valve replacement in
medium long term in aortic stenosis. 2 )
30. A 74 year old man referred hospital from his GP. He had treated essential
hypertension, bendrofluazide 2.5mg amiloride 10mg daily. However, a routine check of
his renal function revealed:
Serum sodium
134 mmol/L
Serum potassium 5.9 mmol/L
Serum urea
7.0 mmol/L
Serum creatinine
100 umol/L
When seen in outpatient clinic, his pressure 134/86 mmHg. A twelve lead
electrocardiogram normal. The GP letter says amiloride was stopped previous day.
Which of these appropriate action?
1
2
3
4
5

)
)
)
)
)

Give 10ml of 10% calcium gluconate intravenously


Give 50% dextrose twelve units of Actrapid insulin intravenously
Repeat serum urea electrolytes in one week
Start low potassium diet
Stop bendrofluazide

Comments:
Bendrofluazide, a thiazide diuretic, inhibits sodium chloride reabsorption in distal convoluted
tubule, resulting in increased sodium free water clearance. A secondary effect loss of
potassium by increased secretion in distal tubule in response increased intraluminal sodium.
Therefore, bendrofluazide may cause hypokalaemia.
Amiloride, a potassium sparing diuretic occasionally prescribed thiazide loop diuretics,
prevent hypokalaemia. It inhibits movement of sodium through channels towards end of distal
tubule collecting ducts, preventing passage of sodium from urinary space into tubular cells.
This action causes hyperpolarisation of apical plasma membrane, preventing secretion of
potassium into collecting ducts.
Hyperkalaemia common (>5%), unaffected by concurrent potassium depleting diuretics.
This patient mild hyperkalaemia, evidence of cardiac toxicity. The management involves
stopping amiloride, repeating U&E in one week. 3 )
31. A 50 year old gentleman presents hospital twenty four hours after suffering an
episode of chest pain. He pain free last 12 hours.
His electrocardiogram shows significant ST elevation in anterior chest leads Q waves
loss of R wave progression.What is the next stage in his management?
1 ) Anticoagulation warfarin
2 ) Exercise test
205

3 ) Primary angioplasty
4 ) Thrombolysis
5 ) Transthoracic echocardiogram
Comments:
The diagnosis a late presenting anterior MI, now painfree. The next stage in his management
establish extent of damage his heart. Thrombolysis primary angioplasty too late myocardial
salvage. An exercise test next step but would be later in admission warfarinization may be
long term treatment. 5 )
32. A 50 year man presented lethargy weight loss.
Investigations showed:
Hb
9.4 g/dL
WBC
54.2 x 109/L
Platelets
79 x 109/L
His film is shown below:

What would be your initial management plan this patient?


1)
Observation
2)
Glivec (Imatinib)
3)
Alpha Interferon
4)
AML type chemotherapy approach
5)
Allogeneic bone marrow transplant
Comments: The film shows chronic myeloid leukaemia. Unlike CLL, CML will usually progress
frank leukaemia quite rapidly, so treatment needed. You can undertake a period of observation,
unless age other co morbidity made treatment inappropriate. NICE have now approved tyrosine
kinase inhibitor Glivec as first line treatment chronic accelerated phase CML. Interferon
longer used as first line, unless in context of a clinical trial. If patient had in blast crisis phase ,
then AML type chemotherapy as well as Glivec would be choice. If remission achieved Glivec,
then in a patient under 60 65 years an allogeneic transplant would be considered if there was a
matched sibling donor, in a 50year patient younger a matched unrelated donor transplant
would be considered too. 2 )
33. An 82 year man presents his General Practitioner a six month history of fatigue
increasing exertional dyspnoea. Investigations show:
206

Haemoglobin
7.5 g/dL
White cells
3.12 x 109/L
Neutrophils
34%
Blasts
1%
Platelets
12 x 109/L
His film is shown below: A bone marrow aspirate, stained an iron stain shows:

1)
Bone marrow aplasia
2)
Chronic myeloid leukaemia
3)
Metastatic bone marrow infiltration
4)
Myelodysplastic syndrome
5)
Myelofibrosis
Comments: The patient a macrocytic anaemia, thrombocytopoenia neutropoenia a small
number of circulating blasts. This suggests a diagnosis of myelodysplastic syndrome. Further
evidence in support of a diagnosis of MDS comes from film showing a dimorphic picture (some
red cells are hypochromic microcytic, while others appear macrocytic), neutrophils are
hypogranular hyposegmented (Pelger Huet cells). The bone marrow shows ring sideroblasts an
iron stain. Ring sideroblasts contain an abnormally high concentration of iron - usually stored in
perinuclear mitochondria. Perls stain (which stains iron) shows this iron deposition as a dark
ring around margin of nucleus. 4 )
207

34. A 19 year man of Afro Caribbean origin presents Accident Emergency department
frank haematuria. On further questioning he gives a history of polyuria nocturia over
several months.
Investigations show:
Hb
6.7g/dL
MCV
58.0fL
WBC
13.1 x 109/L
Neurophils
10.1 x 109/L
Platelets
503 x 109/L
The film shows:

What is them likely cause of the urinary symptoms?


1
2
3
4
5

)
Diabetes mellitus
)
Glomerulonephritis
)
Renal calculi
)
Renal papillary necrosis
)
Urinary tract infection
Comments: The patient Sickle Cell disease. There a hypochromic microcytic anaemia crescent
shaped cells on film; one erythrocyte contains an Howell Jolly body - an indicator of
hyposplenism. The hyposplenism secondary splenic infarcts. Renal papillary necrosis a well
recognized chronic complication of HbSS disease, history, FBC film all support this. 4 )

35. A 29 year old woman was admitted hospital after having a tonic clonic seizure at
home. She had unwell two days, firstly developing diarrhoea then complaining of fever
malaise. Over preceeding twenty four hours she had become slightly confused.
Investigations showed:
Haemoglobin
9.2 g/dL
White cell count
9.4 x 109/L
208

Platelets
Serum sodium
Serum potassium
Serum urea
Serum creatinine
PT
APTT
Fibrinogen
Fibrin Degradation Products
Urine microscopy:
film shown below:

33 x 109/L
135 mmol/L
5.1 mmol/L
34 mmol/L
412 umol/L
12s (Control 11.5 - 15.0s)
38s (Control 30 40s)
2.5 g/L (NR 1.8 - 5.4 g/L)
85 mg/L (NR <100 mg/L)
Blood +++ protein +++ Her

Infection with which of following organisms is likely to be the cause of her illness?
1)
Eschericia coli
2)
Herpes simplex virus 1
3)
Neisseria meningitidis
4)
Salmonella typhi
5)
Streptococcus pneumoniae
Comments: The diagnosis Thrombotic Thrombocytopoenic Purpura (TTP) caused by E. coli
O157:H7 infection. The investigations show anaemia, thrombocytopoenia caogulaopathy
consistent disseminated intravascular coagulation. The film shows a microangiopathic
haemolytic anaemia (MAHA).
Features of MAHA include red cell fragmentation, different red cell sizes (anisocytosis).
Immature erythrocytes are released by bone marrow in an attempt correct anaemia. A
nucleated red cell seen near centre of slide; a neutrophil shown on left.
Causes of MAHA include: 1. TTP 2. Haemolytic uraemic syndrome 3. Disseminated intravascular
coagulation 4. Haemolysis by prosthetic heart valves 5. Malignant hypertension 6. Septicaemia 7.
Vasculitis & connective tissue disease (PAN, Wegeners, SLE) 1 )
36. A 21 year old student presents a ten day history of fevers malaise. There past
history of note. On examination, he enlarged lymph nodes palpable in both supraclavicular
fossae. The nodes are smooth, mobile non tender. There recent history of overseas travel.
He returned from his elective period in Gambia three months previously.
209

His film is shown below:

1
2
3
4
5

)
Dranunculiasis
)
Loa loa infection
)
Onchocerciasis
)
Schistosomiasis
)
West African trypanosomiasis
Comments: The film shows typical trypomastigotes of African trypanosomiasis. The causative
agent Trypanosoma brucei transmitted human by bite of tsetse fly. African trypanosomiasis
two recognised forms: East African sleeping sickness caused by Trypanosoma brucei
rhodesiense; West African sleeping sickness caused by Trypanosma brucei gambiense.
The two forms of disease are distinguishable from their macroscopic appearance. However,
their clinical manifestations are different.
West African trypanosomiasis a slower course. Symptoms start several weeks even months after
tsetse fly bite. The first stage of disease haematolymphatic spread accompanied by fever,
lymphadenopathy (discrete, rubbery, non tender nodes). A rash sometimes occurs mild
hepatosplenomegaly may develop. The second stage meningoencephalitic stage. This occurs
months years after acquisition of infection. Manifestations include personality change
progressive indifference daytime somnolence. Extrapyramidal signs ataxia are common.
East African trypanosomiasis similar clinical features, but progression of disease more rapid starting within days of infection. Death may occur within weeks months. Rash a more
prominent feature lymphadenopathy less frequently present.
Treatment suramin.
5)
37. A 44 year old man presents to Casualty with a two week history of progressive
dyspnoea, dry cough and fever.
Investigations
Haemoglobin
12.5 g/dl
WBC
7.2 x 109/l
Neutrophils
5.9 x 109/l
Lymphocytes
0.4 x 109/l
Monocytes
0.8 x 109/l
Platelets
130 x 109/l
Sodium
144 mmol/l
210

Potassium
4.1 mmol/l
Urea
7.5 mmol/l
Creatinine
75 mol/l
Arterial gases breathing air
pH
7.42
pO2
8.9 kPa (66.8 mmHg)
pCO2
4.7 kPa (35 mmHg)
HCO3
22 mmol/l
His chest x ray shown below.

What is the likely diagnosis?


1
2
3
4
5

)
)
)
)
)

Cytomegalovirus pneumonitis
Mycoplasma pneumonia
Pneumocystis carinii pneumonia
Pulmonary oedema
Severe acute respiratory syndrome (SARS)
211

Comments: His presentation highly suggestive of PCP, a subacute onset predominant


symptoms of dry cough dyspnoea. The lymphopoenia very suggestive of PCP AIDS (and
therefpre low CD4 lymphocyte count), although patients may have low lymphocyte counts other
acute viral infections indeed may be in HIV/AIDS due a compensatory increase in CD8
lymphocyte subset. Hypoxia exercise desaturation a characteristic feature of PCP. The first
choice treatment PCP high dose co trimoxazole (iv po). Alternatives include trimethoprim +
dapsone, intravenous pentamidine. Steroids should be co administered all patients pO2 < 70
mmHg. 3 )
38. A 75 year old man referred outpatient clinic investigation of anaemia. He gives a
six month history of fatigue exertional dyspnoea. He a past history of hypertension which
controlled atenolol, had a prostatectomy aged 70 years benign prostatic hypertrophy. He
smoked 20 cigarettes per day until aged 38 years. He drinks approximately eight units of
alcohol per week.
On examination he appears pale. He becomes mildly dyspnoeic getting onto examination
couch. His pulse 100 beats per minute, regular pressure 125/65. His heart sounds are
murmurs his chest clear. His abdomen soft. A smooth, non tender liver edge palpable
3cm below right costal margin; spleen palpable 10cm below left costal margin.
His peripheral film shown below:

1
2
3
4
5

)
)
)
)
)

Chronic lymphocytic leukaemia


Haemolytic anaemia
Multiple melanoma
Myelofibrosis
Visceral leishmaniasis
212

Comments: The patient anaemic massive splenomegaly a lesser degree of hepatomegaly. The
film shows numerous tear drop cells. One cell (lower left) shows basophilic stippling.
Myelofibrosis a disease of over 50s. As bone marrow fibroses, myelopoiesis switches liver
spleen. Patients are anaemic thrombocytopoenic abnormal clotting. Splenomegaly may be
massive; splenectomy may be required. 4 )

1.
A 43 year old man was admitted hospital cellulitis treatment started intravenous
antibiotics. He had a past history of sever depression was frequently admitted as an
inpatient under care of psychiatry. On third day of his admission he complained of
epigastric pain started vomit bright red.
On examination, his pressure was 95/60 mmHg pulse 120 beats per minute. He admitted
taking a box of injections from treatment room on ward twelve hours previously self
administering them through his intravenous cannula. An empty box of low molecular
weight heparin was subsequently found on floor of his room by nurses.
What specific antidote should be administered?
1
2
3
4
5

)
)
)
)
)

Fresh frozen plasma


None specific antidote exists
Platelet transfusion
Protamine sulphate
Vitamin K

Comments:
The major adverse event related treatment low molecular weight heparin (LMWH) bleeding. If a
patient on LMWH develops a major bleed, further doses should be withheld and, depending on
severity of bleeding, protamine sulphate should be considered.
Protamine sulphate antidote LMWH. This reverses most, but all, of effects of LMWH. The dose
of protamine sulphate given dependent upon dose of LMWH administered time of
administration. If protamine given within 8 hours of LMWH then a maximum neutralizing dose
1mg Protamine/100units (or 1mg) of LMWH given in last dose. If more than 8 hours have
passed since dose of LMWH was given, administer 0.5mg Protamine per 100 units (or 1mg) of
LMWH given. Protamine administered by slow IV infusion (over 10 mins) avoid a hypotensive
reaction. Protamine requires a high level of caution when being prescribed administered. 4 )
2.
A 57 year old gentleman presented his General Practitioner a two month history of
epigastric discomfort was partially relieved antacids. He had past history of note he was
taking any regular medication. He was referred an outpatient upper gastrointestinal
endoscopy, which showed moderate duodenitis, but evidence of peptic ulceration. A test
presence of Helicobacter pylori was positive he was given a seven day course of
omeprazole, amoxicillin clarithromycin. Four weeks later he was seen in outpatient clinic
when he reported he was entirely asymptomatic.
What best option further management of this patient?
1 ) Continue long term acid suppression therapy
213

2
3
4
5

)
)
)
)

Perform H.pylori breath test


Reassure discharge
Repeat endoscopy
Start treatment alginate

Comments:
The main causes of dyspepsia are gastro oesophageal reflux disease (GORD) (15 25%), gastric
duodenal ulcers (15 25%) stomach cancer (2%). The remaining 60% are classified as non ulcer
dyspepsia (NUD).
Current NICE guidelines on treatment of dyspepsia differs according underlying aetiology:
GORD
Patients severe GORD who have a proven pathology (e.g. oesophageal ulceration, Barretts
oesophagus) should be treated a healing dose of a PPI until symptoms have controlled. Once
this achieved, dose should be stepped down lowest dose maintains control of symptoms. A
regular maintenance low dose of PPIs will prevent recurrent GORD symptoms in 70 80% of
patients should be used in preference higher healing dose. Where necessary, should symptoms
re appear, higher dose should be recommenced.
In complicated oesophagitis (stricture, ulcer, haemorrhage), full dose should be maintained.
Patients mild GORD symptoms and/or those who do have a proven pathology can often be
managed alternative therapies including antacids, alginates, H2 receptor antagonists.
Peptic ulceration Patients documented duodenal gastric ulcers testing Helicobacter pylori
should occur. If positive, eradicating infection recommended. Long term acid suppressing
therapy indicated.
NSAID induced ulceration Patients documented NSAID induced ulcers who must continue NSAID
therapy (e.g. those severe rheumatoid arthritis) a PPI should be prescribed. After ulcer healed,
patient should be stepped down a maintenance dose of PPI.
Non ulcer dyspepsia (NUD) Patients NUD may have symptoms caused by different aetiologies
should be routinely treated PPIs. Should symptoms appear be acid related, an antacid lowest
dose of an acid suppressor control symptoms should be prescribed. If they do appear be acid
related, an alternative therapeutic strategy should be employed. The clinical scenario in this
question slightly more difficult as it falls between two groups. The patient H.pylori positive
gastritis but ulcer. He should therefore receive a seven day course of standard eradication
therapy. The absence of symptoms 4 weeks after eradication therapy strong evidence
eradication successful. Confirmation of H.pylori eradication in patients who are asymptomatic
at 4 weeks unnecessary unless there bleeding perforation of a peptic ulcer where there are
ongoing risk factors e.g. anti coagulant NSAID therapy. In such circumstances, H.pylori breath
test best deferred 5 weeks after conclusion of eradication therapy. 3 )
3.
A 48 year old male accountant referred from his General Practitioner a 3 month
history of dry, nocturnal cough. He a ex smoker having given up 5 years ago. He does
produce any sputum an exercise tolerance similar his work colleagues.
He denies any other symptoms of note. Examination reveals he 5' 10" (1.77m) tall weighs
98kg (BMI = 31 kg/m2). Chest clear auscultation.
Results of spirometry are shown below:
FEV1
3.5 L
(Predicted 3.38 L)
FVC
4.3 L
(Predicted 4.40 L)
FEV1/FVC
0.81 L
(Predicted 0.77 L)
214

PEFR
540 L/min
(Predicted 559 L)
What would be appropriate next investigation?
1 ) 24hr oesophageal pH manometry
2 ) bronchoscopy
3 ) flexible nasendoscopy
4 ) peak flow chart
5 ) sleep studies
Comments:
Asthma a reversible condition a lung function test does exclude diagnosis. A variation of
greater than 25% on a peak flow chart would support a diagnosis of asthma. 4 )
4.
A 36 year recently diagnosed thyrotoxicosis presents a sore throat. One month ago
she commenced Carbimazole 40 mg daily plus propranolol 40mg bd began feel better but
over last one week she aware of a sore throat painful swallowing. On examination, her
pulse 80 beats per minute regular she a modest non tender goitre. No other
abnormalities are noted. Her investigations reveal:
Haemoglobin
12.5 g/dl
Platelets
220x109/l
White cell count
3.8x109/l
Neutrophils
1.6x109/l
Lymphocytes
2x109/l
Free T4
23.1 (NR 9 23)
TSH less than
0.05 (NR 0.5 4)
TSH receptor antibody positive What is the appropriate treatment this patient?
1 ) Add prednisolone therapy
2 ) Continue Carbimazole
3 ) Thyroidectomy
4 ) Stop Carbimazole treat Radioactive Iodine
5 ) Stop Carbimazole change Propylthiouracil
Comments:
This patient Graves Disease as suggested by positive TSH receptor antibodies but developed
neutropaenia Carbimazole. The appropriate treatment would be stopping carbimazole treating
radioactive iodine as a definitive treatment in first instance rather than surgery. Like
carbimazole, PTU also associated development of agranulocytosis should be used. 4 )
5.
A 54 year male diabetic a five year history of type 2 diabetes presents at annual
review. He currently receiving glibenclamide 5 mg daily, together Lisinopril 10 mg daily
hypertension. On examination he a BMI of 32.4 kg/m2, a pressure of 132/84 mmHg, all
pulses are palpable he some loss of vibration sensation on both big toes 2 3 dot
haemorrhages in each eye. Investigations reveal:
Hb1Ac
10.2% (NR less than 6)
Total cholesterol
5.2 mmo/l
Triglycerides
2.2 mmol/l
215

Urinalysis
negative
What is the appropriate treatment this patients hyperglycaemia?
1 ) Add Insulin
2 ) Add Metformin
3 ) Add Rosiglitazone
4 ) Increase dose of glibenclamide
5 ) Stop Glibenclamide treat metformin
Comments:
This obese diabetic would more appropriately have treated from outset metformin but
irrespectively finds himself treated a sulphonylurea. His glycaemic control poor this agent
target would be an HbA1c of 7%. Thus an agent would probably need be added his current
sulphonylurea therapy achieve better glycaemic control as stopping SU then adding metformin
would effectively reduce his HbA1c. If this manoeuvre failed then he would probably need
insulin dose limted by concomitant use of metformin. 2 )
6.
A 32 year woman seen in outpatient clinic three weeks after receiving a cadaveric
renal transplant. She currently taking prednisolone ciclosporin. Three days ago her serum
urea electrolytes were normal. She says she feels well.
Repeat serum urea electrolytes show:
Serum sodium
139 mmol/L
Serum potassium
3.9 mmol/L
Serum urea
11 mmol/L
Serum creatinine
168 umol/L
The renal transplant was HLA matched. The patient was CMV IgG negative;
kidney donor was CMV IgG positive.
Which of following best accounts change in renal function?
1 ) Acute cellular rejection
2 ) Ciclosporin toxicity
3 ) CMV infection
4 ) Dehydration
5 ) Pyelonephritis of transplanted kidney
Comments:
This patient had a sudden deterioration in renal function, three weeks following an
uncomplicated renal transplant. Despite this, she clinically well, symptoms.
This lady acute cellular rejection. Approximately 25% of transplant patients will have at least
one episode of rejection mostly between day 7 21, less commonly up three months post
operation. It often clinically silent, only a sharp rise in serum creatinine pointing towards
diagnosis. Doppler ultrasound studies may show a sharp deterioration in graft perfusion, kidney
biopsy will show invading lymphocytes penetrating tubular basement membrane, causing
tubilitis. Treatment IV bolus of high dose steroids. Long term graft function will be
compromised if rejection episode completely reversed.
This lady CMV seronegative receied a graft from a CMV seropositive donor. Although CMV
infection would cause a sudden deterioration in her renal function (as functioning kidney
CMV+), this lady does have a three five fold increased risk of developing severe CMV infection
216

while immunosuppressed. Typical manifestations of infection would include interstitial


pneumonitis, oesophagitis, peptic ulceration, colitis retinitis. CMV infection associated
increased graft rejection renal artery stenosis in renal transplant recipients. Because she well,
early onset of findings CMV+ transplant, this option can be discounted.
Ciclosporin a calcineurin inhibitor which a potent immunosuppressant nephrotoxin.
Ciclosporin can cause a dose dependent increase in urea creatinine in first few weeks of taking,
also long term graft failure. This probably related total amount of ciclosporin taken. There
evidence in question state dose recently increased, therefore this answer can be discounted.
This lady well there suggestion of intercurrent disease causing dehydration.
Pyelonephritis of transplanted kidney a particular problem in early immunosuppressed period.
Pyelonephritis would present low grade pyrexia, tender, swollen kidney deteriorating graft
function. This lady well, evidence of infection. 1 )
7.
A 32 year old man was admitted hospital a fever five days after receiving a cycle of
chemotherapy lymphoma.
On examination, he appeared unwell. He was febrile 39.5oC. Pulse 120 beats per minute,
regular Blood pressure 85/40 mmHg. Heart sounds were added sounds murmurs. His
chest was clear on auscultation his abdomen was soft non tender.
Investigations revealed:
Haemoglobin
White cell count
Neutrophils
Lymphocytes
Monocytes
Eosinophils
Basophils
Platelets

11.2 g/dL
2.0 x 109/L
0.2 x 109/L
1.6 x 109/L
0.15 x 109/L
0.04 x 109/L
0.01 x 109/L
151 x 109/L

What treatment should be started?


1 ) Ceftazidime + ticarcillin/clavulanic acid
2 ) Co trimoxazole
3 ) Flucloxacillin + benzylpenicillin
4 ) Gentamicin + metronidazole
5 ) Piperacillin/tazobactam + gentamicin
Comments:
Assessment of febrile patient neutropoenia should include:
1. Assess likely sites of infection from history clinical examination chest x ray.
2. Comprehensive cultures of blood, urine, sputum, Hickman line (if present).
3. If temperature >40oC 4 6 hours if patient hypotensive, blind broad spectrum antibiotic
therapy should be started (N.B. after taking cultures).
4. Barrier nursing should be instituted if possible.
5. Antibiotics should be continued 2 5 days after fever settled neutrophil count recovered.
6. If fever persists after 48 hours despite antibiotic therapy, fungal (Aspergillus, Candida,
Pneumocystis) viral (CMV) infection should be considered.
217

Although preferred antibiotic regimens neutropoenic fever vary from center center, all operate
on same principles. The aim provide broad spectrum coverage includes cover Pseudomonas
aeruginosa. Dual therapy regimes are usually used. These utilise a broad spectrum antibiotic (an
antipseudomonal penicillin, carbapenem ceftazidime) plus an anti pseudomonal
aminoglycoside.
Typical regimens consist of:
EITHER
Antipseudomonal penicillin:
Piperacillin/tazobactam (Tazocin), or
Ticarcillin/clavulanic acid (Timentin)
OR
Antipseudomonal carbapenem:
Imipenem/cilastin, or
Meropenem
PLUS
Anti pseudomonal aminoglycoside:
Gentamicin, or
Amikacin
If infection Staph aureus suspected, addition of vancomycin should be considered. 5 )
8.
A 76 year old man attended outpatient clinic a routine follow up appointment. He
had presented hospital acute dyspnoea six months reviously had diagnosed severe
aortic regurgitation. During his admission he had undergone aortic valve replacement a
mechanical valve.
During consultation he does report any residual dyspnoea ankle swelling. His INR stable
checked regularly by his General Practitioner. His only complaint of difficulty sleeping,
which he says started when his wife died suddenly one year previously. He wakes at 4am
difficulty getting back sleep. He lives alone enjoyment in life since death of his wife. In
recounting his wifes death he very tearful says he often cries without any apparent
reason.What would be best choice of antidepressant this gentleman?
1
2
3
4
5

)
)
)
)
)

Amitriptyline
Citalopram
Fluoxetine
St Johns Wort
Venlafaxine

Comments:
Selective serotonin reuptake inhibitors (SSRIs; citalopram, fluoxetine) may enhance
anticoagulant effect of warfarin, as may serotonin noradrenaline reuptake inhibitor (SNRI)
venlafaxine. St Johns wort (Hypericum perforatum), an herbal antidepressant available in UK
without a prescription, reduces anticoagulant effect of concomitant use should be avoided.
A tricyclic antidepressant therefore best choice. 1 )
9.
A 60 year old man was seen in Oncology clinic. He had recently diagnosed as having
a rectal adenocarcinoma after presenting anaemia rectal bleeding. He underwent a
colectomy. The resected colon showed a carcinoma had spread serosa but resected
218

margins were free from tumour excised sample lymph nodes did show any evidence of
disease.
What additional treatment, if any, required?
1
2
3
4
5

)
)
)
)
)

Chemotherapy
Immunotherapy
Observation only
Palliative care
Radiotherapy

Comments:
The patient a Stage II (Dukes B) colorectal tumour.
Treatment of colorectal cancers
Stage I (Dukes A) Carcinoma in situ limited mucosa submucosa (T1, N0, M0). Standard
treatment involves surgery remove tumour. Additional treatments are usually needed. For
Dukes stage A tumors involving only mucosa, 5 year survival rate exceeds 90%.
Stage II (Dukes B) Cancer extends into muscularis (B1), into through serosa (B2). Standard
treatment surgical removal of tumour followed by radiotherapy. Radiotherapy shown reduce
rate of recurrence. The role of adjuvant chemotherapy less clear in Dukes B than in Dukes C
(see below). Patients may be offered opportunity of entering into a clinical trial of
chemotherapy, but chemotherapy typically given as standard. For Dukes stage B colon cancers,
5 year survival rate greater than 70% can be greater than 80% if tumor does penetrate
muscularis mucosa.
Stage III (Dukes C) Cancer extends regional lymph nodes (T1 4, N1, M0). Treatment involves
surgery remove tumor, chemotherapy 5 FU leukovorin in some patients radiotherapy may
also be needed (especially if tumor large invading tissue surrounding colon). The 5 year
survival rate Dukes C usually less than 60%.
Stage IV (Dukes D) Cancer metastasized distant sites (T1 4, N1 3, M1). Surgery remove
tumour bypass an obstructing tumour, palliative chemotherapy and/or radiotherapy symptom
relief; use of new agents such as cetumixab (a recombinant human/mouse chimeric epidermal
growth factor inhibitor) becavizumab (a recombinant human anti vascular epidermal growth
factor (VEGF) antibody). Five year survival approximately 5%. 5 )
10. A 32 year old man was referred clinic a ten year history of recurrent chest
infections. He was diagnosed common variable immunedefeciency.
Laboratory investigations showed:
IgG
IgA
IgM

6.5 g/L
0.8 g/L
0.5 g/L

(NR 6.0 13.0)


(NR 0.8 3.0 g/L)
(NR 0.4 2.5)

Which of following options would best prevent recurrent chest infections?


1 ) Chest physiotherapy postural drainage
2 ) Heart lung transplantation
3 ) Intermittent courses of antibiotics when patient feels he developing a chest infection
4 ) Intravenous immunoglobulin
5 ) Lifelong prophylactic antibiotics
219

Comments:
Common variable immunodeficiency (CVID) prevalent of primary immunodeficiency diseases.
The basic defect in CVID a failure of B lymphocyte differentiation into plasma cells produce
various immunoglobulin (Ig) subtypes.
Patients CVID often have marked reduction in serum levels of both IgG IgA; approximately 50%
of these patients also have reduced IgM. However, some patients have IgG levels but functional
antibody deficiency may be present despite IgG subclass levels.
The diagnosis based on exclusion of known causes of defects of humoral immune system.
Most cases are sporadic, although familial cases exist have various inheritance modes (including
autosomal dominant variable penetrance, autosomal recessive, X linked).
Regular intravenous immunoglobulin (IgG) infusions greatly reduce frequency of recurrent
respiratory tract infections.
Clinical manifestations of CVID include:
Recurrent infections Principally recurrent pyogenic infections of upper lower respiratory tract.
This main clinical manifestation of CVID. Symptoms may appear during childhood or, more
often, after adolescence. Bronchiectasis may develop if optimal therapy delayed. Haemophilus
influenzae, Moraxella catarrhalis, Streptococcus pneumoniae, Staphylococcus aureus are
organisms commonly involved. Severe, recurrent infection herpes simplex are also common.
Autoimmune diseases Patients may develop rheumatoid arthritis, hemolytic anemia,
thrombocytopenia, neutropenia, thyroid abnormalities, vitiligo, keratoconjunctivitis sicca. About
20% of these patients have a severe gastroenteropathy severe malabsorption resembling coeliac
sprue, nodular lymphoid hyperplasia, chronic inflammatory bowel disease such as ulcerative
colitis Crohn disease. A smaller number of patients develop achlorhydria pernicious anemia,
autoimmune hepatitis, primary biliary cirrhosis, alopecia totalis, hyperthyroidism, vasculitis,
lymphoid interstitial pneumonia.
Lymphoid hyperplasia granulomatous diseases Atypical lymphoid hyperplasia due clonal
expansion of B T lymphocytes reported in as many as one third of patients CVID. Granulomas
have reported in approximately 5 10% of patients CVID. Granulomas are indistinguishable from
those of classic sarcoidosis are found in lung, liver, spleen, conjunctivae.
Predisposition malignancy Patients CVID have a high risk of developing malignant neoplasms
(most commonly lymphoma (especially NHL), GI carcinoma. Most lymphomas are of these are of
B cell immunophenotype are frequently associated EBV. Lymphoma occurs 300 times more
frequently in women CVID than in affected men. Other malignancies include colon cancer,
breast cancer, prostate cancer, ovarian cancer, oral cancer, melanoma. 4 )
11. After presenting an upper gastrointestinal bleed back pain, a 90 year old lady was
diagnosed gastric cancer. Subsequent investigations revealved multiple bony metastases
in spine metastases in liver lungs. She was discharged home on oramorph pain relief,
but was readmitted within 48 hours because of increased pain. One week after readmission
she developed a productive cough consolidation at right base on chest x ray. Within a few
hours her condition deteriorated rapidly, she became drowsy laboured breathing.
What would be appropriate treatment discuss family?
1
2
3
4
5

)
)
)
)
)

intravenous antibiotics fentanyl patch


intravenous antibiotics intravenous fluids
intravenous antibiotics oramorph
sub cutaneous diamorphine as required
sub cutaneous diamorphine pump
220

Comments:
This elderly lady metastatic gastric carcinoma dying. Her cancer incurable she therefore
needs be kept pain free. Treatment of pneumonia appropriate, but she should be kept free
from respiratory distress. 5 )
12. An 81 year old man was admitted hospital from a residential home after developing
chest pain. He had a past history of ischaemic heart disease, having suffered a myocardial
infarction eight years previously. Following his previous infarction, he had developed
congestive cardiac failure which was well controlled medication.
He had other past history of note, apart from an injury during his time in armed services
sixty years previously when he sustained a chest injury when he was hit by shrapnel while
serving in North Africa. His medications included aspirin, ramipril furosemide. He had
born raised in India, but had lived in UK all of his adult life, where he had worked as a
secondary school teacher latterly a headmaster of a boys boarding school.
His ECG showed lateral ST segment elevation his serum troponin T was elevated. His chest
x ray showed pleural calcification at right base.
Select two likely causes of pleural calcification in this patient
1 ) Adenocarcinoma of bronchus
2 ) Chickenpox
3 ) Childhood measles
4 ) Chronic congestive cardiac failure
5 ) Mesothelioma
6 ) Occupational exposure chalk dust
7 ) Post traumatic
8 ) Primary hyperparathyroidism
9 ) Tuberculosis
10 ) Yellow nail syndrome
Comments:
Unilateral pleural calcification commonly occurs as a chronic change secondary pleural
infection (particularly tuberculous empyema), pyogenic empyema, haemothorax. Unilateral
pleural calcification rarely related asbestos exposure. Bilateral calcified pleural plaques are
usually considered asbestos related. Other rarer causes of bilateral disease include radiation
exposure, hyperparathyroidism, pulmonary infarction, pancreatitis.
The likely explanation pleural calcification in this patient either TB empyema as a result of
haemothorax from his war injury.7) 8)
13. A 34 year old lady presented hospital severe right loin pain, haematuria. She had
noticed some right loin discomfort twenty four hours previously, but acute severe pain
started two hours before admission. She denied having any dysuria, fever, rigors.
She had a past history of Crohns disease, diagnosed when she was 18 years old. One year
previously she had undergone an ileal resection after presenting an acute exacerbation of
Crohns an intestinal perforation. Her regular medication consisted of azathioprine 150mg
daily. She was married two children smoked five cigarettes per day.
On examination she was afebrile. Pulse 100 beats per minute, regular. Blood pressure
115/65 mmHg. Heart sounds were chest was clear. Her abdomen was soft non tender,
right loin was tender palpation.
Investigations:

221

Haemoglobin
White cell count
Platelets
Serum sodium
Serum potassium
Serum urea
Serum creatinine
ESR (Westergren)

11.5 g/dL
8.1 x 109/L
367 x 109/L
139 mmol/L
4.1 mmol/L
6.2 mmol/L
89 umol/L
12 mm/1st hour

Urinalysis showed protein but was negative white cells nitrates.


Microscopy did demonstrate any white cells organisms. A plain x ray of kidneys ureters
bladder (KUB) showed an ovoid opacity, approximately 0.8cm in diameter, adjacent right
kidney.
What likely composition of renal stones?
1
2
3
4
5

) Calcium oxalate
) Calcium phosphate
) Cysteine
) Triple phosphate (calcium, magnesium, ammonium)
) Urate

Comments:
Hyperoxaluria occurs both in patients an ileal resection in patients a short bowel who have had
a distal small bowel resection (e.g. Crohns disease, infracted bowel). It caused by increased
absorption of oxalate by colon. Bile salts in colon increase oxalate absorption. Hyperoxaluria
associated renal stone formation, propensity form stones reduced when citrate intake
reduced.
Radio opaque renal stones
Calcium oxalate
Calcium phosphate
Triple phosphate (calcium, magnesium, ammonium
Cysteine (semi opaque)
Radio lucent renal stones
Urate1 )
14. A 46 year old man was admitted Casualty from home a one week history of mild
global headache. On day of admission he found difficulty in expressing himself verbally.
He had a generalised seizure in ambulance on way hospital.
On examination he was fully conscious a Glasgow coma scale score of 15/15. He was
febrile (38oC) pulse 80 beats per minute in sinus rhythm pressure 130/75 mmHg. There
was nuchal rigidity. Neurological examination an expressive dysphasia mild right sided
weakness.
An MRI scan of his brain showed abnormal signals in both temporal lobes, but was
prominent on left where a degree of mass effect was noted.
A lumbar puncture was performed CSF analysis showed:
222

Opening pressure
CSF protein
CSF glucose
Cells
Gram stain

Normal
Normal
Nomal
9 lymphocytes/mm3
Negative

What is the likely diagnosis?


1 ) Acute disseminated encephalomyelitis
2 ) Bacterial meningitis
3 ) Herpes simplex encephalitis
4 ) Lyme disease
5 ) Tuberculous meningitis
Comments:
This a typical history HSV encephalitis. Characteristic symptoms include a combination of :
fever, headaches, confusion, odd behaviour, dysphasia seizures. Patients may have olfactory
gustatory hallucinations. MRI may show oedema of temporal lobe(s). The diagnosis confirmed
by demonstrating presence of virus in CSF (PCR quickest method).
Acute disseminated encephalomyelitis (post infectious encephalomyelitis) may occur
approximately two weeks after a viral infection (measles, chickenpox, rubella mumps, influenza).
The clinical presentation similar encephalitis fever, convulsions coma +/ involuntary
movements. There prior history of viral infection in this case this option therefore less likely.
Both Borrelia burgdorferi (the causative agent in Lyme disease) can cause encephalitis but there
other supportive evidence Lyme therefore less likely.
The clinical presentation CSF results do support diagnosis of either bacterial meningitis
tuberculous meningitis. 3 )

15. A 17 year old woman presented hospital after suddenly developing pain in her right
calf. The pain started spontaneously while she was rollerblading in a local park. There was
past history of note she was taking any regular medication. She was a non smoker.She
says her cousin a history of blood clotting problems.
On examination she complained of deep pain in her calf, though there was pain on passive
movement of leg ankle. A Doppler ultrasound scan of her leg veins did demonstrate any
evidence of venous thrombosis.
Investigations revealed:
D Dimer
35 mg/L (NR <0.5)
What should be the next step in her management?
1
2
3
4

)
)
)
)

Arrange a lower limb venogram


Discharge analgesia
Repeat Doppler ultrasound scan
Send thrombophilia screen
223

5 ) Start low molecular weight heparin


Comments:
Most D Dimer assays have a high negative predictive value (approx 90%) [NPV probability
patient does have disease given a negative test result i.e. if test negative they are unlikely
have disease]. However positive predictive value often low (approx 15%) [PPV probability
patient will have a condition given a positive test result i.e in case of D Dimer, a positive result
does correlate a high probability of disease]. The test therefore useful exclude DVT rather
than confirm DVT.
Current evidence strongly supports use of a D dimer assay in clinical algorithm of suspected
DVT. In other words, clinical likelihood of a DVT should be taken into account when interpreting
results of D Dimer assay. The value of combining clinical estimation of probability imaging
tests confirmed in several studies. The main utility of assay conserve resources by limiting
number of investigations required.
A negative D dimer assay rules out DVT in patients low to moderate risk. These patients do
require doppler ultrasound scanning.
All patients a positive D dimer assay all patients a moderate to high risk of DVT require a
diagnostic study (duplex ultrasonography).
In this case, clinical history unusual a DVT, onset of calf pain during exercise supporting
clinical features. The pain likely be musculoskeletal in origin patient should be discharged. 2 )
16. A 19 year man was admitted Casualty from inpatient psychiatric unit. He had a
known history of severe depression his admission psychiatric unit had arranged from
community. However, shortly after arriving in psychiatric unit he collapsed. An empty pill
bottle was found in his pocket. On admission Accident Emergency Unit he was semi
conscious. His pupils were equal dilated responded sluggishly light. Multiple muscle
twitches were noted he was globally hyperreflexic. His ECG showed a broad complex
tachycardia.How should this patient be treated?
1
2
3
4
5

)
)
)
)
)

D/C cardioversion
Intravenous lignocaine
Intravenous magnesium sulphate
Intravenous sodium bicarbonate
Oral activated charcoal

Comments:
The history points towards a diagnosis of tricyclic antidepressant (TCA) overdose.
Cardiac effects of TCA overdose
Hypertension results from blockade of norepinephrine reuptake an early transient finding.
Catecholamines are eventually depleted in patients, hypertension mild self limiting best left
untreated.
Orthostasis hypotension are result of direct myocardial depression, catecholamine depletion,
alpha adrenergic blockade, arrhythmias. The combination of decreased contractility
vasodilation produce decreased preload can result in severe refractory hypotension.
Arrhythmias, secondary blockage slowing of fast sodium channels (causing a quinidine like
effect) are serious consequence of TCA overdose. Prolonged conduction along His Purkinje
system produces a QRS interval greater than 100 milliseconds. Progression of ECG changes
relatively predictable related severity of overdose. Mild overdoses produce sinus tachycardia,
mostly as a result of anticholinergic effects. More severe overdoses result in prolonged QRS QTc
224

intervals, followed by a prolonged PR interval, and, finally, ventricular arrhythmias, including


ventricular tachycardia ventricular fibrillation.
Alkalinisation sodium loading are effective in treatment of TCA induced conduction
disturbances, ventricular arrhythmias, hypotension. Sodium bicarbonate attenuates TCA
cardiotoxicity via several mechanisms: alkalinisation of a pH of 7.45 7.55 uncouples TCA from
myocardial sodium channels; also, additional sodium increases extracellular sodium
concentration, thereby improving gradient across channel. 4 )
17. A 64 year presents a two month history of deteriorating thirst weight loss fatigue.
10 years ago she was treated breast carcinoma underwent a right mastectomy since
treated Tamoxifen. She also takes Bendrofluazide 2.5 mg daily a three year history of
hypertension. No specific abnormalities are found on examination, except a pressure of
162/90 mmHg. She otherwise well. Her General Practitioner arranges a series of
biochemical investigations:
Sodium
Potassium
Urea
Creatinine
Calcium
Phosphate
PTH

145 mmol/l (133 144)


3.3 mmol/l (3.5 5.5)
9.4 mmol/l (3 8)
186 micromol/l (50 100)
3.2 mmol/l (2.2 2.6)
0.95 (0.7 1.2)
12 pmol/l (4 8)

What is the likely cause of her hypercalcaemia?


1 ) Drug induced
2 ) Ectopic PTH secretion
3 ) Metastatic bone disease
4 ) Myeloma
5 ) Primary hyperparathyroidism
Comments:
This patient hypercalcaemia associated elevated PTH indicating a diagnosis of primary
hyperparathyroidism. The history of breast Ca a red herring bendrofluazide although
associated hypercalcaemia would cause elevated PTH. 5)
18.
An 80 year old lady was reviewed in outpatient clinic. She was diagnosed polymyalgia
rheumatica eighteen months previously when she presented bilateral shoulder stiffness
generalized myalgia; ESR on presentation had 60 mm/1st hour. She was started on prednisolone
15mg daily along calcium vitamin D supplementation she responded well, resolutaion of her
symptoms within one week. However, her symptoms have persistently relapsed when dose of
prednisolone reduced below her current dose of 12.5mg of prednisolone daily.How should her
disease be managed?
1 ) Continue current dose of prednisolone
2 ) Continue current dose of prednisolone start methotrexate
3 ) Continue taper prednisolone treat any symptoms non steroidal anti inflammatory drugs
4 ) Increase dose of prednisolone add a bisphosphonate
5 ) Stop prednisolone start azathioprine

225

Comments:
Corticosteroids remain mainstay of treatment polymyalgia rheumatica (PMR). The starting dose depends
on patient's weight severity of symptoms. The optimum duration of treatment uncertain largely guided
by response therapy. Calcium vitamin D supplementation should be initiated all patients PMR who are
starting corticosteroid therapy.
The usual starting dose 15mg prednisolone per day. Patients should expect relief of symptoms within 24
72 h. The dose should be increased if symptoms are well controlled within 1 week. The effective starting
dose should be maintained 2 4 weeks after patient becomes asymptomatic. Generally, daily dose can
be lowered by 1.0 2.5 mg every 2 4 weeks find minimum dose needed maintain symptom suppression.
Once patient reduced 10mg per day, daily dose can be tapered by 1 mg every 4 weeks.
Tapering should be guided by clinical response. Normalization of inflammatory markers (especially ESR)
are helpful but should set guidelines decreasing stopping treatment. An isolated increase of ESR
without symptoms during course of treatment a valid reason increase corticosteroid dose; however, a
temporary delay in dosage reduction may be necessary.
Approximately 50 75% of patients can discontinue corticosteroid therapy after 2 years of treatment.
Relapses are more likely occur during initial 18 months of therapy within 1 year of corticosteroid
withdrawal. Patients should be closely monitored recurrence of symptoms throughout period of
corticosteroid tapering until 12 months after therapy stops.
Methotrexate azathioprine have used in patients corticosteroid intolerance as corticosteroid sparing
agents. These are generally reserved patients in whom it difficult reduce prednisolone after prolonged
high dosages (e.g. 10mg more per day more than a year). These agents should be added prednisolone
initially, but a view slowly reduce withdraw prednisolone. As steroid therapy, azathioprine
methotrexate can be discontinued if there sufficient response. 2 )
19.
A 39 year old man was referred renal clinic. Two weeks previously he had an episode of loin
pain, passed a small renal stone. He had one previous episode three years previously, but did seek
advice. He was otherwise fit well.
Haemoglobin
14.5 g/dL
White cells
7.5 x 109/L
Platelets
210 x 109/L
Serum sodium
137 mmol/L
Serum potassium
4.2 mmol/L
Serum urea
6.1 mmol/L
Serum creatinine
100 umol/L
Serum corrected calcium
2.3 (NR 2.2 2.6)
Volume
Calcium

1150 ml/24hr
18 mmol/24hr

(NR 2.5 7.5)

Analysis of stone showed it contain mostly calcium.


He initially advised increase his fluid intake, but he returns your clinic
after one month, having had two further episodes.
Which of following medications would you prescribe?
1
2
3
4
5

)
)
)
)
)

Allopurinol
Loop diuretic
Potassium bicarbonate
Potassium citrate
Thiazide diuretic

Comments:
226

This gentleman had a calcium urinary tract stone. He serum calcium, but raised urinary
excretion of calcium. Idiopathic hypercalciuria often familial, common cause being increased
gastrointestinal absorption of calcium. The common stones are calcium oxalate stones.
In this patient, his urinary output 1150 mls/24hr. He should aim a daily urinary output in
excess of 2000 ml. The correct answer therefore advise him increase his fluid intake. A high
protein intake associated urinary stones, by reducing his dairy dietary intake he may reduce
his GI absorption of calcium, but main initial treatment increase his urine volume.
Allopurinol would reduce incidence of uric acid stones, but would affect formation of calcium
stones.
Potassium citrate potassium bicarbonate can be used alkalinise urine, prevent formation of
cystine containing stones. They would have effect on formation of calcium containing stones.
Thiazide diuretics reduce renal tubular calcium excretion, therefore can prevent calcium stone
formation.
Loop diuretics increase urinary excretion of calcium, therefore would exacerbate calcium renal
stone formation. 5 )
20. A 41 year old woman presented hospital a two day history of severe right side
headache loss of vision. She had a past history of hypertension, which she was taking
bendrofluazide. There was a family history of thyroid problems. There was a past history
of intermittent headaches, though these were regular. Her current headache was worst
she had ever experienced.
On examination she was agitated distressed. Her temperature was 37.5oC, pulse 110 beats
per minute pressure 145/95 mmHg. There was chemosis proptosis of right eye. Upward
downward gaze was paralysed; lateral gaze was preserved. The right pupil was dilated
unresponsive light. The left eye was normal. Fundoscopy showed bilateral papilloedema.
What is the likely diagnosis?
1 ) Cavernous sinus thrombosis
2 ) Chronic paroxysmal hemicrania
3 ) Cluster headache
4 ) Graves ophthalmopathy
5 ) Hypertensive encephalopathy
Comments:
Cavernous sinus thrombosis a syndrome of chemosis, proptosis ophthalmoplegia. The usual
cause infection spreading from face, orbit paranasal sinuses. Symptoms include
periorbital/facial pain, conjunctival periorbital oedema, proptosis, papilloedema blindness
cranial nerve palsies (III, IV, V1, (V2), VI). Untreated, infection may lead subdural empyema,
meningitis infective arteritis of cranial arteries. There a slightly higher incidence in females
(1.3:1) a mean age of onset of 40 years.
Cluster headaches (migrainous neuralgia) characterized by recurrent episodes lasting 15 180
minutes of severe unilateral periorbital pain autonomic features. Episodes tend occur once
twice per day in clusters lasting weeks months, separated by periods of remission of 1 2 years.
Clinical features consist of severe boring unilateral periorbital pain ipsilateral watering
redness of the, miosis ptosis. There frequently ipsilateral rhinorrhoea. Attacks may be
precipitated by nitrates, alcohol calcium channel blockers. Administration of 100% oxygen
frequently terminates attacks within 10 minutes. Migrainous neuralgia much commoner in men
(10:1) a mean age of onset of 20 50 years. Cluster headaches are associated cranial nerve
palsies.
227

Chronic paroxysmal hemicrania (CPH) comer in women (3:1) a mean age of onset in 20s. The
conditionis characterized by frequent brief episodes of severe unilateral periorbital temporal
pain lasting 2 45 minutes recurring five more times per day. It frequently associated
autonomic symptoms (conjunctival injection, lacrimation, rhinorrhoea, ptosis, eyelid oedema)
often responds selectively (within days) indomethacin. The main features differentiating CPH
from cluster headaches (migrainous neuralgia, above) are higher frequency shorter duration of
attacks, higher incidence in women, response treatment indomethacin. CPH associated
cranial nerve palsies.
Graves ophthalmopathy typically bilateral associated features of hyperthyroidism. 1 )
21. A 21 year old man, known have epilepsy, was admitted Casualty having had a series
of tonic clonic convulsions at home. He was taking regular oral phenytoin. On examination,
he was unconscious having continuous seizures. He was placed into a lateral position
maintain his airway 10 L/min of oxygen was given via a high flow mask.
Intravenous access was secured he was given intravenous lorazepam. Five minutes later
he was still having seizures a further bolus of lorazepam was given; a further dose was
given after another five minutes because of continued seizures. The seizures appeared
have settled when, 30 minutes later, he had another generalized convulsion.
Which of following agents should be administered next?
1 ) Intravenous lorazepam
2 ) Intravenous fosphenytoin
3 ) Intravenous clonazepam
4 ) Intravenous thiopental
5 ) Intravenous sodium valproate
Comments:
Management of status epilepticus
1. Protect airway.
2. Give oxygen 10 L/min via high flow mask.
3. Administer benzodiazepine iv rectally. Lorazepam preferred because of long duration of anti
epileptic effect. This effective in ~80% cases.
4. If patient does respond, regime may be repeated after 5 10 minutes using same a different
benzodiazepine.
5. If seizures recur fail respond after 30 minutes a parenteral anti epileptic agent should be
started.
Intravenous phenytoin usually used given as a loading dose of 18mg/kg. Adverse effects are
common include CNS depression cardiac arrhythmias.
Fosphenytoin, a disodium phosphate ester of phenytoin, several advantages over phenytoin: it
can be given iv im (phenytoin can only be given iv) can be given at infusion rates three times
faster than phenytoin; therapeutic levels are achieved within 10 minutes; it a lower incidence of
adverse events than phenytoin. Fosphenytoin a pro drug of phenytoin metabolized in body
phenytoin endogenous phosphates.
If patient already taking phenytoin, either iv phenytoin fosphenytoin should still be given: it
likely plasma levels are subtherapeutic.
The anaesthetic agents thiopental propofol may be effective in controlling SE if above measures
fail (unlicensed indication) but should only be done full intensive care support. 2 )
22. A 65 year old man presented a four week history of pleuritic chest pain associated
shortness of breath dry cough. He also reported weight loss of nearly 10kg in past six
months.
228

He had a past history of myocardial infarction 20 years earlier from which he had made a
good recovery. He did suffer from any exertional chest pain subsequently. He lived alone
had seen his general practitioner two years. He seldom saw his General Practitioner, but
had attended surgery twice recently mild recurrent pain in his left knee responded well
treatment simple analgesia. He was an ex smoker of 15 cigarettes per day, having given
up smoking 20 years previously. His only medication was aspirin.
On examination of his chest he had reduced expansion, dull percussion note decreased
breath sounds on right. A chest X ray confirmed a right sided pleural effusion.
Analysis of a pleural aspirate revealed:
Pleural fluid protein content
42 g/L
Pleural fluid glucose
1.3 mmol/L
What is the diagnosis?
1 ) bronchial carcinoma
2 ) cardiac failure
3 ) mesothelioma
4 ) rheumatoid arthritis
5 ) tuberculosis
Comments:
The pleural fluid protein greater than 30g/l which demonstrates it an exudate effectively
excludes cardiac failure. If pleural fluid protein 25 35 g/l then Lights Criteria more accurate in
determining whether effusion an exudate transudate. It an exudate if 1 more of following
criteria are met (a) pleural fluid protein divided by serum protein > 0.5 (b) pleural fluid LDH
divided by serum LDH > 0.6 (c) pleural fluid LDH > 2/3rds upper limits of serum LDH.
The pleural glucose level very low. Levels less than 3.3 mmol/l are found in empyema,
rheumatoid arthritis, lupus, malignancy, oesophageal rupture tuberculosis. The lowest levels are
found in rheumatoid effusions empyema pleural glucose in rheumatoid effusions rarely being
above 1.6 mmol/l. 4 )
23. A 36 year old gay man presented hospital a four week history of progressively
worsening dyspnoea, dry cough fever. His chest x ray showed bilateral diffuse airspace
shadowing Pneumocystis carinii was isolated from bronchial washings following
bronchoscopy. His CD4 T lymphocyte count was 88 cells/mm3.
He had a past history of rash co trimoxazole was started on treatment Pneumocystis
carinii pneumonia trimethoprim dapsone. His condition improved within twenty four
hours, but five days later he became increasingly breathless again was cyanosed. His
oxygen saturations were measured at 85% by pulse oxymetry his PaO2 was 12.1 kPa.
What is the appropriate immediate course of action?
1 ) Start amoxicillin/clavulanic acid
2 ) Stop dapsone
3 ) Start nebulised pentamidine
4 ) Start prednisolone
5 ) Stop trimethoprim
Comments:
The patient methaemoglobinaemia caused by dapsone. Drugs cause methaemoglobinaemia
include: phenacetin; sulphonamides; dapsone; primaquine; lignocaine; procaine; benzocaine.
In methaemoglobinaemia an abnormally large proportion of iron in haem oxidized ferric state
leading impaired oxygen transport anaemic hypoxia. Patients appear be cyanosed, though
cyanosis does clear when oxygen administered.
229

Clinical features of methaemoglobinaemia depend on MetHb levels in blood. The discolouration


of appearance of cyanosis manifests when MetHb levels reach 1520%. Levels between 2045%
are associated dyspnoea, lethargy, dizziness headaches. MetHb levels above 45% are usually
associated impaired consciousness levels above 55% can cause seizures, coma cardiac
arrhythmias. The lethal concentration adults considered be >70%.
Assessments of oxygenation give conflicting results. Standard pulse oximeters give spuriously
low readings in presence of excess methaemoglobin. The patients cyanosis therefore
attributed hypoxia. Similarly, oxygen saturations measured by gas analysers will also be low
are accompanied by a high PaO2.
The treatment of choice in acquired methaemoglobinaemia a 1% solution of methylene blue.
Additional treatments which may need be instituted consist of hyperbaric oxygen therapy,
exchange transfusion, transfusion of RBCs, dialysis, administration of asorbic acid patients
G6PD deficiency. 2 )
24. A 65 year old man was diagnosed chronic lymphocytic leukaemia. Before starting
chemotherapy fludarabine, what antiobiotic prophylaxis should he receive?
1 ) Aciclovir
2 ) Ciprofloxacin
3 ) Co trimoxazole
4 ) Intravenous immunoglobulins
5 ) Itraconazole
Comments:
Fludarabine a purine analogue phosphorylated intracellularly. All of purine analogues cause
myelosuppression, but there a significantly higher risk of patients developing Pneumocystis
carinii pneumonia while on treatment. Use of prophylactic co trimoxazole (septrin) dramatically
reduced frequency of this severe opportunistic infection in these patients. Co trimoxazole
should be continued after chemotherapy until CD4 counts exceeds 200 cells/mm3 (0.2 x
109/L).
Read more NHS Scotland Haemato Oncology Guidelines on antibiotic prophylaxis in patients
undergoing chemotherapy and/or radiotherapy: 3 )
25. A 25 year old man developed bilateral loin pain frank haematuria. His symptoms had
started 24 hours after developing a sore throat. His pressure was 138/88 mmHg.
Urinalysis was positive (4+) protein (2+). What is the likely diagnosis?
1 ) IgA nephropathy
2 ) Infective endocarditis
3 ) Microscopic polyangiitis
4 ) Nephrolithiasis
5 ) Post streptococcal glomerulonephritis
Comments:
The patient macroscopic haematuria, proteinuria loin pain in association pharyngitis. IgA
nephropathy typically presents as recurrent haematuria in young men, often precipitated by
upper repiratory tract infections; loin pain well described; patients may be normotensive
hypertensive. Although acute post streptococcal glomerulonephritis (APSGN) a recognised
complication of streptococcal pharyngitis, there always a latent period of 1 2 weeks between
streptococcal infection onset of signs symptoms of acute glomerulonephritis (oedema,
hematuria, hypertension); loin pain well recognised. Nephrolithiasis associated loin pain
haematuria, but degree of proteinuria too great in this case. Although clinical presentation of
IgAN varies from asymptomatic urinary abnormalities acute renal failure, 5 different clinical
syndromes are generally recognized. 1 )
230

26. A 26 year old man presented hospital an eight hour history of palpitations
breathlessness at rest. There was past history of ischaemic heart disease. His
electrocardiogram showed atrial fibrillation a ventricular rate of 140 beats per minute his
pressure was 126/59 mmHg.
What best pharmacological agent use cardioversion?
1
2
3
4
5

)
)
)
)
)

Intravenous digoxin
Intravenous esmolol
Oral flecainide
Oral sotalol
Oral verapamil

Comments:
Hemodynamically stable patients AF can be converted sinus rhythm large doses of oral agents
intravenous agents. Large single doses of flecainide (300 mg) propafenone (450 600 mg) given
orally have shown convert patients sinus rhythm.
Several oral agents have used convert atrial fibrillation of recent onset sinus rhythm, including
class IA, class IC, class III antiarrhythmic drugs. The class IC agents flecainide propafenone
have advantage of acting rapidly, their efficacy in converting atrial fibrillation of recent onset
sinus rhythm documented in several placebo controlled trials. Flecainide propafenone are
used in people known suspected ischaemic heart disease, individuals who are already on
antiarrhythmic therapy, those a prolonged QT interval because these agents may have pro
arrhythmic effects (torsade des pointes). 3 )
27. A 54 year old man was brought hospital after collapsing at home. His wife stated he
had complaining of a worsening headach several days. On evening of admission he had a
seizure while watching television subsequently became unresponsive. There was past
history of note he was taking any regular medication.
On examination his Glasgow Coma Scale score was 3/15. His pulse was 110 beats per
minute pressure 220/110 mmHg. Heart sounds were his chest was clear. His abdomen
as soft non tender. Reflexes were brisk symmetrical. Fundoscopy showed bilateral
papilloedema. A CT scan of his brain did show any cerebral infarct haemorrhage.
What treatment should he receive?
1 ) Intravenous furosemide
2 ) Intravenous mannitol
3 ) Intravenous sodium nitroprusside
4 ) Intravenous verapamil
5 ) Sublingual nifedipine
Comments:
The patient characteristic features of malignant hypertension. He markedly hypertensive
encephalopathic. In treating patients malignant hypertension, care should be taken lower
pressure suddenly. Nifedipine of use in patients who are conscious; frusemide can be used
when there concomitant left ventricular failure. However, patients who are encephalopathic
(comatose fitting) need be managed on ICU minute by minute pressure control either sodium
nitroprusside. Labetolol hydralazine. 3 )
28. A 32 year old man referred general outpatient clinic. He his wife had referred by
his GP infertility clinic consideration assisted conception. He generally fit well works
231

as a central heating engineer. He referred because he a chronic productive cough a


history of recurrent chest infections since childhood in addition recurrent siunsitis.
Investigations show:
Sodium sweat test
Serum IgG 7.4 g/L
Serum IgA 1.2 g/L
Serum IgM 3.1 g/L

Normal
(NR 6 13)
(NR 0.8 3.0)
(NR 0.4 2.5)

What is the likely diagnosis?


1 ) Bronchiectasis
2 ) Chediak Higashi syndrome
3 ) Cystic fibrosis
4 ) Primary ciliary dyskinesia
5 ) Situs inversus
Comments:
Primary ciliary dyskinesia inherited as an autosomal disorder characterized by abnormal ciliary
motion impaired mucociliary clearance. The lack of effective ciliary motility, causes abnormal
mucociliary clearance. This leads recurrent persistent respiratory infections (which may lead
bronchiectasis), sinusitis, otitis media, male infertility. In 50% of patients, PCD associated
situs inversus (Kartageners syndrome).
The principal differential diagnoses consider in this case are cystic fibrosis primary ciliary
dyskinesia. The diagnosis of CF based on typical pulmonary and/or gastrointestinal tract
manifestations positive results on sweat test (pilocarpine iontophoresis). A negative sweat test
sufficient evidence exclude CF as a diagnostic possibility.
While patient may have bronchiectasis, this alone sufficient account his other symptoms. 4 )
29. 62 yr man was referred by his General Practitioner an outpatient endoscopy. The
patient gave a four week history of worsening odynophagia dysphagia. He attributed
onset of his symptoms a course of antibiotics he received from his GP a sore throat six
weeks previously. He had past history of note.
Investigations performed by his GP revealed:
Haemoglobin
9.1 g/dL
MCV
94 fL
White cell count
2.05 x 109/L
Neutrophils
1.2 x 109/L
Lymphocytes
0.8 x 109/L
Monocytes
0.7 x 109/L
Eosinophils
0.01 x 109/L
Basophils
0.04 x 109/L
Platelets
124 x 109/L
Upper gastrointestinal endoscopy demonatrated extensive oesophageal candidiasis.
What further investigation, if any, required?
232

1 ) Barium swallow
2 ) Check lymphocyte subsets
3 ) HIV antibody test
4 ) Reassure
5 ) Repeat endoscopy in four weeks
Comments:
Oropharyngeal Candidiasis can result from prolonged broad spectrum antibiotic therapy, but
seldom does Candida affect oesophagus under these circumstances. In this patient, count
shows pancytopoenia. This combination highly suggestive of undiagnosed HIV infection. Where
HIV infection suspected, patient should always be formally tested checking lymphocyte
subsets as a surrogate marker HIV disease unreliable strongly discouraged. 3 )
30. A 28 year old woman epilepsy attended outpatient clinic after having a positive
pregnancy test. She had had period 13 weeks usually had a regular 28 day menstrual
cycle. She had diagnosed epilepsy at 19 years of age when she suffered two tonic clonic
seizures. At time of diagnosis of epilepsy she was started on sodium valproate had taken
this nine years without suffering any further seizures.Given positive pregnancy test, what
action should be taken regard her anticonvulsant therapy?
1 ) Continue sodium valproate
2 ) Convert sodium valproate carbamazepine
3 ) Convert sodium valproate lamotrigine
4 ) Reduce dose of sodium valproate
5 ) Stop sodium valproate
Comments:
Women on anticonvulsant therapy are advised plan their pregnancies carefully so any
adjustments therapy can be made before they become pregnant.
Foetal malformations (neural tube defects, cleft lip palate, VSD) are 25% commoner in babies
born women untreated epilepsy than in women who do have epilepsy; but are 50% commoner
in epileptic women who are taking anticonvulsants.
The risk of neural tube defects skeletal abnormalities highest in babies born women taking
sodium valproate (1 2%) carbamazepine (0.5 1%). The risk of VSD cleft lip/palate highest in
those taking phenobarbitone barbiturates. Most women epilepsy who are taking
anticonvulsants deliver perfectly babies.
Had this woman presented before becoming pregnant, a decision might have made reduce
dose of valproate switch an alternative agent. However, reducing switching therapy needs be
done over a period of several months. Also, these agents are likely cause defects early in foetal
life certainly earlier than 13 weeks therefore best option at this stage continue her current
therapy. 1 )
31. 62 yr man was referred by his General Practitioner an outpatient endoscopy. The
patient gave a four week history of worsening odynophagia dysphagia. He attributed
onset of his symptoms a course of antibiotics he received from his GP a sore throat six
weeks previously. He had past history of note.
Investigations performed by his GP revealed:
Haemoglobin
9.1 g/dL
MCV
94 fL
White cell count
2.79
233

Neutrophils
1.2 x 109/L
Lymphocytes
0.8 x 109/L
Monocytes
0.7 x 109/L
Eosinophils
0.04 x 109/L
Basophils
0.05 x 109/L
Platelets
124 x 109/L
Upper gastrointestinal endoscopy demonatrated extensive oesophageal candidiasis.
What treatment, if any, required?
1
2
3
4
5

)
)
)
)
)

Nystatin oral suspension 5 days


Intravenous amphotericin B
Oral fluconazole 14 21 days
Single dose of oral intraconazole
Reassure treatment required

Comments:
Oropharyngeal Candidiasis can result from prolonged broad spectrum antibiotic therapy, but
seldom does Candida affect oesophagus under these circumstances. In this patient, count
shows pancytopoenia. This combination highly suggestive of undiagnosed HIV infection. Where
HIV infection suspected, patient should always be formally tested checking lymphocyte
subsets as a surrogate marker HIV disease unreliable strongly discouraged.
Although oropharyngeal candidiasis may be treated topical antifungal agents (nystatin,
clotrimazole, amphotericin B oral suspension/lozenges). Candida oesophagitis requires systemic
therapy, usually fluconazole itraconazole at least 14 21 days. 3 )
32. A 62 year old woman was referred endocrinology clinic by her General Practitioner
a lump in her neck. There were accompanying symptoms of hyperthyroidism
hypothyroidism. On clinical examination she had a painless solitary thyroid nodule she
was clinically euthyroid. A fine needle biopsy demonstrated follicular carcinoma of
thyroid.
Which of following treatments should she receive first?
1
2
3
4
5

)
)
)
)
)

Carbimazole
Doxorubicin
External beam radiotherapy
Radioactive iodine
Total thyroidectomy

Comments:
There are four main types of thyroid cancer (in order of frequency):
1. Papillary
2. Follicular
3. Medullary
4. Anaplastic
Follicular thyroid carcinoma (FTC) a well differentiated tumor. In fact, FTC resembles
microscopic pattern of thyroid. FTC originates in follicular cells second common cancer of
234

thyroid after papillary carcinoma. The common presentation of thyroid cancer an asymptomatic
thyroid mass, a nodule, can be felt in neck.
The staging of well differentiated thyroid cancers related age first second stages but related
third fourth stages.
Younger than 45 years:
Stage I: Any T, any N, M0 (Cancer in thyroid only.)
Stage II: Any T, any N, M1 (Cancer spread distant organs.)
Older than 45 years
Stage I: T1, N0, M0 (Cancer in thyroid only may be found in one both lobes.)
Stage II: T2, N0, M0 T3, N0, M0 (Cancer in thyroid only larger than 1.5 cm.)
Stage III: T4, N0, M0 any T, N1, M0 (Cancer spread outside thyroid but outside of neck.)
Stage IV: Any T, any N, M1 (Cancer spread other parts of body.)
Surgery definitive management of thyroid cancer. Various types of operations may be
performed:
Lobectomy isthmectomy
This minimal operation a potentially malignant thyroid nodule. Patients <40 years who have
FTC nodules <1 cm, well defined, minimally invasive, isolated may be treated
hemithyroidectomy isthmectomy.
Subtotal thyroidectomy (small part of contralateral lobe retained)
If feasible, subtotal thyroidectomy preferable since it carries a lower incidence of complications
(eg, hypoparathyroidism, superior and/or recurrent laryngeal nerve injury).
Total thyroidectomy (removal of all thyroid tissue preserving contralateral parathyroid glands)
Approximately 10% of patients who have had total thyroidectomy demonstrate cancer in
contralateral lobe. Total thyroidectomy should be performed in patients who are >40 years FTC
in any patient bilateral disease. Total thyroidectomy recommended any patient a thyroid
nodule a history of irradiation. Some studies show lower recurrence rates increased survival
rates in patients who have undergone total thyroidectomy. This surgical procedure also
facilitates earlier detection treatment of recurrent metastatic carcinoma.
Patients receive radioiodine 4 6 weeks after thyroidectomy detect destroy any metastases any
residual tissue in thyroid.
Following thyroidectomy, patients will need take thyroid replacement therapy.
External beam radiation used in management of FTC if cancer cannot be resected, if there
extension into adjacent structures. Radiotherapy may also be administered postoperatively
reduce risk of local regional recurrence. It may also be used palliatively treat pain from bone
metastases.
Chemotherapy cisplatin doxorubicin limited efficacy. It may be employed when other
treatment modalities have failed. 5 )
33. The nurses on Neurology ward asked Senior House Officer review a patient who
had developed a fever. The patient was a 42 year old man who had on ward several
months had a long term urinary catheter in situ. The patients neurological status was
unchanged since admission, but he was able give any history.
On examination, patient was lying in bed was awake. He was febrile 37.5oC. He appeared
comfortable a respiratory rate of 12 per minute pulse 75 beats per minute, regular. Blood
pressure 120/75 mmHg. His pupils were reactive light, but patient could track a moving
object eye movements appeared be random. He did respond verbal commands, though
he could move all four limbs spontaneously but clear purpose. He did respond any
verbal tactile stimuli but pressure on nail bed elicited a flexor response. Both plantar
responses were extensor. The patient made moaning noises but there was coherent
235

speech. On occasions patient laughed spontaneously but any specific reason. The
nurses reported he had a sleep cycle he was fed by a gastrostomy feeding tube.
Urinalysis showed presence of white cells, protein nitrites microscopy Gram negative
rods.
What neurological diagnosis?
1
2
3
4
5

)
)
)
)
)

Akinetic mutism
Brain death
Coma
Locked in syndrome
Permanent vegetative state

Comments:
Permanent vegetative state (PVS) defined as a state of wakefulness without awareness. The
patient breathes spontaneously without mechanical support, haemodynamically stable cycles
of eye closure opening resemble a sleeping pattern but patient inattentive unaware of
his/her surroundings. Patients may have spontaneous movements (moaning, grunting, teeth
grinding, roving eye movements) may also smile, laugh cry without any apparent reason.
Although there may be eye movement, eyes do track a moving object. Patients may respond
painful stimuli may have myoclonus in response startling stimuli. Primitive reflexes may be
present. Posture may become decorticate plantar responses are commonly extensor.
The condition typically occurs when there irreversible damage cerebral hemispheres but brain
stem remains intact. Causes include head injury, hypoxic injury (cardiac arrest, carbon monoxide
poisoning), stroke, hypoglycaemia, intracranial infection, end stage degenerative brain disease
(e.g. Alzheimers).
Locked in syndrome a de afferented state in which patients are aware of themselves their
environment but are unable respond due loss of motor speech function. The cause usually
either (1) an upper motor neuron lesion of descending corticospinal tracts in brainstem (often
pons) below level of oculomotor nerve nuclei, (e.g. infarction, haemorrhage, tumour,
demyelination, head injury, central pontine myelinolysis after hyponatremia) (2) widespread
lower motor neuron disease (e.g. polyneuropathy such as Guillain Barre syndrome). Clinically,
patient unable speak move but patients may be able open their eyes may blink in an effort
communicate.
Brain death involves irreversible loss of all brainstem function. Unlike PVS, brainstem function
lost mechanical respiratory support required. Brain death is, by definition, irreversible cardiac
arrest usually ensues within hours days of onset of brain death. The diagnosis should be made
by two physicians on at least two separate occasions 12 24 hours apart. Clinically, patient
unconscious. Absent brainstem function demonstrated by pupils dilated (mid fully) unreactive
light. Corneal reflex absent. Dolls eye oculocephalic caloric reflexes are absent. Cough, suck
gag reflexes are absent. Ventilatory reflexes are absent there spontaneous respiration when
patients ventilator switched off sufficient period ensure pCO2 risen above threshold
stimulation of respiration. Muscle tone flaccid there spontaneous movement. In order make
diagnosis, these findings should be consistent.
Akinetic mutism a state of profound apathy in which there evidence of preserved awareness
visual attention tracking. Patients frequently appear as though they are about speak but do
(promise of speech).

236

Coma. Comatose patients are unconscious unresponsive unaware of themselves their


environment cannot be roused into a state of awareness respond their environment. Cyclical
eye opening absent respiratory function usually depressed. 5 )
34. A 34 year old lady presented hospital severe right loin pain, haematuria. She had
noticed some right loin discomfort twenty four hours previously, but acute severe pain
started two hours before admission. She denied having any dysuria, fever, rigors.
She had a past history of Crohns disease, diagnosed when she was 18 years old. One year
previously she had undergone an ileal resection after presenting an acute exacerbation of
Crohns an intestinal perforation. Her regular medication consisted of azathioprine 150mg
daily. She was married two children smoked five cigarettes per day.
On examination she was afebrile. Pulse 100 beats per minute, regular. Blood pressure
115/65 mmHg. Heart sounds were chest was clear. Her abdomen was soft non tender,
right loin was tender palpation.
Investigations:

Haemoglobin
White cell count
Platelets
Serum sodium
Serum potassium
Serum urea
Serum creatinine
ESR (Westergren)

11.5 g/dL
8.1 x 109/L
367 x 109/L
139 mmol/L
4.1 mmol/L
6.2 mmol/L
89 umol/L
12 mm/1st hour

Urinalysis showed protein but was negative white cells nitrates.


Microscopy did demonstrate any white cells organisms. A plain x ray of kidneys ureters
bladder (KUB) showed a radio opaque adjacent right kidney.
She was treated pethidine pain relief given intravenous fluids. Twelve hours later
stones passed spontaneously her symptoms resolved.
What of following would be effective in preventing further renal calculi?
1 ) Allopurinol
2 ) Dietary exclusion of chocolate, tea, rhubarb spinach
3 ) Increase daily oral fluid intake
4 ) Regular lithotripsy
5 ) Thiazide diuretic
Comments:
Hyperoxaluria occurs both in patients an ileal resection in patients a short bowel who have had
a distal small bowel resection (e.g. Crohns disease, infarcted bowel). It caused by increased
absorption of oxalate by colon. Bile salts in colon increase oxalate absorption. Hyperoxaluria
associated renal stone formation, propensity form stones reduced when citrate intake
reduced.
237

Treatment involves having a low oxalate diet taking cholestyramine bind bile salts citrate
prevent stone formation. Low oxalate diets typically exclude cocoa, peanut products, tea, coffee,
wheat germ, rhubarb, beetroot, spinach, tofu soybeans restrict citrus drinks, diet soda drinks,
tomatoes fruit. In addition, calcium excretion can be reduced by using bendofluazide 5mg daily
regular pyridoxine 10mg daily can reduce hyperoxaluria.
Prevention of other renal stones
Calcium phosphate stones can be prevented thiazide diuretics a low calcium diet (restrict dairy
products). Triple phosphate stones often cause staghorn calculi patients require prophylactic
antibiotics prevent recurrent infections; these stones may require surgery removal. Urate
stones may be prevented by giving regular allopurinol by urinary alkalinisation. Prevention of
cysteine stones requires help prevent calcium phosphate stones. Methods of preventing citrate
stones includes adequate hydration (increase daily fluid intake), D penicillamine urinary
alkalinisation. 2 )
35. A 31 year old man was referred dermatology clinic after developing a rash on his
arms legs, predominantly on knees elbows. The rash had present four weeks.
He had a history of hypertension had started on treatment furosemide rampiril by his
General Practitioner six months previously. He also had a long history of bipolar disorder
had started on lithium three months previously by his psychiatrist having taking
chlorpromazine five years. Six weeks previously he had given a course of oxytetracycline
acne.
Which of his medications likely have precipitated rash?
1 ) Chlorpromazine
2 ) Furosemide
3 ) Lithium
4 ) Oxytetracycline
5 ) Ramipril
Comments:
The clinical history gives a good description of psoriasis. Lithium likely precipitant. Several
drugs are known exacerbate trigger onset of psoriasis, including:
1. Beta blockers
2. Lithium
3. Antimalarials
4. NSAIDs
Reactions may occur form less than one month one year after medication initiated. Treatment
of drug induced psoriasis comprises withdrawal of all beta blocking medications, nonsteroidal
anti inflammatory drugs, antimalarials lithium, unless absolutely necessary. Skin punch biopsy
may be performed exclude other forms of erythroderma pustulosis. Bed rest, bland topical
compresses, low potency topical steroids are useful. Frequent emollient use advisable.
Etretinate, methotrexate, phototherapy might be considered.
Chlorpromazine may be associated development of drug induced lupus. 3 )
36. A 30 year old man presented a painless visual loss of his left eye over 24 hours.
Over previous two weeks he had lost vision in his right eye. There had any improvement
of his visual acuity over next 2 months. He denied any other symptoms.
His previous history was unremarkable. He smoked 10 cigarettes per day drank 20 units
of alcohol per week. There was relevant family history. He did take any drugs.

238

Physical examination was unremarkable. His visual acuity was 6/60 on right finger
counting on left. His colour vision iwa impaired in both eyes. He had bilateral optic
atrophy. The remainder of his neurological examination was normal.
Investigations showed:
Full count
Normal
Serum urea & electrolytes
Normal
Liver function tests
Normal
TFTs
Normal
Serum calcium
Normal
Serum glucose
Normal
Autoantibody screen
Normal
VDRL
Negative
Serum vitamin B12
Normal
Chest x ray
Normal
MRI brain orbits
Normal
CSF analysis
Unremarkable
What is the likely diagnosis?
1 ) Multiple sclerosis
2 ) Alcohol/tobacco induced optic neuropathies
3 ) Lebers optic atrophy
4 ) Giant cell arteritis
5 ) Glaucoma
Comments:
Lebers optic atrophy usually affects young men. It causes sequential optic neuropathies in days
weeks. It typical painless severe. Visual acuity fails improve. Optic neuritis usually painful,
visual acuity improves over a matter of weeks. Giant cell arteritis affects elderly patients.
Alcohol/Tobacco optic neuropathies are usually chronic. 3 )
37. A 51 year old man was referred acute intake a two week history of severe pain
around his right eye. The pain had started without any apparent precipitating event had
woken him from his sleep. He described pain as severe boring radiated upwards over
right frontal temporal region. The pain was constant, but seemed start every evening
persist periods of 20 minutes over an hour. The headaches were associated watering of
his right eye a blocked left nostril. There was past history of similar episodes other past
history of note.
On examination he was alert orientated a Glasgow Coma Scale score of 15/15. His
pressure was 125/75 mmHg. He was afebrile there was neck stiffness. The right eye was
red conjunctival injection mild oedema of eyelid. There was a partial right sided ptosis
miosis.
Which of following therapeutic options likely alleviate his symptoms?
1
2
3
4

)
)
)
)

Aspirin
Carbamazepine
Indomethacin
Oxygen 100% FiO2
239

5 ) Verapamil
Comments:
The likely diagnosis cluster headache (migrainous neuralgia). This characterised by recurrent
episodes lasting 15 180 minutes of unilateral periorbital pain associated autonomic features.
Cluster headaches can occur at any age, but are common in age 20 50 years; there a strong
male preponderance (10:1). The pain described as severe boring radiates up over
frontotemporal region and/or down jaw, neck shoulder. There frequently accompanying
misosis without a Horners syndrome. The ipsilateral eye often red watery; there often
associated rhinorrhoea a blocked ipsilateral nostril.
Oxygen 100% (6 8 L/min) often provides relief within 10 minutes. Sumatryptan may also be used
in acute attack. Ergotamine, methysergide, verapamil prednisolone have used as prophylaxis
once acute attack resolved.
Vasodilators (alcohol, nitrates, calcium channel blockers) may precipitate attacks during acute
period, but in remission.
The main differential between cluster headaches chronic paroxysmal hemicrania (CPH; which
treated indomethacin). Features distinguish CPH are: shorter duration of attacks (2 45 mins);
increased frequency of attacks; preponderance selective response treatment indomethacin. 4 )
38. A 56 year old woman was referred hospital by her GP a three day history of malaise,
myalgia, fever pain in her neck. Ten days previously she had had a self limiting upper
respiratory tract infection but had received any antibiotics. She had significant past
history did take any regular medication.
On examination she appeared anxious unwell. Temperature 37.9oC, pulse 130 per minute
regular, pressure 125/75 mmHg. Heart sounds were murmurs her chest was clear.
Examination of abdomen was unremarkable. Neurological examination a fine resting
tremor, but was otherwise unremarkable. The thyroid gland was diffusely enlarged very
tender touch.
Investigations revealed:
Haemoglobin
13.5 g/dL
White cell count
9.3 x 109/L
Platelets
355 x 109/L
ESR (Westergren)
98 mm/1st hour
Plasma
T4 nmol/L (58 174)
Plasma
T3 nmol/L (1.07 3.18)
Plasma TSH
< 0.05 mU/L (0.4 5.0)
Which of following investigation likely be helpful in establishing diagnosis?
1
2
3
4
5

) Blood cultures
) Fine needle aspiration
) Radioactive iodine uptake scan
) Serum anti thyroid antibodies
) Ultrasound scan of neck

Comments:
The patients symptoms signs are typical of De Quervains thyroiditis.
240

De Quervain's Thyroiditis (also known as subacute thyroiditis granulomatous thyroiditis) causes


diffuse, tender enlargement of thyroid gland. The thyroid enlargement typically rapid,
occurring over a period of days. The syndrome often starts after a respiratory tract infection it
likely condition a viral aetiology (although causative infectious agent found). Patients feel
systemically unwell myalgia, fever prostration. Plasma thyroid hormones are greatly elevated as
ESR, but anti thyroid antibodies are present. There are often marked signs symptoms of
thyrotoxicosis. Radioiodine uptake typically less than 1% at 24 hours (Tc 99m uptake similarly
low).
Treatment usually bed rest aspirin reduce inflammation. Occasionally steroids are used
reduce inflammation. After thyroid depleted of thyroid hormone, patients' serum levels of T4
T3 decrease into hypothyroid range. The hypothyroidism usually mild but persists 2 4 months.
A few patients (~5%) remain hypothyroid need long term thyroid hormone replacement.
Recurrences are uncommon.
Thyroid ultrasound alone helpful in distinguishing between abnormalities caused by subacute
thyroiditis other causes of high thyroid hormone levels, including Graves thyroiditis. Ultrasound
useful in assessing discrete nodules. Fine needle aspiration useful when there a solitary
thyroid nodule. 3 )
39. A 36 year old found unresponsive at a home by family members. She a known
history of depression. On examination, she a respiratory rate of 6/min, temperature of
38C, pulse 140 beats/min BP 85/40. Chest was clear bilaterally shallow respiratory
effort. No murmurs were detected she had thready peripheral pulses. Abdomen was soft
non tender. On neurological examination, she had meningism, moved all four limbs deep
pain had symmetrically brisk reflexes.
Investigations:
Hb

13.1 g/dl

WCC

11.9 x 109/l

Platelets

350 x 10 9/l

Plasma sodium

132 mmol/l

Plasma potassium

3.8 mmol/l

Plasma urea

7.1 mmol/l

Plasma creatinine

110 umol/l

Plasma glucose

5.5 mmol/l

Plasma calcium

2.20 mmol/l

Chest x ray
ECG

Normal
Wide complex tachycardia

Arterial gases on air


PaO2

11.2 kPa

PaCO2

5.9 kPa

pH

7.31

CT brain
CSF opening pressure

Normal
16 cm H2O
241

CSF cell count

2 cells/ml

CSF protein

0.3 g/l

CSF glucose

4.5 mmol/l

Shortly after lumbar puncture, she had a grand mal seizure.What is the likely diagnosis?
1
2
3
4
5

)
)
)
)
)

Cerebral hypoperfusion secondary cardiac disease


Tricyclic antidepressant overdose
Herpes simplex encephalitis
Meningoccocal meningitis
Subarachnoid haemorrhage

Comments:
The are localizing neurological signs, therefore suggesting presence of a generalised
disturbance. This, in association seizure development ECG changes of a wide complex
tachycardia, makes diagnosis of tricyclic antidepressant overdose likely. The lumbar puncture
results do support a central nervous system infection a subarachnoid haemorrhage. 2 )
40. A 21 year old student presented hospital two weeks after returning from a three
month elective period in Africa. He gave a seven day history of a non productive cough a
fever. He had seen his General Practitioner previous day had started on a course of
amoxicillin.
He had spent his elective in Kenya, but had also spent time travelling in Malawi South
Africa. He developed a febrile illness diarrhoea two weeks before returning UK was seen
by a doctor in South Africa prescribed a course of metronidazole presumed amoebiasis
his symptoms settled within three days. There was other past history of note.
On examination he was febrile (38.5oC) an urticarial rash was visible over trunk. There
was palpable lymphadenopathy. His pulse was 90 beats per minute in sinus rhythm
pressure 115/65 mmHg. His chest was clear. His abdomen was soft slightly tender in left
hypochondrium, where tip of spleen could be palpated.
Investigations showed:

Haemoglobin
White cell count
Neutrophils
thLymphocytes
Monocytes
Eosinophils
Basophils
Platelets
Chest x ray:
Amoebic serology

14.0 g/dL (NR 13.0 18.0)


9.3 x 109/L (NR 4.0 11.0)
5.3 x 109/L (NR 1.5 7.0)
1.5 x 109/L (NR 1.5 4.0)
0.1 x 109/L (NR <0.8)
1.4 x 109/L (NR 0.04 0.4)
0.1 x 109/L (NR <0.1)
350 x 109/L (NR 150 400)
Normal
Negative
242

What is the likely diagnosis?


1 ) Amoebiasis
2 ) HIV seroconversion illnes
3 ) Schistosomiasis
4 ) Trypanosomiasis
5 ) Typhoid fever
Comments:
The symptoms of acute schistosomiasis are very similar any acute viral bacterial infection it
therefore important establish a travel history an endemic area. Some patients develop a rash at
site of entry of cercariae into skin (swimmers itch) others develop systemic upset. This
systemic upset (Katayama syndrome) what seen in this case. Fever, lethargy, myalgia are
common symptoms; patients may also have cough, headache, anorexia, a generalized rash. If
acute schistosomiasis unrecognized, chronic infection occurs can lead significant morbidity
mortality. The other significant marker in this question presence of an eosinophilia strongly
suggestive of parasitic infection a a feature of any of other choices listed.
Schistosoma mansoni
Geographical distribution: Caribbean, eastern Mediterranean countries, South America,
countries in Africa Causes: Intestinal schistosomiasis liver disease (fibrosis, portal
hypertension). Some patients hepstosplenic disease develop schistosomal cor pulmonale. Spinal
schistosomiasis, presenting as transverse myelitis (travellers myelitis), primarily due S.
mansoni infection
Schistosoma japonicum
Geographical distribution: western Pacific countries (China, Philippines, Indonesia, Thailand).
Causes: Intestinal schistosomiasis liver disease (fibrosis, portal hypertension). Also associated
cerebral schistosomiasis.
Schistosoma haematobium
Geographical distribution: Africa eastern Mediterranean Causes: Urinary tract disease. Rarely
causes intestinal liver disease. Chronic infection associated development of bladder cancer. 3 )
41. A 45 year old man presented diplopia, dysarthria difficulty swallowing. Over next
few days he developed weakness of upper lower limbs. On day 4 he was unable walk
unaided. He denied any sensory symptoms bladder disturbances. His previous history
unremarkable. He a non smoker, does drink alcohol excessively. He does take any drugs.
On examination he was apyrexial. His general examination was normal. His higher mental
function was unremarkable. There were signs of meningism. Cranial nerve examination
showed bilateral dilated fixed pupils. He had binocular diplopia but obvious
ophthalmoplegia. He was dysarthric weak cough. His vital capacity was 3.15 standing 2.00
lying flat. He had lower motor neuron tetraparesis of power 3/5. He was hyporeflexic
sensation was unable walk unaided.
Investgations including full count, urea electrolytes, liver function tests, serum calcium,
thyroid function tests, autoantibody screen, erythrocyte sedimentation rate serum C
reactive protein were normal.
His electrocardiogram chest radiograph were normal. A CT scan of his brain was normal,
as were nerve conduction studies an electromyogram (EMG).
What is the likely diagnosis?
1 ) Guillain Barre Syndrome
2 ) Lyme disease
243

3 ) Myasthenia gravis
4 ) Botulism
5 ) Vasculitis
Comments:
The clinical presentation of descending weakness autonomic dysfunction (fixed dilated pupils)
typical of botulism. It a neuromuscular junction disorder therefore nerve conduction studies
EMG are normal. Repetitive nerve stimulation showing incremental responses, which diagnostic
of Botulism. CSF analysis usually normal. 4 )
42. A 45 year old dockyard worker presented symptoms of fatigue general malaise. He
also reported developing ulcers on his hands after minor trauma.

What is the likely diagnosis?


1 ) Acute intermittent porphyria
2)
Alcoholic haemosiderosis
3)
Contact dermatitis
4)
Haemachromatosis
5)
Porphyria cutanea tarda
Comments: Porphyria cutanea tarda (PCT) associated deficiency of hepatic uroporphyrinogen
(URO) decarboxylase. The major clinical feature cutaneous photosensitivity. Bullae develop on
sun exposed areas lesions heal slowly, leaving scars. Porphyrins are increased in liver, plasma,
urine stool. Porphobilinigen (PBG) normal. Factors contributing PCT are alcohol (the
commonest cause), excess iron excess oestrogens. 5 )
43. A 20 year old woman presented hospital a one week history of increasing fatigue
exertional dyspnoea. She also complained of spontaneous appearance of numerous
bruises. On day of admission she suddenly developed a high fever became drowsy. On
examination she was pale looked unwell. She was drowsy but rousable. Temperature
244

39.5oC, pulse 120 per minute regular, pressure 85/40 mmHg. There was palpable
lymphadenopathy. Heart sounds were her chest was clear auscultation. Several large
bruises were noted on her arms, legs abdomen.
Investigations revealed:
Haemoglobin
7.5 g/dL
White cell count
40.1 x 109/L
Neutrophils
0.1 x 109/L
Lymphocytes
0.8 x 109/L
Monocytes
0.1 x 109/L
Eosinophils
0.098 x 109/L
Basophils
0.002 x 109/L
Promyelocytes
39 x 109/L
Platelets
12 x 109/L
PT
25 sec (Control 11.5 - 15.5)
APTT
50 sec (Control 30 - 40)
Fibrinogen
0.05 g/L (1.8 - 5.4)
D Dimer screen
100 mg/L (NR <0.5)
A high power picture of her film shown below.

Which of following chromosomal abnormalities likely be associated this patients


illness?
1
2
3
4

)
)
)
)

t(8;14)
t(8;21)
t(9; 22)
t(15;17)
245

5)
t(16;16)
Comments: The patients symptoms signs are characteristic of acute promyelocytic leukaemia
(AML M3). Classification cytogenetics of acute myeloid leukaemias The French American British
(FAB) classification system of acute myeloid leukaemias based on type of cell from which
leukemia develops stage of cellular differentiation/maturation. Eight categories of AML are
recognised: M0 (undifferentiated AML) M1 (myeloblastic immature) M2 (myeloblastic
mature) M3 (promyelocytic; acute promyelocytic leukaemia; APL) M4 (myelomonocytic)
M5 (monocytic) M6 (erythroid) M7 (megakaryoblastic) Subtype M2 (acute myeloblastic
leukaemia) common (25% of adult AML) associated a t(8;21) translocation. Subtypes M0, M6,
M7 carry worst prognosis. Acute myelomonocytic leukaemia (AML M4) associated a t(16;16)
translocation. Acute promyelocytic leukaemia AML M3 (acute promyelocytic leukaemia; APL)
usually associated a t(15;17) translocation. Promyelocytes cytoplasm heavily granulated. These
granules contain pro coagulant factors Render patients APL susceptible development of DIC.
Despite this, APL best prognosis of all subtypes of AML. Unlike other AML subtypes, APL
treated all trans retinoic acid (ATRA). The slide in this question shows promyelocytes: cells have
large, dark staining nuclei granular cytoplasm; rod shaped projections in cells are Auer rods.
Auer rods are elongated, bluish red rods composed of fused lysosomal granules, seen in
cytoplasm of myeloblasts, promyelocytes monoblasts. They may be single multiple. The
presence of large numbers of promyelocytes Auer rods cytoplasmic granules in peripheral
circulation typical of promyelocytic leukaemia. Cytogenetics of other haematological
malignancies The acquired chromosomal abnormality 90 95% of patients CML a translocation
between chromosome 9 22 (t(9;22)), resulting in Philadelphia chromosome. 90% of cases of
Burkitt's Lymphoma are due t(8;14) translocation. 4 )
44. A 36 year old presents thirst frequency of micturition. These symptoms have
deteriorated over last three months she now aware of twice nightly nocturia occasionally
needs drink during night. She aware of intermittent frontal headaches over this time
which are generally relieved by paracetamol are without any relationship time of day. She
rather stressed of late since break up of her marriage six months ago. Since separation
she taking Fluoxetine 20 mg daily. No abnormalities are detected upon examination. She
undergoes an 8 hour water deprivation test which reveals following:
Time

Weight (kg)

Plasma Osm

Urine Osm

Urine Vol mls

9am

66.5

285
110

200

10am

200

290

11am

270

220

310

200

1 pm

352

120

2 pm

320

80

420

50

8am

12 midday

66

300

3 pm

4 pm

65.2

303

DDAVP 1 microgram sc given at 4 pm


6pm

66

295

permitted
660

drink
20

DDAVP 1 microgram sc given at 4 pm permitted drink 6pm 66 295 660 20 <P] range
plasma osmolality 275 - 290 mosmol/kg] What is the diagnosis?
246

1
2
3
4
5

)
Complete central diabetes insipidus
)
Fluoxetine induced diabetes insipidus
)
Nephrogenic diabetes insipidus
)
Partial diabetes insipidus
)
Primary polydipsia
Comments: This patient developed haemoconcentration following 8 hrs of dehydration but
although begun concentrate her urine this inadequate constitutes partial DI. She responds
well DDAVP indicating she central DI rather than Nephrogenic DI. Fluoxetine does cause DI
but infrequently associated SIADH. 4 )
45.

What is the diagnosis?


1
2
3
4
5

)
Dermatomyositis
)
Erysipelas
)
Lupus pernio
)
Myasthenia gravis
)
Systemic lupus erythematosus
Comments: The characteristic cutaneous features of DM are heliotrope rash Gottrons papules.
Several other cutaneous features, including malar erythema, poikiloderma (ie, variegated
telangiectasia, hyperpigmentation) in a photosensitive distribution, violaceous erythema on
extensor surfaces, periungual cuticular changes, are characteristic of disease even though they
are pathognomonic. The characteristic heliotrope rash consists of a violaceous erythematous
rash (sometimes oedema) in a symmetrical distribution involving periorbital skin. This
frequently subtle may involve only a mild discoloration along eyelid margin. Since a heliotrope
rash rarely observed in other disorders, its presence highly suggestive of DM. Gottrons
papules are found over bony prominences, particularly metacarpophalangeal joints, proximal
interphalangeal joints, and/or distal interphalangeal joints. Papules may also be found overlying
elbows, knees, and/or feet. The lesions consist of slightly elevated violaceous papules plaques
may be scaly. Nailfold changes consist of periungual telangiectases and/or a characteristic
cuticular change hypertrophy of cuticle small hemorrhagic infarcts this hypertrophic area. 1 )

247

You might also like